PROLOG Reproductive Endocrinology and Infertility, 7e (2015) - (1934984442) - (ACOG)
PROLOG Reproductive Endocrinology and Infertility, 7e (2015) - (1934984442) - (ACOG)
seventh edition
Critique Book
ISBN 978-1-934984-44-4
Copyright 2015 by the American College of Obstetricians and Gynecologists. All rights
reserved. No part of this publication may be reproduced, stored in a retrieval system, posted
on the Internet, or transmitted, in any form or by any means, electronic, mechanical, photo-
copying, recording, or otherwise, without prior written permission from the publisher.
12345/98765
PROLOG Task Force for Reproductive Endocrinology and Infertility, Seventh Edition
COCHAIRS MEMBERS
Bruce R. Carr, MD Victor E. Beshay, MD
Department of Obstetrics and Department of Obstetrics and
Gynecology Gynecology
University of Texas Southwest University of Texas Southwestern School
Medical Center of Medicine
Dallas, Texas Dallas, Texas
Daniel R. Grow, MD Krystene B. DiPaola, MD
Professor and Chair Assistant Professor
Department of Obstetrics and University of Cincinnati
Gynecology UC Center for Reproductive Health
Tufts University School of Medicine West Chester, Ohio
Baystate Medical Center Erika B. Johnston-MacAnanny, MD
Springfield, Massachusetts Medical Director
Wake Forest Center for Reproductive
Medicine
Assistant Professor, Reproductive
Medicine
Office of Women in Medicine and
Science
Wake Forest Baptist Medical Center
Winston-Salem, North Carolina
J. Ricardo Loret de Mola, MD
Professor and Chairman
Department of Obstetrics and
Gynecology
Southern Illinois University
Springfield, Illinois
iii
iv PROLOG
PROLOG Task Force for Reproductive Endocrinology and Infertility, Seventh Edition
(continued)
Jennifer E. Mersereau, MD John T. Queenan Jr, MD
Assistant Professor Professor, Obstetrics and Gynecology
Division of Reproductive University of Rochester Medical Center
Endocrinology and Infertility School of Medicine and Dentistry
Department of Obstetrics and Rochester, New York
Gynecology Spencer S. Richlin, MD
University of North Carolina Surgical Director
Chapel Hill, North Carolina Reproductive Medicine Associates of
Steven T. Nakajima, MD Connecticut
Clinical Professor and Director of the Division Director
IVF Outreach Program Reproductive Endocrinology & Infertility
Co-Director of Clinical Operations, Norwalk Hospital
Reproductive Endocrinology and Norwalk, Connecticut
Infertility
Department of Obstetrics and COLLEGE STAFF
Gynecology Sandra A. Carson, MD
Stanford University Vice President for Education
Palo Alto, California Erica Bukevicz, MBA, MS
Beth J. Plante, MD Senior Director, Educational
Fertility Centers of New England Development and Testing
Reading, Massachusetts Division of Education
Thomas M. Price, MD Christopher T. George, MLA
Associate Professor Editor, PROLOG
Department of Obstetrics &
Gynecology
Division of Reproductive
Endocrinology & Fertility
Duke University School of Medicine
Durham, North Carolina
PROLOG v
Process
The PROLOG series offers the most current information available in five areas of the spe-
cialty: obstetrics, gynecology and surgery, reproductive endocrinology and infertility, gyne-
cologic oncology and critical care, and patient management in the office. A new PROLOG
unit is produced annually, addressing one of those subject areas. Reproductive Endocrinology
and Infertility, Seventh Edition, is the third unit in the seventh 5-year PROLOG series.
Each unit of PROLOG represents the efforts of a special task force of subject experts
under the supervision of an advisory committee. PROLOG sets forth current information as
viewed by recognized authorities in the field of women’s health. This educational resource
does not define a standard of care, nor is it intended to dictate an exclusive course of manage-
ment. It presents recognized methods and techniques of clinical practice for consideration by
obstetrician–gynecologists to incorporate in their practices. Variations of practice that take
into account the needs of the individual patient, resources, and the limitations that are special
to the institution or type of practice may be appropriate.
Each unit of PROLOG is presented as a two-part set, with performance information and
cognate credit available to those who choose to submit their answer sheets for confidential
scoring. The first part of the PROLOG set is the Assessment Book, which contains educa-
tional objectives for the unit and multiple-choice questions, and an answer sheet with a return
mailing envelope. Participants can work through the book at their own pace, choosing to use
PROLOG as a closed- or open-book assessment. Return of the answer sheet for scoring is
encouraged but voluntary.
The second part of PROLOG is the Critique Book, which reviews the educational objec-
tives and items set forth in the Assessment Book and contains a discussion, or critique, of
each item. The critique provides the rationale for correct and incorrect options. Current,
accessible references are listed for each item.
vii
viii PROLOG
Conclusion
PROLOG was developed specifically as a personal study resource for the practicing obste-
trician–gynecologist. It is presented as a self-assessment mechanism that, with its accom-
panying performance information, should assist the physician in designing a personal,
self-directed learning program. The many quality resources developed by the College,
as detailed each year in the College’s Publications and Educational Materials Catalog,
are available to help fulfill the educational interests and needs that have been identified.
PROLOG is not intended as a substitute for the certification or recertification programs of
the American Board of Obstetrics and Gynecology.
Reproductive Endocrinology and Infertility, Seventh Edition, includes the following topics
(item numbers appear in parentheses):
ix
x PROLOG
MEDICAL MANAGEMENT
Acne in an adolescent patient (84)
Alternative therapies for menopause (50)
Ambiguous genitalia (115)
Amenorrhea and galactorrhea (97)
BRCA mutations (43)
Breast cancer (127)
Bulimia nervosa and binge-eating disorder (52)
Chronic pelvic pain (96)
Condom failure and morning-after contraception (65)
Congenital adrenal hyperplasia (45)
Contraception for a patient with BRCA gene mutation (22)
Contraception for a patient with diabetes mellitus (44, 67)
Contraception for a patient with systemic lupus erythematosus (66)
Contraception for a patient with venous thromboembolism (19)
Contraception for an older patient who smokes (55)
Donor egg in vitro fertilization (29)
Dysmenorrhea (42)
Ectopic pregnancy (64, 99)
Endometrial polyp (4)
Endometriosis (37)
Fertility options after tubal ligation (47)
Fertility preservation in a patient undergoing cancer treatment (32)
Fertility preservation techniques (75)
Functional hypothalamic amenorrhea and osteoporosis (108)
Heavy menstrual bleeding (18, 80, 106)
Hirsutism (112)
Hydrosalpinges and in vitro fertilization–embryo transfer (12)
Hysterosalpingography complications (89)
Intrauterine device complications with infection (102)
Intrauterine microinsert and pregnancy (35)
Intrauterine microinsert follow-up (87)
In vitro fertilization and intracytoplasmic sperm (11)
Labial adhesions in children (30)
Leiomyoma in infertility (36)
Leiomyomas and heavy menstrual bleeding (56)
PROLOG xi
PHYSIOLOGY
Ambiguous genitalia (115)
Hormonal changes in pregnancy (139–142)
Low bone mass and bone physiology (94)
Mechanism of action of hormonal contraceptives (146–148)
Normal menstrual cycle (41)
Perimenopausal changes (28)
Physiology of the menstrual cycle (25)
Reproductive function, nutritional status, and protein levels (15)
SURGICAL MANAGEMENT
Bilateral tubal ligation and future fertility (123)
Complications of robotic-assisted surgery (105)
Endometriosis and infertility (14)
Hysteroscopic complications (24)
Laparoscopic surgery complications (159–161)
Müllerian anomalies (23)
Oligospermia (77)
Ovarian cancer in a BRCA1-positive patient (13)
Patient with gastric bypass (9)
COUNSELING
Bilateral tubal ligation and future fertility (123)
Bioidentical hormones (113)
Condom failure and morning-after contraception (65)
Contraception for a patient with BRCA gene mutation (22)
Contraception for a patient with diabetes mellitus (44)
Contraception for a patient with systemic lupus erythematosus (66)
Contraception for a patient with venous thromboembolism (19)
Contraception for an older patient who smokes (55)
Depot medroxyprogesterone acetate and bone loss (58)
Donor oocyte use (126)
Donor sperm use (119)
Intrauterine device complications with infection (102)
In vitro fertilization and intracytoplasmic sperm (11)
Luteal phase deficiency (118)
Multifetal pregnancy reduction (116)
Normal menopausal transition (120)
Obesity and contraceptive choices (17)
Primary ovarian insufficiency (6)
Uterine fibroid embolization (63)
Window of fertilization (121)
A complete subject matter index appears at the end of the Critique Book.
1
In vitro fertilization
A 44-year-old woman comes to your office to discuss in vitro fertilization (IVF). She underwent
a tubal ligation 15 years ago. She now has a new partner with whom she would like to conceive.
She and her partner are not interested in using donor eggs for an IVF cycle. You tell her that in her
age group your clinic’s live-birth rate for IVF without donor eggs is less than 1% per attempt. The
most significant guiding issue to consider before deciding whether to proceed with IVF is
Patient autonomy is defined as an individual’s right to assistance. However, given her age, her chance of preg-
hold views, make choices, and take actions based on his nancy and subsequent live birth with assisted reproductive
or her own personal values and beliefs. A medical ethics technology, such as IVF, using her own eggs is less than
framework can assist patients and health care providers in 1% per attempt. The chance of live birth with donor egg
making difficult medical decisions. In a principle-based IVF, however, is more than 50% per attempt. The deci-
health care ethics system, autonomy is one of the four sion to proceed with third-party reproduction is a compli-
principles used to identify and analyze ethical issues in a cated one for many patients and in itself may have ethical,
medical dilemma: legal, financial, and emotional ramifications. In this case,
• Autonomy the couple is not interested in considering donor egg IVF.
• Beneficence A complicating factor when considering futility is
that patients and health care providers may interpret the
• Nonmaleficence
statistics in different ways. Whereas a health care pro-
• Justice vider may feel that the risk of harm from IVF outweighs
In the case described, the most significant guiding issue the likelihood of live birth, an individual patient may
before deciding whether to proceed with IVF is patient interpret a 1% chance of live birth to be hopeful odds. In
autonomy. addition, patients and health care providers may interpret
In certain cases, the reproductive goals of a particular the final goal differently. For some patients, attempted
patient have a low or nonexistent likelihood of success. conception with IVF, even if the odds of pregnancy are
Generally, the term “futile” refers to treatments that exceedingly low, may provide them with information
have a 1% or less chance of success per cycle. The term about whether or not they can produce eggs and, there-
“very poor prognosis” refers to low odds, ie, a 1–5% fore, could help them feel that they have tried everything
chance of live birth per cycle. Most patients and health before they accept the reality of their situation.
care providers are able to reach a consensus about the A health care provider’s professional interest may
realistic chances of success and to redirect efforts to influence their decisions about cases of medical futility.
options such as third-party assistance in reproduction Ethical concerns arise about the cost of fertility treat-
(eg, egg or sperm donation), adoption, or acceptance ments when the odds of live birth are exceedingly low
of a child-free life. However, there are instances where and in instances when charging patients or insurance
individuals or couples are interested in pursuing whatever providers could become fraudulent. However, given the
treatments are available to achieve their reproductive complexities with interpreting “futility” and “very poor
goals, regardless of the odds of success. In these situa- prognosis,” there are no clear guidelines about when it
tions, ethical issues may arise that force choices between is unacceptable to receive payment for treatments that
patient autonomy and the health care provider’s medical have little chance of success. A patient’s ability to pay
recommendation. for fertility treatments, regardless of the odds of suc-
Because of her prior tubal ligation, the described cess, should not guide the clinician in making medical
patient is unable to attempt conception without medical recommendations.
Additional questions may arise about the rights and guiding factor when deciding about potentially futile IVF
legal duties a health care provider has in regard to pro- treatments.
ceeding with or refusing treatment. The relationship The ethics committee at the American Society for
between the health care provider and patient is consen- Reproductive Medicine recommends that decisions about
sual. That is, the patient and the health care provider have treating or refusing to treat patients should be patient-
the right to refuse to pursue treatment with which they do focused to promote patient autonomy. If a patient has
not feel comfortable, as long as they do not violate laws been counseled adequately about the low chance of suc-
about impermissible discrimination. In fact, health care cess and has reviewed the risks and benefits of treatment,
providers are permitted to terminate the patient–physi- it is considered reasonable for health care providers to
cian relationship as long as they provide timely notice proceed with fertility treatments. Strong consideration
and information about other health care providers who should be given to referral for psychologic counseling.
may have better success rates for individual situations, if Clinics should consider developing explicit policies to
applicable. guide decisions about pursuing futile fertility treatments.
Nonmaleficence is the principal of avoiding medical This will allow consistency and prevent having to make
care that may cause harm to a patient. The actual medical decisions on a case-by-case basis. Ideally, these policies
risks of IVF treatment are minimal. Many of the medi- should be discussed with patients when the initial deci-
cations are injectable and have a small risk of bruising sion about treatment is being made.
and pain. A remote risk exists of complications, such as
ovarian hyperstimulation syndrome, pelvic injury dur- Ethical decision making in obstetrics and gynecology. ACOG
ing an egg retrieval, infection, ovarian torsion, ectopic Committee Opinion No. 390. American College of Obstetricians and
Gynecologists. Obstet Gynecol 2007;110:1479–87.
pregnancy, and multiple gestation. However, the risk of
Fertility treatment when the prognosis is very poor or futile: a commit-
serious complications from an IVF cycle is low enough tee opinion. Ethics Committee of American Society for Reproductive
that the principal of nonmaleficence should not be a Medicine. Fertil Steril 2012;98:e6–9.
Reproductive Endocrinology and Infertility 3
2
Ultrasonographic findings in a patient who takes tamoxifen citrate
(A) hyperplasia
(B) cancer
(C) cystic atrophy
* (D) polyps
(E) submucosal leiomyoma
Tamoxifen citrate is a selective estrogen receptor modu- changes include endometrial polyps (with the incidence
lator with a complex mechanism of action including between 8% and 36% versus 0–10% in untreated women)
antiestrogenic activity in the breast and estrogenic effects and endometrial hyperplasia (1.3–20% versus 0–10% in
in other tissues such as the endometrium. It is widely untreated women). The risk of endometrial carcinoma
used for the treatment of breast cancer and for chemo- for treated women is 1.3–7.5-fold higher than the risk
prevention in high-risk premenopausal and postmeno- for untreated women; cystic atrophy, adenomyosis, and
pausal women. Tamoxifen is approved by the U.S. Food leiomyomas also have been reported. For the described
and Drug Administration for breast cancer prevention patient, the most common abnormality that is likely to
in women aged 35 years or older. Tamoxifen reduces be identified by means of ultrasonography is endometrial
the risk of breast cancer by nearly 50%. This effect is polyps. The most appropriate treatment for her is hystero-
observed even in women with up to three first-degree scopic resection.
relatives with breast cancer, but the medication only Detailed guidelines are available for the follow-up of
reduces the risk of estrogen receptor-positive breast patients who take tamoxifen for the treatment of breast
cancer. cancer. Currently, there is no active screening for asymp-
The finding of a decrease in contralateral breast cancer tomatic patients treated with tamoxifen other than routine
incidence after tamoxifen administration for adjuvant gynecologic care.
therapy has led to its use in breast cancer prevention. Four
large prevention trials of tamoxifen and cancer have dem- Davies C, Pan H, Godwin J, Gray R, Arriagada R, Raina V, et al.
Long-term effects of continuing adjuvant tamoxifen to 10 years versus
onstrated the benefits of using tamoxifen. In the National stopping at 5 years after diagnosis of oestrogen receptor-positive breast
Surgical Adjuvant Breast and Bowel Project’s Breast cancer: ATLAS, a randomised trial. Adjuvant Tamoxifen: Longer
Cancer Prevention Trial, tamoxifen reduced the risk of Against Shorter (ATLAS) Collaborative Group [published erratum
invasive breast cancer by 49%. A recent study suggests appears in Lancet 2013;381:804]. Lancet 2013;381:805–16.
extension of therapy to 10 years. Palva T, Ranta H, Koivisto AM, Pylkkanen L, Cuzick J, Holli K. A
double-blind placebo-controlled study to evaluate endometrial safety
One important determinant of tamoxifen safety is the and gynaecological symptoms in women treated for up to 5 years with
possible effects of the drug on the endometrium. The first tamoxifen or placebo—a substudy for IBIS I Breast Cancer Prevention
report to connect tamoxifen and endometrial cancer was Trial. Eur J Cancer 2013;49:45–51.
published in 1989, and since then, several reports have Pinkerton JV, Goldstein SR. Endometrial safety: a key hurdle for
described the endometrial changes in postmenopausal selective estrogen receptor modulators in development. Menopause
2010;17:642–53.
breast cancer patients treated with tamoxifen. The results
Polin SA, Ascher SM. The effect of tamoxifen on the genital tract.
also indicated that even though the risk-reducing effect Cancer Imaging 2008;8:135–45.
of tamoxifen appeared to persist for at least 10 years,
Tamoxifen and uterine cancer. Committee Opinion No. 601. Amer-
most of the adverse effects did not continue after the ican College of Obstetricians and Gynecologists. Obstet Gynecol
5-year treatment period. The most common endometrial 2014;123:1394–7.
4 PROLOG
3
Antiphospholipid syndrome
A 28-year-old woman, gravida 3, para 0, comes to your office for evaluation of recurrent pregnancy
loss. Her history is significant for a deep vein thrombosis at age 21 years. You are concerned that
she might have antiphospholipid syndrome (APS). In addition to testing for lupus anticoagulant and
anticardiolipin antibodies, the factor that you should screen for is
Antiphospholipid syndrome is an autoimmune disorder complex. A number of other factors can cause prolonged
characterized by venous or arterial thrombosis, recur- clotting times, so additional testing is necessary if the
rent fetal loss, or placental insufficiency in the setting initial screening is positive. Lupus anticoagulant cannot
of antiphospholipid antibodies. Antiphospholipid anti- be quantified and, thus, is ultimately only reported as
bodies are present in approximately 5% of the general present or absent.
reproductive-aged population and in 15% of patients with Anticardiolipin antibodies and anti-β2-glycoprotein I
recurrent miscarriages. The syndrome is characterized antibodies usually are detected using enzyme-linked
by at least one clinical feature plus at least one specific immunosorbent assays. The IgG and IgM isotypes should
laboratory criterion. It can occur on its own or together be measured. Results are reported in international stan-
with another autoimmune disease, infectious disease, dard units: G phospholipid and M phospholipid for IgG
malignancy, or certain medications; up to 37% of patients
with systemic lupus erythematosus also will have APS.
The diagnostic criteria for APS have changed over BOX 3-1
time but are currently based on a consensus statement
published in 2004 and revised in 2006 (Box 3-1 and Laboratory Criteria for the Diagnosis of
Antiphospholipid Syndrome
Box 3-2). In addition to including one clinical feature,
the criteria for APS require the persistent presence for 1. Lupus anticoagulant is present in the plasma
more than 12 weeks of medium-to-high titers of at on two or more occasions at least 12 weeks
least one specific autoantibody of immunoglobulin G apart. It is interpreted as either present or
(IgG) or immunoglobulin M (IgM) isotype for anti- absent. Testing for lupus anticoagulant is ide-
ally performed before the patient is treated
β2-glycoprotein I or anticardiolipin, or the presence of with anticoagulants, or
lupus anticoagulant. Transient positive tests may occur 2. Anticardiolipin antibody of immunoglobulin
and, therefore, two positive laboratory tests at least G and/or immunoglobulin M isotype is in the
12 weeks apart are required to diagnose APS. Many dif- serum or plasma, present in medium or high
ferent antiphospholipid antibodies have been identified titer (ie, greater than 40 G phospholipid or M
phospholipid, or greater than the 99th per-
that are related to each other but still distinctly different.
centile) on two or more occasions at least
Only lupus anticoagulant, anticardiolipin, or anti-β2- 12 weeks apart, or
glycoprotein I can be used in diagnosing APS. Testing for 3. Anti-β2-glycoprotein I of immunoglobulin G
other antibodies, such as phosphatidylserine antibodies, and/or immunoglobulin M isotype is present in
annexin V antibodies, or phosphatidylinositol antibodies, the serum or plasma (in titer greater than 99th
is not recommended. percentile for a normal population as defined
by the laboratory performing the test) on two or
Lupus anticoagulant is associated with thrombosis and more occasions at least 12 weeks apart.
not anticoagulation. The presence of lupus anticoagulant
is assessed indirectly with a series of tests being required Modified from Miyakis S, Lockshin MD, Atsumi T, Branch
DW, Brey RL, Cervera R, et al. International consensus
to confirm its presence. Initially, a combination of sensi-
statement on an update of the classification criteria for
tive clotting assays are used to screen for lupus antico- definite antiphospholipid syndrome (APS). J Thromb
agulant. Lupus anticoagulant, if present, prolongs clotting Haemost 2006;4:295–306.
times by interfering with the assembly of the prothrombin
Reproductive Endocrinology and Infertility 5
4
Endometrial polyp
A 30-year-old woman, gravida 2, para 2, comes to your office with bleeding between menstrual
periods. Her Pap test 3 months ago was normal and speculum examination in your office identi-
fies a normal cervix and vaginal mucosa. The urinary human chorionic gonadotropin test result is
negative. Saline sonohysterography reveals a 1-cm endometrial polyp. She is interested in future
fertility. The treatment option that is most likely to improve her symptoms is
Endometrial polyps are benign, focal overgrowths of benefit of surgical polypectomy in regard to fertility also
endometrial tissue that are covered by epithelium and is debated. Theorized mechanisms by which endometrial
contain variable amounts of glands, stroma, and blood polyps could adversely affect reproductive performance
vessels. They may commonly cause intermenstrual bleed- include irregular intraendometrial bleeding; creation of
ing and are diagnosed easily by two-dimensional ultraso- an inflammatory endometrial response similar to an intra-
nography or saline-infused sonohysterography. Because uterine device; an obstructive defect that inhibits sperm
polyps also may be asymptomatic, their causative role in transport; a physical surface area effect that prevents
abnormal menstrual bleeding has been questioned. The exposure of the embryo to the endometrium; and an endo-
prevalence of polyps in women who report abnormal crine surface area effect in which increased endometrium
uterine bleeding ranges from 13% to 50%. Approximately surface area results in increased secretion of glycodelin,
10% of asymptomatic premenopausal women were found which has been shown to inhibit sperm binding to the
to have small endometrial polyps. zona pellucida. When evaluated retrospectively, surgical
The likelihood of an endometrial polyp being malignant treatments to remove polyps have been noted to result in
is small, even with symptomatic irregular bleeding. The improvement in bleeding symptoms and in high satisfac-
lifetime risk of developing adenocarcinoma of the endo- tion rates.
metrium is approximately 2–3%. Somewhat higher rates Progestin withdrawal triggers release of lysosomal
are observed in obese women with unopposed estrogen, enzymes in the endometrial stromal cells, which promote
such as patients with polycystic ovary syndrome. Oral enzymatic digestion of all the components of the glands,
progestin would induce cyclic menses in this population stroma, and connective tissue of the endometrial func-
but would not cause eradication of the endometrial polyp. tionalis. This cyclical process of autodigestion occurs
Endometrial biopsy is helpful in evaluation of irregular monthly after progesterone production stops with involu-
bleeding secondary to endometrial hyperplasia; however, tion of the corpus luteum.
in the described patient, an endometrial polyp has been
confirmed with sonohysterography. A dilation and curet- Lieng M, Istre O, Qvigstad E. Treatment of endometrial polyps: a sys-
tematic review. Acta Obstet Gynecol Scand 2010;89:992–1002.
tage without hysteroscopy may have the potential to miss
Perez-Medina T, Bajo-Arenas J, Salazar F, Redondo T, Sanfrutos L,
resection of the pathology. Endometrial ablation is contra- Alvarez P, et al. Endometrial polyps and their implication in the preg-
indicated if future fertility is desired. nancy rates of patients undergoing intrauterine insemination: a prospec-
Hysteroscopic resection of endometrial polyps usually tive, randomized study. Hum Reprod 2005;20:1632–5.
is performed in order to exclude atypical or malignant Silberstein T, Saphier O, van Voorhis BJ, Plosker SM. Endometrial
polyps in reproductive-age fertile and infertile women. Isr Med Assoc
endometrial changes, to relieve abnormal uterine bleed- J 2006;8:192–5.
ing, or to potentially improve fertility. The effect that Tjarks M, Van Voorhis BJ. Treatment of endometrial polyps. Obstet
an endometrial polyp has on fertility is unclear. The Gynecol 2000;96:886–9.
Reproductive Endocrinology and Infertility 7
5
Evaluation of male factor infertility
A 38-year-old nulligravid woman requests your help to achieve pregnancy. Her male partner is
noted to have azoospermia on semen analysis, but with normal volume; the result is confirmed on
repeat testing. His general physical examination does not reveal any additional abnormalities. The
next step in evaluation of the male partner should include assessment of
In addition to the male partner’s history, semen analy- TABLE 5-1. Normal Semen Parameters by World Health
sis is the mainstay of the male evaluation. Although it Organization Five Criteria
should not be considered a test of fertility, an abnormal
Parameter Value
semen analysis suggests that the probability of achieving
fertility is lower than normal. It should be performed Volume 1.5 mL
after 2–3 days of abstinence. The semen should be col- Concentration 15 million/mL
lected by masturbation and without the use of lubricants. Progressive motility 32%
Couples may opt to use intercourse for collection; if this Total motility 40%
is their preference, a special collection condom should Normal forms (morphology) 4% by Kruger strict
be provided. The sample should be transported to the criteria
laboratory within 1 hour of collection while kept at body Cooper TG, Noonan E, von Eckardstein S, Auger J, Baker HW,
temperature (specimen cup should be placed close to the Behre HM, et al. World Health Organization reference values
skin during transport). The laboratory then assesses the for human semen characteristics. Hum Reprod Update 2010;
specimen for time needed to liquefy, sperm count, sperm 16:231–45.
motility, sperm morphology, and number of round cells,
which may signify the presence of either white blood
cells or immature sperm. stimulating hormone and luteinizing hormone levels with
Table 5-1 shows normal semen parameters according a low testosterone level) or hypogonadotropic hypogo-
to the World Health Organization. In cases in which an nadism (ie, low levels of both hormones with or without
abnormality in the semen parameters is detected, the high prolactin, and low testosterone level). Genetic
semen analysis should be repeated 2–3 months later, testing with chromosomal analysis and Y chromosome
given that spermatogenesis takes 60–80 days to complete. microdeletion analysis also should be performed because
Thus, a semen analysis collected today reflects biologic the chance of encountering a genetic abnormality is
influences 60–80 days ago. approximately 15% in men with azoospermia. These tests
Azoospermia (no sperm detected in the specimen) also should be considered in severe oligospermia where
can be the result of either obstructive or nonobstructive the risk of genetic abnormalities is approximately 5%.
causes. In cases where azoospermia is encountered, a Fructose testing will differentiate between obstructive
physical examination should be performed to rule out and nonobstructive causes of azoospermia. In addition,
obstructive problems such as congenital absence of the results can be obtained rapidly during a semen analysis.
vas deferens, which may be bilateral and is commonly Cystic fibrosis testing will be of value if obstructive
seen in individuals affected by cystic fibrosis gene muta- azoospermia is suspected and the physical examination
tions. Fructose, the energy source in sperm, is absent in reveals absent vas deferens. White blood cells may affect
cases of obstructive azoospermia. The next step in man- sperm function but would not be the cause of azoosper-
agement of the described male partner is to evaluate for mia. Antisperm antibodies are a cause of low motility but
fructose in the semen. not azoospermia. Postejaculatory urinalysis for sperm
In addition to fructose testing, evaluation of gonado- should not be performed except in cases of presence
tropins, prolactin, and sex hormones needs to be done in of fructose in semen, which indicates a nonobstructive
order to evaluate for testicular failure (ie, high follicle- etiology.
8 PROLOG
Anguiano A, Oates RD, Amos JA, Dean M, Gerrard B, Stewart C, et Reijo R, Lee TY, Salo P, Alagappan R, Brown LG, Rosenberg M,
al. Congenital bilateral absence of the vas deferens. A primarily genital et al. Diverse spermatogenic defects in humans caused by Y chromo-
form of cystic fibrosis. JAMA 1992;267:1794–7. some deletions encompassing a novel RNA-binding protein gene. Nat
Genet 1995;10:383–93.
Cooper TG, Noonan E, von Eckardstein S, Auger J, Baker HW, Behre
HM, et al. World Health Organization reference values for human
semen characteristics. Hum Reprod Update 2010;16:231–45.
6
Primary ovarian insufficiency
A 29-year-old nulligravid woman is concerned about her fertility and wants a second opinion. She
has noticed spacing out of her menstrual intervals to every 3 months along with some vasomotor
symptoms and insomnia. She tells you that her younger sister was diagnosed with “premature
menopause,” but she is not sure of the specifics. Her primary care provider obtained a day-3 follicle-
stimulating hormone (FSH) level of 44.3 mIU/mL and a 46,XX karyotype. A repeat day-3 FSH was
56.1 mIU/mL and estradiol level was 16 pg/mL. She was told by her primary care provider that
she has entered menopause and can no longer conceive. You inform her that the best next step to
explain the findings is to screen for
The diagnosis of secondary amenorrhea as a result of Premature ovarian failure was first noted to be associ-
ovarian insufficiency is made when a patient younger ated with heterozygotic carriers of the fragile X premuta-
than 40 years has amenorrhea for 3 or more months with tion in the early 1990s. This syndrome is now known
laboratory findings of an elevated FSH level in the meno- as primary ovarian insufficiency. Affected individuals
pausal range (greater than 40 mIU/mL on two separate will have expansion of the CGG tandem repeat on the X
occasions weeks apart). Any patient in this age group who chromosome in a region that has been named the fragile
experiences secondary amenorrhea should be evaluated, X mental retardation 1 (FMR1) gene. Approximately
especially if she has symptoms of estrogen deficiency 16–24% of women with this premutation will go on
such as hot flushes or vaginal dryness. to develop primary ovarian insufficiency. In contrast,
The causes of primary ovarian insufficiency include women with primary ovarian insufficiency have a 3% risk
genetic abnormalities of the X chromosome (eg, 45,X of fragile X premutation with no family history and up
and fragile X premutations), enzyme defects such as to 12% when one or more siblings are affected. Women
17α-hydroxylase deficiency, aromatase deficiency, or gal- with infertility due to primary ovarian insufficiency who
actosemia. Other etiologies include chemotherapy, radia- have a normal karyotype should be tested for the fragile
tion, viral infection, autoimmune disorders, and unknown X premutation. The best next step to explain this patient’s
causes. results is to screen for fragile X syndrome.
This patient has primary ovarian insufficiency and Cystic fibrosis is an autosomal recessive disorder that
there is reason to suspect that her younger sister also is leads to the production of thickened secretions that can
affected, which would point to a genetic cause. Her chro- affect pulmonary, gastrointestinal, and reproductive func-
mosomal analysis is 46,XX. A karyotype was obtained tions in individuals who are homozygous for the disease.
in order to rule out the presence of a Y chromosome. Niemann–Pick disease describes a group of diseases that
Because of the potential risk of developing germ cell are inherited in an autosomal recessive manner. Affected
tumors, the presence of a Y chromosome is an indication patients typically have deficient acid sphingomyelinase
to remove abnormal intra-abdominal gonads. activity or defective transport of low-density lipoprotein
Reproductive Endocrinology and Infertility 9
cholesterol. Gaucher disease patients have defective acid Current evaluation of amenorrhea. Practice Committee of the Ameri-
can Society for Reproductive Medicine. Fertil Steril 2006;86(suppl):
beta glucosidase activity. Each of these conditions is an S148–55.
autosomal recessive disorder and could be seen in two Testing and interpreting measures of ovarian reserve: a committee
siblings. However, cystic fibrosis, Niemann–Pick dis- opinion. Practice Committee of the American Society for Reproductive
ease, and Gaucher disease do not cause primary ovarian Medicine. Fertil Steril 2012;98:1407–15.
insufficiency.
7
Migraine and use of oral contraceptives
A 31-year-old woman has a history of frequent migraines without aura around the time of her men-
ses. She has read that an oral contraceptive (OC) might help control these headaches. She reports
that she experiences throbbing, centrally located pain; numbness of her fingers; and nausea that
occur 1–3 days before and during menses. The symptom that she experiences that will influence
the choice of OC is
Intermittent headaches are not a contraindication to the are preceded by nausea and occasional vomiting. Affected
use of OCs unless the patient has migraine headaches individuals also demonstrate a marked sensitivity to light,
preceded by an aura that often includes the presence of smell, touch, or sound. The aura may manifest as the
associated focal neurologic signs. A migraine headache perception of a strange light or an unpleasant smell, or it
preceded by an aura increases twofold the risk of a stroke may begin with confusing thoughts or experiences. It is
in an otherwise healthy individual. not uncommon for an affected individual to have multiple
The presence of an isolated headache can be due to different auras before their headache, but many have the
a number of factors. Some headaches can be associated same aura symptom(s) before their migraine attack.
with the fall of estrogen and progesterone levels 1–3 days Migraines may be precipitated by stress, alcohol, and
before menses (in the late luteal phase), which can lead foods that are rich in tyramine and tryptophan such as red
to menstrual-related headache, sometimes referred to as wine, chocolate, and ripe cheeses. Migraines are usually
catamenial headache. cyclical in frequency, but can be completely absent for
Other headaches can be classified as migraine. The a period. Women with a history of migraine headaches
headache is usually the most prominent feature of a num- without an aura are not at increased risk of stroke unless
ber of associated neurologic and systemic symptoms that they have other major risk factors, such as tobacco smok-
compose a migraine attack. Migraine attacks are common ing, hypertension, diabetes mellitus, or age greater than
and occur in approximately 43% of women at some time 35 years. Headaches also may be exacerbated after initia-
in their life. Patients with a migraine headache often have tion of OCs.
a throbbing, centrally located pain that begins slowly and Women with migraines and an associated preceding
worsens over 1–2 hours. aura often have associated focal neurologic symptoms.
Migraines associated with an aura are designated “clas- These symptoms are the most severe and worrisome.
sic migraines,” whereas migraines without an aura are Focal neurologic deficits can affect the functions of the
known as “common migraines.” The aura that precedes a brain, spinal cord, and central nervous system, which
classic migraine is a perceptual disturbance, often a repet- leads to alterations in movement (paralysis, weakness,
itive telltale sensation that occurs for a few minutes or up tremor), sensation (paresthesia, numbness), speech,
to an hour before the headache. The aura may persist for vision, and hearing. In the described patient, the symptom
the duration of the migraine, often leaving the patient dis- that will influence the choice of OC is the reported numb-
oriented and confused. Migraine attacks with aura often ness of her fingers.
10 PROLOG
Women with a history of migraine headaches with Charles A. Advances in the basic and clinical science of migraine. Ann
Neurol 2009;65:491–8.
aura should use a non-estrogen-mediated contraceptive
Understanding and using the U.S. Selected Practice Recommendations
method, such as the progestin-only OC, an intrauterine for Contraceptive Use, 2013. Committee Opinion No. 577. American
device, a progestin injection or implant, or barrier meth- College of Obstetricians and Gynecologists. Obstet Gynecol 2013;122:
ods to prevent unintended conception. 1132–3.
Other management options for menstrual migraine U.S. Medical Eligibility Criteria for Contraceptive Use, 2010. Centers
for Disease Control and Prevention. MMWR Recomm Rep 2010;59
include initiation of nonsteroidal antiinflammatory drugs (RR-4):1–86.
begun 7 days before the onset of menses and continued U.S. Selected Practice Recommendations for Contraceptive Use, 2013:
throughout the patient’s menses. Triptan medications, adapted from the World Health Organization selected practice recom-
such as sumatriptan succinate or naratriptan hydrochlo- mendations for contraceptive use, 2nd edition. Division of Reproductive
Health, National Center for Chronic Disease Prevention and Health
ride, often are used in conjunction with nonsteroidal Promotion, Centers for Disease Control and Prevention. MMWR
antiinflammatory drugs. The triptan medication is started Recomm Rep 2013;62(RR-05):1–60.
2–3 days before menses and continued for a total treat- Use of hormonal contraception in women with coexisting medical
ment period of 5–6 days. conditions. ACOG Practice Bulletin No. 73. American College of
Obstetricians and Gynecologists. Obstet Gynecol 2006;107:1453–72.
Bonnema RA, McNamara MC, Spencer AL. Contraception choices in
women with underlying medical conditions. Am Fam Physician 2010;
82:621–8.
8
Assisted reproductive technology and ovarian hyperstimulation syndrome
A 36-year-old patient with primary infertility undergoes in vitro fertilization (IVF). She develops
abdominal pain, bloating, and oliguria after her egg retrieval. The risk factor most commonly asso-
ciated with these complications is
Ovarian hyperstimulation syndrome is a complication reduced end-organ perfusion, especially to the kidneys.
of assisted reproductive techniques. This disease state is Additionally, third spacing into the pleural space can lead
characterized by enlarged ovaries and acute fluid shifts to respiratory distress syndrome. Vascular changes associ-
from the intravascular space into the extravascular space ated with ovarian hyperstimulation syndrome ultimately
(peritoneal cavity and thoracic cavity). These therapies lead to an increased risk of thromboembolic events in
can include IVF and, less frequently, clomiphene citrate these patients.
(low risk) and aromatase inhibitors, such as tamoxifen The primary symptoms of ovarian hyperstimulation
citrate and letrozole. Ovarian hyperstimulation syndrome syndrome are abdominal fullness and shortness of breath.
is caused by increased vascular permeability through The incidence of moderate-to-severe ovarian hyper-
ovarian hypersecretion of vascular endothelial growth stimulation syndrome can vary between 5% and 8%
factor (VEGF) activating VEGF receptor 2 (VEGF-2); of all total IVF cycles. Mild symptoms typically com-
VEGF levels are increased after human chorionic gonado- prise lower abdominal pain, nausea, vomiting, and diar-
tropin (hCG) is administered in assisted reproductive tech- rhea. Moderate-to-severe symptoms include persistent
nology (ART) cycles to help induce ovulation and final and worsening ascites. Serious symptoms consist of
maturation of the growing oocytes. As levels of VEGF rapid weight gain, tense ascites, hemodynamic instability
increase after hCG administration, it stimulates vascular from orthostatic hypotension, tachypnea, and progres-
permeability by interacting with the VEGF-2 receptor. sive oliguria. Severe symptoms, seen in up to 1.4% of all
Massive extravasation caused by ovarian hyperstimu- women who undergo ART cycles, can be life threatening
lation syndrome can lead to hemoconcentration with (Table 8-1). Laboratory findings associated with serious
Reproductive Endocrinology and Infertility 11
Classification of Ovarian
Hyperstimulation
Syndrome Symptoms Clinical Features Laboratory Values
Mild Abdominal discomfort/distention
Mild nausea/vomiting
Diarrhea
Enlarged ovaries
Moderate Mild features plus
Ultrasonographic evidence of ascites Elevated hematocrit greater than 41%
Elevated leukocytes greater than
15,000/mm3
Severe Mild and moderate features plus
Clinical evidence of ascites Hematocrit greater than 51%
Hydrothorax Elevated leukocytes greater than
25,000/mm3
Severe dyspnea Creatinine clearance less than 50 mL/min
Oliguria/anuria Creatinine greater than 1.6 mL/min
Intractable nausea/vomiting Sodium less than 135 mEq/L
Tense ascites Potassium greater than 5 mEq/L
Low blood pressure/central venous Elevated liver enzymes
pressure
Rapid weight gain
Syncope
Severe abdominal pain
Critical Mild, moderate, and severe features plus
Anuria/acute renal failure
Arrhythmia
Thromboembolism
Pericardial effusion
Massive hydrothorax
Arterial thrombosis
Acute respiratory distress syndrome
Sepsis
this medication can significantly reduce the incidence to eventual ischemic injury of the ovary and loss of the
of moderate ovarian hyperstimulation syndrome, as well affected ovary.
as pelvic fluid accumulation and hemoconcentration. Polycystic ovaries are the risk factor most com-
Implantation and ongoing pregnancy–live-birth rates of monly associated with the described patient’s symptoms.
IVF patients at risk of ovarian hyperstimulation syndrome Advanced maternal age, diminished ovarian reserve, and
appear not to be affected by cabergoline administration. obesity are not the factors most commonly associated
Cabergoline administration is warranted in women at risk with these complications.
of ovarian hyperstimulation syndrome, and such women
should be offered this medication to help reduce the inci- Gomez R, Soares SR, Busso C, Garcia-Velasco JA, Simon C, Pellicer
A. Physiology and pathology of ovarian hyperstimulation syndrome.
dence of ovarian hyperstimulation syndrome. Semin Reprod Med 2010;28:448–57.
Patients who undergo ART are at risk of adnexal tor- Humaidan P, Quartarolo J, Papanikolaou EG. Preventing ovarian
sion. The risk is believed to be less than 1%. Patients who hyperstimulation syndrome: guidance for the clinician. Fertil Steril
have undergone an ART cycle have enlargement of the 2010;94:389–400.
ovaries. These patients are at an 11-fold increased risk of Maxwell KN, Cholst IN, Rosenwaks Z. The incidence of both serious
and minor complications in young women undergoing oocyte donation.
adnexal torsion. This is especially true for patients whose Fertil Steril 2008;90:2165–71.
treatment was complicated by ovarian hyperstimulation Ovarian hyperstimulation syndrome. Practice Committee of American
syndrome. Delayed diagnosis of ovarian torsion can lead Society for Reproductive Medicine. Fertil Steril 2008;90:S188–S193.
9
Patient with gastric bypass
A 34-year-old nulligravid woman comes to your office 18 months after Roux-en-Y gastric bypass
surgery. Her body mass index is 28 (calculated as weight in kilograms divided by height in meters
squared). She now has regular cycles and her hypertension has resolved. She is a vegetarian.
Before attempting pregnancy, the most appropriate immediate management for this patient is
Bariatric surgery has become an alternative surgical urinary incontinence, and degenerative joint disease.
choice for morbidly obese patients who cannot lose Children of obese mothers are at risk of childhood obe-
weight through nonsurgical approaches. A patient with sity. Surgical and nonsurgical preconception weight loss
a body mass index of 40 or higher (or 35 or higher with help break this cycle.
comorbidities) is a candidate for bariatric surgery. The This patient now has regular cycles and should attempt
two most common bariatric surgical procedures are Roux- pregnancy on her own with timed intercourse or ovula-
en-Y gastric bypass (restrictive and malabsorptive) and tion predictor kits. Weight loss 2 years after bariatric
gastric banding (restrictive). Because these procedures surgery ranges between 20 kg and 40 kg. More than 70%
are becoming more common, understanding their man- of anovulatory patients will resume regular menses after
agement and complications is vital for preconception and losing 20 kg. Because the described patient is a vegetarian
conception care in patients of reproductive age. with recent bariatric surgery, a nutritional status evalu-
Quality of life in general improves after bariatric ation is indicated before she attempts pregnancy. When
surgery. This is important because it sets the stage for a her nutritional status is optimized, she then can attempt
healthier pregnancy for the mother and the fetus. Bariatric pregnancy.
surgery can lead to a general improvement in health and Nutritional deficiencies can occur after bariatric
a decrease specifically in gestational diabetes mellitus, surgery. This patient had Roux-en-Y gastric bypass
hypertension, preeclampsia, sleep apnea, gastric reflux, surgery. Nutritional deficiencies mostly are seen with
Reproductive Endocrinology and Infertility 13
malabsorptive procedures because part of the intestine is year, the rate of loss decreases and stabilizes. The
bypassed. Serious nutritional deficiencies include vitamin American College of Obstetricians and Gynecologists
D, vitamin B12, vitamin B1 (thiamine), magnesium, folic advises bariatric surgery patients to delay pregnancy for
acid, calcium, and ferritin deficiencies. Patients most at 12–18 months after surgery, during the rapid weight-loss
risk of nutritional deficits include vegetarians such as phase. Waiting to conceive may help the patient avoid
the described patient. In a vegetarian, an evaluation for nutritional deficiencies and intrauterine growth restric-
micronutrient deficiencies is warranted. Preconception tion. Postponing pregnancy an additional 6 months is not
evaluation and consultation with a nutritionist should be warranted. Studies have compared pregnancy outcomes
considered for a patient who has had bariatric surgery. among patients in the first year after bariatric surgery. In
Complications of bariatric surgery carried out during patients who became pregnant in the first year after bar-
pregnancy include intestinal obstruction, gastrointestinal iatric surgery, no increases were observed in pregnancy
hemorrhage, hernias, anastomotic leaks, and band compli- complications or comorbidities, such as hypertension,
cations. Abdominal pain, nausea, and vomiting in a preg- diabetes mellitus, and congenital malformations.
nant patient who has undergone bariatric surgery need to
Bariatric surgery and pregnancy. ACOG Practice Bulletin No.
be evaluated carefully. Early consultation with a bariatric 105. American College of Obstetricians and Gynecologists. Obstet
surgeon is warranted. Gynecol 2009;113:1405–13.
Pelvic ultrasonography would not be the most appro- Merhi ZO. Impact of bariatric surgery on female reproduction. Fertil
priate immediate need for this patient. However, pelvic Steril 2009;92:1501–8.
ultrasonography should be part of the overall evaluation Moran LJ, Norman RJ. The effect of bariatric surgery on female repro-
ductive function. J Clin Endocrinol Metab 2012;97:4352–4.
of any infertile patient.
Sheiner E, Edri A, Balaban E, Levi I, Aricha-Tamir B. Pregnancy out-
The first year after bariatric surgery is when most come of patients who conceive during or after the first year following
weight loss occurs (catabolic phase). In the second bariatric surgery. Am J Obstet Gynecol 2011;204:50.e1–6.
14 PROLOG
10
Macroadenoma and galactorrhea
A 28-year-old nulligravid woman comes to your clinic after 6 months of amenorrhea. Her men-
strual irregularity started initially as irregular cycles 1 year ago, followed by amenorrhea. On
evaluation, she is not pregnant, but her prolactin level is elevated to 85 ng/mL. She has a normal
thyroid-stimulating hormone level. Repeat testing shows a persistently high prolactin elevation of
91 ng/mL. Magnetic resonance imaging reveals the image shown in Figure 10-1. The serum test
level most likely to be elevated in further evaluations is
Loss of gonadotropin-releasing hormone pulsatility results patient is free from neurologic symptoms that require
in abnormal luteinizing hormone and FSH secretion with surgical intervention. Surgical intervention may be used if
resultant anovulation and oligomenorrhea or amenorrhea. the response to medical management of a macroadenoma
The long-term lack of estradiol production may result in is not adequate.
osteopenia or osteoporosis. For this patient, the serum test most likely to be
Hyperprolactinemia can be diagnosed in the presence elevated is IGF-1, given that GH-producing adenomas
of an elevated serum prolactin level. Thyroid function are the second most common type of pituitary macroad-
evaluation and a pregnancy test (to rule out pregnancy enoma. Although FSH and morning cortisol levels can
as a physiologic cause of hyperprolactinemia) should be evaluated, they are less likely to reveal the diagnosis
be performed. Magnetic resonance imaging of the brain because FSH-secreting and adrenocorticotropic hormone-
should be obtained in any patient with a significantly secreting macroadenomas are rare. Measurement of the
elevated prolactin level. However, some practitioners antimüllerian hormone level is not relevant to the evalua-
advocate imaging for all patients with hyperprolactinemia tion of hyperprolactinemia but is helpful in the evaluation
because mildly elevated prolactin levels may be due to the of ovarian reserve.
pituitary stalk compression by a non-prolactin-secreting Beshay VE, Beshay JE, Halvorson LM. Pituitary tumors: diagnosis,
pituitary macroadenoma or a craniopharyngioma. management, and implications for reproduction. Semin Reprod Med
2007;25:388–401.
Prolactinomas may be treated medically or surgically.
Davis JR. Prolactin and reproductive medicine. Curr Opin Obstet
Medical management with the use of dopamine receptor Gynecol 2004;16:331–7.
agonist is the primary approach for treatment of prolac- Shibli-Rahhal A, Schlechte J. Hyperprolactinemia and infertility.
tinomas, including macroprolactinomas, as long as the Endocrinol Metab Clin North Am 2011;40:837–46.
11
In vitro fertilization and intracytoplasmic sperm
A 36-year-old nulligravid woman with primary infertility comes to your office for evalua-
tion. She has been attempting pregnancy for 12 months and has had regular menstrual cycles.
Hysterosalpingography is normal and basal antral follicle count is 15. Ovarian reserve testing
reveals a follicle-stimulating hormone level of 6.5 mIU/mL and an antimüllerian hormone level of
2.5 ng/mL. Semen analysis concentration is 3 million/mL with 15% motility and 1% strict morphol-
ogy. Repeat semen analysis is similar. Male karyotype, Y chromosome microdeletion, total testos-
terone level, and follicle-stimulating hormone level are all normal. You consider recommending in
vitro fertilization (IVF) with intracytoplasmic sperm injection (ICSI). You counsel the patient that
a complication unique to ICSI is
The described patient has a negative infertility evalua- infertility. Once initial screening reveals a low concen-
tion. Her partner’s initial and repeat semen analyses show tration of sperm (5–10 million/mL), a reproductive uro-
oligoasthenoteratozoospermia. Based on World Health logic consultation is warranted. The initial evaluation will
Organization guidelines, a normal sperm concentration include a medical history and physical examination in
has greater than 15 × 106 spermatozoa/mL, greater than addition to a paternal karyotype, Y chromosome analysis,
40% motility, and greater than 4% normal forms by strict transrectal ultrasonography, and an endocrine evaluation.
Kruger morphology. The semen analysis is the most Genetic evaluation of the patient’s partner is indicated
important initial step in the evaluation of male factor before performing ICSI with his sperm. Approximately
16 PROLOG
permeability with intravascular fluid moving into third in subsequent IVF cycles. Thus, ICSI can restore fertil-
space compartments. Mildly affected patients often ization rates by overcoming either an oocyte or sperm
experience bloating, shortness of breath, nausea, and function problem.
decreased urination. One of the main mediators is
increased vascular endothelial growth factor, which is Diagnostic evaluation of the infertile male: a committee opinion.
Practice Committee of American Society for Reproductive Medicine
driven by high estradiol levels and human chorionic [published erratum appears in Fertil Steril 2013;99:951]. Fertil Steril
gonadotropins. Risk factors include an elevated anti- 2012;98:294–301.
müllerian hormone level, previous history of ovarian Forman EJ, Hong KH, Treff NR, Scott RT. Comprehensive chromo-
hyperstimulation syndrome, polycystic ovary syndrome, some screening and embryo selection: moving toward single euploid
blastocyst transfer. Semin Reprod Med 2012;30:236–42.
young age, high antral follicle count, rapidly increasing
Multiple gestation associated with infertility therapy: an American
estradiol levels, and estrogen levels above 2,500 pg/mL. Society for Reproductive Medicine Practice Committee opin-
The key to prevent ovarian hyperstimulation syndrome is ion. Practice Committee of American Society for Reproductive
to recognize risk factors and choose an ovulation induc- Medicine. Fertil Steril 2012;97:825–34.
tion protocol and gonadotropin dose carefully. Ovarian hyperstimulation syndrome. Practice Committee of American
Society for Reproductive Medicine. Fertil Steril 2008;90:S188–93.
The use of ICSI increases fertilization rates in properly
Vermeiden JP, Bernardus RE. Are imprinting disorders more prevalent
selected patients. Patients who have had poor or failed after human in vitro fertilization or intracytoplasmic sperm injection?
fertilization in a prior cycle have an indication for ICSI Fertil Steril 2013;99:642–51.
12
Hydrosalpinges and in vitro fertilization–embryo transfer
A 29-year-old nulligravid woman with regular menstrual cycles comes to your office after
18 months of infertility. Her husband had a normal semen analysis. Hysterosalpingography 1 year
ago showed large bilateral hydrosalpinges and distal obstruction (Appendix B). Before proceeding
with in vitro fertilization (IVF), the best next step in management is
Hydrosalpinges, large dilated fallopian tubes that are laparoscopic salpingectomy for hydrosalpinges before
damaged intrinsically and no longer function properly, IVF. Pregnancy rates with IVF return to normal levels
may be observed by means of ultrasonography, hys- after surgically excising the damaged tubes. Hysteroscopic
terosalpingography, or both. They contain fluid that is closure of hydrosalpinges using an implantable contracep-
believed to retrograde spill back within the uterus and tive device has been theorized to yield similar results. A
lead to decreased implantation of transferred embryos. recent pooled analysis shows good efficacy, but evidence
The hydrosalpinx fluid may interfere mechanically with from a randomized controlled trial is lacking.
embryonic apposition or reduce endometrial receptiv- The described patient is not a candidate for reassurance
ity. In the presence of hydrosalpinges, the expression of because hydrosalpinges are a marker for permanent tubal
endometrial receptivity markers (integrin α, β, and γ [eg, disease. Hydrosalpinges will not resolve spontaneously
integrin]) is reduced. This fluid is believed to be toxic to and repeat imaging with hysterosalpingography or saline
embryos. sonohysterography is not necessary. Therefore, the hydro-
A Cochrane review and an American Society for salpinges must be surgically addressed. Neosalpingostomy
Reproductive Medicine practice committee guideline of dilated tubes is not the treatment of choice because it
have noted that pregnancy rates are decreased by approxi- is unlikely to result in a successful intrauterine preg-
mately 50% in patients with hydrosalpinges. The Coch- nancy. Bilateral salpingectomy is the best choice for the
rane review demonstrated improved pregnancy rates with described patient. Antibiotic therapy alone would not
18 PROLOG
address the deleterious effects of the toxic fluid on the with and without hydrosalpinx: a meta-analysis of published compara-
tive studies. Hum Reprod 1999;14:1243–9.
embryos. Therefore, this is not a reasonable treatment
modality. Johnson N, van Voorst S, Sowter MC, Strandell A, Mol BWJ. Surgical
treatment for tubal disease in women due to undergo in vitro fertilisa-
tion. Cochrane Database of Systematic Reviews 2010, Issue 1. Art. No.:
Arora P, Arora RS, Cahill D. Essure® for management of hydrosalpinx
prior to in vitro fertilization—a systematic review and pooled analysis. CD002125. DOI: 10.1002/14651858.CD002125.pub3.
BJOG 2014;121:527–36. Kontoravdis A, Makrakis E, Pantos K, Botsis D, Deligeoroglou E,
Bildirici I, Bukulmez O, Ensari A, Yarali H, Gurgan T. A prospective Creatsas G. Proximal tubal occlusion and salpingectomy result in
evaluation of the effect of salpingectomy on endometrial receptivity similar improvement in in vitro fertilization outcome in patients with
in cases of women with communicating hydrosalpinges. Hum Reprod hydrosalpinx. Fertil Steril 2006;86:1642–9.
2001;16:2422–6. Strandell A, Lindhard A, Waldenstrom U, Thorburn J. Hydrosalpinx
Camus E, Poncelet C, Goffinet F, Wainer B, Merlet F, Nisand I, et al. and IVF outcome: cumulative results after salpingectomy in a random-
Pregnancy rates after in-vitro fertilization in cases of tubal infertility ized controlled trial. Hum Reprod 2001;16:2403–10.
13
Ovarian cancer in a BRCA1-positive patient
A 33-year-old nulligravid woman comes to your office for her annual well-woman examination.
She tells you that her mother is battling breast cancer. She is of Ashkenazi Jewish heritage and
reports early onset of breast and ovarian cancer throughout her mother’s family. The patient and her
mother have tested positive for the BRCA1 gene mutation. She currently is taking oral contracep-
tives (OCs) and is afraid that she is increasing her chances of developing cancer. The contraceptive
that offers the most noncontraceptive health benefits for this patient is
(A) progestin-only OC
* (B) combination estrogen–progestin OC
(C) medroxyprogesterone acetate
(D) intrauterine device
Breast and ovarian cancer are among the leading causes Oral contraceptives can reduce the risk of ovarian
of cancer death in the United States. Most patients with cancer by approximately 50% in BRCA1 mutation car-
breast and ovarian cancer have sporadic cancer forma- riers. The protective effects of OCs appear to be similar
tion. Only a small percentage of all cases of breast and in women without a BRCA1 mutation compared with
ovarian cancer (approximately 4–7%) are considered to women who are BRCA1 mutation carriers. The breast
be familial. During their lifetime, BRCA1 mutation car- cancer risk for BRCA1 mutation carriers and the use of
riers have an approximate 18–60% chance of developing OCs is subject to debate. Studies have demonstrated
ovarian cancer and a 54–85% chance of developing breast conflicting results for and against increased risk of breast
cancer. cancer. Mastectomy and bilateral salpingo-oophorectomy
Controversy continues about the role of OCs and birth confer the highest risk reduction of breast and ovar-
control. Good evidence exists that OCs have a protective ian cancer in BRCA1 mutation carriers. However, the
effect against ovarian cancer. The described patient is described patient has not completed childbearing. Until
relatively young and nulliparous. Women who use OCs then, OCs offer a significant risk reduction of ovarian
have been found to have approximately a 46% reduction cancer even in this patient population.
in risk of ovarian cancer compared with women who have Medroxyprogesterone acetate has only been shown to
never taken OCs. The risk reduction appears to be associ- decrease the risk of endometrial cancer. The levonorg-
ated with longer length of use, and the protective effect estrel intrauterine device can be used to help treat malig-
has been demonstrated to be as long as 30 years after nancy of the endometrium, although varying degrees of
discontinuation of use. The risk reduction does not appear treatment effectiveness have been described. Progestin
to be related to high-, moderate-, or low-dose contracep- therapy is particularly successful to help maintain fertility
tives. All formulations of OCs seem to confer ovarian in patients who have been diagnosed with endometrial
cancer protection to users. cancer and who wish to maintain fertility.
Reproductive Endocrinology and Infertility 19
Finch A, Beiner M, Lubinski J, Lynch HT, Moller P, Rosen B, et al. Kauff ND, Domchek SM, Friebel TM, Robson ME, Lee J, Garber JE, et
Salpingo-oophorectomy and the risk of ovarian, fallopian tube, and peri- al. Risk-reducing salpingo-oophorectomy for the prevention of BRCA1-
toneal cancers in women with a BRCA1 or BRCA2 mutation. Hereditary and BRCA2-associated breast and gynecologic cancer: a multicenter,
Ovarian Cancer Clinical Study Group. JAMA 2006;296:185–92. prospective study. J Clin Oncol 2008;26:1331–7.
Gunderson CC, Fader AN, Carson KA, Bristow RE. Oncologic and McLaughlin JR, Risch HA, Lubinski J, Moller P, Ghadirian P, Lynch H,
reproductive outcomes with progestin therapy in women with endo- et al. Reproductive risk factors for ovarian cancer in carriers of BRCA1
metrial hyperplasia and grade 1 adenocarcinoma: a systematic review. or BRCA2 mutations: a case–control study. Hereditary Ovarian Cancer
Gynecol Oncol 2012;125:477–82. Clinical Study Group. Lancet Onc 2007;8:26–34.
14
Endometriosis and infertility
A 25-year-old nulligravid woman has been attempting to become pregnant for 2 years. She was
asymptomatic when she was taking combination oral contraceptives. Since she stopped taking
oral contraceptives, she has experienced progressively worsening dysmenorrhea, dyspareunia,
and pelvic pain, which significantly interfere with her daily life. Bimanual examination reveals
evidence of right adnexal fullness and tenderness. Transvaginal ultrasonography shows a normal
uterus and a 5-cm right ovarian cyst with a homogeneous “ground glass” appearance suggestive
of an endometrioma. The best surgical treatment to improve her fertility and decrease her pain is
laparoscopy with
Endometriosis is characterized by lesions of endometrial- especially in women with minimal or mild endometriosis.
like tissue outside the uterus; the condition is associated Laparoscopy also allows for staging based on the 1997
with concurrent inflammation, pelvic pain, and infertil- revised American Society for Reproductive Medicine
ity. The mechanisms responsible for endometriosis are scoring system.
believed to be retrograde menstruation, celomic meta- Because of a lack of data from sufficient random-
plasia, and lymphatic spread in immunologically and ized controlled clinical trials, the best way to treat
genetically susceptible individuals. It is recognized that endometriosis at different clinical stages is unclear. The
the stage and extent of disease may not correlate with World Endometriosis Society sponsored the Montpellier
its symptoms, reproductive outcome, or recurrence risk. Consortium Consensus conference to bring together rep-
Although the cause of endometriosis is unknown, it is resentatives of multinational societies to reach a consen-
likely to be multifactorial in origin, including genetic fac- sus on the management of endometriosis. A process was
tors with possible epigenetic influences promoted through developed to score the current evidence to make recom-
environmental exposures. Endometriosis is associated mendations for future care and treatment. Whenever pos-
with pelvic–abdominal pain and infertility. Other symp- sible, laparoscopic surgery should always be performed
toms include dysmenorrhea, dyspareunia, heavy menstrual rather than laparotomy. In cases of severe endometriosis,
bleeding, nonmenstrual pelvic pain, pain at ovulation, gynecologic surgeons may consider the option of limit-
dyschezia, and dysuria. Endometriosis-related infertility ing surgical excision at an initial operation and refer the
is associated with the severity of disease, and although the patient to a surgeon experienced in endometriosis sur-
exact mechanism for infertility is unknown, it is related gery. Studies suggest that the first surgical intervention
to impaired tubo-ovarian function, inflammation, ovarian delivers the greatest benefit compared with subsequent
endometriomas, reduced egg quality, and implantation. surgical procedures, with pain improvement at 6 months
The standard for making the diagnosis of endometriosis of approximately 83% for the first excisional procedure
is histologic confirmation because the false-positive rate versus 53% for subsequent surgeries. Excessive numbers
with laparoscopic visualization alone may approach 50%, of repeat laparoscopic procedures should be avoided.
20 PROLOG
The Montpellier conference emphasized consideration stasis to control bleeding is recommended, as is the use
of the ovarian reserve before surgery in women who of suturing. The best surgical treatment to improve the
are experiencing infertility. Growing evidence suggests described patient’s fertility and to decrease her pain is
that surgical treatment of endometriomas contributes to laparoscopy with resection of the endometrioma cyst
reduced ovarian reserve. Laparoscopic surgical removal wall.
of stage I and II endometriosis improves fertility. It is Oophorectomy should be avoided because there is
unclear whether excision is better than ablation; how- normal ovarian tissue that can be preserved and that will
ever, it is recommended to excise lesions where possible, improve the patient’s future fertility. Likewise, excessive
especially in cases of deep endometriosis associated with use of cautery, although effective for ablating endome-
pain. To date, no controlled clinical trials have addressed triosis, leads to excessive destruction of normal ovarian
whether surgery improves fertility for stage III and IV tissue, reducing ovarian reserve. Insufficient data exist
endometriosis. at this time in regard to the value of robotic surgery in
Adjunct therapy with gonadotropin-releasing hormone endometriosis care. However, limited recent studies have
agonist after laparoscopic surgery has not been shown suggested that robotic surgery appears to be at least com-
to benefit fertility. Use of gonadotropin-releasing hor- parable with conventional laparoscopic surgery but with
mone agonist after surgery is not recommended because higher cost and prolonged operative times.
it delays pregnancy at a time when fertility has been
improved by surgery. Recent evidence suggests that Hart RJ, Hickey M, Maouris P, Buckett W. Excisional surgery versus
ablative surgery for ovarian endometriomata. Cochrane Database
laparoscopic excision (cystectomy) should be performed of Systematic Reviews 2008, Issue 2. Art. No.: CD004992. DOI:
whenever possible for endometriomas greater than 4 cm 10.1002/14651858.CD004992.pub3.
in diameter because it improves fertility more than abla- Johnson NP, Hummelshoj L. Consensus on current management of
tion (drainage and coagulation). However, the surgeon endometriosis. World Endometriosis Society Montpellier Consortium.
needs to take great care to identify the tissue planes and Hum Reprod 2013;28:1552–68.
to carefully dissect the endometrioma to avoid remov- Somigliana E, Berlanda N, Benaglia L, Vigano P, Vercellini P, Fedele L.
Surgical excision of endometriomas and ovarian reserve: a systematic
ing normal ovarian tissue, which will affect the ovarian review on serum antimüllerian hormone level modifications. Fertil
reserve. Reducing the amount of electrosurgical hemo- Steril 2012;98:1531–8.
Reproductive Endocrinology and Infertility 21
15
Reproductive function, nutritional status, and protein levels
A 17-year-old female gymnast comes to your office with secondary amenorrhea. She is 1.63 m
(64 in.) tall and weighs 41 kg (91 lb). She reports exercise in excess of 21 hours per week. She is
virginal and has a negative pregnancy test. Her pelvic examination is normal. She has a follicle-
stimulating hormone level of 3.0 and an undetectable luteinizing hormone level. Negative energy
balance in this patient causes low serum levels of
* (A) leptin
(B) ghrelin
(C) cortisol
(D) corticotropin-releasing hormone
(E) adrenocorticotropic hormone
A relationship exists between nutritional status and repro- Leptin also has been associated with the maintenance of
ductive function. Profound caloric restriction can lead to luteinizing hormone pulsatility and the preovulatory
poor reproductive function. Leptin is a 16kDa protein that surge associated with ovulation. Leptin levels in female
acts as a satiety signal. Leptin exhibits a diurnal pattern athletes were observed over a 20-week period and noted
that corresponds to consumption of meals. Women with to be decreased over the 20-week study period. Women
eating disorders and low caloric intake have low circulat- with exercise-induced amenorrhea have lower leptin
ing levels of leptin (Fig. 15-1). levels than ovulatory females. In this study, leptin lev-
Leptin-deficient animals exhibit impaired gonadotro- els of women with exercise-induced amenorrhea were
pin-releasing hormone secretion and are anovulatory. compared with those with ovulatory controls, and, when
Restoration of leptin levels restores ovulatory status. controlling for body fat levels among the two groups,
Stress anorexia
↓ Leptin
↑ Ghrelin
CNS ↓ Insulin
+
↑ Cortisol ↑ Endorphins
Amenorrhea
FIG. 15-1. Relationship of various hormone levels in women and stress anorexia, eating, weight loss, and amenorrhea.
Abbreviations: CNS, central nervous system; CRH, corticotropin-releasing hormone; GnRH, gonadotropin-releasing
hormone; NPY, neuropeptide Y; RT3, reverse triiodothyronine. (Speroff L, Fritz MA. Clinical gynecologic endocrinology
and infertility. 8th ed. Philadelphia [PA]: Lippincott Williams and Wilkins; 2011. p. 875.)
22 PROLOG
little difference was observed between the two patient corticotropin-releasing hormone and adrenocorticotropic
populations. This suggests that factors other than exercise hormone secretion. The most likely laboratory finding to
are important in the modulation of leptin levels. explain this patient’s secondary amenorrhea is low levels
Ghrelin is a 28-amino acid peptide that provides of serum leptin.
energy balance. It can cross the blood–brain barrier and
can modulate the vagus nerve and nucleus tractus solitar- Chou SH, Chamberland JP, Liu X, Matarese G, Gao C, Stefanakis R,
et al. Leptin is an effective treatment for hypothalamic amenorrhea.
ius at the level of the brain. Ghrelin levels are decreased Proc Natl Acad Sci U S A 2011;108:6585–90.
by food intake and obesity. Its production is increased in De Souza MJ, Leidy HJ, O’Donnell E, Lasley B, Williams NI. Fasting
fasting and caloric-restrictive states. High ghrelin levels ghrelin levels in physically active women: relationship with men-
are seen in women with anorexia, bulimia, and exercise- strual disturbances and metabolic hormones. J Clin Endocrinol Metab
2004;89:3536–42.
induced amenorrhea.
Thin, amenorrheic individuals with caloric restriction Lawson EA, Donoho D, Miller KK, Misra M, Meenaghan E, Lydecker
J, et al. Hypercortisolemia is associated with severity of bone loss and
can have a nutritional state of anorexia. These patients depression in hypothalamic amenorrhea and anorexia nervosa. J Clin
often exhibit hypercortisolism in response to elevated Endocrinol Metab 2009;94:4710–6.
16
Unexplained infertility
A 32-year-old nulligravid woman comes to your office with her partner after failing to achieve a
pregnancy despite unprotected intercourse for the past 18 months. They are both healthy and take
no medications. She has regular monthly menstrual cycles. You recommend that she proceed with
hysterosalpingography and ovarian reserve testing, both of which are normal. Her partner’s semen
analysis is also normal. Given these findings, the most appropriate cost-effective first step is
Approximately 25% of couples that are evaluated for fer- fecundity rate tends to be lower for couples in which
tility concerns do not have an identifiable diagnosis and the woman is older or the length of infertility is longer.
instead have “unexplained infertility.” The basic infertil- Treatment plans should be tailored to each couple, and
ity evaluation occurs after 6–12 months of attempted if the proposed plan does not result in a pregnancy after
conception. The evaluation includes confirmation of three to four cycles, alternative treatments should be con-
ovulatory status, testing to evaluate tubal patency and sidered. Lifestyle changes should be part of the conversa-
the uterine cavity, semen analysis, and testing of ovarian tion, with the aim to eliminate cigarette smoking, achieve
reserve. In the past, other tests such as a postcoital test, a normal body mass index, and reduce caffeine intake.
laparoscopy, and endometrial dating were performed, but These lifestyle changes can be beneficial for couples who
such steps have been shown to be of no assistance in terms are attempting conception on their own or with assistance.
of improving pregnancy rates. The natural decline in Although many couples with unexplained infertility may
fertility due to increasing age clearly contributes to chal- conceive on their own if they are given enough time, the
lenges with conception, especially because the average low likelihood of conception combined with their steadily
age of first-time mothers increased from age 21.4 years in declining fecundity due to age warrants that such patients
1970 to age 25 years in 2006. be offered treatment rather than expectant management.
In couples with otherwise unexplained infertility, the Treatments, such as clomiphene, are used commonly
monthly fecundity rate without treatment varies between for conditions associated with anovulation. Women with
1.8% and 3.8%, which is much lower than couples with- unexplained infertility are already ovulating on a reg-
out fertility challenges (20–25% per month). The monthly ular basis. Consideration should be given to such
Reproductive Endocrinology and Infertility 23
medications to achieve superovulation. Clomiphene and Treatments such as IVF can prove to be very success-
other medications have the potential to produce two to ful in women with unexplained infertility, depending
three mature follicles during a given cycle. Many studies on the woman’s age. In 2011, the live-birth rate in the
have evaluated the benefit of clomiphene with intercourse United States for couples younger than 35 years with
in cases of unexplained infertility and have found little unexplained infertility and who used IVF was 42.5%.
benefit compared with no treatment. Given the lack of Although IVF is clearly a more effective treatment per
benefit and the increased risk of twins and, on rare occa- cycle than other options, it is usually not the first-line
sion, higher-order multiples, care should be taken before option for couples with unexplained infertility because of
routinely using clomiphene with intercourse. its significant cost and relatively complex treatment plan.
Intrauterine insemination involves washing semen with
sterile placement of a concentrated volume of motile Bhattacharya S, Harrild K, Mollison J, Wordsworth S, Tay C, Harrold
A, et al. Clomifene citrate or unstimulated intrauterine insemination
sperm into the uterine cavity at the approximate time of compared with expectant management for unexplained infertility: prag-
ovulation. In a recent trial to compare IUI in natural cycles matic randomised controlled trial. BMJ 2008;337:a716.
with expectant management, no appreciable improvement Centers for Disease Control and Prevention. Assisted reproductive
in pregnancy rates was observed with IUI alone. technology. 2011 National summary report. Section 2: ART cycles
using fresh, nondonor eggs or embryos (part B). Atlanta (GA): CDC;
The use of clomiphene combined with IUI to achieve 2014. Available at: http://www.cdc.gov/art/ART2011/section2b.htm.
superovulation has been studied and found to have some Retrieved March 20, 2014.
benefit. The Fast Track and Standard Treatment trial Fisch P, Casper RF, Brown SE, Wrixon W, Collins JA, Reid RL, et al.
evaluated approximately 500 couples with unexplained Unexplained infertility: evaluation of treatment with clomiphene citrate
and human chorionic gonadotropin. Fertil Steril 1989;51:828–33.
infertility who were treated with clomiphene in combina-
Reindollar RH, Regan MM, Neumann PJ, Levine BS, Thornton KL,
tion with IUI. During 1,294 cycles, the monthly fecun- Alper MM, et al. A randomized clinical trial to evaluate optimal treat-
dity was 9.5%, a significant increase compared with the ment for unexplained infertility: the fast track and standard treatment
expected 2–4% monthly fecundity with expectant man- (FASTT) trial. Fertil Steril 2010;94:888–99.
agement. Overall, given the low cost and relative simplic- Speroff L, Fritz MA. Female infertility. In: Speroff L, Fritz MA,
editors. Clinical gynecologic endocrinology and infertility. 8th ed.
ity of using clomiphene together with IUI, this is the most Philadelphia (PA): Lippincott Williams and Wilkins; 2011. p. 1185–90.
cost-effective first step for the treatment of couples with
unexplained infertility.
24 PROLOG
17
Obesity and contraceptive choices
A 29-year-old obese woman with a body mass index of 32 (calculated as weight in kilograms
divided by height in meters squared) has just undergone an elective abortion and is considering her
contraceptive options. The best reversible contraceptive choice for this woman compared with a
nonobese woman is
In the United States, obesity is increasing; it is estimated TABLE 17-1. Contraceptive Method Chosen After
that approximately 61% of adult women are overweight Elective Abortion
or obese. Obesity is a major contributor to many medi-
Method Overweight Obese
cal problems, including type 2 diabetes mellitus, coro-
nary artery disease, stroke, and pulmonary dysfunction. Intrauterine OR =1.3 OR =1.6
These health conditions often are magnified when a device 95% CI, 1.28–1.32 95% CI, 1.59–1.61
P<.01 P<.01
woman becomes pregnant. Pregnancy places the patient
at risk of developing gestational diabetes, preeclampsia, Combination OR =1.4 OR =1.6
a venous thromboembolic event, or a delivery complica- contraceptive 95% CI, 1.28–1.52 95% CI, 1.57–1.63
ring P=.04 P<.01
tion. Pregnancies complicated by maternal obesity also
are associated with an increased prevalence of macroso- Laparoscopic OR =1.5 OR =2.9
tubal ligation 95% CI, 1.44–1.62 95% CI, 2.79–3.01
mic infants, stillbirths, and neonatal deaths. Given these
P=.03 P<.01
increased health risks for pregnant obese women, it is
important to recognize differences in contraceptive meth- Injectable OR =0.70 OR =0.52
progestin 95% CI, 0.59–0.81 95% CI, 0.48–0.56
ods practiced by obese women compared with nonobese P=.02 P<.01
women.
Abbreviations: CI, confidence interval; OR, odds ratio.
In a recent retrospective review of 7,262 charts of
Scott-Ram R, Chor J, Bhogireddy V, Keith L, Patel A.
women who had a first-trimester termination of a preg-
Contraceptive choices of overweight and obese women in
nancy, women were counseled after their abortion on a publically funded hospital: possible clinical implications.
the use of contraceptive options, such as combination Contraception 2012;86:122–6.
hormonal contraceptives (OCs, ring, and patch), the
intrauterine device (IUD), the progestin implant, inject-
able progestin, condoms, and laparoscopic tubal ligation. obese individual than estrogen-containing contraceptive
After performing a multivariable analysis adjusting for methods, such as combination OCs, the patch, or the ring.
age, education, race, and tobacco use, overweight and Obese individuals have an inherent increased baseline
obese women were more likely to select the IUD, com- risk of thrombosis, which may be further exacerbated
bination hormonal ring, or tubal ligation and were less by estrogen-containing contraceptive methods. Although
likely to choose injectable progestin (Table 17-1). No a recent study found no difference in the unintended
relation was found between being overweight or obese pregnancy rate of overweight and obese users of OCs,
and the choice of condoms, OCs, or the progestin implant the patch, and the ring, these methods may increase mor-
after a first-trimester abortion. These findings suggest bidity because of occult coexistent conditions, such as
that after counseling following an elective termination of hypertension and diabetes, which may have escaped ini-
pregnancy, overweight and obese women select different tial detection. Although injectable progestin use would be
contraception options than nonobese women. Knowledge safer, its association with increased weight gain may be
of these predispositions may help the health care provider responsible for overweight and obese women adversely
to foster improved patient adherence to their postabortion selecting it after an abortion.
contraceptive method.
McNicholas C, Zhao Q, Secura G, Allsworth JE, Madden T, Peipert
Use of long-acting reversible contraception, such as an JF. Contraceptive failures in overweight and obese combined hormonal
IUD or a progestin implant, may be a better choice in an contraceptive users. Obstet Gynecol 2013;121:585–92.
Reproductive Endocrinology and Infertility 25
Scott-Ram R, Chor J, Bhogireddy V, Keith L, Patel A. Contraceptive Skouby SO. Hormonal contraception in obesity, the metabolic syn-
choices of overweight and obese women in a publically funded hospital: drome, and diabetes. Ann N Y Acad Sci 2010;1205:240–4.
possible clinical implications. Contraception 2012;86:122–6.
18
Heavy menstrual bleeding
A 48-year-old multiparous woman comes to your office with a multiple-year history of heavy men-
ses that adversely affects her daily activities. Her examination is negative. Ultrasonography reveals
posterior uterine thickening consistent with adenomyosis. You categorize her bleeding as abnormal
uterine bleeding (AUB) with adenomyosis. She requests nonsurgical, nonhormonal treatment. The
nonhormonal treatment with the highest likelihood of decreasing her menstrual flow and improving
her quality of life is
(A) ibuprofen
(B) mefenamic acid
* (C) tranexamic acid
(D) black cohosh
(E) ethamsylate
Heavy menstrual bleeding is an important cause of mor- difference has been shown between NSAIDs such as
bidity in premenopausal women. In an effort to simplify naproxen and mefenamic acid.
the nomenclature and move away from such terms as Tranexamic acid, a synthetic derivative of the amino
menorrhagia, menometrorrhagia, and polymenorrhea, the acid lysine, is an antifibrinolytic agent that acts by bind-
International Federation of Gynecology and Obstetrics ing to plasminogen and blocking the interaction with
developed a classification system for causes of AUB that fibrin, thereby preventing the dissolution of the fibrin
classifies the etiologies of AUB into two categories: clot. In multiple large randomized trials, tranexamic acid
significantly reduced blood loss compared with placebo in
1. Structural causes, ie, polyp, adenomyosis, leiomyo-
a variety of surgical procedures, including joint replace-
mata, malignancy (PALM)
ments, cardiopulmonary bypass, and prostatectomy. In
2. Coagulopathy, ovulatory dysfunction, endometrial, many instances, tranexamic acid reduced the number of
iatrogenic, and “not yet classified” (COEIN) blood transfusions associated with surgery, and reduced
The two sets of causes constitute the PALM–COEIN clas- all-cause mortality and death due to bleeding in trauma
sification system (Appendix C). patients. In Europe, the use of tranexamic acid in gynecol-
Although hormonal preparations and surgery are effec- ogy has become commonplace.
tive therapies, other medical therapies have established A recent randomized controlled trial tested the effec-
efficacy and are preferred by some patients. Nonsteroidal tiveness of tranexamic acid in women with AUB. Patients
antiinflammatory drugs (NSAIDs) reduce prostaglandin were prescribed tranexamic acid (n=115) or placebo
levels, which often are elevated in women with AUB. (n=78) for the first 5 days of menses for 6 treatment
Studies have shown that NSAIDs have a beneficial effect months after quantification of menstrual flow was deter-
on dysmenorrhea. A recent Cochrane review analyzed mined at baseline. Women who received tranexamic
the data from 17 randomized controlled trials that exam- acid had significantly greater reduction in AUB (40%
ined the effects of NSAIDs on the reduction of AUB. As decrease) compared with placebo (8% decrease), had
a group, NSAIDs were more effective than placebo at significantly improved quality-of-life scores, and had no
reducing AUB, but all were less effective than tranexamic difference in adverse events compared with placebo. Even
acid, danazol, and the levonorgestrel intrauterine device. though tranexamic acid is an antifibrinolytic agent and
Although NSAIDs were shown to be as effective as there appears to be no increased risk of thromboembolic
oral contraceptives and ethamsylate, most studies were events with combined use of tranexamic acid and oral
underpowered. In terms of efficacy in reducing AUB, no contraceptives, their concomitant use is contraindicated.
26 PROLOG
Tranexamic acid appears to be well tolerated and effec- Diagnosis of abnormal uterine bleeding in reproductive-aged
women. Practice Bulletin No. 128. American College of Obstetricians
tive. Other studies with much larger cohorts of women and Gynecologists. Obstet Gynecol 2012;120:197–206.
showed 80% satisfaction with tranexamic acid therapy at Lethaby A, Duckitt K, Farquhar C. Non-steroidal anti-inflammatory
3 months. In terms of the described patient, the treatment drugs for heavy menstrual bleeding. Cochrane Database of Systematic
that has the highest likelihood of markedly decreasing her Reviews 2013, Issue 1. Art. No.: CD000400. DOI: 10.1002/14651858.
AUB and improving her quality of life is tranexamic acid. CD000400.pub3.
Ethamsylate is a hemostatic agent used in the treatment Lukes AS, Moore KA, Muse KN, Gersten JK, Hecht BR, Edlund M, et
al. Tranexamic acid treatment for heavy menstrual bleeding: a random-
of heavy menstrual bleeding, although its effectiveness ized controlled trial. Obstet Gynecol 2010;116:865–75.
appears to be less than that of tranexamic acid. Black McCormack PL. Tranexamic acid: a review of its use in the treatment
cohosh has limited effectiveness for the treatment of AUB. of hyperfibrinolysis. Drugs 2012;72:585–617.
19
Contraception for a patient with venous thromboembolism
A 25-year-old woman develops a pulmonary embolism during her first pregnancy and is found to
be heterozygous for a factor V Leiden mutation. After 6 months of postpartum therapy with warfa-
rin sodium, anticoagulation therapy is discontinued. She is planning on becoming pregnant in the
next 3 years. The most appropriate contraceptive for her is
The risk of venous thromboembolism in women correlates antibodies and lupus anticoagulant also have increased
with aging, obesity, smoking, genetic thrombophilias, risk of venous thromboembolism.
autoimmune disorders, pregnancy, and estrogen expo- Estrogens and the synthetic ethinyl estradiol in combi-
sure. The annual incidence of venous thromboembolism nation OCs primarily increase the risk of venous thrombo-
is approximately 4 in 10,000 nonhormone users in their embolism by altering hemostatic factor production by the
twenties, increasing to approximately 1 in 1,000 nonhor- liver. Procoagulant changes include increased prothrom-
mone users at age 45 years. This incidence has increased bin fragments, D-dimers, soluble fibrin, fibrinogen, factor
with time, most likely because of increasing body mass VII, factor VIII, Ristocetin cofactor activity, and fibrino-
index in the population. Obesity and smoking indepen- gen degradation. Additionally, there is a decrease in anti-
dently increase the odds of venous thromboembolism thrombin, protein S, and plasminogen activator inhibitor
approximately twofold to threefold. Genetic thrombo- 1, which also increases coagulation. The estrogen-induced
philias are inherited as autosomal recessive disorders and changes in coagulation are dose dependent. In the absence
include factor V Leiden mutation, prothrombin G20201A of estrogen, progesterone and progestins have negligible
mutation, and mutations leading to decreased protein C, effects on hemostatic factors.
protein S, and antithrombin. Other conditions include Combination OCs increase the risk of venous throm-
hyperhomocysteinemia, acquired activated protein C boembolism by approximately threefold, ie, to approxi-
resistance, and elevated factor VIIIc. Of these genetic mately seven venous thromboembolism events in 10,000
thrombophilias, factor V Leiden mutation is the most women per year. The risk is dramatically higher in women
common type. Most of these conditions increase the risk with a thrombophilia, making it a contraindication for
of venous thromboembolism in reproductive-aged women the use of combination OCs. For example, the risk of
who do not use hormones by threefold to fivefold except a venous thromboembolism in a woman with factor V
for prothrombin G20201A mutation, which has the least Leiden heterozygote increases from threefold in a woman
risk. Compound heterozygous and homozygous variants who does not use combination OCs (compared with indi-
of these conditions carry a greater risk than single muta- viduals without mutations) to 15-fold in a woman who
tions. Patients with significant titers of anticardiolipin uses combination OCs.
Reproductive Endocrinology and Infertility 27
The type of progestin in combination OCs changes implant, the levonorgestrel intrauterine device (IUD), and
the risk of venous thromboembolism. Third-generation depot medroxyprogesterone acetate (DMPA). Studies
progestins (norgestimate, desogestrel) and particularly show no increase in venous thromboembolism risk with
the progestin drospirenone in combination OCs may have norethindrone OCs or the levonorgestrel IUD. The etono-
higher venous thromboembolism risks compared with the gestrel implant has not been adequately studied, but hemo-
first-generation progestin, norethindrone, and the second- stasis factors are not changed with use of the implant,
generation progestin, levonorgestrel. The increased risk suggesting a low likelihood of venous thromboembolism
correlates with the increase in sex hormone-binding glob- risk. Traditionally, use of DMPA was not thought to
ulin (SHBG) associated with combination OCs that contain impose a venous thromboembolism risk because DMPA
these progestins. In addition, SHBG levels correlate with does not have substantial effects on coagulation factors.
the degree of activated protein C resistance. Combination This idea has been challenged by more recent data that
OCs with norgestimate and desogestrel compared with show an increase in venous thromboembolism in women
levonorgestrel result in higher circulating prothrombin who use DMPA compared with women who use nonhor-
fragments, factor VII, and decreased protein C. monal contraceptive methods. Current information would
Originally it was thought that nonoral administration suggest that the least risky hormonal contraceptives for
of combination contraceptives, including use of the patch women at high risk of venous thromboembolism are pro-
and the vaginal ring, would yield less risk because the gestin-only contraceptives, such as norethindrone OCs,
increase in hemostatic factors was expected to be less. the levonorgestrel IUD, and the etonogestrel implant.
No clinical study to date has proven this hypothesis. The Although any IUD would be safe and effective, her desire
contraceptive patch containing ethinyl estradiol and norel- for a pregnancy within 3 years makes this approach less
gestromin has a higher risk of venous thromboembolism appropriate.
compared with combination OCs containing the similar
progestin, norgestimate. The reason may be related to Plu-Bureau G, Maitrot-Mantelet L, Hugon-Rodin J, Canonico M.
Hormonal contraceptives and venous thromboembolism: an epidemio-
higher circulating levels of ethinyl estradiol from the logical update. Best Pract Res Clin Endocrinol Metab 2013;27:25–34.
patch. Based on limited data, the vaginal ring releasing Raps M, Helmerhorst F, Fleischer K, Thomassen S, Rosendaal F, Rosing
ethinyl estradiol and etonogestrel (an active metabolite of J, et al. Sex hormone-binding globulin as a marker for the thrombotic
desogestrel) may have a higher venous thromboembolism risk of hormonal contraceptives. J Thromb Haemost 2012;10:992–7.
risk compared with desogestrel-containing combination Wu O, Robertson L, Langhorne P, Twaddle S, Lowe GD, Clark P, et al.
Oral contraceptives, hormone replacement therapy, thrombophilias and
OCs. To date, this product has been less well studied than risk of venous thromboembolism: a systematic review. The Thrombosis:
other combination OCs. Risk and Economic Assessment of Thrombophilia Screening (TREATS)
Progestin-only contraceptive products available in the Study. J Thromb Haemost 2005;94:17–25.
United States include norethindrone OCs, the etonogestrel
28 PROLOG
20
Premature thelarche
A 2-year-old girl is brought to your clinic by her mother. The girl has a 3-month history of breast
growth without vaginal bleeding. Physical examination shows that, compared with girls her age,
she is in the 75th percentile for height and the 90th percentile for weight. In addition, she has
Tanner stage 3 breast development and Tanner stage 1 pubic hair development. The most appropri-
ate combination of tests to assess her problem is
Thelarche is the estradiol-dependent growth of breast with premature thelarche will subsequently develop pre-
tissue with stages originally described by Marshall and cocious puberty. The initial presenting symptom is breast
Tanner (Fig. 20-1). Breast development is typically the development, most commonly Tanner stage 2, which may
initial physical sign of puberty and is dependent on nutri- be asymmetrical or unilateral. There should be no signs or
tional status and adiposity. Compared with the mid-1800s, symptoms of sexual hair development or uterine bleeding.
pubertal development now occurs approximately 4 years The differential diagnoses include initial presentation of
earlier. In addition, significant ethnic differences in the precocious puberty, iatrogenic causation from exogenous
onset of pubertal development have been observed, ie, steroids, and the rare possibility of an estrogen-producing
on average, African American girls experience thelarche tumor.
at ages 8.9–9.5 years, Mexican American girls at age Initial evaluation of premature thelarche should include
9.8 years, and white girls at ages 10–10.4 years. Other a medical history, review of growth records, and physi-
classifications of breast development are available based cal examination. Initial diagnostic testing includes blood
on areolar diameter, pigmentation, and contour. estradiol and TSH levels and bone age. Hypothyroidism
Traditionally, premature thelarche is defined as breast may be associated with premature puberty and functional
development before age 8 years. Yet, given the mean ovarian cysts. For the described patient, blood estradiol
difference in thelarche among girls of different eth- and TSH levels plus bone age would be the most appro-
nicities, some experts suggest that premature thelarche priate combination of tests at this time.
be redefined as breast development before age 6 years Girls with premature thelarche have normal to slightly
in African American girls and age 7 years in white girls. increased growth velocity and normal to slightly advanced
This criterion would hold true as long as other factors are bone age, but less than 2 standard deviations. If these
not present, such as rapid progression of puberty, ie, bone variables are normal, close follow-up every 4–6 months
age greater than 2 years ahead of chronologic age and pre- is reasonable. Any change in signs or symptoms or
dicted height of less than 150 cm or 2 standard deviations abnormal laboratory findings may require further testing
or more below the genetic target height; central nervous for early precocious puberty, including baseline FSH and
system findings, eg, headaches, seizures, or other neuro- LH levels and a GnRH stimulation test. Normal levels of
logic symptoms; and pubertal progression that affects the estradiol, FSH, and LH do not exclude early precocious
emotional health of the family or the girl. puberty and require a GnRH stimulation test. Premature
Premature thelarche most commonly presents in two thelarche is characterized by an FSH-dominant response,
age categories: 1) from age 6 months to 2 years and whereas central precocious puberty is characterized by
2) age 5–7 years. Approximately 60% of cases are in the an LH-dominant response with GnRH stimulation. In the
younger age group. Approximately 50% will have regres- United States, GnRH is no longer available and stimula-
sion within 4 years, 30–35% will persist without progres- tion testing is performed with the use of a GnRH agonist.
sion, and 10–15% will develop precocious puberty. No Pelvic ultrasonography also may result in findings that
criterion or test is available that can predict which girls discriminate premature thelarche from central precocious
Reproductive Endocrinology and Infertility 29
21
Recurrent pregnancy loss
A 25-year-old woman seeks your help after three consecutive first-trimester miscarriages. As part
of your evaluation, you send her blood and her partner’s blood for chromosome testing. If a trans-
location is detected, the most likely scenario is
(A) patient—reciprocal
* (B) patient—robertsonian
(C) partner—reciprocal
(D) partner—robertsonian
Recurrent pregnancy loss is defined as two or more failed psychologic benefit of the couple. This approach has sev-
clinical pregnancies (ie, pregnancies visualized by ultra- eral pitfalls. The material may fail to grow in a culture,
sonography). Evaluation of pregnancy loss should be ini- leading to a lack of diagnosis, and there is also a poten-
tiated after two first-trimester pregnancy losses. The most tial for maternal contamination. Even if a cytogenetic
common contributors to recurrent pregnancy loss are abnormality in the product of conception is identified,
maternal anatomic abnormalities, maternal antiphospho- this does not exclude other causes of recurrent preg-
lipid syndrome, and cytogenetic abnormalities in either nancy loss, which should still be considered. In addition,
partner. Cytogenetic abnormalities make up approxi- approximately one half of all first-trimester pregnancy
mately 5% of etiologies for recurrent pregnancy loss. If losses may be associated with sporadic aneuploidies
recurrent pregnancy loss is diagnosed, blood should be (trisomies or monosomies). Aneuploidies do not tend to
collected from both parents and sent for karyotyping. be repetitive and also may be related to maternal age, but
Reciprocal and robertsonian translocations can be iden- maternal age may not be the only factor leading to these
tified in couples with recurrent pregnancy loss. Reciprocal aneuploidies. Treatment for couples with parental cytoge-
translocation, the most common chromosomal rearrange- netic abnormalities should start with genetic counseling.
ment in humans, refers to the exchange of chromo- Couples also may opt for preimplantation genetic screen-
some material between nonhomologous chromosomes. ing with transfer of unaffected embryos.
Robertsonian translocation refers to whole arm exchange
between acrocentric chromosomes (chromosomes 13, 14, Evaluation and treatment of recurrent pregnancy loss: a committee
opinion. Practice Committee of the American Society for Reproductive
15, 21, and 22). Carriers of reciprocal translocations can Medicine. Fertil Steril 2012;98:1103–11.
produce balanced and unbalanced gametes. Robertsonian
Hassold T, Hunt P. Maternal age and chromosomally abnormal preg-
translocation carriers tend to produce more unbalanced nancies: what we know and what we wish we knew. Curr Opin Pediatr
gametes. Translocations are more common in women 2009;21:703–8.
than in men. It is therefore more likely to detect a rob- Hirshfeld-Cytron J, Sugiura-Ogasawara M, Stephenson MD. Manage-
ertsonian translocation in the female partner if a chromo- ment of recurrent pregnancy loss associated with a parental carrier of
a reciprocal translocation: a systematic review. Semin Reprod Med
somal abnormality exists. However, this should not be a 2011;29:470–81.
reason for excluding the male partner from evaluation.
Stephenson MD, Awartani KA, Robinson WP. Cytogenetic analysis of
Cytogenetic evaluation of the products of concep- miscarriages from couples with recurrent miscarriage: a case–control
tion can be performed for the evaluation as well as the study. Hum Reprod 2002;17:446–51.
Reproductive Endocrinology and Infertility 31
22
Contraception for a patient with BRCA gene mutation
A 22-year-old nulligravid woman is a BRCA2 mutation carrier. She consults you about her con-
traceptive options. She desires future children and is interested in reliable, reversible birth control.
You inform her that oral contraceptives (OCs) decrease the risk of cancer of the endometrium and
cancer of the
(A) cervix
(B) breast
* (C) ovary
(D) colon
In recent decades, awareness of a genetic link to breast small increased risk of breast cancer, although the risk
cancer has become better understood by the scientific com- does not appear to be greatly increased or statistically
munity, and today screening tests for BRCA1 and BRCA2 significant but rather is comparable with the effect in OC
mutations are frequently performed. Approximately 0.04– users who were not carriers of BRCA mutations. A recent
0.2% of U.S. women and 2% of women of Ashkenazi meta-analysis found that new formulations of OCs are not
Jewish descent are estimated to be BRCA mutation associated with an increased risk of breast cancer among
carriers. Reproductive factors may influence the risk of BRCA mutation carriers compared with older formula-
developing breast and ovarian cancer in women who carry tions used before 1975. Given the uncertainty of a mildly
BRCA mutations. Menarche before age 12 years, low par- increased risk of breast cancer, mutation carriers should
ity, younger age at first childbirth, and breastfeeding all consider the substantially decreased risk of other cancers
have been shown to modulate the risk of breast cancer. when weighing the risk–benefit ratio of OC use.
Many mutation carriers will opt for risk-reduction surgery, Several investigations have demonstrated that OCs are
such as bilateral prophylactic mastectomy and salpingo- protective against ovarian cancer in the general popula-
oophorectomy, when they have completed their reproduc- tion. A large meta-analysis of 23,257 cases and 87,303
tive goals. As BRCA mutation screening becomes more controls confirmed these findings and found a significant
common, more women may be identified as carriers at a reduction in ovarian cancer in women who had ever used
younger age. Given the national trend toward older age at OCs. The risk of ovarian cancer decreased by 20% for
the time of the first full-term pregnancy, BRCA mutation every 5 years of OC use. This risk reduction has been
carriers may struggle with decisions about contraception confirmed in BRCA mutation carriers. Another large
options. Many of these women will opt for nonhormonal meta-analysis of BRCA1 and BRCA2 mutation carriers
options such as the copper intrauterine device or barrier who did or did not use OCs found a significant reduc-
contraception. However, when considering hormonal tion in ovarian cancer risk among OC users (summary
contraceptive options in BRCA mutation carriers, it is relative risk, 0.5; 95% confidence interval, 0.33–0.75).
important to weigh the benefits and risks. This benefit became more significant for every additional
An increased risk of cervical cancer has been demon- 10 years of OC use in mutation carriers.
strated among OC users in the general population. Some Data remain limited and inconsistent in regard to the
studies have shown an increased risk of cervical cancer risk of colon cancer in BRCA mutation carriers. Guide-
in BRCA mutation carriers, although this association has lines for colon cancer screening for mutation carriers
not been confirmed conclusively. Women who are BRCA remain the same as those for the general population.
mutation carriers should be screened for cervical cancer The use of OCs does not alter the risk of colon cancer in
in a manner similar to that for the general population, BRCA mutation carriers.
with Pap tests and human papillomavirus testing. Data
Cibula D, Gompel A, Mueck AO, La Vecchia C, Hannaford PC,
do not demonstrate that use of OCs modulates the risk of Skouby SO, et al. Hormonal contraception and risk of cancer. Hum
cervical cancer in BRCA mutation carriers. Reprod Update 2010;16:631–50.
Many studies have attempted to evaluate the effect of Hereditary breast and ovarian cancer syndrome. ACOG Practice Bulletin
OC use on breast cancer development in BRCA mutation No. 103. American College of Obstetricians and Gynecologists and
Society of Gynecologic Oncologists. Obstet Gynecol 2009;113:957–66.
carriers; these studies have shown conflicting results.
Iodice S, Barile M, Rotmensz N, Feroce I, Bonanni B, Radice P, et al.
Most of the studies are retrospective with potential recall Oral contraceptive use and breast or ovarian cancer risk in BRCA1/2
bias or small sample size. Most such studies show a carriers: a meta-analysis. Eur J Cancer 2010;46:2275–84.
32 PROLOG
23
Müllerian anomalies
A 19-year-old woman with müllerian agenesis would like to become sexually active. Upon physi-
cal examination, you find she has Tanner stage V breast and pubic hair development and a 2-cm
blind-ending vagina. The best next step in management to create a functional vagina for her is
Müllerian dysgenesis refers to patients with the absence Several surgical interventions are available, the most
of a vagina and variations in the presence or absence of a common being the creation of a neovagina with a skin
uterus. The term “müllerian agenesis” implies complete graft (McIndoe operation). It is a highly successful pro-
absence of all müllerian structures, and typically a patient cedure in motivated patients but is still a major opera-
will present with different degrees of development. tion entailing hospitalization, anesthesia, and creation of
Müllerian agenesis occurs in approximately 1 in 4,000 the graft. It can result in postoperative pain. If the graft
births and also is known as Mayer–Rokitansky–Küster– is not used with certain frequency it can constrict, and
Hauser syndrome. Müllerian agenesis occurs in an other- reoperations are fairly difficult for the patient to handle.
wise phenotypically normal female with amenorrhea and Variations of the McIndoe operation are available with
a normal 46,XX karyotype. Characteristically, in addition the utilization of a loop of the small or large bowel, buc-
to vaginal agenesis, the uterus is hypoplastic or absent cal mucosa, amnion, and an absorbable adhesion barrier
and, if present, the fallopian tubes are rudimentary. The to create the neovagina. However, each procedure carries
ovaries are normal in structure and function because they similar risks to those of the original procedure and would
derive from a different embryologic source. Associated certainly not be the first choice today. Vaginal estrogen
anomalies may include renal aplasia, spinal anomalies, creams may help as a lubricant to introduce dilators, and
anorectal malformations, and deafness. may increase tissue growth, healing, and pliability of the
When treating a patient with these anomalies, the cor- tissues, but use of a cream alone will not help develop a
rect diagnosis and assessment of the structures and length neovagina.
of the vagina are important. Evaluations of associated Another technique is the Vecchietti operation, which
anomalies and the urinary tract are essential. Assessment uses a traction device that is attached and wired laparo-
can be accomplished without surgical intervention by scopically to the vaginal “dimple.” Progressive traction is
using ultrasonography or magnetic resonance imaging. placed, allowing for invagination. The technique requires
Given the varying degrees of the anomalies, proper regular use of dilators and coitus. The Vecchietti opera-
preparation, follow-up, and emotional assessment are also tion has been reported with high success rates; however,
critical. It is important to emphasize to the patient and it still requires a laparoscopic procedure with its inherent
her family that she has reproductive and sexual options. complications, anesthesia, and several days of hospitaliza-
Fertility treatments need to be discussed, including the use tion in order to increase the traction on the device.
of a gestation carrier. The most successful and least invasive procedure
Several treatment options are currently available to to achieve a functional vagina is the use of sequential
develop a functional vagina for sexual intercourse. How- vaginal dilators with pressure applied by the patient in
ever, the concept of developing a new vagina requires 20-minute intervals over several months. The pressure can
significant emotional support and should be attempted be applied by the patient (Frank method) or by sitting on
only when the patient is ready to start sexual intimacy and a bicycle seat that has been previously fitted with dilators
is emotionally mature. Patients who have been monitored that are changed to progressively enlarge the vagina to the
after developing a neovagina have reported some degree desired size (Ingram method). These methods are under
of dyspareunia at penetration but otherwise report satisfy- the control of the patient and can be done at home with-
ing sexual activity. out the need for surgery or anesthesia. Sequential vaginal
Reproductive Endocrinology and Infertility 33
dilators have been demonstrated to be effective and to Frank RT. The formation of an artificial vagina without an operation.
Am J Obstet Gynecol 1938;35:1053–5.
provide patients with a satisfying intimate life within a
Ingram JM. The bicycle seat stool in the treatment of vaginal agenesis
matter of months. and stenosis: a preliminary report. Am J Obstet Gynecol 1981;140:
867–73.
Brucker SY, Rall K, Campo R, Oppelt P, Isaacson K. Treatment of Nakhal RS, Creighton SM. Management of vaginal agenesis. J Pediatr
congenital malformations. Semin Reprod Med 2011;29:101–12. Adolesc Gynecol 2012;25:352–7.
24
Hysteroscopic complications
A 33-year-old woman visits your office with heavy menstrual bleeding. Her evaluation includes
saline infusion ultrasonography showing a 2-cm mass consistent with a type 0 submucosal myoma.
A hysteroscopic myomectomy is recommended. The most common complication with hystero-
scopic resection of a type 0 submucosal myoma using glycine as a distention media is
Operative hysteroscopy has become common in gynecol- blood would not disperse in the solution, high viscosity,
ogy for many procedures, including endometrial ablation, nonelectrolyte dextran 70 was used commonly in the past
submucosal leiomyomata removal, lysis of intrauterine to improve image quality. Dextran 70, however, is used
adhesions, polypectomy, incision of uterine septum, tubal infrequently now because of the higher risk of anaphy-
sterilization, tubal cannulation for infertility, and removal laxis, pulmonary edema, and disseminated intravascular
of intrauterine devices. With improvements in equipment, coagulation compared with other agents.
many procedures can be performed in an office or an out- Instruments for hysteroscopy include operative scis-
patient surgery center with local anesthesia or conscious sors, graspers, biopsy forceps, electrocautery devices,
sedation. and morcellators. Electrocautery devices include unipolar
The necessary equipment includes a hysteroscope, loops for cutting and ablation, which require a nonelec-
light source, camera and monitor, distention medium, trolyte solution as mentioned previously. Bipolar devices
and instruments. Hysteroscopes are available in varying for cutting and ablation also are available that can be
diameters for diagnostic (2.7 mm) or operative (7–10 mm) used with NS or lactated ringers. Morcellator systems
use and as flexible or rigid scopes. Rigid hysteroscopes are available with a company-specific hysteroscope. The
are available with 0-, 12-, and 30-degree lenses. Operative morcellating attachment consists of two hollow tubes. The
hysteroscopes typically have a single operating chan- inner tube rotates and oscillates within the outer tube to
nel for placement of semirigid or rigid instruments. A remove tissue through a distal opening in the outer tube.
single inflow channel may be used with diagnostic scopes, This device is used for hysteroscopic polypectomy and
whereas dual inflow and outflow channels for continuous myomectomy.
flow are found in operative scopes. Anesthesia for hysteroscopy depends upon the degree
Distention media for diagnostic hysteroscopy include of needed cervical dilation, the degree of needed uter-
carbon dioxide (CO2) and normal saline (NS) or lactated ine distention, and the length of the case. Diagnostic
ringers, although CO2 is used less often because of hysteroscopy with a flexible scope typically requires no
increased operative time, shoulder pain, and vasovagal anesthesia. Polypectomy, IUD removal, and tubal steril-
reactions. Media for operative hysteroscopy include NS, ization may be performed with a paracervical block with
lactated ringers, nonelectrolyte solutions of 1.5% glycine, or without conscious sedation. More lengthy procedures,
3% sorbitol, 5% mannitol, and 5% dextrose. Because such as extensive lysis of adhesions, septum incision, and
34 PROLOG
myomectomy usually require monitored anesthesia care. and keeping the intrauterine pressure below the mean
General anesthesia may be needed in cases of concomi- arterial pressure. Intracervical injection of a dilute vaso-
tant laparoscopy or simultaneous transabdominal ultraso- pressin solution at 12-, 3-, 6-, and 9-o’clock also will
nography performed for hysteroscopic lysis of adhesions, decrease fluid absorption. Inadvertent intravascular injec-
septum incision, myomectomy, and tubal cannulation. tion of vasopressin may result in severe hypertension and
Hysteroscopy is a relatively safe procedure with a bradycardia.
complication rate of 0.95–3%. The most frequent com- Cervical laceration is most likely caused by a single-
plication is uterine perforation, with an incidence of tooth tenaculum with strong countertraction during cervi-
0.7–3%. Perforation is most common with hysteroscopic cal dilation. Depending on the depth and bleeding, this
lysis of adhesions followed by myomectomy and uterine may be controlled with application of silver nitrate or
septum incision. Perforation is recognized by a sudden require suture repair. Pretreatment with vaginal misopro-
decrease in or loss of distention, increased bleeding, or stol before surgery may ease cervical dilation in difficult
visualization of bowel or omentum. A small fundal per- cases. Adverse effects may include nausea, diarrhea, and
foration with a sound, small dilator or hysteroscope may abdominal pain.
be observed for clinical signs of intraperitoneal bleeding. Bleeding during hysteroscopy may be due to a uter-
Lateral perforations and perforations that occur with ine perforation, cervical laceration, or interruption of
sharp or electrocautery instruments are more likely to a myometrial vessel. Heavy bleeding in the absence of a
result in bleeding or bowel injury and may require lapa- coagulation or platelet disorder is uncommon. Bleeding
roscopic evaluation. without evidence of perforation or laceration is best con-
Excessive fluid absorption occurs in 0.2–0.76% of trolled by placement of an intrauterine balloon. This may
surgical cases. Factors that increase fluid absorption be accomplished with a specific triangular stent or a
include type of procedure (eg, myomectomy, lysis of Foley catheter.
adhesions, or septum incision), intrauterine pressure, and Air embolism is a rare but often fatal hysteroscopic
length of surgery. Defects created in the endometrium, complication. Studies show that air enters the vasculature
such as hitting the fundus during cervical dilation, may during most hysteroscopic procedures. Thus, the dif-
increase fluid absorption. The main risk with isotonic ficulty is not the mere presence of air but the volume of
solutions is fluid overload, which can cause pulmonary air that enters. Methods to decrease air entering the uterus
edema. Generally, the surgery should be concluded with include minimizing the degree of Trendelenburg position,
a total fluid deficit of 2,500 mL, remembering to account flushing all tubing with media, and minimizing repetitive
for intravenous and hysteroscopic fluids. Nonelectrolyte instrument insertions through the cervix that can cause
media add an additional risk of hyponatremia. For this piston-like transmission of air into the uterus.
reason, the fluid deficit should be limited to no more than
1,500 mL. As the deficit approaches 750 mL or greater, Cooper NA, Khan KS, Clark TJ. Local anaesthesia for pain control
during outpatient hysteroscopy: systematic review and meta-analysis.
intraoperative blood sodium levels should be considered BMJ 2010;340:c1130.
and the procedure stopped with the onset of hyponatre-
Jansen FW, Vredevoogd CB, van Ulzen K, Hermans J, Trimbos JB,
mia. Fluid absorption can be minimized by maintaining Trimbos-Kemper TC. Complications of hysteroscopy: a prospective,
a continuous fluid flow (not having the outflow closed) multicenter study. Obstet Gynecol 2000;96:266–70.
Reproductive Endocrinology and Infertility 35
25
Physiology of the menstrual cycle
Hormone therapy prescribed for a variety of conditions may mimic physiologic menstrual events.
The administration of cyclic progesterone for a patient with polycystic ovary syndrome mimics
Menstruation is a result of the profound tissue remodel- progesterone-producing corpus luteum. Response from
ing that occurs each month in women of reproductive the endometrium involves tissue breakdown and menses.
age. Endometrial tissue responds to the sex steroid If pregnancy takes place, hCG is responsible for main-
hormones produced in the follicular and luteal phases taining the corpus luteum and progesterone production.
of the ovarian cycle. Estradiol, produced by the ovaries The administration of cyclic progesterone for a patient
at approximately days 4–5 of the cycle, induces growth with polycystic ovary syndrome is not mimicked by
and proliferation of the endometrium. The epithelial and hCG rescue of the corpus luteum. Neither the action of
stromal cells undergo mitoses and multiply. hormone-mediated proliferation of the endometrium nor
After withdrawal of steroid hormone support, spe- that of pulsatile luteinizing hormone from the anterior
cifically involution of the corpus luteum, and decline in pituitary mimics the administration of cyclic progester-
progesterone, the functionalis layer of the endometrium one in such a patient.
undergoes extensive changes, resulting in complete tis-
sue breakdown. The underlying basalis layer contains the Beshay VE, Carr B. The normal menstrual cycle and the control of
ovulation. Available at http://www.endotext.org/. Retrieved December
progenitor cells that regenerate the functionalis layer in 4, 2013.
each cycle. Figure 25-1 illustrates the normal menstrual Matteson KA, Rahn DD, Wheeler TL 2nd, Casiano E, Siddiqui NY,
cycle. Harvie HS, et al. Nonsurgical management of heavy menstrual bleed-
To reproduce this cyclical exposure and withdrawal ing: a systematic review. Society of Gynecologic Surgeons Systematic
Review Group. Obstet Gynecol 2013;121:632–43.
from progesterone, a short course of oral progestin may
Nair AR, Taylor HS. The mechanism of menstruation. In: Amenorrhea:
be given. Completion of the course of progestin results a case-based clinical guide. Contemporary endocrinology. New York
in decreasing serum levels, mimicking involution of the (NY): Humana Press; 2010. p. 21–34.
36 PROLOG
FSH
LH
International E2 P
Units/L pg/mL ng/mL
20 500 10
18 9 P
16 400 8 LH
14 7
12 300 6
10 5 FSH
8 200 4
6 3
4 2 E2
100
2 1
0 0 0
2 4 6 8 10 12 14 16 18 20 22 24 26 28
Menses Ovulation
The endometrial
cycle
FIG. 25-1. The normal menstrual cycle. Abbreviations: E2, estradiol; FSH, follicle-stimulating hormone; LH, luteinizing
hormone; P, progesterone. (Fritz MA. Evaluation of the female: ovulation. Cedars Ml, editor. Infertility: practical path-
ways in obstetrics and gynecology. New York (NY): The McGraw-Hill Companies, Inc.; 2005.)
Reproductive Endocrinology and Infertility 37
26
Pregnancy termination
(A) etonogestrel
(B) medroxyprogesterone acetate
(C) norethindrone
* (D) mifepristone
(E) dienogest
Abortions in the second trimester (ie, at 13–24 weeks) statistically significant shortening of their induction time
can be performed either surgically by means of dilation to abortion with mifepristone pretreatment. The medical
and evacuation (D&E) or medically by use of misopros- protocol consisted of ingestion of either oral mifepristone
tol, a synthetic orally active E1 prostaglandin. Because of or a placebo at home 1 day before a 15-hour hospital
a shortage in the number of individuals who are skilled in admission. In the hospital, women used buccal misopro-
performing D&E procedures and an unequal distribution stol every 3 hours up to five doses or until the fetus and
of these health care providers throughout the country, placenta were expelled. Pretreatment with mifepristone
some women do not have a surgical option and must resulted in more than twice the chance of complete uter-
use medical therapy for their second-trimester abortions. ine evacuation within 15 hours (79.8% versus 36.9%,
The operating health care provider’s prior experience relative risk, 2.16; 95% confidence interval, 1.70–2.75).
and technique in performing D&E procedures are vital. In a recent systematic review of the mifepristone to
Although second-trimester abortion procedures account misoprostol dosing interval and its effect on induction
for only 10–15% of all abortions worldwide, they are abortion times, the authors concluded that shortening the
associated with two thirds of the major complications. mifepristone–misoprostol interval to 12–24 hours did not
One way to shorten the induction time (first misopro- compromise the safety or efficacy of the second-trimester
stol dose to abortion) is to pretreat these women with medical termination procedure and markedly reduced the
mifepristone 12–48 hours before use of the misoprostol. total abortion time.
Mifepristone is an antiprogestin that was approved by Etonogestrel, medroxyprogesterone acetate, norethin-
the U.S. Food and Drug Administration for the ter- drone, and dienogest are all progestins and pretreatment
mination of first-trimester pregnancies in September with any of them would not improve the efficacy of
2000. Mifepristone binds to the progesterone receptor misoprostol alone for medical therapy for a second-
with a five-times-greater affinity than progesterone. trimester abortion. Etonogestrel (3-ketodesogestrel) is the
Mifepristone is a corticosteroid receptor antagonist and progestin that resides within the implant used for contra-
has a three-times-greater affinity for the corticosteroid ception. Medroxyprogesterone acetate and norethindrone
receptor than dexamethasone. Mifepristone has less are orally active synthetic progestins. Dienogest is an
affinity (only 25%) for the testosterone receptor and does orally active progestin utilized in a recently introduced
not bind to the estrogen or mineralocorticoid receptors. oral contraceptive pill.
In blocking the progesterone receptor, mifepristone is
thought to increase uterine sensitivity to misoprostol and Ngoc NT, Shochet T, Raghavan S, Blum J, Nga NT, Minh NT, et
is associated with cervical softening and pliability. In al. Mifepristone and misoprostol compared with misoprostol alone
for second-trimester abortion: a randomized controlled trial. Obstet
first-trimester medical abortions, mifepristone pretreat- Gynecol 2011;118:601–8.
ment promotes endometrial instability and sloughing. In
Shaw KA, Topp NJ, Shaw JG, Blumenthal PD. Mifepristone-
a randomized, placebo-controlled, double-blind trial of misoprostol dosing interval and effect on induction abortion times: a
mifepristone use before misoprostol use, women had a systematic review. Obstet Gynecol 2013;121:1335–47.
38 PROLOG
27
Müllerian dysgenesis
A 25-year-old woman with primary amenorrhea visits your office to discuss her future reproduc-
tive options. She has müllerian dysgenesis with a functional vagina through progressive dilation.
She is concerned about her future reproductive potential and wishes to have a safe approach that
maximizes her genetic participation. The best option is
Müllerian dysgenesis, also referred to as müllerian agen- mentary uterine structure. Cyclical pelvic pain may occur
esis or Mayer–Rokitansky–Küster–Hauser syndrome, initially when functional endometrium is present, at
occurs in 1 in 4,000–10,000 females and is characterized times necessitating surgical removal to relieve pain and
by a 46,XX karyotype and normal ovaries that produce decrease the potential for endometriotic implants.
the normal array of female hormones. It is the second An uncommon presentation of primary amenorrhea is
most common etiology of primary amenorrhea, with cervical agenesis in which the patient has a normal mid-
gonadal dysgenesis being the first and androgen insen- line uterus with no cervix. One solution to this problem
sitivity the third. Müllerian dysgenesis occurs in patients is to carry out ZIFT. Pregnancy has been reported using
in the early teenage years or adolescence with primary ZIFT with laparoscopic embryo transfer into the fallopian
amenorrhea. On physical examination, patients with this tube. Because the described patient does not have a mid-
condition have normal height, breast development, exter- line uterus, ZIFT is not an option.
nal genitalia, and pubic and axillary hair. The vagina is In vitro fertilization with a gestational carrier is the
absent or may be a short pouch without a cervix. preferred modality for patients with müllerian dysgenesis
The patient has successfully created a functional to have their own genetic offspring. Offspring are likely
vagina through progressive dilation. The nonsurgical to have normal müllerian anatomy at birth. Practices
approach to vaginal creation is the first-line therapy. Up may have mentors or patients with müllerian dysgenesis
to 90% of motivated patients are able to dilate and create who can talk to other patients with the condition and
a functional vagina. Counseling is very important to the serve as role models. Local and national organizations
success of these patients. Most literature reports normal offer patient referral, support groups, and information
sexual desire, arousal, and orgasm when these patients (see www.ghr.nlm.nih.gov/condition/mayer-rokitansky-
are compared with controls. A subset of women with kuster-hauser-syndrome).
müllerian dysgenesis are not successful at self-dilation or The described patient has questions about her future
do not wish to self-dilate. For such women, surgical cre- reproductive potential. Proper compassionate counsel-
ation of a neovagina usually is performed when patients ing is the most important goal for this consultation.
are ready for vaginal intercourse. The most common Gestational surrogates can be a family member (sister,
surgical approach is the McIndoe operation: creation of a cousin) or an unrelated carrier. Ovulation induction is fol-
neovagina using a skin graft. Postoperatively, a dilator is lowed by oocyte retrieval. Ovaries can be lateral and near
used to keep the space patent. Other operations also are the pelvic brim, making transvaginal oocyte retrieval dif-
available for neovaginal creation. The choice depends on ficult. Some patients may require laparoscopic or transab-
the experience of the operating surgeon. dominal oocyte retrieval. Embryo transfer will occur in a
Even though patients have normal ovaries, one or both hormonally primed surrogate carrier. Pregnancy rates for
ovaries may be located at the pelvic brim, in the inguinal IVF are excellent and are related to the chronologic age
canal, or may be hypoplastic. The location is important of the woman’s oocyte.
for the patient who desires in vitro fertilization and will Using donor oocytes is not warranted at this time. This
dictate the need for abdominal or vaginal oocyte retrieval. patient is aged 25 years and is actively interested in her
Most women with müllerian dysgenesis will have a reproductive options. When she is ready to build a fam-
nonfunctioning rudimentary uterus. Up to 7% of patients ily, she can use her own oocytes. Donor oocytes may be
will have a functional endometrium present in the rudi- needed in the rare scenario in which a patient presents
Reproductive Endocrinology and Infertility 39
with müllerian dysgenesis in her 40s or has ovaries that Müllerian agenesis: diagnosis, management, and treatment. Committee
Opinion No. 562. American College of Obstetricians and Gynecologists.
are not amenable to oocyte retrieval. Obstet Gynecol 2013;121:1134–7.
Adoption is an option for patients with müllerian dys- Reichman DE, Laufer MR. Mayer–Rokitansky–Kuster–Hauser syn-
genesis who are not interested in IVF with a surrogate drome: fertility counseling and treatment. Fertil Steril 2010;94:1941–3.
carrier or for whom IVF is not an option. For this patient, Thijssen RF, Hollanders JM, Willemsen WN, van der Heyden PM, van
it is a future option that needs to be discussed. Adoption Dongen PW, Rolland R. Successful pregnancy after ZIFT in a patient
with congenital cervical atresia. Obstet Gynecol 1990;76:902–4.
is a valid tool for family building.
28
Perimenopausal changes
A 48-year-old woman with irregular menstrual cycles comes to your office to discuss the peri-
menopausal transition. You inform her that the finding most characteristic of the early menopausal
transition is
Perimenopause is defined as the interval of a woman’s The hallmark of the early menopausal transition, Stage
life before actual menopause until 12 months after the –2, part of the perimenopause stage, is the presence of
final menstrual period. Because the timing of the final increased variability in menstrual cycle length, defined
menstrual period is established retrospectively, it is as a persistent difference of 7 days or more in the length
difficult to prospectively estimate when the perimeno- of consecutive cycles. Persistence was defined as recur-
pausal stage will end. The hallmark of this phase is the rence within 10 cycles of the first variable length cycle.
occurrence of intermittent anovulation that leads to the The early menopausal transition also is characterized by
clinical sign of irregular vaginal bleeding. The vaginal elevated but variable early follicular phase follicle-stimu-
bleeding may have no pattern of frequency, varying lating hormone (FSH) levels, low antimüllerian hormone
from infrequent menses every 12 weeks to daily vaginal levels, and a low antral follicle count. The overall duration
bleeding and spotting. Traditionally, the World Health of the early menopausal transition is thought to be vari-
Organization has defined the beginning of perimenopause able among women.
as a break in a woman’s regular menstrual cycles, but not The late menopause transition, Stage –1, part of the
more than 3 months without a period. When a woman has perimenopause stage, is marked by the occurrence of
3–11 months of amenorrhea, she is considered to be in the amenorrhea of 60 days or longer. Menstrual cycles are
late perimenopausal stage. characterized by increased variability in cycle length,
In 2001, the Stages of Reproductive Aging Workshop extreme fluctuations in hormone levels, and increased
recommended terminology and established a staging prevalence of anovulation. During a random blood draw,
system for reproductive aging. That became the criterion FSH levels usually will be greater than 25 international
standard for characterizing reproductive aging through units/L, but if associated with an elevated estradiol level,
menopause. In 2011, at the follow-up workshop, the FSH levels sometimes may be lower than 25 international
Stages of Reproductive Aging Workshop + 10, modi- units/L. Associated vasomotor symptoms are likely to
fications were made to the 2001 staging system and occur during the late menopausal transition. The overall
the perimenopausal interval was divided into early and duration of the late menopausal transition is believed to
late phases of the menopausal transition (Table 28-1). last 1–3 years. The late menopausal transition ends at the
TABLE 28-1. The Stages of Reproductive Aging Workshop + 10 Staging System for Reproductive Aging in Women
Supportive Criteria
Endocrine
FSH Low Variable* ↑Variable* ↑25 international ↑Variable Stabilizes
AMH units/L or more†
Inhibin B Low Low Low Low Low Very low
Low Low Low Low Very low
Antral follicle Low Low Low Low Very low Very low
count
Descriptive Characteristics
Symptoms Vasomotor Vasomotor Increasing
symptoms symptoms symptoms of
likely most likely urogenital atrophy
final menstrual period and is dated retrospectively. The not to occur during the perimenopausal transition, but in
perimenopause stage extends into the early postmeno- the late postmenopausal stage (Stage +2).
pausal stage and is defined to end 1 year after the final
Harlow SD, Gass M, Hall JE, Lobo R, Maki P, Rebar RW, et al.
menstrual period, ie, Stage +1a, part of the perimeno- Executive summary of the Stages of Reproductive Aging Workshop
pause stage. + 10: addressing the unfinished agenda of staging reproductive
Given the aforementioned definitions of perimeno- aging. STRAW + 10 Collaborative Group. J Clin Endocrinol Metab
2012;97:1159 – 68.
pause, the presence of vasomotor symptoms and an
Soules MR, Sherman S, Parrott E, Rebar R, Santoro N, Utian W, et
interval of amenorrhea of 60 days or longer would be al. Executive summary: Stages of Reproductive Aging Workshop
consistent with the changes seen in the late menopausal (STRAW). Fertil Steril 2001;76:874 – 8.
transition. The presence of urogenital atrophy is thought
29
Donor egg in vitro fertilization
* (A) echocardiography
(B) confirmation of rubella immunity
(C) mammography
(D) diabetes mellitus screening
(E) Pap test
In recent decades, women who are past the age at which preeclampsia; hemolysis, elevated liver enzymes, and low
natural conception is generally possible have been able to platelet count (HELLP) syndrome; abnormal placenta-
consider IVF with an egg donor. The method is available tion; and premature rupture of membranes. One study of
to 74 women aged 45–56 years who received donor eggs
• women with primary ovarian insufficiency before age found that the incidence of pregnancy complications
40 years was 38%.
Ideally, all women who consider conception, either
• women in their 40s who have decreased fecundity
with assistance or on their own, should have a preconcep-
• women who have loss of fertility potential (eg, loss of tion evaluation with their health care provider. A precon-
fertility caused by oophorectomy or cancer therapy) ception visit is an opportunity to address behavioral and
For such women, becoming pregnant after donor egg IVF medical conditions that may affect the safety of preg-
does not pose any significant additional risk compared nancy for the mother and child. Many women do not see a
with a spontaneous conception. Egg donation dispro- health care provider before conception. However, infertile
portionately occurs in women older than 40 years, and women who seek assistance in conception in particular
that alone often results in their pregnancies being clas- should receive a thorough preconception evaluation with
sified as high risk. In addition, the frequency of multiple consideration of use of preconception folic acid. For an
gestations is high with the use of donor egg cycles. In older patient who wishes to receive donor eggs, this coun-
2010, approximately 38% of pregnancies that resulted seling could be provided by a maternal–fetal medicine
from donor egg IVF were multiple gestations, most of specialist who can review the added risks associated with
which were twins. Other maternal and fetal complications the woman’s age.
can occur during pregnancy in older women, even with It is recommended that women with certain known
singleton pregnancies. Such complications include pre- health problems have an appropriate evaluation before con-
term labor; gestational hypertension; diabetes mellitus; sideration of pregnancy. Women with chronic hypertension
42 PROLOG
should confirm that they are taking medication that is is appropriate to make sure the patient has had her routine
safe for their pregnancy and will control their blood Pap test and mammography; any concerning test results
pressure before conception. Women who have a history are more easily pursued before pregnancy. This should be
of poorly controlled hypertension, as in the described a routine part of a preconception visit and is not unique
patient, should be evaluated for serious sequelae that to this specific case.
can affect the pregnancy. Echocardiography is the most Given that maternal hyperglycemia can present a det-
essential preconception test for this patient because it can rimental fetal risk, screening women at risk of diabetes
identify left ventricular hypertrophy that could cause life- mellitus before pregnancy is essential. The described
threatening cardiomyopathy during pregnancy. patient is not at an especially high risk of diabetes; she
Rubella, also known as German measles, is a child- should have routine screening as part of her preconcep-
hood disease that is now rare in much of the world. tion evaluation, but diabetes screening is not the most
Most individuals in the United States received rubella essential preconception test for her.
vaccination as children, but for some individuals, the
American College of Obstetricians and Gynecologists. Hypertension in
vaccine loses efficacy as they age. The rubella vaccine is pregnancy. Washington, DC: American College of Obstetricians and
a live, attenuated vaccine and is contraindicated during Gynecologists; 2013. Available at: http://www.acog.org/Resources_
pregnancy, so testing for immunity and subsequent vac- And_Publications/Task_Force_and_Work_Group_Reports/Hyper
tension_in_Pregnancy. Retrieved March 20, 2014.
cination must occur during the preconception evaluation.
Centers for Disease Control and Prevention. Assisted reproductive
Although confirmation of rubella immunity is important technology. 2011 National summary report. Section 2: ART cycles
before attempting pregnancy, it is not the most important using fresh, nondonor eggs or embryos (part B). Atlanta (GA): CDC;
preconception test for this patient. 2014. Available at: http://www.cdc.gov/art/ART2011/section2b.htm.
Retrieved March 20, 2014.
Screening for breast and cervical cancer is essential for
Sauer MV, Paulson RJ, Lobo RA. Oocyte donation to women of
all women in their 40s, whether they are attempting preg- advanced reproductive age: pregnancy results and obstetrical outcomes
nancy or not. Before beginning an egg donation cycle, it in patients 45 years and older. Hum Reprod 1996;11:2540–3.
Reproductive Endocrinology and Infertility 43
30
Labial adhesions in children
A 5-year-old girl has had difficulty voiding over the past 6 weeks. Genital examination reveals that
the labia are fused with a white band of adhesions. The urethra and clitoris are not visualized, but
you observe a small pinpoint opening at the introitus. The most appropriate management is
Labial adhesions, also termed labial agglutination, labial exists regarding treatment in patients with asymptomatic
fusion, or vulvar synechiae, are a relatively uncommon labial adhesions, given that most cases will resolve spon-
condition. The condition occurs in up to 1.8% of female taneously within 1 year of puberty.
prepubertal patients, with a peak incidence at approxi- Patients with thin adhesions can be treated with topical
mately 13–23 months of age. Although most patients estrogen but patients with dense, fibrous adhesions are
are asymptomatic, some experience complications such less likely to respond to topical estrogen therapy alone.
as local inflammation, dysuria, urinary tract infection, If conservative therapy with topical estrogen has failed
and obstruction. Labial adhesions also can occur with or if a girl presents with urinary retention, additional
chronic urinary tract infection, chronic inflammation measures are necessary. Manual or surgical separation is
such as vulvovaginitis, genital trauma, and hypoestrogen- then indicated and should be performed under anesthesia
ism. All patients with labial adhesions should therefore to limit potential emotional and psychologic trauma from
be screened for urinary tract infections. During labial the procedure.
trauma, denuding of the upper squamous epithelial layer As an antibiotic cream, clindamycin is primarily used
of labial mucosa leads to formation of connective tissue for the treatment of bacterial vaginosis, which would not
bridges between the healing labia because of vulvo- be expected in a pediatric patient except in the case of
vaginitis and poor local hygiene. Approximately 80% sexual abuse. Topical clobetasol cream is used to treat
of labial adhesions resolve spontaneously within 1 year, lichen sclerosus and historically has been thought to
and persistence after the onset of puberty is very rare. It be ineffective in the treatment of labial adhesions. The
is important to note that the bulk of medical literature on authors of one small retrospective study reported modest
this subject consists of case series and other observational success with topical betamethasone valerate. They found
studies. estrogen and betamethasone to be equally efficacious.
Management options range from reassurance for However, the small number of study subjects limits the
asymptomatic patients to surgical treatment for severe strength of their conclusions. For now, estrogen cream
cases and those resistant to conservative treatment. The remains the first-line treatment for labial adhesions.
finding of labial adhesions in pubertal girls also may sug-
gest sexual abuse and warrants a careful disclosure inter- Berkoff MC, Zolotor AJ, Makoroff KL, Thackeray JD, Shapiro RA,
view and examination by skilled personnel to determine Runyan DK. Has this prepubertal girl been sexually abused? JAMA
2008;300:2779–92.
if there is evidence of sexual interference.
Eroglu E, Yip M, Oktar T, Kayiran SM, Mocan H. How should we treat
Locally applied estrogen cream changes the vaginal prepubertal labial adhesions? Retrospective comparison of topical treat-
epithelium from a thin and atrophic environment to a ments: estrogen only, betamethasone only, and combination estrogen
thicker environment rich in glycogen with more acidic and betamethasone. J Pediatr Adolesc Gynecol 2011;24:389–91.
vaginal secretions that prevent inflammation and infec- Kumar RK, Sonika A, Charu C, Sunesh K, Neena M. Labial adhesions
in pubertal girls. Arch Gynecol Obstet 2006;273:243–5.
tion. For symptomatic patients, conservative treatment
with estrogen cream is recommended as the first-line Tebruegge M, Misra I, Nerminathan V. Is the topical application of
oestrogen cream an effective intervention in girls suffering from labial
treatment. Treatment may be required for several weeks adhesions? Arch Dis Child 2007;92:268–71.
to achieve separation of the labia minora. Controversy
44 PROLOG
31
Risks and benefits of hormone therapy
The Women’s Health Initiative (WHI) study was the largest study of its kind to examine hormone
therapy in menopausal women. The study included two randomized clinical trials: a study of the use
of estrogen–progestin in women with a uterus and a study of the use of estrogen alone in women
without a uterus. The study showed that estrogen–progestin has no benefit in terms of protection
against the primary outcome of
(A) dementia
* (B) cardiovascular disease
(C) colon cancer
(D) hip fracture
The WHI study was designed to assess the effect of and the proximity of use to the onset of menopause. The
estrogen–progestin on cardiovascular events in healthy current recommendations from regulatory agencies and
postmenopausal women. A total of 161,809 women professional associations, such as the American College
aged 50–79 years were enrolled in the study. The study of Obstetricians and Gynecologists, advise the use of
was planned to run for 8 years, but it was stopped after hormone therapy only for moderate-to-severe hot flushes.
5.2 years when the number of new breast cancer cases Osteoporosis may be treated with estrogens when other
reached a preset safety threshold for harm. treatment modalities have failed or are poorly tolerated.
The WHI study had two randomized controlled trials The risks of estrogen–progestin in younger meno-
of postmenopausal estrogen–progestin: 1) daily estrogen– pausal women are reassuring. Table 31-1 illustrates that
progestin and 2) unopposed estrogen in hysterectomized in women aged 50–54 years, the individuals most likely
women. The estrogen–progestin arm was discontinued to benefit from receiving estrogen–progestin, this ther-
first because of a statistically significant increase in apy carries a small increase in absolute risk, ie, 1.56 net
invasive breast cancer and a trend toward increased adverse events per 1,000 women-years. By comparison,
cardiovascular events. The estrogen-only arm was dis- for estrogen-only therapy the net increase was 0.08 events
continued after 6.8 years in a controversial decision. per 1,000 women-years. Estrogen-only therapy and estro-
Unlike the combined estrogen–progestin arm, none of the gen–progestin therapy are associated with a 20% increase
preset boundaries had been crossed, although there was an in stroke. For women aged 50–54 years, pulmonary
increase in stroke risk as well as a trend toward increased embolism, coronary heart disease, and breast cancer were
risk of dementia or cognitive impairment. Interim statisti- increased only in the estrogen–progestin arm of the WHI
cal analysis also had concluded that the primary endpoint, study.
coronary heart disease reduction, would not have been
realized even if the study had continued to its planned Anderson GL, Limacher M, Assaf AR, Bassford T, Beresford SA,
Black H, et al. Effects of conjugated equine estrogen in postmenopausal
endpoint. women with hysterectomy: the Women’s Health Initiative randomized
The WHI investigators reported an overall increase controlled trial. Women’s Health Initiative Steering Committee. JAMA
in stroke in the estrogen–progestin arm. This large and 2004;291:1701–12.
well-designed randomized clinical trial demonstrated that Grady D. Clinical practice. Management of menopausal symptoms. N
Engl J Med 2006;355:2338–47.
daily estrogen–progestin does not have any cardioprotec-
Postmenopausal estrogen therapy: route of administration and risk of
tive benefits against cardiovascular disease. In fact, there venous thromboembolism. Committee Opinion No. 556. American
was an increase in cardiovascular events in this arm of the College of Obstetricians and Gynecologists. Obstet Gynecol 2013;121:
study. The findings of the WHI study were consistent with 887–90.
the vast majority of smaller clinical trials that preceded it. Rossouw JE, Anderson GL, Prentice RL, LaCroix AZ, Kooperberg C,
Stefanick ML, et al. Risks and benefits of estrogen plus progestin in
However, the cardiovascular results were an exception. healthy postmenopausal women: principal results From the Women’s
The WHI study showed that estrogen–progestin had no Health Initiative randomized controlled trial. Writing Group for the
benefit in terms of protection against the incidence of Women’s Health Initiative Investigators. JAMA 2002;288:321–33.
cardiovascular disease. Wassertheil-Smoller S, Hendrix SL, Limacher M, Heiss G, Kooperberg
C, Baird A, et al. Effect of estrogen plus progestin on stroke in post-
It has been postulated that the role of estrogen– menopausal women: the Women’s Health Initiative: a randomized trial.
progestin varies with the timing of initiation of therapy WHI Investigators. JAMA 2003;289:2673–84.
Reproductive Endocrinology and Infertility 45
TABLE 31-1. Relative Risks of Disease Outcomes from the Women’s Health Initiative Trials and Estimates of Absolute
Differences in Risk Among Women Aged 50–54 Years*
32
Fertility preservation in a patient undergoing cancer treatment
A 27-year-old nulligravid woman is referred to your office because she has a recent diagnosis of
breast cancer. She and her partner desire future childbearing. The treatment most likely to preserve
their fertility is
In vitro fertilization is a process in which exogenous therefore relocation of her ovaries offers her no fertility
gonadotropins are administered to a patient, eggs are sur- preservation.
gically extracted and fertilized using either partner sperm In vitro maturation is a technique to mature oocytes in
or donor sperm, and the resultant embryos are frozen for vitro that are either retrieved from unstimulated ovaries
future use. The technique has been used successfully for or from ovarian tissue. This technique has shown early
couples undergoing infertility treatment. Until recently, promise; however, the culture system associated with
embryo cryopreservation was the main recognized treat- this technique is complex and still requires validation
ment for reproductive-aged cancer patients whose treat- through published evidence-based research. At this time,
ment may render them sterile. it is not a reliable fertility preservation option for cancer
Before 2012, oocyte cryopreservation was considered patients.
experimental. In 2012, the experimental designation was Ovarian tissue freezing is a technique in which the
lifted by the American Society for Reproductive Med- cortex of an ovary is surgically stripped from the ovary.
icine, which reports that there are limited data on thawed The obtained ovarian tissue is subsequently frozen for
cryopreserved oocytes. However, the limited data suggest future use. The technique comprises the freezing of ovar-
that there is no increased risk of chromosomal abnormali- ian tissue and reimplantation of the tissue to the patient
ties, birth defects, or developmental deficits in offspring once she is cancer free or in remission. The main concern
born with the use of this technology. in regard to this technique is possible reintroduction into
Modifications in the cryopreservation techniques for the body of cancer-containing cells. An additional issue is
oocytes have improved the survival of cells. The vit- that only a limited series of studies have been published
rification freezing technique allows the oocyte to be to demonstrate that ovarian tissue freezing is a reliable,
cryopreserved in a glass-like state. The technology also reproducible, and successful technique.
is available to those who do not have a partner or to Researchers have theorized that gonadal suppres-
individuals who are unwilling to cryopreserve embryos. sion with GnRH agonist preserves hormonal function
Oocyte cryopreservation is not as successful as embryo and fertility. However, randomized control trials have
cryopreservation. Large observational clinical studies shown mixed results. As a consequence, GnRH agonist
demonstrate that implantation and pregnancy rates may would not be the patient’s best choice for reliable fertility
be lower when using frozen oocytes compared with fro- preservation.
zen embryos. Embryos are multicellular and can survive
the freeze and thaw process well. Although egg freezing Badawy A, Elnashar A, El-Ashry M, Shahat M. Gonadotropin-releasing
hormone agonists for prevention of chemotherapy-induced ovarian
is no longer considered experimental, ongoing research damage: prospective randomized study. Fertil Steril 2009;91:694–7.
is still necessary to assess the long-term effects of such Mature oocyte cryopreservation: a guideline. Practice Committees of
treatments. At this time, embryo cryopreservation offers the American Society for Reproductive Medicine and the Society for
the best treatment to preserve this patient’s fertility. Assisted Reproductive Technology. Fertil Steril 2013;99:37–43.
Ovarian transposition is a surgical technique to move Sverrisdottir A, Nystedt M, Johansson H, Fornander T. Adjuvant gos-
erelin and ovarian preservation in chemotherapy treated patients with
ovaries outside the field of radiation in cancer patients early breast cancer: results from a randomized trial. Breast Cancer Res
who require radiation treatment to the pelvis. The des- Treat 2009;117:561–7.
cribed patient does not require pelvic irradiation, and
Reproductive Endocrinology and Infertility 47
33
Turner syndrome
A 22-year-old nulligravid woman with secondary amenorrhea visits your clinic. Her follicle-
stimulating hormone level is 45 international units/L. She is 1.6 m (63 in.) tall and experienced
normal pubertal development with spontaneous onset of menstrual bleeding at age 13 years. The
most likely karyotype finding is
(A) 46,X,del(Xp)
(B) 45,X/46,XY
(C) 45,X
* (D) 45,X/46,XX/47,XXX
Turner syndrome was originally a clinical syndrome of and 68% of 45,X/46,XX/47,XXX Turner syndrome
phenotypic findings described by Henry Turner in 1938 individuals have spontaneous onset of menses. In one
before the discovery that its genetic basis is due to the review, Turner syndrome patients with a 46,X,del(Xp)
loss of a X chromosome, which results in a 45,X karyo- karyotype did not initiate puberty. It appears that dele-
type. The classic physical findings of Turner syndrome tion of the short arm of one of the X chromosomes is
are short stature of less than 1.47 m (58 in.), lack of sufficient to lead to the Turner syndrome phenotype.
secondary sexual development, webbed neck, and cubitus Typically, the karyotype in a Turner syndrome patient
valgus. The incidence of Turner syndrome is approxi- is only determined from the DNA isolated from periph-
mately 1 in 2,500 live female births. Approximately 50% eral venous blood leukocytes. The karyotype also can
of Turner patients have a pure 45,X cell line without be obtained from other tissue such as fibroblasts from
detectable mosaicism; the remaining 50% of patients a muscle biopsy or ovarian cells from an ovarian tissue
have a wide variety of mosaicism. biopsy. Karyotypes have been found to differ depending
The presentation of Turner syndrome is variable and on the source of the tissue.
is not uniformly correlated with the presence or degree
of mosaicism. The most important characteristics of
Turner syndrome include the cardiac and renal anoma- BOX 33-1
lies. The cardiac defects include coarctation of the aorta,
bicuspid aortic valve, and dilated aortic root. Dissection Phenotypic Findings in Turner Syndrome
Patients
with or without rupture of the aorta can occur in any
Turner syndrome patient at any time in her life without • Short stature less than 1.47 m (58 in.), sexual
identifiable risk factors. Turner syndrome patients with infantilism, webbed neck, and cubitus valgus
hypertension or a dilated aortic root are at greater risk • Cardiac defects (eg, coarctation of the aorta,
of aortic dissection. Patients with Turner syndrome who bicuspid aortic valve, and dilated aortic root)
conceive spontaneously or through oocyte donation are at • Renal anomalies (eg, horseshoe kidney and
double or cleft renal pelvis)
greater risk of maternal morbidity and mortality because
• Edema of the hands and feet
of the increased cardiovascular demands of pregnancy. • Low posterior hairline
Furthermore, after pregnancy, such individuals remain • Nail dysplasia or hypoplasia
at increased risk of cardiovascular events. The maternal • Rotated ears
mortality rate for Turner syndrome patients who con- • Small mandible
ceive is estimated to be 2%, which represents a 150-fold • Multiple pigmented nevi
increase compared with the general population. Box 33-1 • Broad shield chest leading to widely spaced
lists phenotypic findings in Turner syndrome patients. nipples
Besides short stature, most Turner syndrome patients • Short fourth metacarpal
have absent or delayed puberty, but the presentation can • High arched palate
be quite variable. Even in 45,X Turner syndrome indi- Bondy CA. Care of girls and women with Turner syn-
viduals, 11% can have puberty with spontaneous onset of drome: a guideline of the Turner Syndrome Study Group.
menses at the appropriate time. The occurrence of puber- Turner Syndrome Study Group. J Clin Endocrinol Metab
tal changes with menses appears to increase with the pres- 2007;92:10–25.
ence of mosaicism. Approximately 34% of 45,X/46,XX
48 PROLOG
Genetic abnormalities are likely in the described patient Bondy CA. Care of girls and women with Turner syndrome: a guideline
of the Turner Syndrome Study Group. Turner Syndrome Study Group. J
who has secondary amenorrhea and an elevated follicle- Clin Endocrinol Metab 2007;92:10–25.
stimulating hormone level suggestive of premature ovar- Bondy CA, Matura LA, Wooten N, Troendle J, Zinn AR, Bakalov VK.
ian failure. Because she had appropriate pubertal changes The physical phenotype of girls and women with Turner syndrome is not
with spontaneous menses, a deletion of the short arm of X-imprinted. Hum Genet 2007;121:469–74.
the X chromosome, ie, 46,X,del(Xp), is unlikely. Patients Karnis MF. Fertility, pregnancy, and medical management of Turner
syndrome in the reproductive years. Fertil Steril 2012;98:787–91.
with a 46,X,del(Xp) karyotype have not been reported
Zhong Q, Layman LC. Genetic considerations in the patient with
to undergo puberty. She is also taller than patients with Turner syndrome—45,X with or without mosaicism. Fertil Steril 2012;
a pure 45,X karyotype, suggesting a mosaic karyotype. 98:775–9.
In comparing a 45,X/46,XX/47,XXX karyotype with
the 45,X/46,XY karyotype, the 45,X/46,XX/47,XXX
karyotype is more likely to be associated with a history of
pubertal change and spontaneous menses.
34
Postmenopausal uterine bleeding and hormone therapy
A 63-year-old woman, gravida 2, para 2, has been taking hormone therapy (HT) for 11 years. She
tells you that she has been experiencing vaginal spotting in recent weeks. She has not been sexually
active for more than 10 years and her pelvic examination shows no signs of infection. Transvaginal
ultrasonography indicates no evidence of polyps, leiomyomas, or ovarian enlargement. The endo-
metrial stripe is 3 mm. Your next step in management is
* (A) observation
(B) saline sonohysterography
(C) endometrial sampling
(D) hysteroscopy
(E) dilation and curettage
Endometrial abnormalities in asymptomatic women are endometrial thickness of less than or equal to 4 mm.
rare. Patients who have postmenopausal bleeding often In patients with an endometrial thickness of more than
will have abnormalities of the endometrium that are either 4 mm, endometrial sampling is warranted to rule out
benign or malignant. The clinical approach to postmeno- malignancy. If the endometrial thickness is less than or
pausal bleeding requires prompt evaluation for endome- equal to 4 mm, endometrial sampling is not required.
trial carcinoma. Bleeding that occurs in patients who are Observation would be the appropriate management for
taking HT may be caused by poor adherence, inadequate the described patient at this time. Saline sonohysterog-
dosing, or poor absorption of the medication. Clinicians raphy, endometrial sampling, hysteroscopy, and dilation
should use endometrial biopsy or transvaginal ultrasonog- and curettage are all more invasive and are not clinically
raphy to assess patients with postmenopausal bleeding. indicated.
Cancer of the endometrium is the most common gyne- Endometrial thickness is measured as the maximum
cologic malignancy among U.S. women. Although vagi- anterior–posterior distance of the endometrial stripe on
nal bleeding is the presenting sign in more than 90% of a long-axis transvaginal view of the uterus. Adequate
postmenopausal patients with endometrial carcinoma, the visualization of the endometrium by means of transvagi-
vast majority of cases of postmenopausal bleeding are nal ultrasonography may not be possible in all patients
due to atrophic changes of the vagina or endometrium. because of anatomic changes such as prior surgery, obe-
The incidence of endometrial cancer in patients who sity, or myomas. Transvaginal ultrasonography can be
present with postmenopausal bleeding is approximately useful in the triage of patients who underwent endometrial
1–14% depending on age and risk factors. The probabil- sampling and had insufficient tissue for diagnosis. No fur-
ity of malignancy is greatly reduced in patients with an ther evaluation is needed if transvaginal ultrasonography
50 PROLOG
35
Intrauterine microinsert and pregnancy
* (A) 25%
(B) 40%
(C) 55%
(D) 70%
(E) 85%
Female sterilization with the use of a hysteroscopi- would demonstrate complete tissue encapsulation 4 years
cally placed permanent contraceptive microinsert was (48 months) later would appear to be a conservative one.
approved by the U.S. Food and Drug Administration In the time since the initial introduction of the con-
in 2002. After undergoing the procedure, some women traceptive microinsert system, clinicians have extended
have desired to conceive and many have considered the indications for the system to include its off-label use
using IVF–ET to resolve their tubal factor infertility. By to occlude a hydrosalpinx proximally as an alternative
design and variations in the placement technique of the to a salpingectomy procedure before an embryo trans-
microinserts, the number of outer coils of the microinsert fer. The hysteroscopic occlusion may be preferable to
that extend into the uterine cavity might vary from one laparoscopic tubal ligation or salpingectomy because of
individual to another. The manufacturer recommends either the patient’s prior history of abdominal surgeries or
that 3–8 expanded outer coils protrude into the uterine obesity, as well as to minimize operative anesthesia time.
cavity after initial placement. Two potential areas of con- Prior studies have demonstrated an improved implanta-
cern for a woman who is considering an embryo transfer tion and delivery rate after surgical removal of proximal
with the microinsert in place are 1) a possible decreased tubal occlusion of a hydrosalpinx.
implantation rate per embryos transferred and 2) the pos- In a small prospective, single-arm, clinical study of
sible disruption of the normal development of a fetus if 20 women with unilateral or bilateral hydrosalpinx vis-
any remnant of the outer coils protrudes into the uterine ible by transvaginal ultrasonography, microinserts were
cavity. To address these concerns, the manufacturer has placed in all patients in an ambulatory setting without
stated in the package insert that “Effects, including risks, complications. Clinicians inserted the microinserts with a
of . . . [the] inserts on IVF have not been evaluated. Risks mean number of three outer coils protruding into the uter-
to the patient, fetus and continuation of pregnancy are ine cavity (range: 1–4 outer coils). This deeper placement
unknown.” of the microinsert rather than the 3–8 expanded outer
To quantify these concerns, investigators in one study coils recommended by the manufacturer was done to
performed a second-look hysteroscopy procedure in minimize a potential effect of the microinsert on embryo
22 patients (20 with uterine bleeding and 2 pre-IVF). They implantation and development. Proximal tubal occlusion
demonstrated that 25% of the women (4 of 16 patients) was confirmed 3 months after placement by hysterosal-
had complete tissue encapsulation of the microinsert pingography in 95% of patients (19 out of 20) and 96%
13–43 months after microinsert placement. Complete of treated fallopian tubes (26 out of 27). After the hys-
tissue encapsulation was defined as endometrial tissue terosalpingography, 45 embryo transfer procedures were
ingrowth covering the microinsert. It was hypothesized performed in 19 patients, resulting in 18 pregnancies with
that the endometrial tissue around the tubal ostium used 12 live births, 6 miscarriages, and 1 premature delivery
the trailing coils of the microinsert as a scaffolding with eventual death of the infant for a 63% cumulative
structure to gradually cover over the microinsert and live-birth rate per patient and a 27% cumulative live-birth
exclude it from the uterine cavity. The investigators also rate per embryo transfer. To further evaluate the safety
demonstrated that over time the trailing coils protruding and efficacy of this procedure, the authors are conduct-
into the uterine cavity shortened. Given the fact that these ing a multicenter randomized controlled trial to compare
individuals were observed for a variable time interval of microinsert placement with salpingectomy for the treat-
13–43 months, the estimate that at least 25% of the women ment of a hydrosalpinx before IVF–ET.
52 PROLOG
Kerin JF, Munday D, Ritossa M, Rosen D. Tissue encapsulation of Mijatovic V, Dreyer K, Emanuel MH, Schats R, Hompes PG.
the proximal Essure micro-insert from the uterine cavity follow- Essure® hydrosalpinx occlusion prior to IVF–ET as an alternative
ing hysteroscopic sterilization. J Minim Invasive Gynecol 2007;14: to laparoscopic salpingectomy. Eur J Obstet Gynecol Reprod Biol
202–4. 2012;161:42–5.
36
Leiomyoma in infertility
A nulliparous 40-year-old woman comes to your office with a 1-year history of heavy menstrual
bleeding and a normal hemoglobin level. She has no pain or additional symptoms. Office ultra-
sonography reveals a thickened endometrium measuring 22 mm. Follow-up sonohysterography
is suggestive of the presence of a 1.5-cm submucosal leiomyoma type 2 based on the leiomyoma
subclassification system (Fig. 36-1), a subset of the International Federation of Gynecology and
Obstetrics classification system for causes of abnormal uterine bleeding in nongravid women of
reproductive age (Appendix C). She would like to preserve her future fertility and requests nonsur-
gical options for management. The most effective long-term treatment option for her is
Polyp Coagulopathy
Adenomyosis Ovulatory dysfunction
Submucosal
Leiomyoma Endometrial
Other
Iatrogenic
Malignancy and hyperplasia
Not yet classified
FIG. 36-1. Classification system for abnormal uterine bleeding including leiomyoma subclassification system in non-
gravid women of reproductive age. (Munro MG. Uterine leiomyomas, current concepts: pathogenesis, impact on
reproductive health, and medical, procedural, and surgical management. Obstet Gynecol Clin North Am 2011;38:
703–31 copyright 2011 with permission from Elsevier. Adapted from Munro MG. Abnormal uterine bleeding. New York:
Cambridge University Press, 2010.)
Reproductive Endocrinology and Infertility 53
Uterine leiomyomas are common uterine neoplasms found Insufficient evidence exists in regard to the use of
in approximately 70–80% of premenopausal women. It medroxyprogesterone acetate, a systemic progestin, for
has been estimated that leiomyomas account for 30–40% women with leiomyomas. Systemic progestins may tem-
of the approximately 600,000 hysterectomies performed porarily stabilize the endometrium, but are ultimately
annually in the United States. Growth of leiomyomas is ineffective as a treatment for leiomyomas.
largely dependent on female gonadal steroids, especially Aromatase inhibitors, such as letrozole, inhibit the
estrogens and progesterone, and after menopause they physiological conversion of androgens to estrogens in the
can spontaneously regress. Women with leiomyomas ovary and peripheral tissues. The relative hypoestrogen-
experience symptoms such as infertility, abnormal uter- emia induced by these medications can reduce leiomyoma
ine bleeding, and pelvic pain or pressure; however, most volume without vasomotor symptoms. Long-term use of
cases of leiomyomas are asymptomatic. Although there such medications in premenopausal women can induce
has been an increase in the number of minimally invasive folliculogenesis and ovarian cysts. To date, there have
procedures to surgically address these neoplasms, the use been no large-scale trials of aromatase inhibitors to treat
of appropriate nonsurgical therapy should be discussed leiomyomas.
with the patient as a viable alternative. The best option to treat the described patient’s leio-
The use of GnRH agonist induces amenorrhea second- myoma is a levonorgestrel IUD. The levonorgestrel IUD
ary to suppression of gonadotropin release and halted fol- delivers 20 micrograms per day of active hormone into
liculogenesis, creating a hypoestrogenic state that leads to the uterine cavity over a period of 5 years with minimal
size reduction. This may be helpful for short-term courses systemic absorption. It is a very effective contraceptive
to reduce the size of the leiomyoma before surgery and and also has been shown to be useful in the control of
to be able to convert an otherwise difficult or open case menorrhagia. Endometrial suppression occurs over a
into a minimally invasive one. It has been shown that period of 3–4 months when most women will experi-
GnRH agonist results in a reduction of the leiomyoma and ence irregular bleeding or spotting. By the sixth month,
total uterine volume by a mean of approximately 50% amenorrhea and oligomenorrhea occur in 50% and 25%
by 12 weeks of treatment. However, the effect is tem- of women, respectively, and usually persist for the 5-year
porary because the leiomyoma will return to baseline duration of therapy. A prospective clinical trial that
level within a few months after the cessation of therapy. included women with abnormal uterine bleeding and at
Therefore, GnRH agonist therapy is not a long-term solu- least one type II submucosal leiomyoma of 5 cm or less
tion to leiomyomas. Adverse effects of GnRH agonist demonstrated a 90% reduction in menstrual blood loss at
therapy associated with hypoestrogenemia include vaso- 3 months, 6 months, and 12 months after insertion and a
motor symptoms, vaginal atrophy, and osteopenia if the low expulsion rate. These results compare favorably with
therapy lasts longer than 6 months. those seen for the endometrial ablation thermal balloon.
The use of mifepristone, a selective progesterone recep-
tor modulator, has been shown to reduce or even eliminate Munro MG. Uterine leiomyomas, current concepts: pathogenesis,
impact on reproductive health, and medical, procedural, and surgical
abnormal uterine bleeding associated with leiomyomas. management. Obstet Gynecol Clin North Am 2011;38:703–31.
Some studies have shown that endometrial hyperplasia is Soysal S, Soysal ME. The efficacy of levonorgestrel-releasing intrauter-
an adverse effect of the use of mifepristone. Large-scale ine device in selected cases of myoma-related menorrhagia: a prospec-
clinical trials are necessary to further elucidate the value tive controlled trial. Gynecol Obstet Invest 2005;59:29–35.
of this approach.
54 PROLOG
37
Endometriosis
A 30-year-old nulligravid woman reports that she and her partner have been trying to conceive
for 18 months without success. She has severe dysmenorrhea and deep dyspareunia. Transvaginal
ultrasonography identifies a 6-cm ovarian cyst consistent with an endometrioma. The best next
step in management to improve her symptoms and maximize her chances of conception in the
next 6 months is
Women with endometriosis typically have cyclic pelvic but significant improvement in live-birth rates. In patients
pain, dyspareunia, adnexal masses, and infertility. Clas- with stage III–IV endometriosis, a nonrandomized study
sical studies suggest that approximately 25–50% of demonstrated that the cumulative pregnancy rates in
infertile women have endometriosis and that 30–50% of 216 infertile patients monitored up to 2 years after surgi-
women with endometriosis are infertile. Among women cal resection were 45–63%. Laparoscopic cystectomy
with pelvic pain, the prevalence of endometriosis ranges for ovarian endometriomas greater than 4 cm improved
from 30% to 80%. Although a direct causal mecha- fertility compared with cyst drainage and coagulation.
nism for the role of endometriosis in infertility has not The health care provider should make all attempts to
been established, a reasonable body of evidence sup- preserve functioning ovarian tissue. The possible adverse
ports the association between the two disease processes. consequences of loss of ovarian cortex and periovarian
Mechanisms suggested include altered pelvic anatomy adhesion formation must be weighed against the possible
impairing oocyte release or inhibiting ovum capture, improvement in fecundity and pain symptoms. After
altered peritoneal function, altered hormonal and cell- the first infertility operation, particularly in the setting
mediated function, abnormal uterotubal transport, poor of diminished ovarian reserve, patients with suspected
oocyte quality, impaired implantation, and endocrine and stage III–IV endometriosis may be better served by
ovulatory abnormalities. Intrauterine insemination or in using assisted reproductive technologies as the first-line
vitro fertilization alone would not improve this patient’s therapy.
pain symptoms. Although gonadotropin-releasing hor-
mone agonists would improve her pain symptoms, they Bulun SE. Endometriosis. N Engl J Med 2009;360:268 –79.
would impair her fertility. Chapron C, Vercellini P, Barakat H, Vieira M, Dubuisson JB.
Management of ovarian endometriomas. Hum Reprod Update 2002;
The American Society for Reproductive Medicine 8:591–7.
classification system for endometriosis is the most widely Crosignani PG, Vercellini P, Biffignandi F, Costantini W, Cortesi I,
accepted staging system for endometriosis. Medical Imparato E. Laparoscopy versus laparotomy in conservative surgical
therapy to suppress endometriosis has not been shown to treatment for severe endometriosis. Fertil Steril 1996;66:706 –11.
improve fecundity rates and may result only in a delay Endometriosis and infertility: a committee opinion. Practice Committee
of the American Society for Reproductive Medicine. Fertil Steril 2012;
in the use of more effective treatments to achieve preg- 98:591–8.
nancy. Surgery for stage III or IV endometriosis can be Revised American Society for Reproductive Medicine classification of
helpful to treat pelvic adhesions that may affect reproduc- endometriosis: 1996. Fertil Steril 1997;67:817–21.
tive function.
In stage I–II endometriosis, laparoscopic ablation of
endometrial implants has been associated with a small
Reproductive Endocrinology and Infertility 55
38
Male factor infertility
A 36-year-old nulligravid woman comes to your office and reports that she has spent 8 months try-
ing to conceive without success. Her male partner is unable to provide a semen specimen because
of erectile dysfunction (ED). He reports low energy, low libido, and generalized fatigue; evaluation
reveals hyperprolactinemia and a pituitary microadenoma. The treatment most likely to improve
his symptoms is
Hyperprolactinemia, although a rare condition, often is may require human chorionic gonadotropin and human
associated with ED, diminished libido, and orgasmic menopausal gonadotropin injection in addition to dopa-
or ejaculatory dysfunction. The condition should not be mine agonist therapy to enhance spermatogenesis.
neglected in men because many cases result from pitu- Direct-to-consumer marketing for testosterone supple-
itary tumors. All types of hyperprolactinemia (idiopathic, mentation has increased in recent years and has created
tumoral, or drug induced) can inhibit most aspects of male a subset of male fertility patients who have unknowingly
sexual behavior. A literature review that encompassed suffered from iatrogenic hypogonadotropic hypogonad-
studies that included a total of more than 300 hyperpro- ism. Although testosterone supplementation may help
lactinemic men found sexual dysfunction in 88% of the hypogonadotropic male patients to improve libido and
men, including ED in almost every case. Typically, ED arousal, spermatogenesis will not improve.
was found to be associated with reduced sexual desire. Sildenafil citrate is an effective treatment to improve
The nonsexual symptoms of hyperprolactinemia were erectile performance in the male patient with dysfunc-
less frequent: reduced body hair in approximately 40% of tion. The described patient’s symptoms are a result
the men, gynecomastia in 21%, and galactorrhea in 13%. of low testosterone that will not be affected by silde-
Systematic determinations of serum prolactin in patients nafil treatment. Clomiphene citrate treatment requires a
with ED found very low levels of hyperprolactinemia functioning hypothalamic–pituitary–testicular axis to
(1–5%). The mechanism suggested for prolactin-induced provide any clinical results. The described patient has
sexual dysfunction is a decrease in testosterone secre- hypothalamic–pituitary–testicular axis dysfunction due to
tion. Hyperprolactinemia impairs pulsatile luteinizing hyperprolactinemia. Dopamine agonist is the treatment
hormone release, which results in a decrease of serum that is most likely to improve his symptoms.
testosterone secretion.
Dopamine agonist therapy is the first-choice treatment Buvat J. Hyperprolactinemia and sexual function in men: a short
for prolactin-induced sexual dysfunction. In some cases of review. Int J Impot Res 2003;15:373–7.
pituitary adenoma, hypogonadism persists despite return Chao J, Hwang TI. Contemporary management of erectile dysfunction.
Urol Sci 2013;24:35–40.
of a normal prolactin level because of definitive interrup-
tion of the hypothalamic–pituitary–testicular connection, Colao A, Di Sarno A, Guerra E, De Leo M, Mentone A, Lombardi G.
Drug insight: Cabergoline and bromocriptine in the treatment of hyper-
destruction of the pituitary gonadotrophs by the pituitary prolactinemia in men and women. Nat Clin Pract Endocrinol Metab
tumor, or as a result of pituitary resection. Such patients 2006;2:200–10.
56 PROLOG
39
Use of donor sperm for same-sex couples
A 33-year-old woman comes to your office with her 30-year-old female partner to discuss using a
known sperm donor for insemination. You meet with the 25-year-old donor to discuss the process
of donating his sperm and to obtain a medical and family history from him. He is a healthy man
but has two cousins who died in their 20s from sudden cardiac arrest; no further evaluation was
performed. You are concerned that this potential donor may be at risk of a genetic condition that
would predispose him and his offspring to cardiac arrest. The most appropriate next step would be
Artificial insemination is the technique of introducing together, and it is recommended that legal issues (such as
sperm into the vagina or uterus without sexual inter- parental rights and responsibilities) be addressed before
course. Clinically, this technique is performed for several donor insemination because such legal issues may vary
indications, eg, in cases of unexplained or male factor from state to state. All donors must undergo an extensive
infertility, when it usually is used with the male partner’s history (personal, medical, sexual, and family history) as
sperm. A therapeutic donor insemination is performed well as a physical examination and routine blood tests.
using sperm from a man who is not the patient’s partner. Guidelines require that sperm samples from anonymous
A number of circumstances may call for use of thera- male donors be quarantined for at least 6 months to allow
peutic donor insemination, eg, in men with severe male for repeat sexually transmitted infection testing of the
factor infertility, couples that carry a heritable disease, donor before release of the sperm for donor insemination.
and, commonly, women who do not have a male part- Female recipients who use a known sperm donor may
ner. The American Society for Reproductive Medicine defer this quarantine in selected situations if they recog-
Ethics Committee guidelines state that single, gay, and nize and accept the potential risk of undiagnosed sexually
lesbian patients should have access to fertility services. transmitted infections.
Lesbian couples often use therapeutic donor insemination The guidelines also recommend that potential donors
to attempt pregnancy. A 2001 study found that approxi- be screened for common genetic mutations, such as cystic
mately 75% of U.S. fertility clinics provide such services fibrosis, regardless of ethnic background. Other genetic
to single and lesbian women. testing should be considered based on the donor’s ethnic-
Many patients select an anonymous donor from a ity and family history. Karyotype testing sometimes is
sperm bank, although some elect to have a “known” recommended, although it is not a routine requirement.
sperm donor. Regardless of plans to use an anonymous or In cases where the medical or family history of a
known donor for donor insemination, recommendations potential donor raises suspicion for an inheritable disease,
are available on how to select donors in a fashion that is further testing may be warranted before the individual is
safe for the donor and the recipient. The American Society accepted as a donor. It is not necessary to reject a donor
for Reproductive Medicine’s “Guidelines for Gamete and solely based on concern for carrier status before appropri-
Embryo Donation” provide a summary of the current ate confirmatory testing is completed. In the described
recommendations, including information from the U.S. scenario, the potential donor’s history raises suspicion
Food and Drug Administration about screening donors for for an inherited heart disease, such as hypertrophic car-
communicable diseases. For example, donors must be of diomyopathy (HCM), given that he had two cousins with
legal age (older than 17 years) and ideally younger than early sudden cardiac arrest and death. In one reported
40 years. Donors must undergo a psychologic assessment case, the sperm from an anonymous sperm donor with
by a mental health professional to address the emotional, undiagnosed HCM resulted in at least 22 offspring. After
psychologic, and ethical aspects of being a sperm donor. he was diagnosed with HCM, the children were tested
In cases in which the sperm donor is a friend or a fam- and at least eight were found to be affected. That case and
ily member of the recipient couple, the donor and recipi- other similar cases raise issues about the proper protocol
ent often will meet with the mental health professional for genetic testing for potential donors.
Reproductive Endocrinology and Infertility 57
Many sperm and egg donors are young men and counseling be part of the testing process to allow individ-
women in their 20s who have not had the need or inclina- uals to make educated decisions about whether to proceed
tion to contemplate their own genetic health. Given the with testing. In addition, they need information about dis-
complexities associated with a diagnosis of an adult-onset ease risk if indeed they are carriers of a mutation as well
disorder, such as HCM, breast cancer, and Alzheimer dis- as residual risk if they are not found to be carriers. When
ease, donors should be given the opportunity to make an a health care provider is evaluating a potential donor, the
informed choice about proposed genetic testing and their donor becomes their patient, and the health care provider
desire to learn the results. Therefore, in the described is then responsible for recommendations about his or her
scenario, it is not appropriate to proceed with genetic test- health and testing. The health care provider should thus
ing without first addressing the implications of the testing proceed in a fashion that protects the rights and choices
with the donor. Similarly, although the recipient couple of the donor.
likely has an interest in genetic testing for the selected
Daar JF, Brzyski RG. Genetic screening of sperm and oocyte donors:
donor, they are not the ultimate decision makers about ethical and policy implications. JAMA 2009;302:1702–4.
whether the donor proceeds with this testing. If he opts Maron BJ, Lesser JR, Schiller NB, Harris KM, Brown C, Rehm HL.
not to pursue testing, they may decide not to use him as Implications of hypertrophic cardiomyopathy transmitted by sperm
their donor, given the gap in information about genetic donation. JAMA 2009;302:1681–4.
risk for their offspring. Recommendations for gamete and embryo donation: a committee
opinion. Practice Committee of American Society for Reproductive
For targeted genetic testing for high-risk individu- Medicine and the Practice Committee of Society for Assisted
als, it is recommended that pretest and posttest genetic Reproductive Technology; Fertil Steril 2013;99:47–62.
40
Phantom human chorionic gonadotropin results
A 28-year-old woman, gravida 4, para 3, underwent methotrexate treatment for an ectopic preg-
nancy 3 months ago. She has followed up with monthly serum β-hCG quantitative values. These
values have consistently been between 127 mIU/mL and 132 mIU/mL. She has been taking oral
contraceptives since her most recent visit. The best next step in her management is
False-positive or phantom serum β-hCG test results can on farms, and women who regularly work close to or with
lead to misdiagnosis and mismanagement of the patient. animal tissue.
It is critical to consider false-positive β-hCG test results Heterophilic antibodies can cross-react with immuno-
when monitoring patients with pregnancy of uncertain globulins from other species. Patients who have hetero-
location, molar gestation, or gestational trophoblastic philic antibodies may have false-positive serum β-hCG
neoplasia. test results. Heterophilic antibodies tend to interfere with
Phantom β-hCG can be caused by the presence of two-sided immunometric assays commercially used for
heterophilic antibodies that can result in false elevation β-hCG measurements as well as with other hormonal
of serum β-hCG. Heterophilic antibodies are human anti- assays.
bodies that react against animal-derived antigens used in To evaluate patients for phantom β-hCG, serum and
immunoassay testing. Women who have a history of expo- urine samples should be assessed. Patients with phantom
sure to animals are at risk of heterophilic antibody develop- β-hCG usually have no measurable β-hCG in a parallel
ment over their lifetime. Such individuals include women urine sample. The main limitation in this technique is that
laboratory workers who work with animals, women raised patients with low β-hCG levels may have urine levels
58 PROLOG
too low to detect the true presence of β-hCG. Typically, to order a chest X-ray at this point in her management
serum β-hCG values less than 25 mIU/mL are considered given that metastatic gestational trophoblastic disease
too low to detect in urine. is unlikely. Additionally, it is very rare to have gesta-
Flooding the sample with a mixture of nonspecific ani- tional trophoblastic disease after an ectopic pregnancy.
mal antibodies or antibodies against heterophilic antibod- Performing ultrasonography of the uterus is only appro-
ies is another strategy to assess if β-hCG levels are false. priate if the β-hCG value is noted to be real and not a
If subsequent testing of the sample does not demonstrate false elevation.
a β-hCG value, then heterophilic antibodies are likely the It is inappropriate to administer an additional dose of
cause for false β-hCG elevation. methotrexate to this patient. She has already been treated
Patients with suspected heterophilic antibodies should medically for an ectopic pregnancy. Other causes for per-
have β-hCG levels measured in serial dilutions. The sistent elevations (phantom β-hCG) should be considered.
serial dilutions of a true serum β-hCG value should fol-
Avoiding inappropriate clinical decisions based on false-positive
low a linear β-hCG curve. Serial dilutions that do not human chorionic gonadotropin test results. ACOG Committee Opinion
follow a linear curve suggest the presence of heterophilic No. 278. The American College of Obstetricians and Gynecologists.
antibodies and, therefore, a false or phantom β-hCG Obstet Gynecol 2002;100:1057–9.
value (see Table 40-1). Berkowitz RS, Goldstein DP. Current management of gestational tro-
phoblastic diseases. Gynecol Oncol 2009;112:654–62.
The patient is not a candidate to repeat her β-hCG
Cole LA, Kohorn EI. The need for an hCG assay that appropriately
level in 2 days because it is established that she has had detects trophoblastic disease and other hCG-producing cancers. J
consistent β-hCG levels every month. It is inappropriate Reprod Med 2006;51:793–811.
Test Explanation
Urine Assess β-hCG quantitative or qualitative Heterophilic antibodies are not in the urine. If urinary
urinary β-hCG. β-hCG test is negative and serum testing is positive,
heterophilic antibodies are likely present.
Serum Rerun serum β-hCG with serial dilutions Serial dilutions of serum measuring β-hCG should
of the serum. show a linear curve. Lack of linear β-hCG curve
confirms the presence of heterophilic antibodies.
Heterophilic antibodies react with reagents in the
immunoassay and not β-hCG.
Serum Laboratory to add heterophilic antibody If β-hCG is not detected after adding heterophilic
blocking agents to serum. antibody blocking agents, heterophilic antibodies can
explain false β-hCG elevations.
Avoiding inappropriate clinical decisions based on false-positive human chorionic gonadotropin test results. ACOG Committee
Opinion No. 278. American College of Obstetricians and Gynecologists. Obstet Gynecol 2002;100:1057–9.
Reproductive Endocrinology and Infertility 59
41
Normal menstrual cycle
A patient with menstrual cycles between 26 days and 30 days comes to your office for evaluation.
You tell her that the phase of the menstrual cycle that most influences cycle length is the
The menstrual cycle involves the interaction of different anterior pituitary due to prolonged exposure to estradiol.
hormones and endocrine glands as well as a responsive Ovulation typically takes place approximately 24 hours
uterus. The cycle occurs in three phases: 1) follicular after the luteinizing hormone peak. It is also constant
(proliferative), 2) ovulatory, and 3) luteal (secretory). and does not contribute to the variability in the menstrual
Follicular and luteal refer to ovarian changes and follicle cycle length.
development whereas proliferative refers to endometrial The luteal phase starts after ovulation. During this
changes. Menstruation is part of the proliferative phase phase, the remaining granulosa cells that are not released
of the menstrual cycle. The menstrual cycle also can with the oocyte become the corpus luteum, which pre-
be described based on the number of days between the dominantly secretes progesterone. Peak progesterone
onset of menstrual bleeding in one cycle and the onset production is noted 1 week after ovulation takes place
of bleeding in the next cycle. Most women have a men- (midluteal). Progesterone converts the proliferative endo-
strual cycle length between 25 days and 30 days, with the metrium into a secretory endometrium, allowing for pos-
median duration being 28 days. The variability in length sible embryo implantation. If a pregnancy takes place,
of a menstrual cycle is based on the variable length of human chorionic gonadotropin will maintain the corpus
the follicular phase; the luteal phase is constant in most luteum. However, if a pregnancy fails to occur, luteolysis
women at 14 days in length. Therefore, the luteal phase takes place and the corpus luteum is lost. The loss of the
does not contribute to the variability of the overall men- corpus luteum and progesterone lead to menstruation due
strual cycle length. However, disorders such as hyper- to the instability and sloughing of the endometrium. The
prolactinemia can decrease the length of this phase of the length of days that a patient bleeds does not contribute to
menstrual cycle. the total length of the cycle.
Folliculogenesis takes place during the follicular phase
Cahill DJ, Wardle PG, Harlow CR, Hull MG. Onset of the preovulatory
of the menstrual cycle. As follicles grow, a dominant luteinizing hormone surge: diurnal timing and critical follicular prereq-
follicle is selected and destined to ovulate. The growth uisites. Fertil Steril 1998;70:56–9.
of follicles is dependent on follicle-stimulating hor- Presser HB. Temporal data relating to the human menstrual cycle. In:
mone stimulation. During this follicle growth, estradiol Ferin M, Halber F, Richart RM, Van de Wiele R, editors. Biorhythms
and human reproduction. New York: John Wiley and Sons; 1974. p.
is produced and is responsible for the proliferation of 145–60.
the endometrial lining of the uterus. Ovulation happens Treloar AE, Boynton RE, Behn BG, Brown BW. Variation of the human
in response to the luteinizing hormone surge, which is menstrual cycle through reproductive life. Int J Fertil 1967;12:77–126.
released in a positive feedback mechanism from the Vollman RF. The menstrual cycle. Philadelphia: WB Saunders; 1977.
60 PROLOG
42
Dysmenorrhea
A 16-year-old adolescent woman comes to your office with dysmenorrhea, headache, diarrhea, and
back pain during menses that interferes with her ability to attend school. On bimanual examination
and ultrasonography, there is no evidence of gynecologic pathology. She used oral contraceptives
(OCs) in the past and developed minor depression. She now wishes to avoid using any hormonal
preparations. She requests information about other treatment options. The best next step for pain
relief from dysmenorrhea in this patient is
(A) acetaminophen
* (B) nonsteroidal antiinflammatory drugs (NSAIDs)
(C) presacral neurectomy
(D) opioids
(E) uterine nerve ablation
Dysmenorrhea is the occurrence of painful menstrual Dysmenorrhea may be treated with the use of NSAIDs,
cramps and other associated symptoms. It is one of the such as aspirin, propionic acid derivatives (eg, naproxen
most common gynecologic problems in reproductive- and ibuprofen), and fenamates (eg, mefenamic acid and
aged women. After ovulation, prostaglandins may be diclofenac). It is known that NSAIDs inhibit the cyclo-
produced in abundance in the late luteal phase, leading oxygenase (COX)-1 and COX-2 enzymes. Pain relief
to myometrial contractions and pain. Patients also may comes from their ability to block endometrial prostaglan-
experience additional symptoms such as headache, nau- din production. Clinical trials have shown the effective-
sea, backache, and diarrhea. Such additional symptoms ness of NSAIDs to be in the 70–90% range. Long-term
can be attributed to the release of prostaglandin com- use of NSAIDs may increase the risk of serious gastroin-
pounds into the systemic circulation. Primary dysmenor- testinal adverse effects such as ulceration, bleeding, and
rhea occurs in the absence of any demonstrable disease. perforation. Selective COX-2 inhibitors (eg, celecoxib)
Secondary dysmenorrhea refers to menstrual pain result- have been developed for patients at high risk of seri-
ing from pelvic pathology, such as endometriosis, uterine ous gastrointestinal tract adverse effects. These agents,
fibroids, or adenomyosis. however, are expensive and carry an increased risk of
Dysmenorrhea is most common in adolescence. serious and potentially fatal cardiovascular thrombotic
Studies have reported a prevalence of 60–90% among events, such as myocardial infarction and stroke. Because
adolescents. Incidence of the disease has been observed COX-2 inhibitors interfere with the formation of lipox-
to decrease with increasing age. Research has shown ins and prostacyclin in blood vessels, vasoconstriction
that women with dysmenorrhea have higher levels of and platelet aggregation can occur, resulting in possible
prostaglandins than asymptomatic females. Endometrial excess clot formation and hypertension. Long-term use
prostaglandin production reaches a peak in the first can lead to increased rates of myocardial infarction and
48 hours of menstruation, which coincides with the time life-threatening thrombotic events.
of greatest menstrual discomfort. Medical therapy for Epidemiologic studies have shown that women who
dysmenorrhea includes OCs and NSAIDs, both of which take OCs have a lower prevalence of dysmenorrhea. This
act by suppressing prostaglandin production and reducing is thought to be because OCs allow for an atrophic and
the volume of menstrual flow. Although these treatments decidualized endometrium that produces less prostaglan-
are very effective, a 20–25% failure rate leads patients to dins. By decreasing menstrual flow and prostaglandin-
seek additional treatments or surgery. mediated pain, OCs offer a safe and effective choice for
The secretory endometrium is rich in arachidonic acid, the treatment of dysmenorrhea.
leukotrienes, and prostaglandins. Release of these agents Numerous clinical trials have been performed to assess
into the systemic circulation leads to uterine smooth- the effectiveness of NSAIDs and OCs for dysmenorrhea.
muscle contractions. The result is crampy, spasmodic Systematic reviews of randomized controlled trials have
lower abdominal or back pain and myometrial ischemia. shown that NSAIDs are superior to acetaminophen or
Constriction of the uterine vessels reaches a peak on the placebo and are an effective treatment for dysmenorrhea.
first day of menstrual flow. Insufficient evidence is available to demonstrate the
Reproductive Endocrinology and Infertility 61
superiority of any individual NSAID for either pain relief Women who fail to gain relief from NSAIDs or OCs
or safety. To date, no studies have compared the perfor- have a variety of alternatives. Therapies that have been
mance of combination OCs with NSAIDs. Systematic proposed for the relief of dysmenorrhea include exercise,
review of cyclic OCs for dysmenorrhea has offered behavioral modification strategies, dietary modification,
limited evidence for pain improvement for low-dose herbal therapies, vitamins, and application of heat to the
and medium-dose OC preparations. No evidence has affected area. Surgery may be an option when medical
been found of a difference in efficacy between differ- treatment has failed.
ent OC preparations. The data suggest NSAIDs should Uterine nerve ablation and presacral neurectomy
be the treatment of choice for primary dysmenorrhea. have been used for dysmenorrhea. Both procedures are
These agents usually are used for short-term therapy just designed to interrupt most cervical sensory pain nerve
before menses and continued for up to 4–6 days. Oral fibers. Surgical interruption of pelvic nerve pathways
contraceptives may be a wise choice for any patient who also has been studied by systematic review. Insufficient
does not tolerate NSAIDs or who concurrently desires evidence is available to recommend the use of nerve
contraception. interruption in the management of dysmenorrhea, regard-
The best next step for pain relief from dysmenor- less of the cause.
rhea for the described patient is NSAIDs, given that
she has intolerance for the adverse effects of OCs. Archer DF. Menstrual-cycle related symptoms: a review of the ratio-
nale for continuous use of oral contraceptives. Contraception 2006;
Acetaminophen and presacral neurectomy would be less 74:359–66.
effective in her case than NSAIDs. Opioids should be Dawood MY. Primary dysmenorrhea: advances in pathogenesis and
avoided because of their adverse effects, potential for management [Review]. Obstet Gynecol 2006;74:439–45.
abuse, and possible dependency with long-term use. Majoribanks J, Proctor M, Farquhar C, Derks RS. Nonsteroidal anti-
Patients with secondary dysmenorrhea should have inflammatory drugs for dysmenorrhoea. Cochrane Database Syst Rev
2010;1:CD001751. DOI: 10.1002/14651858.CD001751.pub2.
further investigation of their underlying pathology. Such
Proctor M, Latthe P, Farquhar C, Khan K, Johnson N. Surgical interrup-
an investigation will likely guide treatment. Many of the tion of pelvic nerve pathways for primary and secondary dysmenorrhea.
therapies for conditions that cause dysmenorrhea will treat Cochrane Database Syst Rev 2005;4:CD001896.
the pain and the underlying pathology. Depot medroxy- Speroff L, Fritz MA. Menstrual disorders. In: Speroff L, Fritz MA.
progesterone acetate and the levonorgestrel intrauterine Clinical gynecologic endocrinology and infertility. 8th ed. Philadelphia
(PA): Lippincott Williams and Wilkins; 2011. p. 579–83.
device are effective in the treatment of endometriosis
Wong CL, Farquahar C, Roberts H, Proctor M. Oral contraceptive pill
and dysmenorrhea. Long-acting gonadotropin-releasing for primary dysmenorrhoea (Review). Cochrane Database Syst Rev
hormone agonists, such as depot leuprolide acetate, are 2009;4:CD002120. DOI: 10.1002/14651858.CD002120.pub3.
effective for endometriosis and dysmenorrhea.
62 PROLOG
43
BRCA mutations
A 25-year-old woman who is a known BRCA1 mutation carrier comes to your office for counseling.
You discuss risk-reduction strategies, including surgery, but she tells you that she is not ready to
consider mastectomy or salpingo-oophorectomy. In addition to semiannual clinical breast examina-
tions, you advise her that the recommended breast cancer screening strategy for a known BRCA1
mutation carrier would be annual
(A) mammography
(B) breast ultrasonography
(C) breast magnetic resonance imaging (MRI)
(D) mammography and breast ultrasonography
* (E) mammography and breast MRI
Hereditary breast and ovarian cancer syndrome is a gen- Prophylactic salpingo-oophorectomy decreases the risk
etic cancer-susceptibility syndrome. It usually is character- of ovarian cancer, fallopian tube cancer, and peritoneal
ized by multiple family members with breast cancer, cancer by 85–90% and simultaneously decreases the risk
ovarian cancer, or both; an individual with breast and ovar- of breast cancer by 50%. Similarly, prophylactic mastec-
ian cancer; or an individual with breast cancer at an early tomy lowers the risk of breast cancer by more than 90%.
age. Under appropriate circumstances, genetic testing can For patients who delay or decline surgery, screening
help physicians to identify patients who are at increased for breast and ovarian cancer is recommended. Screening
risk of breast and ovarian cancer. For at-risk individuals, for ovarian cancer has not been shown to decrease mor-
screening and prevention strategies should be discussed. tality or to improve survival, but, given the risk associ-
The BRCA1 and BRCA2 genes are tumor suppressor ated with BRCA1 and BRCA2 mutations, screening is
genes. Mutations in these genes are associated with ele- recommended. Consensus groups recommend periodic
vated risks of breast and ovarian cancer. In families with screening with CA 125 and transvaginal ultrasonography
hereditary breast and ovarian cancer syndrome, BRCA1 starting at approximately age 30 years or 5–10 years
and BRCA2 mutations account for the overwhelming earlier than the youngest age of diagnosis of ovarian
majority of cases. In the general population, however, cancer in an affected relative. Breast cancer surveillance
BRCA1 and BRCA2 mutations only account for approxi- is more reliable than ovarian cancer surveillance. Starting
mately 10% of cases of ovarian cancer and 3–5% of cases at age 25 years (or earlier based on the youngest age of
of breast cancer. For ovarian cancer, BRCA1 mutations diagnosis), BRCA1 and BRCA2 mutation carriers should
carry a 39–46% risk, whereas for breast cancer, BRCA1 undergo semiannual clinical breast examinations in addi-
and BRCA2 mutations carry a risk of approximately tion to annual mammography and breast MRI. Screening
65–74%. In the general population, between 1 in 300 with MRI is more sensitive than mammography. The
individuals and 1 in 800 individuals are estimated to carry combination of MRI, mammography, and clinical breast
a BRCA1 or BRCA2 mutation. In order to help physicians examination has the greatest sensitivity for detecting
determine which individuals should be offered genetic breast cancer in BRCA mutation carriers.
testing, the Society of Gynecologic Oncologists has Therefore, the most appropriate breast cancer screen-
established screening guidelines (Box 43-1). ing for the described BRCA1 mutation carrier, in addition
If a patient is found to be a carrier of a mutation, she to a semiannual clinical breast examination, is a combina-
and her family members should be offered genetic coun- tion of mammography and breast MRI. Mammography
seling. Mastectomy and salpingo-oophorectomy are the alone would be insufficient. Her needs also would not be
most effective strategies to reduce the risk of breast and met by breast MRI alone or a combination of mammogra-
ovarian cancer. Risk-reducing salpingo-oophorectomy phy and breast ultrasonography. Breast ultrasonography
is recommended for BRCA1 or BRCA2 mutation carri- is not recommended for screening high-risk individuals
ers by age 40 years or when childbearing is complete. but is useful for imaging dense masses.
Reproductive
Reproductive Endocrinology
Endocrinology and
and Infertility 3
Infertility 63
Hereditary
incidence breast and cancer.
of breast ovarian Prophylactic
cancer syndrome. ACOG Practice
mastectomy low-
BOX 43-1 Bulletin No. 103. American College of Obstetricians and Gynecologists.
ers the risk of breast cancer
Obstet Gynecol 2009;113:957–66. more than 90%. Prophylactic
Society of Gynecologic Oncologists salpingo-oophorectomy
Kriege lowers
M, Brekelmans CT, Boetes C,the risk of
Besnard PE,ovarian cancer
Zonderland HM,
Guidelines for Genetic Testing Obdeijn IM, et al. Efficacy of MRI and mammography for breast
by 96% while simultaneously lowering the risk of breast-
cancer
cancerscreening
by 50%.in women with a familial or genetic predisposi-
Patients with greater than approximately 20–25% tion. Magnetic Resonance Imaging Screening Study Group. N Engl J
chance of having an inherited predisposition to
The most appropriate screening for this 20-year-old
Med 2004;351:427–37.
breast and ovarian cancer and for whom genetic daughter of aPowell
Lancaster JM, known CB,BRCA1
Kauff ND,carrier
Casswho hasLM,
I, Chen beenLutreated
KH, et
risk assessment is recommended for breast cancer is a combination of mammography
al. Society of Gynecologic Oncologists Education Committee state- and
ment
breaston MRI.
risk assessment for inherited gynecologic
Other procedures cancer cytology
such as ductal predisposi-
• Women with a personal history of both breast tions. Society of Gynecologic Oncologists Education Committee.
and ovarian cancer and breast
Gynecol Oncolultrasonography
2007;107:159–62. are not recommended for
• Women with ovarian cancer* and a close rela- screening high-risk individuals. Breast ultrasonography
tive† with breast cancer at 50 years or less or is useful for imaging dense masses.
ovarian cancer at any age
• Women with ovarian cancer* at any age who Antoniou A, Pharoah PD, Narod S, Risch HA, Eyfjord JE, Hopper JL,
are of Ashkenazi Jewish ancestry et al. Average risks of breast and ovarian cancer associated with BRCA1
or BRCA2 mutations detected in case Series unselected for family his-
• Women with breast cancer at age 50 years or tory: a combined analysis of 22 studies. Am J Hum Genet 2003;72:
less and a close relative† with ovarian or male 1117–30.
breast cancer at any age.
Chen S, Parmigiani G. Meta-analysis of BRCA1 and BRCA2 pene-
• Women of Ashkenazi Jewish ancestry and trance. J Clin Oncol 2007;25:1329–33.
breast cancer at age 40 years or less King MC, Marks JH, Mandell JB. Breast and ovarian cancer risks due
• Women with a first- or second-degree relative to inherited mutations in BRCA1 and BRCA2. New York Breast Cancer
with a known BRCA1 or BRCA2 mutation Study Group. Science 2003;302:643–6.
Patients with greater than approximately 5–10% Kriege M, Brekelmans CT, Boetes C, Besnard PE, Zonderland HM,
Obdeijn IM, et al. Efficacy of MRI and mammography for breast-can-
chance of having an inherited predisposition to cer screening in women with a familial or genetic predisposition. N
breast and ovarian cancer and for whom genetic Engl J Med 2004;351:427–37.
risk assessment may be helpful ‡‡
Lancaster JM, Powell CB, Kauff ND, Cass I, Chen LM, Lu KH, et al.
• Women with breast cancer at 40 years or less Society of Gynecologic Oncologists Education Committee statement
• Women with bilateral breast cancer (particularly on risk assessment for inherited gynecologic cancer predispositions.
Society of Gynecologic Oncologists Education Committee. Gynecol
if the first cancer was at 50 years or less) Oncol 2007;107:159–62.
• Women with breast cancer at 50 years or less
and a close relative† with breast cancer at
50 years or less
• Women of Ashkenazi Jewish ancestry with
breast cancer at 50 years or less
• Women with breast or ovarian cancer at any
age and two or more close relatives† with breast
cancer at any age (particularly if at least one
breast cancer was at 50 years or less)
• Unaffected women with a first- or second-
degree relative that meets one of the above
criteria
44
Contraception for a patient with diabetes mellitus
A 19-year-old woman with a 5-year history of well-controlled type 1 diabetes mellitus has had
infrequent vaginal intercourse for 2 years. Although you counsel her about the benefits of long-
acting reversible contraception and depot medroxyprogesterone acetate, she rejects those options.
Therefore, the best contraceptive method for this patient would be
Women with type 1 diabetes mellitus who conceive and (10–35 micrograms) ethinyl estradiol OCs do not cause
continue an unplanned pregnancy during periods of metabolic disturbance of a woman’s diabetes and are safe
poor glycemic control are at increased risk of develop- for women with well-controlled diabetes. In typical use,
ing major maternal and fetal complications. Pregnancy approximately 9% of women who use OCs will experi-
should be contemplated when the patient’s hemoglobin ence an unintended pregnancy during the first year of use.
A1c level is optimal, but studies demonstrate that less In comparison, the failure rates for a number of other
than 50% of women with diabetes plan their pregnancies. methods are all higher with typical use: use of the con-
Moreover, most diabetes specialists feel unqualified to traceptive sponge in a nulliparous woman (12%), male
provide effective contraceptive counseling and defer this condom (18%), periodic abstinence during ovulation
responsibility to other women’s health care providers. (24%), and use of a vaginal spermicide (28%). The fail-
In a study of 107 women with diabetes who answered ure rate with typical use of other methods often used by
a research questionnaire survey on their contraceptive women with diabetes included 85% for no method of
knowledge and methods used, 36% of women stated they contraception and 9% for the progestin-only OC, despite
had not received any contraceptive advice in the past the fact that the progestin-only OC only inhibits ovulation
year; 8% of women recalled obtaining information from in approximately 50% of menstrual cycles. The following
their diabetes specialist; and the remaining women said methods eliminate or decrease the necessity of patient
they received information from their general practitioner adherence to the method and all have a lower failure
or family planning clinic. Women in this study had a rate than OCs: depot medroxyprogesterone acetate (6%),
mean age of 31 years with an age range of 16–44 years. levonorgestrel IUD (0.2%), copper IUD (0.8%), and pro-
They used the following methods for contraception: gestin implant (0.05%). Because this patient rejects these
combination oral contraceptives (OCs) (24%), condoms methods, the most efficacious contraceptive method for
(16%), progestin-only OCs (7%), intrauterine devices her would be a combination hormonal contraceptive.
(IUDs) (6%), natural family planning (6%), sterilization
Shawe J, Smith P, Stephenson J. Use of contraception by women with
(female or male partner) (5%), depot medroxyproges- type 1 or type 2 diabetes mellitus: ‘it’s funny that nobody really spoke
terone acetate (1%), and progestin implant (1%). Of the to me about it’. Eur J Contracept Reprod Health Care 2011;16:350–8.
women in the study, 34% used no contraceptive method. Use of hormonal contraception in women with coexisting medical
conditions. ACOG Practice Bulletin. No. 73. American College of
Although these women were taking insulin and other Obstetricians and Gynecologists. Obstet Gynecol 2006;107:1453–72.
medications to control their diabetes, the women had con- U.S. Selected Practice Recommendations for Contraceptive Use,
cerns about using any additional medication and elected 2013: adapted from the World Health Organization selected practice
not to use hormonal methods of contraception. Some recommendations for contraceptive use, 2nd edition. Division of
Reproductive Health, National Center for Chronic Disease Prevention
of this bias may be due to older OC formulations with and Health Promotion, Centers for Disease Control and Prevention.
higher doses of estrogen and progestin. Current low-dose MMWR Recomm Rep 2013;62(RR-05):1–60.
Reproductive Endocrinology and Infertility 65
45
Congenital adrenal hyperplasia
A 30-year-old woman, gravida 2, para 1, comes to your office for fertility treatment. She has a
history of bilateral tubal blockage. She underwent in vitro fertilization (IVF) 3 years ago and gave
birth to a severely virilized female infant with 21-hydroxylase deficiency. The child now has
learning disabilities. She wishes to avoid pregnancy termination. The best way to prevent virilized
female infants in subsequent pregnancies is
Congenital adrenal hyperplasia (CAH) is caused by a and clitoromegaly. Despite the virilization of the female
defect in one of the five enzymatic steps in the adrenal external genitalia, the ovaries, uterus, and fallopian tubes
cortex necessary in the conversion of cholesterol to cor- are normal. Affected males have normal external genital
tisol. The most common defect is related to 21-hydroxy- development and may escape initial diagnosis; however,
lase deficiency, an autosomal recessive disorder due to both genders experience varying degrees of electrolyte
mutations in the CYP21A2 gene, which accounts for abnormalities at birth that require immediate attention
approximately 90–95% of all cases or 1 in 14,500 new- because of a significant risk of sudden death in the
borns. The CYP21A2 gene is part of a complex major newborn.
histocompatibility locus on chromosome 6p21. To date, Beginning in the 1980s, supraphysiologic corticoste-
more than 170 CYP21A2 mutations have been identified. roid treatment was used to decrease the virilization of the
Because CAH is autosomal recessive, a patient who gives external genitalia of affected female fetuses. However,
birth to a child with CAH has a one in four chance of giv- recent clinical observations and animal studies have
ing birth to a subsequent child with CAH. There is also raised concerns regarding the safety of this prenatal
a two in four chance that the fetus will be unaffected but treatment.
carry the mutation, and a one in four chance that the fetus The fetal adrenal cortex is composed of a fetal zone and
will not be a carrier. Given that one half of conceptions a definitive (adult) zone. The fetal zone primarily secretes
are female, the risk of having a female with a genital tract dehydroepiandrosterone and dehydroepiandrosterone
anomaly is one in eight. The 21-hydroxylase enzyme sulfate, which are converted to estriol by the placenta.
converts 17-hydroxyprogesterone to 11-deoxycortisol in Fetal cortisol synthesis is low for much of the gesta-
the synthesis of cortisol, in the zona fasciculata of the tion, and the placenta prevents most transfer of maternal
adrenal cortex; therefore, increased levels of 17-hydroxy- cortisol to the fetus. Corticosteroids administered to the
progesterone are used to make the diagnosis. In the pregnant woman can cross the placenta. In the placenta,
zona glomerulosa, 21-hydroxylase converts progester- the HSD11B2 gene encodes the 11β-hydroxysteroid
one to deoxycorticosterone in the synthesis of aldoste- dehydrogenase enzyme, which is responsible for the
rone. Deficient aldosterone synthesis is more deleterious inactivation of maternal cortisol. However, when not
to the newborn infant, which causes hyponatremia, inactivated by placental HSD11B2, dexamethasone can
hyperkalemia, acidosis, hypovolemia, and, if untreated, have effects on the female fetus at levels 60 times greater
death. than the usual fetal levels.
Deficient cortisol synthesis leads to increased produc- Prenatal treatment of CAH has different levels of suc-
tion of adrenocorticotropin and increased steroidogen- cess. When dexamethasone was given throughout the
esis, which, in the absence of 21-hydroxylation, results gestation in 49 pregnancies, of the 25 female fetuses who
in the accumulation of androgenic precursor steroids received dexamethasone before week 9 of pregnancy,
leading to virilization of the external genitalia in females. 11 had normal female genitalia, 11 had minimal viriliza-
The virilization may be sufficiently severe to result in tion (Prader stages 1–2), and 3 were virilized (Prader
errors in sex assignment in the delivery room, including stage 3). Among 24 female fetuses in whom treatment
defects in the urogenital sinus and scrotum, labial fusion, was begun after week 9, the genitalia averaged a Prader
66 PROLOG
score of 3.0; among untreated females, the score was knows she is pregnant. To start dexamethasone treatment
3.75. The success rate was approximately 80–85%, thus, at 12 weeks of gestation will be ineffective. Genetic diag-
even if treatment is initiated in a timely fashion, not all nosis by chorionic villus sampling or by amniocentesis
fetuses will escape the presence of genital anomalies. usually is not done until 10–12 weeks or after 14 weeks
Prenatal treatment of CAH is associated with other of gestation, respectively, and will lead to unnecessary
risks to the pregnant woman and her child, including exposure of the pregnancy.
greater pregnancy-associated weight gain, more striae Because the described patient will undergo IVF because
and edema, mild hypertension, and gestational diabetes of bilateral tubal obstruction, the best method to prevent
mellitus. Because of documented teratogenic effects other severely virilized female infants is to perform pre-
produced by high doses of dexamethasone administered implantation genetic diagnosis. This prenatal genetic
to pregnant animals and human patients (eg, cleft lip and testing of the parents will identify the specific mutations
palate), the U.S. Food and Drug Administration classi- carried by the mother and father. In carrying out IVF with
fies dexamethasone as a category C drug. Administered preimplantation genetic analysis of all embryos, the only
corticosteroids can promote differentiation of the fetal embryos to be transferred will be embryos not affected
lung and a single course of antenatal corticosteroids by the condition, or male or females who will be known
can improve pulmonary outcome in preterm infants, but to be carriers at birth but will not suffer from genital or
the use of multiple courses is associated with decreased metabolic anomalies.
weight, length, and head circumference at birth. Low
birth weight has been shown to be associated with Barker DJ, Eriksson JG, Forsen T, Osmond C. Fetal origins of adult
disease: strength of effects and biological basis. Int J Epidemiol 2002;
increased risks of adulthood hypertension, type 2 diabetes 31:1235–9.
mellitus, and cardiovascular disease. In addition, children Feldman-Witchel S, Miller WL. Prenatal treatment of congenital
who have received short-term prenatal dexamethasone adrenal hyperplasia—not standard of care. J Genet Counsel 2012;21:
had poor verbal working memory, poor self-perception 615–24.
of scholastic competence, and increased self-rated social Fernandez-Balsells MM, Muthusamy K, Smushkin G, Lampropulos
JF, Elamin MB, Elnour NOA, et al. Prenatal dexamethasone use for
anxiety. Some observers have characterized the prenatal the prevention of virilization in pregnancies at risk for classical con-
treatment of CAH as an unresolved ethical dilemma, eg, genital adrenal hyperplasia because of 21-hydroxylase (CYP21A2)
exposure to high-dose dexamethasone in seven of eight deficiency: a systematic review and meta-analyses. Clin Endocrinol
fetuses (four out of four males and three out of four 2010;73:436–44.
females) to treat one affected female. Speiser PW, Azziz R, Baskin LS, Ghizzoni L, Hensle TW, Merke DP,
et al. Congenital adrenal hyperplasia due to steroid 21-hydroxylase
Prenatal treatment to prevent CAH in patients at risk deficiency: an Endocrine Society clinical practice guideline. J Clin
of a virilized child must be started as soon as the woman Endocrinol Metab 2010;95:4133–60.
Reproductive Endocrinology and Infertility 67
46
Macroadenoma
A 23-year-old woman visits your clinic with a 6-month history of headaches, amenorrhea, and
bilateral galactorrhea. Laboratory results show a prolactin level of 325 ng/mL and a thyroid-
stimulating hormone (TSH) level of 1.25 mU/L. Cranial magnetic resonance imaging (MRI) shows
a 13-mm enclosed pituitary adenoma. Formal visual field testing is normal. Additional hormone
testing shows normal levels of insulin-like growth factor 1, free thyroxin, and morning and after-
noon cortisol. By contrast, follicle-stimulating hormone and luteinizing hormone levels are abnor-
mally low. The most appropriate management for this patient is
Macroadenomas are benign pituitary tumors greater than with sudden expansion. Predisposing factors include
10 mm in diameter. The morbidity from these tumors hypertension, sudden hypotension, closed head injury,
includes direct oversecretion of active anterior pituitary anticoagulation therapy, previous irradiation, treatment
hormones, impairment of hormone secretion by the nor- with a dopamine agonist, and dynamic pituitary function
mal pituitary tissue, and mass effect on the surrounding testing. Presenting symptoms include severe headache
central nervous system structures. Elevated circulat- with nausea and vomiting, visual changes, lethargy, and
ing levels of all anterior pituitary hormones have been possible altered consciousness. Unrecognized panhy-
reported with macroadenomas. Prolactin and growth hor- popituitarism with loss of adrenal function may result
mone elevations are the most commonly seen increases, in mortality. Because of the risk of pituitary apoplexy,
and many anterior pituitary tumors will cosecrete both patients with a macroadenoma should wear a medical
hormones. With prolactin-secreting macroadenomas, the alert device until treatment has resulted in microadenoma
prolactin level correlates with tumor size and inva- or tumor removal.
siveness. Clinically active adrenocorticotropic hormone, Central nervous system symptoms, primarily headache
TSH, and gonadotropin tumors are much less common. or abnormal anterior pituitary hormone findings, typically
Tumors that do not produce active hormones have been lead to the diagnosis of macroadenoma using MRI. The
referred to as nonsecreting, but most of these tumors finding of a macroadenoma requires complete evaluation
secrete glycoprotein chains, including free alpha subunit, of the anterior pituitary function. Baseline hormones
follicle-stimulating hormone beta subunit, or luteinizing should include prolactin, insulin-like growth factor 1 to
hormone beta subunit. Free alpha subunit may be assayed evaluate growth hormone function, morning and after-
and used as a tumor marker if present. Compression on noon cortisol, adrenocorticotropic hormone to evaluate
the normal pituitary, interference with blood supply, and the cortisol axis, TSH level, free thyroxin level for thy-
feedback from secreted hormones may impair normal roid function, and free alpha subunit as a tumor marker.
pituitary function. Mass effect on the optic chiasm may Abnormally low levels may require dynamic testing for
lead to peripheral vision loss. Bitemporal vision loss is the an absolute diagnosis of deficiency. Additionally, abnor-
most common vision loss but the degree is typically asym- mally high levels may require suppression testing for
metrical. Blurred vision is the most common symptom, verification. All patients with a macroadenoma should
but it is not unusual for patients not to realize that they have formal visual field testing by ophthalmology.
have vision loss. Macroadenomas also may invade the The treatment of macroadenomas depends on the
cavernous sinus (a collection of thin-walled veins lateral type of hormone secreted and the degree of mass effect.
to the sella turcica through which runs the internal carotid Although each case has to be individualized, prolactin-
artery and cranial nerves III, IV, V, and VI). Symptoms secreting macroadenomas (Fig. 46-1) are most com-
from involvement of the cavernous sinus are unusual but monly treated medically, whereas tumors that secrete
decrease the chance of complete surgical resection. other hormones or that are nonsecreting are treated surgi-
Pituitary apoplexy is the most serious complication cally. Prolactin-secreting macroadenomas may be treated
of a macroadenoma, although it is uncommon (approxi- initially with a dopamine agonist, such as bromocriptine
mately 2% of cases). It involves bleeding into the tumor or cabergoline. Cabergoline may be preferable because of
68 PROLOG
FIG. 46-1. Patient with amenorrhea/galactorrhea due to a prolactin-secreting macroadenoma. Despite compression of
the optic chiasm (small arrow), visual field testing was normal. Follow-up magnetic resonance imaging 3 months after
initiation of dopamine agonist therapy shows tumor shrinkage away from the chiasm. (Price TM, Bates GW. Adolescent
amenorrhea. In: Carpenter SEK, Rock JA, editors. Pediatric and adolescent gynecology. 2nd edition. Philadelphia:
Lippincott, Williams & Wilkins; 2000. p. 203.)
its greater potency and generally less pronounced adverse commonly precludes complete resection. Complications
effects of nausea and orthostatic hypotension. Response from transsphenoidal resection include vision loss (1.5%),
to a dopamine agonist tends to be rapid. Approximately infection (0.5%), vascular injury or stroke (0.6%), cere-
three quarters of women will return to normal prolactin brospinal fluid rhinorrhea (3.3%), and oculomotor nerve
levels within 3 months. A decrease in prolactin typi- palsy (0.6%). Persistent hyperprolactinemia after the
cally indicates a decrease in tumor size, but the degree resection of prolactin-secreting macroadenomas is seen
of decrease does not correlate with the magnitude of in most cases, requiring continued medical therapy.
tumor shrinkage. A repeat MRI examination is reason- Radiation therapy usually is reserved for tumors that
able as early as 3 months after initiation of therapy but cannot be controlled medically or surgically. Some
no later than 6 months. Either an increase in tumor size degree of hypopituitarism is commonly seen after radia-
or no change in tumor size by 6 months suggests the tion therapy.
need for surgery. Many adenomas express somatostatin Patients with a macroadenoma should avoid pregnancy
receptors. Macroadenomas that cosecrete prolactin and until the tumor has regressed in size to less than 10 mm
growth hormone may have a better response to a dopa- in diameter. With prolactin-secreting macroadenomas,
mine agonist and a somatostatin analog. Discontinuation approximately 39% will increase in size with pregnancy.
of treatment with a dopamine agonist also may result in Thus, continuation of dopamine agonist therapy is rea-
rebound growth of a macroadenoma, so patient education sonable in the event of pregnancy. A change in symptoms
is crucial. For the described patient, the most appropriate warrants reevaluation with MRI and pituitary function
management is treatment with a dopamine agonist. testing.
In macroadenomas that are not secreting prolactin,
transsphenoidal resection is usually the treatment of Ikeda H, Watanabe K, Tominaga T, Yoshimoto T. Transsphenoidal
microsurgical results of female patients with prolactinomas. Clin
choice. The success of complete resection correlates Neurol Neurosurg 2013;115:1621–5.
with the expertise of the neurosurgeon and with whether
Klibanski A, Zervas NT. Diagnosis and management of hormone-
the tumor remains within the sella turcica. Published secreting pituitary adenomas. N Engl J Med 1991;324:822–31.
success rates vary from 6% to 74% for initial complete Nawar RN, AbdelMannan D, Selman WR, Arafah BM. Pituitary tumor
removal. Expansion of the tumor into the cavernous sinus apoplexy: a review. J Intensive Care Med 2008;23:75–90.
Reproductive Endocrinology and Infertility 69
47
Fertility options after tubal ligation
A 28-year-old woman, gravida 3, para 3, is seeking to conceive after tubal sterilization. She under-
went laparoscopic tubal occlusion with the use of the titanium clip lined with silicone rubber 3 years
ago. She requests tubal reversal in order to conceive. The most important factor in the success of
tubal reanastomosis is
Tubal sterilization is a commonly used method of con- achieve pregnancy. Nonetheless, women between ages
traception. After tubal sterilization, patients sometimes 40 years and 45 years have a cumulative pregnancy rate of
desire restoration of their fertility. The most common rea- approximately 45–50%. The data show that tubal reanas-
son that a patient requests fertility restoration is a change tomosis remains a viable option for women after age
in her marital status. Some women also may encounter 40 years. In assessing what is the most important influ-
regret after tubal sterilization. Women who desire fertility ence on the success of tubal reanastomosis, the patient’s
restoration after tubal ligation have the option of in vitro age appears to play the most major role. Type of steriliza-
fertilization or microsurgical tubal reanastomosis in order tion (eg, tubal ligation versus titanium clip), location of
to conceive. After the introduction of microsurgical tech- the tubal occlusion (isthmic versus isthmic–ampullary),
niques, the successful outcomes of tubal reanastomosis and length of the tube affect pregnancy rates after tubal
procedures have improved. Pregnancy rate after a tubal reanastomosis. However, their role is not as significant as
reanastomosis is influenced by many factors, including the patient’s age at the time of tubal reanastomosis.
the patient’s age, her prior fertility, the type of steriliza-
tion procedure performed, and tubal length after repair of Gordts S, Campo R, Puttemans P, Gordts S. Clinical factors determining
pregnancy outcome after microsurgical tubal reanastomosis. Fertil Steril
the tubes as well as her partner’s semen parameters. These 2009;92:1198–202.
factors should all be considered in the preoperative evalu- Sreshthaputra O, Sreshthaputra RA, Vutyavanich T. Factors affecting
ation of a woman seeking tubal reanastomosis. pregnancy rates after microsurgical reversal of tubal sterilization. J
Cumulative pregnancy rates 12 months after tubal Reconstr Microsurg 2013;29:189–94.
reanastomosis range between 57% and 84%. However, Yoon TK, Sung HR, Kang HG, Cha SH, Lee CN, Cha KY. Lapar-
oscopic tubal anastomosis: fertility outcome in 202 cases. Fertil Steril
patients older than 36 years have been shown to have a 1999;72:1121–6.
lower pregnancy rate and to require a longer interval to
70 PROLOG
48
Androgen insensitivity syndrome
A 17-year-old adolescent comes to your office with primary amenorrhea. She has Tanner stage 4
breast development and a blind-ending vagina with no palpable uterus. Her pubic hair is Tanner
stage 1 and she has no axillary hair. The most likely diagnosis is
Androgen insensitivity syndrome (AIS) occurs in patients are slightly taller than their peers and have a short vaginal
with a normal male karyotype (46,XY) and is caused pouch, which usually can be lengthened either by the use
by an inactivating mutation in the gene that encodes of an intravaginal dilator or by intercourse. Some cases
the intracellular androgen receptor. Anecdotal reports might require surgical creation of a neovagina.
suggest that AIS has been recognized for a number of Key issues in this disorder include decisions on sexual
centuries. Joan of Arc and Queen Elizabeth I of England assignment, the timing of gonadectomy, and genetic and
are thought to have had this syndrome. psychologic counseling. Because the disorder is caused
Affected individuals have testes that make testosterone by an X-linked mutation, the patient may have female
and antimüllerian hormone, but the mutation makes the siblings with the same disorder. In terms of sexual assign-
androgen receptor insensitive to androgen action. As a ment, opinions diverge in regard to when and how a
result, male internal and external genitalia fail to undergo woman with androgen insensitivity should be told about
normal embryologic development. At the time of puberty, her diagnosis. Most experts recommend that the decision
estrogen, derived from peripheral conversion of high cir- to have such a conversation be individualized based on
culating testosterone levels, causes feminization because the age and maturity of the patient. It should be empha-
there is no androgen action to oppose it. In the complete sized that in complete AIS, the assignment is female.
form of AIS, the phenotype is female. Incomplete andro- Psychologic counseling should be recommended.
gen resistance is associated with a variety of clinical Gonads carry a risk of gonadoblastoma or invasive
presentations that usually involve sexual ambiguity at cancer. This risk appears to increase with age. More
birth. The biologic action of androgen is mediated by a recent data has suggested that the overall risk of malig-
single-gene–encoded intracellular androgen receptor on nancy may be lower than was previously believed.
the long arm of the X chromosome, Xq. Mutations in Because of the low incidence of tumor development
this gene result in varying degrees of androgen receptor before puberty, gonadectomy should be planned after
dysfunction; however, the phenotypes often show poor sexual maturation and breast development are complete.
concordance with the genotype. After surgical removal, estrogen therapy is recommended
The incidence of the syndrome may be as high as 1 in to maintain secondary sexual characteristics.
20,000 individuals. In one series of primary amenorrhea The differential diagnosis of AIS is relatively uncom-
cases, androgen insensitivity was the third most common plicated. The typical patient with AIS has breasts,
cause of primary amenorrhea after gonadal dysgenesis normal-to-above-average height, absent pubic and axil-
and müllerian dysgenesis. Diagnosis at the time of men- lary hair, a blind-ending vagina, and possibly an inguinal
arche is usually straightforward. The classic presentation hernia. The condition that is most often confused with
involves a phenotypic female who presents with primary AIS is müllerian dysgenesis (Table 48-1). Patients with
amenorrhea, normal breasts, little or no pubic or axillary müllerian dysgenesis usually exhibit normal axillary
hair, an absent uterus, a 46,XY karyotype, and serum and pubic hair, a short vagina and absent uterus, normal
testosterone levels in the normal male range. The inter- female serum testosterone levels, and a normal female
nal genitalia are characterized by the near absence of all karyotype (46,XX).
structures except testes. The testes may be located in the The described patient does not have Perrault syndrome
abdomen, inguinal canal, or labia majora and may present (46,XX; gonadal dysgenesis and sensorineural deafness)
as a hernia. The lack of androgen action results in scant or Swyer syndrome (46,XY; gonadal dysgenesis). In
or absent pubic and axillary hair. Most of these patients Perrault syndrome and Swyer syndrome, the gonads fail
Reproductive Endocrinology and Infertility 71
Androgen Insensitivity
Syndrome Müllerian Dysgenesis
Karyotype 46,XY 46,XX
Breasts ++ ++
Uterus – –
Pubic hair – +
Axillary hair – +
Testosterone levels Male Female
+, present; –, absent.
to develop and the patients do not undergo puberty or Hughes IA, Deeb A. Androgen resistance. Best Pract Res Clin
Endocrinol Metab 2006;20:577–98.
develop breasts because of the lack of estrogen action. In
Sultan C, Lumbroso S, Paris F, Jeandel C, Terouanne B, Belon C, et al.
addition, patients with either Perrault syndrome or Swyer Disorders of androgen action. Semin Reprod Med 2002;20:217–28.
syndrome would be expected to have a uterus because of Wisniewski AB, Migeon CJ, Meyer-Bahlburg HF, Gearhart JP,
the lack of production of antimüllerian hormone. Berkovitz GD, Brown TR, et al. Complete androgen insensitivity syn-
drome: long-term medical, surgical, and psychosexual outcome. J Clin
Cools M, Drop SL, Wolffenbuttel KP, Oosterhuis JW, Looijenga LH. Endocrinol Metab 2000;85:2664–9.
Germ cell tumors in the intersex gonad: old paths, new directions, mov-
ing frontiers. Endocr Rev 2006;27:468–84.
72 PROLOG
49
Depot medroxyprogesterone acetate and unscheduled vaginal bleeding
Erratic, “unscheduled” vaginal bleeding continues to be Long-acting progestin exposure alters endometrial vas-
the most common adverse effect associated with wide- cular development leading to irregularly distributed,
spread use of long-acting, progestin-only methods of superficially enlarged, fragile capillaries and venules
contraception. As a consequence, it is also the main embedded in a collapsed stromal extracellular matrix. On
reason for premature discontinuation of these methods. transvaginal ultrasonographic evaluation, the endometrial
Progestin contraceptives have the advantage of offering stripe would appear thin and homogeneous, typically
long-term contraception, which improves efficacy and less than 5 mm. It is unlikely that a patient who is taking
is highly acceptable to many women. During the first DMPA would develop an endometrial polyp, adenomyo-
months of use, episodes of unpredictable bleeding and sis, a myoma, or a thickened endometrium.
spotting lasting 7 days or longer are common. At 1 year, Mifepristone and other progesterone antagonists or
approximately 50% of users experience amenorrhea, receptor modulators may be effective treatments for
increasing to approximately 75% with long-term use. abnormal bleeding with progestin-only methods. In new
Research has increased knowledge about the multiple users of DMPA, mifepristone taken once every 2 weeks
mechanisms that contribute to the appearance of superfi- was found to reduce unscheduled bleeding. Some studies
cial, thin-walled fragile vessels within the endometrium have shown that estrogen supplementation may terminate
of many women with troublesome bleeding. Progestin a current bleeding episode but does not improve long-
exposure affects all cellular elements of the endome- term bleeding patterns.
trium, and it is likely that unscheduled bleeding arises
because of changes in the tightly regulated system of D’Arcangues C. Management of vaginal bleeding irregularities induced
by progestin-only contraceptives. Hum Reprod 2000;15(suppl 3):24–9.
vascular, stromal, and epithelial interaction seen during
Hickey M, Fraser IS. Iatrogenic unscheduled (breakthrough) endome-
the normal menstrual cycle. The exact mechanisms are trial bleeding. Rev Endocr Metab Disord 2012;13:301–8.
not fully understood, but superficial vascular fragility is Jain JK, Nicosia AF, Nucatola DL, Lu JJ, Kuo J, Felix JC. Mifepristone
a key feature. In addition, changes in endometrial ste- for the prevention of breakthrough bleeding in new starters of depo-
roid response, structural integrity, tissue perfusion, and medroxyprogesterone acetate. Steroids 2003;68:1115–9.
local angiogenesis all contribute to unscheduled bleeding.
Reproductive Endocrinology and Infertility 73
50
Alternative therapies for menopause
A 53-year-old woman underwent a mastectomy for invasive breast cancer without metastasis
2 years ago. She comes to your office to report that she is experiencing severe, frequent hot flushes.
She reports that she has more than 14 hot flushes per week and asks about non-estrogen-containing
therapy. The most efficacious therapy with the fewest adverse effects for reduction in the number
of hot flushes is
* (A) venlafaxine
(B) gabapentin
(C) fluoxetine
(D) clonidine
Vasomotor symptoms are reported to be more severe, of hot flushes. Although patients significantly preferred
distressing, and of greater duration in breast cancer sur- venlafaxine to gabapentin, both medications were asso-
vivors than in control women. This may be because of ciated with adverse effects. Treatment with venlafaxine
the patient’s age at diagnosis (frequently older than age was associated with greater nausea (P=.02), appetite loss
50 years), abrupt discontinuation of existing use of (P=.003), and constipation (P=.05) and fewer negative
hormone therapy, induction of premature menopause mood changes (P=.01) than gabapentin. Compared with
secondary to therapy (chemotherapy with or without venlafaxine, gabapentin was associated with more dizzi-
oophorectomy), or predisposition to estrogen deficiency ness (P=.005) and increased appetite (P<.001).
due to therapy with tamoxifen citrate or aromatase In two studies to compare the use of clonidine with
inhibitors. To prevent hormonal stimulation of potential placebo in breast cancer patients who took tamoxifen,
residual breast cancer cells, non-estrogen-containing neither study achieved statistical significance for the
therapy often is prescribed to alleviate the number of reduction of hot flush frequency or severity. The cloni-
hot flushes that occur in breast cancer survivors. Various dine group reported an increase in sleep problems com-
antidepressant medications from the selective serotonin pared with the placebo group. In breast cancer survivors,
reuptake inhibitor (SSRI) and selective noradrenergic the SSRI fluoxetine has been effective for vasomotor
reuptake inhibitor classes have been found to be effective symptoms, but the use of some SSRI and selective nor-
for the treatment of vasomotor symptoms. These medica- adrenergic reuptake inhibitor medications may inhibit
tions include the SSRI fluoxetine and the selective nor- the cytochrome P450 2D6 isoenzymes (CYP2DG) that
adrenergic reuptake inhibitor venlafaxine. Gabapentin, a are important in converting tamoxifen to its active
gamma-aminobutyric acid analog, also has been found to metabolite, endoxifen. The SSRIs fluoxetine, paroxetine,
be effective against hot flushes. Clonidine is a centrally and bupropion have been shown to adversely affect the
acting α2 adrenergic agonist used to treat hypertension. circulating endoxifen concentration level. The selective
In a multicenter, randomized, crossover trial of venla- noradrenergic reuptake inhibitor venlafaxine appears to
faxine versus gabapentin for the treatment of hot flushes have less effect on endoxifen levels and appears to be
in breast cancer survivors, both were found to be effective safer than the SSRI medications.
in reducing vasomotor symptoms (66% reduction), but The described patient should be counseled that the
patients significantly preferred venlafaxine (68%) over leading patient-preferred non-estrogen-containing ther-
gabapentin (32%), (P=.01). The study was an open-label, apy for the reduction in the number of hot flushes is
4-week treatment group-sequential trial of venlafaxine. A venlafaxine. In 2013, low-dose paroxetine was approved
total of 56 women used either venlafaxine or gabapentin by the U.S. Food and Drug Administration for the treat-
initially and then were switched to the other medication. ment of hot flushes. To date, few comparative studies of
Participants had baseline levels of at least 14 bother- the efficacy of low-dose paroxetine for the treatment of
some hot flushes per week in the month before study hot flushes have been reported.
enrollment. During the study, women kept a diary to
record the incidence, duration, and severity of their hot Bordeleau L, Pritchard KI, Loprinzi CL, Ennis M, Jugovic O, Warr D,
et al. Multicenter, randomized, cross-over clinical trial of venlafaxine
flushes. While the study was in progress, patients were versus gabapentin for the management of hot flashes in breast cancer
not allowed to use any other therapy for the alleviation survivors. J Clin Oncol 2010;28:5147–52.
74 PROLOG
Hall E, Frey BN, Soares CN. Non-hormonal treatment strategies for function during treatment of vasomotor symptoms associated with
vasomotor symptoms: a critical review. Drugs 2011;71:287–304. menopause. Menopause 2014 Feb 17. [Epub ahead of print]
Kelly CM, Juurlink DN, Gomes T, Duong-Hua M, Pritchard KI, Austin Simon JA, Portman DJ, Kaunitz AM, Mekonnen H, Kazempour
PC, et al. Selective serotonin reuptake inhibitors and breast cancer mor- K, Bhaskar S, et al. Low-dose paroxetine 7.5 mg for menopausal
tality in women receiving tamoxifen: a population based cohort study. vasomotor symptoms: two randomized controlled trials. Menopause
BMJ 2010;340:c693. 2013;20:1027–35.
Morrow PK, Mattair DN, Hortobagyi GN. Hot flashes: a review Tan O, Pinto A, Carr BR. Hormonal and non-hormonal management of
of pathophysiology and treatment modalities. Oncologist 2011;16: vasomotor symptoms: a narrated review. J Endocrinol Diabetes Obes
1658–64. 2013;1(2):1009 –24.
Portman DJ, Kaunitz AM, Kazempour K, Mekonnen H, Bhaskar S,
Lippman J. Effects of low-dose paroxetine 7.5 mg on weight and sexual
51
Obesity and pregnancy
A 27-year-old, gravida 2, para 0, comes to your office to discuss her recent pregnancy losses.
Within the past year, she has had two first-trimester pregnancy losses (under 10 weeks) while tak-
ing clomiphene citrate. She has a body mass index (BMI) of 36 (calculated as weight in kilograms
divided by height in meters squared) and has irregular menstrual cycles. Her thyroid-stimulating
hormone and prolactin levels are normal, as is transvaginal ultrasonography. She shows no evi-
dence of diabetes mellitus on a 2-hour glucose tolerance test. She has no clinical signs of hyperan-
drogenemia. In addition to initiating a recurrent pregnancy loss evaluation, the best next step is to
Increasing data indicate that obesity may increase the The corpus luteum provides progesterone, which
risk of miscarriage. A greater risk of miscarriage occurs induces a secretory maturation of the endometrium in
in obese women after natural conception, ovulation preparation for implantation. A normal luteal phase is
induction, and assisted reproduction. The mechanism for 14 days, whereas a short luteal phase is less than 10 days.
early miscarriage in the obese patient has not been fully Abnormal thyroid-stimulating hormone and prolactin
elucidated. Obesity in pregnancy is a risk for the woman levels can be associated with a short luteal phase. In this
and her fetus. Maternal complications include mortality, patient with oligo-ovulation, ovulation induction with
gestational diabetes mellitus, preeclampsia, hypertension, clomiphene will correct a luteal phase deficiency by
cesarean delivery, thromboembolism, postpartum hemor- increasing follicular phase follicle-stimulating hormone
rhage, and anesthesia complications. Fetal complications levels, which in turn causes multiple ovulation. Increased
include miscarriage, stillbirth, congenital malformations, pituitary gonadotropin release induces follicular develop-
and fetal growth abnormalities. These complications may ment and produces sufficient progesterone levels from the
be preventable with preconception lifestyle modifications. corpus luteum for adequate endometrial maturation.
Many reproductive centers around the world are imple- Ordering a thrombophilia panel is not indicated for
menting BMI guidelines that postpone care until a desired the described patient. She had two miscarriages before
BMI is met. A team approach (nutritionist, nurse, physi- 10 weeks of gestation, which is before placental function
cian) is needed to help support and motivate these patients is present. The vast majority of first-trimester losses are
to lose weight and become healthier. Often the goal of due to chromosomal anomalies. A thrombophilia panel
family building can motivate an obese patient to adopt a is indicated when losses occur after 10 weeks, when
healthier lifestyle. With weight loss, menstrual irregulari- thrombosis can occur in the vasculature of the placenta.
ties often revert to regular ovulatory cycles. In addition Metformin hydrochloride is an insulin sensitizer used
to initiating a recurrent pregnancy loss evaluation, this in the treatment of impaired glucose tolerance and
patient should join a nutritional weight-loss program. type 2 diabetes mellitus. Obese women with or without
Reproductive Endocrinology and Infertility 75
polycystic ovary syndrome (PCOS) are often insulin Pregnancy is a hypercoagulable state. Aspirin, an
resistant, resulting in an increased risk of developing anticoagulant, has been used for women with recur-
impaired glucose tolerance and type 2 diabetes mel- rent pregnancy loss to try to increase the subsequent
litus. When PCOS is suspected (menstrual irregulari- live-birth rate in women with inherited thrombophilias,
ties, hyperandrogenism, polycystic-appearing ovaries), antiphospholipid syndrome, and unexplained recurrent
a 2-hour oral glucose tolerance test is recommended. pregnancy loss. There are no data to suggest that empiric
Although a hemoglobin A1c test may be a potential sub- aspirin therapy decreases pregnancy loss associated with
stitute for the 2-hour glucose test, to date, the efficacy of obesity. Therefore, empiric aspirin use in this patient is
the hemoglobin A1c test in patients with PCOS has not not indicated.
been completely proven.
Metformin is indicated if impaired glucose tolerance de Jong PG, Goddijn M, Middeldorp S. Testing for inherited throm-
bophilia in recurrent miscarriage. Semin Reprod Med 2011;29:540–7.
or type 2 diabetes mellitus is diagnosed. Metformin can
Kaandorp S, Di Nisio M, Goddijn M, Middeldorp S. Aspirin or antico-
decrease the progression of impaired glucose tolerance agulants for treating recurrent miscarriage in women without antiphos-
to diabetes mellitus. This patient has irregular menstrual pholipid syndrome. Cochrane Database of Systematic Reviews 2009,
cycles but is not clinically hyperandrogenic and does not Issue 1. Art. No.: CD004734. DOI: 10.1002/14651858.CD004734.pub3.
have polycystic-appearing ovaries. The next step in her Obesity in pregnancy. Committee Opinion No. 549. American
College of Obstetricians and Gynecologists. Obstet Gynecol 2013;
management would not be to initiate metformin unless 121:213–7.
she has impaired glucose tolerance or type 2 diabetes
mellitus.
52
Bulimia nervosa and binge-eating disorder
A 24-year-old woman comes to your office for her annual well-woman examination. When review-
ing her medical history, she tells you that she has experienced several months of bulimia nervosa
for which she has not sought treatment. You discuss nutritional counseling. She has a body mass
index of 22 (calculated as weight in kilograms divided by height in meters squared) and her physi-
cal examination is normal. The first-line treatment that you recommend is
Bulimia nervosa is characterized by recurrent episodes anorexia nervosa and is not associated with the recurrent
of eating unusually large amounts of food accompanied use of inappropriate compensatory behavior as observed
by feelings of loss of control and recurrent inappropri- with bulimia nervosa. Binge-eating disorder may occur in
ate compensatory behaviors in order to prevent weight patients with normal weight but often occurs in conjunc-
gain. With bulimia nervosa, binge eating and inappropri- tion with obesity.
ate compensatory behaviors occur on average at least Bulimia nervosa and binge-eating disorder are associ-
once a week for 3 months. The disturbance does not ated with a number of medical complications. Patients
occur exclusively during episodes of anorexia nervosa. often hide their eating disorder from their physician and
Compensatory behaviors to prevent weight gain include may present with other nonspecific complaints, such as
self-induced vomiting; misuse of laxatives, diuretics, and fatigue, menstrual irregularity, abdominal pain and bloat-
enemas; fasting; or excessive exercise. The individual’s ing, or constipation. Physical examination findings that
self-esteem is typically strongly influenced by body shape may suggest underlying bulimia nervosa include tachy-
and weight. cardia, parotid gland swelling, hypotension, dry skin,
By contrast, with binge-eating disorder, the binge eat- and dental enamel erosion. Associated medical complica-
ing does not occur during the course of bulimia nervosa or tions may involve multiple organ systems, specifically
76 PROLOG
gastrointestinal, cardiovascular, renal, musculoskeletal, recovery. The remaining patients exhibit responses that
and endocrine disorders. Severe complications, such as range from greatly improved to no improvement. For
electrolyte abnormalities, esophageal rupture, or cardiac patients with bulimia nervosa who are overweight or
arrhythmias, are rare but can be life threatening. obese, cognitive–behavioral therapy is likely to help
Several approaches have been proposed in regard to reduce binge eating and purging but may not result in
the long-term outpatient treatment of eating disorders. weight loss. Patients with personality disorders, par-
Nutritional counseling and rehabilitation are important ticularly borderline personality disorder, may be less
but are unlikely to be effective alone. The goal of nutri- responsive to cognitive–behavioral therapy or to therapy
tional rehabilitation is to restore a consistent eating pat- in general.
tern that typically consists of three meals and two snacks When cognitive–behavioral therapy is not sufficient,
a day. A Cochrane review including more than 100 trials antidepressant drugs, particularly selective serotonin
supports the efficacy of cognitive–behavioral therapy as a reuptake inhibitors, are effective in treating bulimia ner-
first-line therapy in the treatment of individuals with buli- vosa. Higher doses than those used to treat major depres-
mia nervosa. The therapy focuses on modifications of the sion are recommended. These medications typically
specific behaviors and ways of thinking that contribute result in a dramatic reduction in the frequency of binge
to the patient’s eating disorder. The cognitive aspect of eating and purging as well as an improvement in mood.
therapy focuses on modifying the dysfunctional thoughts Bupropion hydrochloride is contraindicated in the treat-
and attitudes associated with bulimia nervosa. The behav- ment of bulimia nervosa because there is an increased
ioral component strives to improve the patient’s behav- seizure risk in patients who are actively purging.
ioral responses to their dysfunctional thoughts. Typically,
approximately 20 individual sessions over 4–5 months American Psychiatric Association. Diagnostic and statistical manual
of mental disorders: DSM-5. 5th ed. Arlington (VA): American
are necessary. Psychiatric Association; 2013.
Cognitive–behavioral therapy has demonstrated low Hay PPJ, Bacaltchuk J, Stefano S, Kashyap P. Psychological treatments
relapse rates and appears to be superior to other forms of for bulimia nervosa and binging. Cochrane Database of Systematic
psychotherapy. Remission rates with cognitive–behav- Reviews 2009, Issue 4. Art. No.: CD000562. DOI: 10.1002/14651858.
CD000562.pub3.
ioral therapy are significantly greater than with inter-
Mehler PS. Clinical practice. Bulimia nervosa. N Engl J Med 2003;
personal psychotherapy, behavioral therapy, hypnosis 349:875–81.
plus behavioral therapy, or supportive psychotherapy. Treasure J, Claudino AM, Zucker N. Eating disorders. Lancet 2010;
One third to one half of patients who undergo cognitive– 375:583–93.
behavioral therapy will make a complete and lasting
Reproductive Endocrinology and Infertility 77
53
Postpartum thyroiditis
A 24-year-old woman, gravida 1, para 1, comes to your clinic for a follow-up visit at 6 months
postpartum. She has nothing to report, but her thyroid is noted to be symmetrically enlarged with-
out nodules. Her pulse is 90 beats per minute, her thyroid-stimulating hormone (TSH) level is low,
and her free thyroxine level is 2.4 ng/dL. The best treatment option is
Thyroid disease is a common clinical endocrine disorder thyroiditis. Similarly, the prevalence of postpartum thy-
in women of reproductive age. Hypothyroidism and roiditis also is increased in women with Graves disease
hyperthyroidism can occur during pregnancy and are in remission and in women with chronic viral hepati-
associated with maternal and fetal morbidity and mortal- tis. Consequently, the TSH level should be checked at
ity if undiagnosed or untreated. 3 months and 6 months postpartum in patients with any
Postpartum thyroiditis is defined as a transient auto- of these conditions.
immune thyroid disorder that may occur in 5–10% of Transient hyperthyroidism results from excessive
all women during the first year after childbirth. It occurs release of thyroid hormone from the glandular disrup-
as thyrotoxicosis, hypothyroidism, or thyrotoxicosis fol- tion caused by the antibodies. Symptoms may include
lowed by hypothyroidism in the first year postpartum in a painless but enlarged goiter, palpitations, or fatigue.
women who demonstrated no clinical evidence of thyroid Approximately 30% of patients with postpartum thyroid-
disease before pregnancy. It appears that an autoimmune- itis are asymptomatic during the thyrotoxic phase. This
induced release of preformed hormone occurs from the phase usually occurs between 1 month and 6 months
thyroid. Postpartum thyroiditis almost exclusively affects postpartum and generally resolves after 1–2 months. The
thyroid antibody-positive women. Among women who hypothyroid phase usually occurs between 3 months and
are euthyroid in the first trimester but who test posi- 8 months postpartum and typically lasts 4–6 months.
tive for thyroid autoantibodies, approximately 50% will The best treatment option for the described patient is
develop postpartum thyroiditis. Consequently, women observation. Antithyroid drugs would not be indicated or
who are known to be positive for thyroid peroxidase anti- required. Patients that develop significant hypothyroid
bodies should have their TSH level measured at 6 months symptoms during the hypothyroid transient period may
postpartum or sooner if clinically indicated. Postpartum benefit from low-dose levothyroxine. Asymptomatic
thyroiditis has an approximately 70% risk of recurrence women with postpartum thyroiditis who have an elevated
and also can occur after a pregnancy loss. TSH level that is less than 10 mIU/L and who are not
Postpartum thyroiditis may be related to a change in immediately planning another pregnancy do not neces-
the immune system in postpartum women. In general, sarily require intervention but should be remonitored in
the syndrome initially presents as transient hyperthyroid- 4–8 weeks. If the patient’s TSH remains elevated, treat-
ism within 13 weeks after delivery followed by transient ment should be initiated with levothyroxine. Because
hypothyroidism. This can be associated with thyroid the disorder may last up to 1 year, levothyroxine can be
enlargement. If the thyroid is biopsied, a lymphocytic continued and then tapered off. Patients may redevelop
infiltration may be observed. Some of the symptoms that hypothyroidism later in life and yearly assessment of the
may occur after delivery are vague and nonspecific, such TSH level is recommended. Beta blockers, propylthio-
as depression and memory impairment. Such symptoms uracil, radioactive iodine, and methimazole would be of
can make it difficult to differentiate postpartum thyroid- use only if the patient exhibits signs and symptoms of
itis from postpartum depression or the postpartum blues. overt hyperthyroidism.
Primary risk factors other than thyroid antibodies include
a history of thyroid dysfunction or a family history of De Groot L, Abalovich M, Alexander EK, Amino N, Barbour L, Cobin
RH, et al. Management of thyroid dysfunction during pregnancy and
thyroid disease or autoimmune disease. Up to 25% of postpartum: an Endocrine Society clinical practice guideline. J Clin
women with type 1 diabetes mellitus develop postpartum Endocrinol Metab 2012;97:2543–65.
78 PROLOG
Lucas A, Pizarro E, Granada ML, Salinas I, Foz M, Sanmarti A. Thyroid disease in pregnancy. ACOG Practice Bulletin No. 37.
Postpartum thyroiditis: epidemiology and clinical evolution in a nonse- American College of Obstetricians and Gynecologists. Obstet Gynecol
lected population. Thyroid 2000;10:71–7. 2002;100:387–96.
Stagnaro-Green A, Pearce E. Thyroid disorders in pregnancy. Nat Rev
Endocrinol 2012;8:650–8.
54
Anorexia and nutrition
A 28-year-old nulligravid woman with oligo-ovulation comes to your office with a 2-year history of
primary infertility. Her body mass index is 19 (calculated as weight in kilograms divided by height
in meters squared). Hysterosalpingography, thyroid-stimulating hormone level, and partner semen
analysis are normal. She is a compulsive exerciser. Her physical examination reveals dry skin with
normal blood pressure. The most appropriate next step in the management of this patient is
Eating disorders affect approximately 5–7% of women of questions (Box 54-1). A score of 2 or more indicates
childbearing age. The three categories of eating disorders a likely case of anorexia nervosa or bulimia nervosa.
are 1) anorexia nervosa, 2) bulimia nervosa, and 3) eating This patient has initiated an evaluation, and the goal of
disorders not otherwise specified. The vast majority of motherhood is often a motivating force for patients to
patients with eating disorders do not disclose their disor- initiate treatment of these disorders. Once she is in remis-
der to their physician. One study noted that 58% of infer- sion, her cycles may normalize as her hypothalamic–
tility patients with menstrual irregularities had clinical pituitary–ovarian axis is released from inhibition. She
indicators for eating disorders, but none of these patients can use timed intercourse or an ovulation predictor kit to
disclosed them to their health care provider. help her become pregnant. Attempting pregnancy before
Anorexic women have a distorted body image and remission increases the risk of maternal malnutrition and
refuse to maintain a normal body weight for their height.
They have a fear of gaining weight even though they are
underweight and are often amenorrheic. Anorexic patients BOX 54-1
binge and purge or restrict their food intake. Commonly,
anorexic patients use compulsive exercise to lose weight The SCOFF Questions*
or to maintain body weight. The common thread among • Do you make yourself Sick because you feel
all eating disorders is the continual drive to be thin. uncomfortably full?
Treatment of anorexia hinges on a team that includes a • Do you worry you have lost Control over how
nutritionist, a mental health professional, and a clinical much you eat?
coordinator. Weight gain with a decrease in exercise is • Have you recently lost Over 14 lb (6.4 kg) in a
3-month period?
the ultimate goal in this patient. The goal can be accom- • Do you believe yourself to be Fat when others
plished with the use of a team approach. say you are too thin?
Eating disorders often manifest as secondary amen- • Would you say that Food dominates your life?
orrhea. This may be the clinician’s first clue to a
*One point for every “yes”; a score of 2 or more indicates
possible eating problem. The described woman is oli-
a likely case of anorexia nervosa or bulimia.
gomenorrheic and is thin with a body mass index of 19.
Morgan JF, Reid F, Lacey JH. The SCOFF questionnaire:
In addition, she is a self-proclaimed intense exerciser. assessment of a new screening tool for eating disorders.
Because her cycles are irregular, she is having diffi- BMJ 1999;319:1467–8. Copyright 1999 BMJ Publishing
culty timing intercourse and becoming pregnant. The Group Ltd., reproduced with permission.
SCOFF questionnaire is a simple set of five yes or no
Reproductive Endocrinology and Infertility 79
giving birth to a newborn who is small for gestational The patient with oligo-ovulation needs to be screened for
age. thyroid disease. Checking a thyroid-stimulating hormone
Clomiphene citrate should not be used as a first-line level in this patient is part of her evaluation for her eat-
agent in an anovulatory patient with an eating disorder. ing disorder and oligo-ovulation. Thyroid disease needs
Efforts should be made to correct the patient’s eating to be corrected before attempting pregnancy because
disorder before ovulation is initiated. Without weight hypothyroidism and hyperthyroidism can affect maternal
gain, clomiphene is only occasionally effective. Anorexic and fetal health.
patients often have hypothalamic amenorrhea and should All women of childbearing age should take preconcep-
be treated accordingly. tional folic acid. In addition to a balanced diet, women
A trial of antidepressant medication, such as a selec- need extra iron and folic acid during pregnancy. To reduce
tive serotonin reuptake inhibitor, is reserved for the the risk of neural tube defects, a minimum of 0.4 mg
patient who, in addition to anorexia, has a psychologic of folic acid should be taken daily before pregnancy and
comorbidity. The anorexic patient should be screened then through the first 12 weeks of gestation.
for depression and anxiety. If depression or anxiety is
detected, the problem should be treated in conjunction Andersen AE, Ryan GL. Eating disorders in the obstetric and gyneco-
logic patient population. Obstet Gynecol 2009;114:1353–67.
with the eating disorder. The risks and benefits of psychi-
Fritz MA, Speroff L. Amenorrhea. In: Fritz MA, Speroff L, editors.
atric medications in pregnancy need to be discussed with Clinical gynecologic and infertility. 8th ed. Philadelphia (PA):
the patient because there may be fetal risks. Selective Lippincott Williams & Wilkins; 2011. p. 486–9.
serotonin reuptake inhibitors are part of the management Hoffman ER, Zerwas SC, Bulik CM. Reproductive issues in anorexia
for patients with bulimia nervosa but not for patients with nervosa. Expert Rev Obstet Gynecol 2011;6:403–14.
anorexia.
80 PROLOG
55
Contraception for an older patient who smokes
A 37-year-old patient continues to smoke cigarettes despite your advice to quit. You counsel her
that the contraceptive method that presents the highest risk of morbidity and mortality for her is
Thromboembolic events are classified into two types: In OC users who smoke, the increased thrombotic
1) venous thromboembolism and 2) arterial thrombosis. risk exists because of a nicotine-mediated decrease in
Venous thromboembolism is characterized by a low-flow circulating prostacyclin and an increase in thromboxane
circulatory system, a high circulating fibrinogen level, levels, leading to increased platelet aggregation. In OC
and a low platelet count. Clinically, venous thromboem- users, procoagulatory increases in fibrinogen and fibrino-
bolism manifests as either a deep vein thrombosis or a peptide A levels are balanced by a compensatory increase
pulmonary embolism. Arterial thrombosis is character- in antithrombin III levels. However, in an OC user who
ized by a high-flow circulatory system, a low circulating smokes cigarettes, there is no corresponding increase in
fibrinogen level, and a high platelet count. Clinically, antithrombin III levels, which predisposes this patient to
arterial thrombosis manifests as either a myocardial an increased hypercoagulable state.
infarction (MI) or a stroke. When examining the risk of arterial thrombosis,
Because of their pharmacologic levels of estrogen, women who are older than 35 years and smoke cigarettes
estrogen-containing contraceptive methods (pill, patch, or should not use combination OCs. In a 15-year Danish
ring) increase the production of clotting factors, which in historical cohort study, the absolute risk of thrombotic
turn increases the risk of thrombotic events. The route of stroke and MI was increased by a factor of 0.9–1.7 with
estrogen administration does not affect this increased risk 20 micrograms of ethinyl estradiol OCs and by a factor
of thrombosis. When compared with pregnancy, the use of of 1.3–2.3 with 30–40 micrograms of ethinyl estradiol
an estrogen-containing contraceptive method is less likely OCs, with relatively small differences in risk according
to lead to a nonfatal venous thromboembolism event. to the type of associated progestin used. In contrast, none
Women who have risk factors for hypercoagulability of the women who used progestin-only contraceptive
or who are 35 years or older and smoke cigarettes are at methods (progestin-only OCs, progestin-containing IUD,
increased risk of thromboembolic events and should not or progestin implant) had a significant increased risk of a
use estrogen-containing contraceptive methods. The type thrombotic stroke or MI.
of progestin also can influence venous thromboembolism Women older than 35 years who smoke should be
risk. In a recent case–control study of 186 newly diag- counseled to use a non-estrogen-containing contracep-
nosed idiopathic cases compared with 681 controls, the tive because of the increased risk of a thromboembolic
incidence rates for venous thromboembolism were 30.8 event with the use of estrogen-containing contraceptive
(95% confidence interval [CI], 25.6–36.8) per 100,000 methods. Acceptable contraceptive methods include the
woman-years among users of OCs containing drospire- progestin-only OC, the depot medroxyprogesterone ace-
none and 12.5 (95% CI, 9.61–15.9) per 100,000 woman- tate injection, the progestin implant, and either the
years among users of OCs containing levonorgestrel. Use progestin-containing IUD or the copper-containing IUD.
of OCs increases risk of venous thromboembolism over
baseline; the age-adjusted incidence ratio for a venous Jick SS, Hernandez RK. Risk of non-fatal venous thromboembolism
in women using oral contraceptives containing drospirenone compared
thromboembolism event with a drospirenone-containing with women using oral contraceptives containing levonorgestrel: case-
OC compared with a levonorgestrel-containing OC is control study using United States claims data. BMJ 2011;342:d2151.
2.8 (95% CI, 2.1–3.8). Prior studies involving the con- Lidegaard O, Lokkegaard E, Jensen A, Skovlund CW, Keiding N.
traceptive patch also have shown a twofold to threefold Thrombotic stroke and myocardial infarction with hormonal contracep-
tion. N Engl J Med 2012;336:2257–66.
increased risk of venous thromboembolism compared
with users of levonorgestrel-containing contraceptives.
Reproductive Endocrinology and Infertility 81
56
Leiomyomas and heavy menstrual bleeding
A 30-year-old nulligravid woman is trying to conceive. She has a history of heavy menstrual bleed-
ing. Pelvic ultrasonography and endometrial biopsy fail to reveal any anatomic abnormality. The
medical treatment most likely to improve her heavy menstrual bleeding is
The U.S. Food and Drug Administration recently with the induction of labor. Misoprostol is not used for
approved an oral formulation of tranexamic acid for the treatment of AUB.
clinical use in the United States. The pharmacologic Norethindrone is a synthetic progestin used in com-
mechanism of tranexamic acid involves reversibly block- bination oral contraceptives and for the induction of
ing lysine-binding sites on plasminogen, which in turn menses. Although it can be used to control anovulatory
stops the interaction of plasmin and fibrin polymer. This amenorrhea, for the prevention of endometrial hyper-
prevents fibrin degradation and leads to stabilization of plasia, and for cycle control, it is not as effective as
clots and reduced bleeding. Tranexamic acid has been tranexamic acid for the management of AUB.
used extensively to reduce blood loss and the need for Mefenamic acid is a NSAID that is used to treat
blood transfusion during and after surgical procedures, pain, including dysmenorrhea. It decreases uterine
such as coronary artery bypass, scoliosis surgery, and contractions by an unknown mechanism. It has been
knee arthroplasty. Tranexamic acid has been used for the shown that NSAIDs are effective in reducing blood loss
treatment of abnormal uterine bleeding (AUB) outside of compared with placebo in cases of heavy menstrual
the United States for decades. Tranexamic acid is supe- bleeding. A recent meta-analysis, however, has shown
rior to placebo, diclofenac, mefenamic acid, ethamsylate, such drugs to be less effective than tranexamic acid.
and luteal phase norethisterone for reduction of blood Medroxyprogesterone acetate is also less effective than
loss in idiopathic AUB and reduction of blood loss asso- tranexamic acid.
ciated with intrauterine devices. A recent study indicates
that tranexamic acid appears to reduce heavy bleeding, Lethaby A, Duckitt K, Farquhar C. Non-steroidal anti-inflammatory
drugs for heavy menstrual bleeding. Cochrane Database of Systematic
including bleeding caused by leiomyomas.
Reviews 2013, Issue 1. Art. No.: CD000400. DOI: 10.1002/14651858.
Use of a GnRH analog would place the patient in a CD000400.pub3.
state of amenorrhea and improve her AUB but would not Lukes AS, Moore KA, Muse KN, Gersten JK, Hecht BR, Edlund M, et
be advisable given the adverse effects and cost. Therapy al. Tranexamic acid treatment for heavy menstrual bleeding: a random-
with a GnRH analog also would be a temporary measure; ized controlled trial. Obstet Gynecol 2010;116:865–75.
it cannot be given for extended periods of time without Munro MG. Uterine leiomyomas, current concepts: pathogenesis,
impact on reproductive health, and medical, procedural, and surgical
estrogen add-back therapy. management. Obstet Gynecol Clin North Am 2011;38:703–31.
Misoprostol is a synthetic prostaglandin E1 analog that
Naoulou B, Tsai MC. Efficacy of tranexamic acid in the treatment of
is used for the prevention of nonsteroidal antiinflamma- idiopathic and non-functional heavy menstrual bleeding: a systematic
tory drug (NSAID)-induced gastric ulcers and to help review. Acta Obstet Gynecol Scand 2012;91:529–37.
82 PROLOG
57
Absolute risk versus relative risk
A 23-year-old healthy nonsmoking nulligravid woman has been taking combined oral contracep-
tives (OCs) for the past 2 years with no problems. However, she became concerned after her friend
informed her that individuals who use OCs have a risk of venous thromboembolism twice as high
as those who do not use OCs. She has no personal or family history of venous thromboembolism.
You inform her that her risk of venous thromboembolism is higher while she is taking OCs com-
pared with women who do not use OCs, although her actual risk while she is taking OCs is only
0.0015% per year. This concept is described as
Women who use estrogen-containing OCs have increased The terms positive predictive value (PPV) and nega-
risks of venous thromboembolism compared with non- tive predictive value (NPV) are used when evaluating
users. Although the risk of venous thromboembolism how well a certain test identifies patients who do and do
is lower with currently available OCs, which contain not have a disease. Specifically, PPV refers to the likeli-
a much lower concentration of estrogen, the risk is not hood that a person who has a positive test actually has the
eliminated. Certain factors increase the risk. For example, disease in question and NPV refers to the likelihood that
obese women, older women, and women who smoke a person who has a negative test actually does not have
have a significantly increased risk of venous thromboem- the disease. The predictive values (negative and positive)
bolism if they use OCs. Progestin-only pills appear not to depend on the prevalence of the disease in the population
increase the risk of venous thromboembolism. in question. However, PPV and NPV are terms reserved
It is important for health care providers to have mean-
ingful conversations with patients to review the risks and
TABLE 57-1. Relative Risk and Actual Incidence of
benefits of OC use. Absolute risk is a number or rate,
Venous Thromboembolism
whereas relative risk (also known as risk ratio) is a ratio.
Relative risk describes the risk compared with a standard. Incidence
Commonly, cohort studies will present data demonstrat- (per 10,000
ing the risk of disease in exposed individuals versus Population Relative Risk Women per Year)
unexposed individuals. The risk for unexposed persons is Young women, 1 5–10
generally set as equal to one and the experimental group general population
risk is expressed as a fraction below or above that value. Pregnant women 12 60–120
Absolute risk or incidence is the number of cases of High-dose OCs 6–10 30–100
disease per population size per unit of time. In this case,
Low-dose OCs 2 10–20
the relative risk of venous thromboembolism in young
Factor V Leiden 6–8 30–80
women who use low-dose OCs is 2.0 compared with the mutation carrier
risk in young women who do not use OCs. However, the
Factor V Leiden 10–15 50–100
absolute risk of venous thromboembolism for nonusers mutation carrier
and users is low: 5–10 per 10,000 women per year versus and OCs
10–20 per 10,000 women per year, respectively. Because Leiden mutation, 80 400–800
venous thromboembolism is a comparatively rare event homozygous
in young women, discussions with such patients about Abbreviation: OCs, oral contraceptives.
OC use should include discussion of the absolute and rel- Speroff L, Fritz MA. Oral contraception. In: Speroff L, Fritz MA,
ative risks of venous thromboembolism while taking OCs editors. Clinical gynecologic endocrinology and infertility. 8th ed.
to permit them to make educated decisions (Table 57-1). Philadelphia (PA): Lippincott Williams and Wilkins; 2011. p. 981.
Reproductive Endocrinology and Infertility 83
for evaluation of a test’s utility and are not applicable in error) refers to the probability of incorrectly concluding
the described case. that a statistically significant difference exists between
Similarly, the terms sensitivity and specificity refer the groups, and type 2 error refers to the probability
to measures of the performance of a diagnostic test. of incorrectly assuming that there was no difference
Sensitivity is the number of patients with a positive test between groups in a study. These terms are reserved for
who have a disease divided by all the patients who have considerations about the validity of hypothesis testing
the disease. Specificity is the number of patients who and conclusions when interpreting study results.
have a negative test and who do not have the disease
divided by the total number of patients without the dis- Heit JA, O’Fallon WM, Petterson TM, Lohse CM, Silverstein MD, Mohr
DN, et al. Relative impact of risk factors for deep vein thrombosis and pul-
ease. The interdependence of sensitivity and specificity monary embolism: a population-based study. Arch Intern Med 2002;162:
often are shown using a receiver operating characteristic 1245–8.
curve. The concepts of sensitivity and specificity are Lidegaard O, Nielsen LH, Skovlund CW, Skjeldestad FE, Lokkegaard
related to test properties and are not related to risk of E. Risk of venous thromboembolism from use of oral contraceptives
disease from a specific exposure. containing different progestogens and oestrogen doses: Danish cohort
study, 2001–9. BMJ 2011;343:d6423.
Type 1 error is a term used when making assumptions
Speroff L, Fritz MA. Oral contraception. In: Speroff L, Fritz MA,
about the validity of data. There is a presumed truth about editors. Clinical gynecologic endocrinology and infertility. 8th ed.
a situation (such as the true relation between OC use Philadelphia (PA): Lippincott Williams and Wilkins; 2011. p. 980–1.
and venous thromboembolism), and authors of studies White RH. The epidemiology of venous thromboembolism. Circulation
attempt to gather data that will approximate this truth. 2003;107:I-4–I-8.
When testing a hypothesis, type 1 error (also called alpha
58
Depot medroxyprogesterone acetate and bone loss
A 36-year-old nulligravid woman with a body mass index of 26.5 (calculated as weight in kilograms
divided by height in meters squared) comes to your office for her annual well-woman examination.
She is sexually active and has been adherent to her depot medroxyprogesterone acetate (DMPA)
regimen for the past 2 years. She smokes one pack of cigarettes a day. She is concerned about
reports that point to the danger of low bone mineral density (BMD) in women who take DMPA.
You counsel her that the best strategy to reduce her risk of low BMD is
Cigarette smoking is a risk factor for BMD loss, but the compared with nonsmokers. For the described patient,
mechanisms are not completely understood. Less efficient smoking cessation at her age will limit recidivism and
calcium absorption in smokers compared with nonsmok- help counteract the increased risk of BMD changes typi-
ers has been suggested as a possible cause, as have local cally seen after menopause.
and systemic toxic effects on bone collagen synthesis and In recent years, observations of reduced BMD in
alterations in metabolism of adrenal, cortical, and gonadal current DMPA users have led to concerns that DMPA-
hormones. Longitudinal studies indicate that rates of induced bone loss might lead to osteopenia and increase
bone loss are approximately 1.5–2 times greater among the long-term risk of fractures, particularly in young
current smokers compared with individuals who do not women who have not yet attained their peak bone mass
smoke. Rates of bone loss, particularly in postmenopausal and among perimenopausal women who may be start-
smokers, are higher when observed over the same period ing to lose bone mass. The effect of DMPA on BMD is
84 PROLOG
related to its contraceptive action through inhibition of Furthermore, bisphosphonates are contraindicated in
gonadotropic secretion, ie, DMPA prevents follicular women who may become pregnant because of concerns
maturation and ovulation and causes endometrial thin- about possible teratogenesis. Similarly, calcium supple-
ning. This also results in suppression of ovarian estradiol mentation may not be needed at this time. Providing
production. Bone resorption via osteoclast activity is dietary guidance to the patient on nutritional sources of
down-regulated by estrogens, and in hypoestrogenemia, calcium and how to meet the recommended daily allow-
bone resorption exceeds bone formation. This imbalance ance will be sufficient. Weight loss would have no sig-
results in decreased BMD. nificant effect on her risk of low BMD.
Meta-analyses of available literature on BMD and
DMPA suggest that transient but reversible declines in Gbolade BA. Depo-Provera and bone density. J Fam Plann Reprod
Health Care 2002;28:7–11; quiz 11, 50.
BMD occur in premenopausal women who take DMPA.
Kaunitz AM, Arias R, McClung M. Bone density recovery after
However, the advantages of DMPA contraceptive use depot medroxyprogesterone acetate injectable contraception use.
generally outweigh the theoretical concerns regarding Contraception 2008;77:67–76.
skeletal harm, particularly because there are no data to Krall EA, Dawson-Hughes B. Smoking increases bone loss and decreases
indicate that such women are at increased fracture risk. intestinal calcium absorption. J Bone Miner Res 1999;14:215–20.
Oral bisphosphonates may be prescribed for patients Pitts SA, Feldman HA, Dorale A, Gordon CM. Bone mineral density,
fracture, and vitamin D in adolescents and young women using depot
who have osteoporosis. The described patient likely medroxyprogesterone acetate. J Pediatr Adolesc Gynecol 2012;25:
has normal BMD, so this intervention is not indicated. 23–6.
59
Ovulation induction in a patient with polycystic ovary syndrome
A 29-year-old nulliparous woman has a history of irregular menstruation every 70–90 days. Her
body mass index is 29 (calculated as weight in kilograms divided by height in meters squared).
She is hirsute in appearance and has an ovarian volume of 12 cm3. Her male partner has a normal
semen analysis. The ovulation induction medication most likely to result in high-order multiple
gestation is
Polycystic ovary syndrome (PCOS) is the most common in prescribing metformin alone. No additional benefit was
ovulation disorder; it affects approximately 5–10% of found when metformin was combined with clomiphene
women of reproductive age. At a meeting in Rotterdam, compared with clomiphene alone.
Netherlands in 2003, international experts on PCOS Clomiphene remains the treatment of choice for induc-
reached a consensus regarding diagnosis of the syn- tion of ovulation in anovulatory women with PCOS. It is
drome. The meeting was endorsed by the European inexpensive, is taken orally, is patient friendly, has few
Society for Human Reproduction and Embryology and adverse effects, and requires little ovarian-response moni-
the American Society for Reproductive Medicine, and toring. Abundant clinical data are available in regard to
its proceedings were published. In 2007, in Thessaloniki, its safety. The mechanism of action is not entirely known,
Greece, an American Society for Reproductive Medicine but it is thought to involve the blockade of the negative
and European Society for Human Reproduction and feedback mechanism in the pituitary gland and hypo-
Embryology-Sponsored PCOS Consensus Workshop thalamus that results in increased secretion of follicle-
met to agree on how to manage infertility in patients stimulating hormone. The main factors that predict
with PCOS and incorporate concepts from a recent outcome of treatment are obesity, hyperandrogenemia, and
PCOS randomized treatment protocol organized by the age. Ovarian volume and menstrual status are additional
Reproductive Medicine Network, which found no value factors that help to predict responsiveness to clomiphene.
Reproductive Endocrinology and Infertility 85
The starting dose of clomiphene generally should be to document any advantage to the use of thiazolidinedio-
50 mg/d for 5 days. Confirmation of ovulation should be nes over metformin.
obtained, and if the patient is ovulatory, treatment of up The ovulation induction medication most likely to
to 6 months is appropriate. If the patient is nonovulatory, result in high-order multiple gestation is gonadotropin.
the dose may be increased up to a maximum of 150 mg/d Gonadotropin therapy initiates and maintains follicle
for 5 days because there is no clear evidence of efficacy growth by increasing follicle-stimulating hormone above
at higher doses. Approximately 75–80% of patients with a threshold dose for sufficient duration to generate a lim-
PCOS will ovulate after clomiphene, with a conception ited number of developing follicles. Patients with PCOS
rate of up to 22% per cycle. are prone to excessive multiple-follicle development,
Insulin-sensitizing agents currently are being used to which increases multiple births. Therefore, gonadotropin
treat diabetes mellitus and include metformin, a bigu- therapy is discouraged as a first-line agent for use in
anide, and thiazolidinediones, such as pioglitazone and patients with PCOS.
rosiglitazone. There is considerable interest in the use of Letrozole is a promising medication to induce ovula-
metformin for ovulation induction given earlier reports tion. Recent data have been reassuring with regard to
that indicated an increased rate of ovulation in patients its safety and ability to induce ovulation at a similar or
taking metformin, but this did not translate to significantly higher rate to clomiphene. However, this medication is
higher pregnancy rates. Two randomized controlled trials not approved in the United States for noncancer use and
have indicated that metformin does not increase live-birth it is also more expensive than clomiphene.
rates higher than those observed with clomiphene alone
Bates GW Jr, Propst AM. Polycystic ovarian syndrome management
in either obese or normal-weight women with PCOS. options. Obstet Gynecol Clin North Am 2012;39:495–506.
The Reproductive Medicine Network trial demonstrated Consensus on infertility treatment related to polycystic ovary syn-
a selective disadvantage to metformin compared with drome. Thessaloniki ESHRE/ASRM-Sponsored PCOS Consensus
clomiphene and no apparent advantage to adding metfor- Workshop Group. Fertil Steril 2008;89:505–22.
min to clomiphene, except perhaps in women who have a Legro RS, Barnhart HX, Schlaff WD, Carr BR, Diamond MP, Carson
SA, et al. Clomiphene, metformin, or both for infertility in the polycys-
body mass index greater than 35 and in those with clomi- tic ovary syndrome. Cooperative Multicenter Reproductive Medicine
phene resistance. In addition, there are insufficient data Network. N Engl J Med 2007;356:551–66.
60
Congenital adrenal hyperplasia
A 24-year-old nulligravid woman comes to your office for prenatal counseling. Her medical his-
tory is significant for congenital adrenal hyperplasia (CAH). The most common form of congenital
adrenal hyperplasia is deficiency in
(A) 17α-hydroxylase
* (B) 21-hydroxylase
(C) 11β-hydroxylase
(D) 3β-hydroxysteroid dehydrogenase
The virilizing congenital adrenal hyperplasias are a the deficient enzyme, and increased adrenal androgen
group of autosomal recessive disorders characterized by secretion. The most common form of CAH that accounts
mutations in steroidogenic enzyme genes. The virilizing for approximately 90–95% of cases is 21-hydroxy-
forms of CAH are due to decreased activity of 21-hydrox- lase deficiency. The reported incidence of classic 21-
ylase (P450c21), 11β-hydroxylase (P450c11B1), or hydroxylase deficiency ranges from 1 in 9,000 to 1 in
3β-hydroxysteroid dehydrogenase type 2. The common 15,000 with a variation between ethnic and racial back-
features of these disorders are impaired cortisol pro- grounds. The incidence of nonclassic CAH is approxi-
duction, loss of cortisol-negative feedback inhibition, mately 1 in 1,000 but also varies among populations.
increased pituitary adrenocorticotropic hormone secre- In female fetuses with CAH, the adrenal gland pro-
tion, accumulation of steroid intermediates proximal to duces supraphysiologic doses of androgens during the
86 PROLOG
61
Androgen disorders
A 5-year-old girl is brought to your clinic by her mother. She is short for her age and walks with a
slight limp. Her mother asks you about her significant skin discoloration. On examination, she has
Tanner stage 1 breasts and hyperpigmented lesions on her face, chest, and left arm (Fig. 61-1; see
color plate). Imaging shows that her ovaries and brain are normal. Follicle-stimulating hormone
and luteinizing hormone levels are low. The best medical treatment for her is
McCune–Albright syndrome is a rare endocrinologic dis- evidence of McCune–Albright syndrome, often char-
order characterized by a triad of cutaneous pigmentations acterized by hyperpigmented skin lesions. Laboratory
(café au lait spots which have irregular borders and stop evaluation consists of testing for gonadotropin levels
at the midline), polyostotic fibrous dysplasia, and mul- (follicle-stimulating hormone and luteinizing hormone),
tiple endocrine abnormalities. At least two of these three sex steroid level (estradiol), and thyroid hormone levels.
features must be present in a patient in order to make Pelvic ultrasonography may be ordered to evaluate for
the diagnosis. Fibrous dysplasia can involve a single or ovarian masses or cysts. In addition to laboratory testing,
multiple skeletal sites. Patients often exhibit a limp with X-ray of the dominant hand should be done to evaluate
or without pain and, occasionally, a pathologic fracture. for bone age.
Scoliosis is common and may be progressive. The treatment goal is to preserve the height potential of
Precocious puberty is classified as gonadotropin the child as well as to minimize secondary sexual charac-
dependent (central or true) or gonadotropin independent teristics to avoid psychologic effects when the child com-
(peripheral or pseudo). Central and peripheral preco- pares herself with her peers. In cases of central precocious
cious puberty are differentiated by the level of activity puberty, the goal is to suppress the excessive gonadotro-
of the hypothalamic–pituitary–ovarian axis. The hypo- pin production. This can be achieved through the use of
thalamic–pituitary–ovarian axis is active in physiologic GnRH analogs such as leuprolide acetate. Leuprolide
puberty as well as central precocious puberty. Peripheral will cause suppression of the gonadotropin production
precocious puberty is independent of gonadotropin secre- from the pituitary, which will in turn reduce sex steroid
tion from the pituitary and there is no activation of the production from the ovaries. Because peripheral preco-
hypothalamic–pituitary–ovarian axis. The source of the cious puberty is independent of gonadotropin production,
sex steroids is either exogenous (accidental ingestion) or leuprolide therapy will not be effective in halting the pro-
endogenous (McCune–Albright syndrome or sex steroid- gression of pubertal development. Therefore, the under-
producing tumor). lying cause of sexual precocity should be addressed first.
The most common endocrinopathy is precocious Aromatase inhibitors have been effectively used to treat
puberty due to overfunction of the ovary. Other endocrine sexual precocity in McCune–Albright syndrome since the
abnormalities include excessive function of the thyroid 1980s. These medications inhibit aromatase activity by
gland, adrenal gland, and pituitary gland (over production competitively binding to the cytochrome P450 subunit
of growth hormone or prolactin). The presence of higher of the aromatase enzyme, decreasing estrogen biosyn-
levels of sex steroids in peripheral precocious puberty thesis. Other aromatase inhibitors, such as testolactone,
may cause the activation of the hypothalamic–pituitary– a first-generation aromatase inhibitor, have been shown
ovarian axis and the patient may develop central preco- to be somewhat effective, but high doses are required to
cious puberty. achieve the desirable effect. The newer third-generation
Evaluation of a patient who presents with premature aromatase inhibitors, anastrozole and letrozole, have the
development of secondary sexual characteristics starts advantage of once-daily dosing.
with an examination that pays close attention to the In the described case, because the cause of precocious
Tanner stages of the breasts and pubic hair as well as puberty is secondary to McCune–Albright syndrome,
88 PROLOG
which causes precocious puberty independent of the tions of McCune–Albright syndrome: a case series review. J Pediatr
Adolesc Gynecol 2012;25:e9–e13.
hypothalamus and pituitary, use of GnRH agonists or
Brito VN, Latronico AC, Arnhold IJ, Mendonça BB. Update on the
antagonists would not be useful. The use of medroxypro- etiology, diagnosis and therapeutic management of sexual precocity
gesterone acetate in McCune–Albright syndrome does (Erratum in Arq Bras Endocrinol Metabol 2008;52:576). Arq Bras
not prevent closure of the epiphyses and patients may Endocrinol Metabol 2008;52:18–31.
remain short. Haddad N, Eugster E. An update on the treatment of precocious puberty
in McCune-Albright syndrome and testotoxicosis. J Pediatr Endocrinol
Alves C, Silva SF. Partial benefit of anastrozole in the long-term treat- Metab. 2007;20:653–61.
ment of precocious puberty in McCune-Albright syndrome. J Pediatr Mieszczak J, Lowe ES, Plourde P, Eugster EA. The aromatase inhibi-
Endocrinol Metab 2012;25:323–5. tor anastrozole is ineffective in the treatment of precocious puberty
Berberoglu M. Precocious puberty and normal variant puberty: defini- in girls with McCune-Albright syndrome. J Clin Endocrinol Metab
tion, etiology, diagnosis and current management. J Clin Res Pediatr 2008;93:2751–4.
Endocrinol 2009;1:164–74. Wit JM, Hero M, Nunez SB. Aromatase inhibitors in pediatrics. Nat
Bercaw-Pratt JL, Moorjani TP, Santos XM, Karaviti L, Dietrich JE. Rev Endocrinol 2011;8:135–47.
Diagnosis and management of precocious puberty in atypical presenta-
62
Premature adrenarche
A 6-year-old white girl comes to your office with her mother. The girl has a 6-month history of
pubic hair growth. Physical examination shows a body mass index of 27 (calculated as weight in
kilograms divided by height in meters squared). She has Tanner stage 1 breast development and
Tanner stage 3 pubic hair development. Bone age is slightly advanced. Blood tests reveal normal
levels of androstenedione, testosterone, 17-hydroxyprogesterone, and thyroid-stimulating hormone.
Her dehydroepiandrosterone sulfate (DHEAS) level is slightly elevated for her age. The disease that
this patient will most likely develop as an adult is
Premature adrenarche has historically been defined as activity remains low, this decreases the conversion of
pubic or axillary hair development in girls before age Δ5 compounds (pregnenolone, 17-hydroxypregnenolone,
8 years and refers to early activation of the hypotha- DHEA) to the respective Δ4 compounds (progesterone,
lamic–pituitary–adrenal axis with increased production hydroxyprogesterone, androstenedione). Physical mani-
of the adrenal C19 steroids, DHEAS, and androstenedi- festations of adrenarche include development of pubic
one (Fig. 62-1). Normal adrenarche occurs around age and axillary hair and increased apocrine sweat gland
6–8 years with increased production of dehydroepian- activity resulting in body odor.
drosterone (DHEA) and DHEAS from the adrenal reticu- In the general population, an increase in adrenal C19
laris. The DHEA is metabolized to androstenedione and, steroids is seen in girls aged 6–8 years, but racial varia-
subsequently, to testosterone in peripheral tissues and tions in axillary and pubic hair development exist. (Mean
in the gonads. Although production of DHEA is under ages for pubic hair growth: African Americans, 8.8 years;
the control of the adrenocorticotropic hormone (ACTH), whites, 10.5 years. Mean ages for axillary hair growth:
adrenarche is not associated with an increase in ACTH. African Americans, 10 years; whites, 11.8 years.) Normal
Thus, the increase in DHEA is due to changes in ste- sexual hair development is dependent on androgen pro-
roidogenic enzyme activities, including 17α-hydroxylase duction by the adrenal glands and the gonads. Decreased
(converts pregnenolone to 17-hydroxypregnenolone) androgen production by either organ, such as with pri-
and 17,20 lyase (converts 17-hydroxypregnenolone to mary hypoadrenalism or with primary ovarian failure
DHEA). Because 3β-hydroxysteroid dehydrogenase (Turner syndrome), results in decreased sexual hair
Reproductive Endocrinology and Infertility 89
Cholesterol
1
Pregnenolone 6 17-OH Pregnenolone 7 Dehydroepiandrosterone
2 2 2
6 7
Progesterone 17-OH Progesterone Androstenedione
3 3
Deoxycorticosterone 11 Deoxycortisol
4 4
Corticosterone Cortisol
5
Aldosterone
but not the total absence of it. Traditionally, activation signs of virilization. Pelvic ultrasonography is indicated in
of the hypothalamic–pituitary–adrenal axis (adrenarche) patients with testosterone levels greater than 200 ng/dL.
and the hypothalamic–pituitary–gonadal axis (gonad- Adrenal imaging may be necessary in patients with abnor-
arche) have been considered independent because either mal cortisol levels and in individuals with greatly elevated
can occur without the other in pathologic conditions. DHEAS levels (greater than 700 micrograms/dL). Height
Yet physiologically they are interrelated. Peripheral and bone age are advanced for chronologic age but normal
conversion of DHEA to androgens and estrogens affects for pubertal age, and predicted adult height is normal. In
gonadotropin-releasing hormone pulsatility. Children contrast to central precocious puberty, follicle-stimulating
with early adrenarche have slightly early pubertal onset. hormone and luteinizing hormone levels are consistent
The term premature adrenarche is often incor- with age and show a prepubertal response to gonadotropin-
rectly used interchangeably with premature pubarche. releasing hormone stimulation.
Premature adrenarche consists of abnormally elevated Children with premature adrenarche are more likely to
DHEA and DHEAS levels in addition to physical find- develop adult hyperandrogenemia with clinical criteria
ings of body odor, oily skin, acne, or sexual hair consistent with PCOS. Prepubertal children with pre-
development. The differential diagnoses for premature mature adrenarche, compared with controls, may show
adrenarche include idiopathic premature adrenarche, biochemical characteristics of PCOS, including insulin
congenital adrenal hyperplasia due to enzyme defects resistance, dyslipidemia, increased inflammatory mark-
(21-hydroxylase deficiency, 11β-hydroxylase deficiency, ers, and abnormal adipose distribution. These findings
and 3β-hydroxysteroid dehydrogenase deficiency), are more prominent with associated obesity. Other con-
Cushing disease, virilizing adrenal or ovarian tumors, and ditions (eg, CAH, Cushing disease, ovarian tumors, and
exogenous exposure to androgens. Central precocious adrenal tumors) may cause premature adrenarche but
puberty may rarely initially present with pubarche. By are not more likely in adults with a history of childhood
far the most cases are idiopathic premature adrenarche. premature adrenarche.
Evaluation for premature adrenarche includes a his- A girl with idiopathic premature adrenarche may
tory, physical examination, review of growth records, be at higher risk as an adult of PCOS along with other
and laboratory testing for DHEAS, follicle-stimulating metabolic derangements. Additional testing for impaired
hormone, luteinizing hormone, testosterone, thyroid- glucose tolerance and lipid abnormalities may be consid-
stimulating hormone, and morning 17-hydroxyproges- ered in children with premature adrenarche and obesity
terone levels. Typically, DHEAS levels are elevated or a history of low birth weight with subsequent catch-up
compared with chronologic age but are normal for the growth.
pubertal stage. An ACTH stimulation test for CAH should
be performed in cases with screening 17-hydroxypro- Barker DJ, Winter PD, Osmond C, Margetts B, Simmonds SJ. Weight in
gesterone levels greater than 200 ng/dL. In patients with infancy and death from ischaemic heart disease. Lancet 1989;2:577–80.
hypertension, additional testing should include blood Ibanez L, Dimartino-Nardi J, Potau N, Saenger P. Premature adre-
narche--normal variant or forerunner of adult disease? Endocr Rev
deoxycortisol, deoxycorticosterone levels to evaluate for 2000;21:671–96.
11β-hydroxylase deficiency, and 24-hour urinary free cor-
Idkowiak J, Lavery GG, Dhir V, Barrett TG, Stewart PM, Krone N,
tisol to evaluate for Cushing disease. Adrenal and ovarian et al. Premature adrenarche: novel lessons from early onset androgen
tumors are rare and most often will be accompanied by excess. Eur J Endocrinol 2011;165:189–207.
90 PROLOG
63
Uterine fibroid embolization
A 40-year-old woman with symptomatic leiomyomas is considering her treatment options. She is
interested in learning more about uterine fibroid embolization. You counsel her that the most com-
mon complication that patients experience after uterine fibroid embolization is
(A) infection
(B) hemorrhage
(C) chronic malodorous vaginal discharge
* (D) postembolization syndrome
(E) vaginal passage of a myoma
Uterine fibroid embolization, also known as uterine Vaginal passage of a myoma occurs in up to 10% of
artery embolization, is a radiologic technique to decrease patients after uterine fibroid embolization. This com-
the size and symptoms of uterine myomas. Polyvinyl plication also appears to be unique to uterine fibroid
alcohol particles or microspheres are injected via a embolization and would not be expected with surgical
transcutaneous femoral artery approach into both uter- myomectomy.
ine arteries. Supplemental metal coils also may be used The safety of uterine fibroid embolization in women
to augment vascular occlusion. The goal is to produce who desire future childbearing has not been established.
ischemia, devascularization, and involution of myomas In patients who wish to preserve their fertility, some
without damaging normal uterine tissue. unique factors need to be taken into consideration. Tran-
Postembolization syndrome is the most common com- sient or permanent ovarian failure has been described in
plication experienced by patients who undergo uterine up to 15% of patients after uterine fibroid embolization.
fibroid embolization. It is characterized by pelvic pain, The etiology is unclear, but embolic material may travel
cramping, nausea, vomiting, fever, fatigue, myalgias, to the ovarian blood supply. Women older than 45 years
malaise, and leukocytosis. The symptoms occur in the may be particularly susceptible to ovarian insufficiency
first 48 hours after the procedure and will improve after undergoing the procedure. In one investigation, repro-
gradually over the next week. Because of the intensity of ductive-aged women younger than 45 years who under-
postembolization syndrome, most patients are hospital- went uterine fibroid embolization showed no change
ized overnight for pain control, and they are administered in basal follicle-stimulating hormone testing, whereas
antipyretics and antiemetics. women older than 45 years exhibited an elevation in
Other complications of uterine fibroid embolization follicle-stimulating hormone levels after the procedure.
include infection, hemorrhage, adhesions, chronic mal- Although uterine fibroid embolization is less likely
odorous vaginal discharge, vaginal passage of a myoma, to adversely affect ovarian reserve in reproductive-aged
and ovarian insufficiency. Infection is possible with women younger than 45 years, this possibility should be
uterine fibroid embolization or surgical myomectomy, discussed when counseling women who are considering
but this complication is uncommon with both procedures. the procedure. Intrauterine adhesions are another concern
Hemorrhage rarely occurs with uterine fibroid emboliza- after either uterine fibroid embolization or myomectomy
tion and is more of a concern with surgical myomectomy. (abdominal or hysteroscopic). Ischemic endometrium
Pelvic and intrauterine adhesions commonly occur after after uterine fibroid embolization may predispose these
uterine fibroid embolization and surgical myomectomy patients to intrauterine adhesion formation and future
and studies suggest incidence rates as high as 59% for placentation abnormalities. Patient selection is critical in
pelvic adhesions. Chronic malodorous vaginal discharge determining the appropriateness of uterine fibroid embo-
is unique to uterine fibroid embolization and would not lization. Surgical myomectomy remains first-line therapy
be expected with surgical myomectomy. Approximately for women with symptomatic myomas who desire fertil-
4–7% of patients experience chronic malodorous vaginal ity preservation. In addition, women with submucosal
discharge after uterine fibroid embolization, presum- myomas or subserosal myomas with small stalks are not
ably due to fluid extrusion from infarcted myomas into ideal candidates.
the endometrial cavity. The discharge will ultimately A number of studies have examined the long- and short-
resolve spontaneously in most cases, but some patients term outcomes of uterine fibroid embolization. Overall,
will require hysteroscopic removal of the necrotic tissue. 85–95% of women report improvement in abnormal
Reproductive Endocrinology and Infertility 91
bleeding and 60–96% report improvement in bulk-related Edwards RD, Moss JG, Lumsden MA, Wu O, Murray LS, Twaddle S,
et al. Uterine-artery embolization versus surgery for symptomatic uter-
symptoms. In terms of long-term outcomes, 75% of ine fibroids. Committee of the Randomized Trial of Embolization versus
patients report improved or normal uterine bleeding Surgical Treatment for Fibroids. N Engl J Med 2007;356:360–70.
5 years or more after uterine fibroid embolization, but Toor SS, Jaberi A, Macdonald DB, McInnes MD, Schweitzer ME,
20% of patients report that they have undergone a second Rasuli P. Complication rates and effectiveness of uterine artery emboli-
zation in the treatment of symptomatic leiomyomas: a systematic review
procedure to manage leiomyoma-related symptoms. In and meta-analysis. AJR Am J Roentgenol 2012;199:1153–63.
appropriately selected candidates, uterine fibroid emboli- van der Kooij SM, Hehenkamp WJ, Volkers NA, Birnie E, Ankum
zation is a safe and effective option for the management WM, Reekers JA. Uterine artery embolization vs hysterectomy in the
of uterine myomas. treatment of symptomatic uterine fibroids: 5-year outcome from the
randomized EMMY trial. Am J Obstet Gynecol 2010;203:105.e1–13.
Alternatives to hysterectomy in the management of leiomyomas.
ACOG Practice Bulletin No. 96. American College of Obstetricians and
Gynecologists. Obstet Gynecol 2008;112:387–400.
64
Ectopic pregnancy
A 22-year-old woman, gravida 1, para 0, comes to your office 8 weeks after her last menstrual
period. Ultrasonography demonstrates a 4-cm left-sided adnexal mass. There is a discernible fetal
heart rate, and she has a β-hCG level of 15,000 mIU/mL. The most appropriate next step is
Early pregnancy failure is the most common complication of 2,000 mIU/L may be an ectopic pregnancy, a multiple
of pregnancy, and approximately 1–2% of all pregnancies pregnancy not yet discernible by ultrasonography, or a
will be ectopic. It is a leading cause of maternal morbid- failed pregnancy.
ity and mortality. An expedient diagnosis of an ectopic Once a concern for ectopic pregnancy is established,
pregnancy may be lifesaving in many circumstances. the patient must be monitored carefully. Most ectopic
Approximately 25% of recognized pregnancies end in pregnancies occur within the fallopian tube. However,
miscarriage. there have been reported cases within the ovary, cervix,
Human chorionic gonadotropin is the most widely stud- uterine corneal region, and intra-abdominal sites. Patients
ied test of early pregnancy. As a single value it is nondi- are at high risk of intra-abdominal bleeding and hemor-
agnostic; however, serial serum β-hCG levels are helpful rhage. This puts the patient at ultimate risk of death if
in identifying patients who require closer surveillance hemorrhage remains untreated.
for early pregnancy failure or ectopic pregnancy. The The described patient has a clear diagnosis of ectopic
expected β-hCG rise in 48 hours for a viable intrauterine pregnancy. Therefore, the patient should not have serial
pregnancy is at least 53%, although a recent study sug- serum β-hCG levels because the location of the ectopic
gests that the optimal accuracy for correctly identifying an pregnancy is clear.
intrauterine pregnancy may require a more conservative Methotrexate is an antimetabolite that impairs DNA
threshold of a 35% rise in 48 hours. Patients who demon- replication and cell proliferation acting through inhibition
strate a slow increase in β-hCG require vigilant care until of the dihydrofolate reductase enzyme. It acts on tropho-
an ectopic pregnancy can be excluded. blastic and normal cells. Methotrexate injected either into
The β-hCG discriminatory zone for a singleton intrauter- the gestational sac or intramuscularly is contraindicated
ine pregnancy is between 1,500 mIU/L and 2,500 mIU/L. in this patient because she has a 4-cm gestational sac and
The absence of an intrauterine gestational sac of approxi- embryonic cardiac motion. Other contraindications for
mately 5 mm in size in the presence of a β-hCG value methotrexate include a gestational sac greater than 3.5 cm
92 PROLOG
and visible cardiac activity on ultrasonography. See Box The described patient would be best served by surgical
64-1 for criteria for the use of intramuscular methotrexate management of her ectopic pregnancy. Compared with
in tubal pregnancies. surgery, medical therapy is less likely to succeed. It may
lead to ectopic pregnancy rupture and a life-threatening
clinical scenario that places the patient’s life at risk and
BOX 64-1 compromises future fertility.
Criteria for the Use of Intramuscular Barnhart K, Sammel MD, Chung K, Zhou L, Hummel AC, Guo
Methotrexate in Tubal Pregnancies W. Decline of serum human chorionic gonadotropin and spontane-
ous complete abortion: defining the normal curve. Obstet Gynecol
Absolute indications 2004;104:975–81.
• Hemodynamically stable without active bleed- Medical management of ectopic pregnancy. ACOG Practice Bulletin
ing or signs of hemoperitoneum No. 94. American College of Obstetricians and Gynecologists. Obstet
• Nonlaparoscopic diagnosis Gynecol 2008;111:1479–85.
• Patient desires future fertility Morse CB, Sammel MD, Shaunik A, Allen-Taylor L, Oberfoell NL,
• General anesthesia poses a significant risk Takacs P, et al. Performance of human chorionic gonadotropin curves
in women at risk for ectopic pregnancy: exceptions to the rules. Fertil
• Patient is able to return for follow-up care Steril 2012;97:101–106e2.
• Patient has no contraindications to methotrexate
Relative indications
• Unruptured mass 3.5 cm or less at its greatest
dimension
• No fetal cardiac motion detected
• Patients whose β-hCG level does not exceed a
predetermined value (6,000–15,000 mIU/mL)
Absolute contraindications
• Breastfeeding
• Overt or laboratory evidence of immunodefi-
ciency
• Alcoholism, alcoholic liver disease, or other
chronic liver disease
• Preexisting blood dyscrasias (eg, bone marrow,
hypoplasia, leukopenia, thrombocytopenia, or
significant anemia)
• Known sensitivity to methotrexate
• Active pulmonary disease
• Peptic ulcer disease
• Hepatic, renal, or hematologic dysfunction
Relative contraindications
• Gestational sac larger than 3.5 cm
• Embryonic cardiac motion
65
Condom failure and morning-after contraception
A 19-year-old woman calls you to report that she engaged in intercourse 4 days earlier and the
condom she used broke. She has heard that there are pills to take to prevent pregnancy. You counsel
her that, in this situation, the most appropriate oral morning-after contraception is
Emergency contraception, also called the morning-after Single-dose ulipristal acetate is efficacious as emer-
pill or postcoital contraception, is used to prevent preg- gency contraception up to 120 hours after unprotected
nancy after unprotected or insufficiently protected sexual intercourse. Unlike levonorgestrel regimens, efficacy of
intercourse. Most commonly, the need for emergency ulipristal acetate does not appear to decrease over time (up
contraception arises when OC use is inconsistent, condom to 120 hours after intercourse). For the patient described,
failure occurs, or no contraceptive has been used. ulipristal acetate is the most appropriate oral option given
In 1974, Yuzpe and colleagues described the first emer- the time interval since she and her partner experienced
gency contraception method, which comprised high-dose condom failure. A copper-containing IUD is also very
combination OCs. In 1998, the U.S. Food and Drug effective as emergency contraception. Published trials
Administration approved the first product specifically report IUD insertions for emergency contraception up to
intended for emergency contraception. This method con- 5 days after unprotected intercourse with pregnancy rates
sisted of four tablets of ethinyl estradiol 50 micrograms ranging from 0% to 0.2%.
and norgestrel 0.5 mg, with two tablets taken initially and Hormonal forms of emergency contraception work by
two more tablets taken 12 hours later. Other methods of inhibiting or delaying ovulation. Emergency contraception
emergency contraception that have since been approved is not effective once a pregnancy has been established and
include progestin-only OCs, antiprogestin OCs, and the should not be confused with medical abortion. If hormonal
copper-containing intrauterine device (IUD). emergency contraception is inadvertently taken after a
The current levonorgestrel-only method for emergency pregnancy is already established, there is no increased ter-
contraception consists of a 1.5 mg total dose of levo- atogenic risk to the developing embryo. Emergency con-
norgestrel. The single-dose protocol involves taking two traception, particularly levonorgestrel-only emergency
0.75 mg levonorgestrel pills at the same time. The contraception, is considered safe and should be available
two-dose regimen requires the patient to take one pill to all women, including women with contraindications
immediately and another 12 hours later. Both protocols to conventional combined OCs. The Centers for Disease
are equally efficacious, but the single-dose regimen may Control and Prevention’s “Medical Eligibility Criteria for
improve compliance. Both levonorgestrel-only regimens Contraceptive Use” lists no conditions in which the risks
appear to be more effective than combination estro- of emergency contraception outweigh the benefits.
gen–progestin regimens and also are associated with The American College of Obstetricians and Gynecol-
less nausea and vomiting. Because of these advantages, ogists recommends that emergency contraception be
levonorgestrel-only regimens are preferred over combina- taken as soon as possible after an episode of unprotected
tion estrogen–progestin options. Levonorgestrel regimens or inadequately protected intercourse. Although efficacy
are highly efficacious up to 72 hours after intercourse is highest with early use, emergency contraception should
(although moderate efficacy may persist up to 120 hours be made available to patients who request it up to 5 days
after intercourse). Even in the first 72 hours, the timing after unprotected intercourse. Additionally, neither a
of treatment is important. When levonorgestrel regi- physical examination nor a pregnancy test is required
mens are used in the first 24 hours, 95% of pregnancies before providing emergency contraception, although
are prevented. When treatment is delayed 25–48 hours counseling regarding long-term contraceptive methods is
after intercourse, 85% of pregnancies are prevented. At recommended. In particular, the copper-containing IUD
49–62 hours after intercourse, only 58% of pregnancies may be appropriate for some women who desire either
are prevented. Efficacy also may decline with obesity. emergency contraception or long-acting contraception.
94 PROLOG
Emergency contraception. Practice Bulletin No. 112. American College Raine TR, Harper CC, Rocca CH, Fischer R, Padian N, Klausner JD,
of Obstetricians and Gynecologists; Obstet Gynecol 2010;115:1100–9. et al. Direct access to emergency contraception through pharmacies and
Fine P, Mathe H, Ginde S, Cullins V, Morfesis J, Gainer E. Ulipristal effect on unintended pregnancy and STIs: a randomized controlled trial.
acetate taken 48-120 hours after intercourse for emergency contracep- JAMA 2005;293:54–62.
tion. Obstet Gynecol 2010;115:257–63.
Glasier AF, Cameron ST, Fine PM, Logan SJ, Casale W, Van Horn J,
et al. Ulipristal acetate versus levonorgestrel for emergency contra-
ception: a randomised non-inferiority trial and meta-analysis. Lancet
2010;375:555–62.
66
Contraception for a patient with systemic lupus erythematosus
A 32-year-old obese woman, gravida 1, para 1, with systemic lupus erythematosus (SLE) requests
reversible contraception. Her disease has been complicated by lupus nephritis, vasculitis, and
antiphospholipid antibodies. You counsel her that the reversible contraceptive with the greatest
risk for her is
For women with coexisting medical conditions, reliable For the described obese patient with antiphospholipid
contraception is especially important. No option is com- antibodies, combination hormonal contraceptives carry
pletely risk free, so health care providers must weigh the the greatest risk of the available options. In nonobese
risks of contraception against the risks of an unplanned women with uncomplicated SLE, however, combination
pregnancy. In SLE, avoiding disease flares or disease hormonal contraceptives are a reasonable choice based on
progression is particularly important. In many cases of data from two different clinical trials. For patients with
SLE, the coexistence of antiphospholipid antibodies complicated SLE, progestin-only contraceptives (oral
increases a woman’s baseline risk of thromboembolism. contraceptives, injections, implants, and hormonal IUDs)
Complications of SLE, such as systemic vascular disease, or nonhormonal contraceptives (copper IUD or permanent
nephritis, or uncontrolled hypertension, also can make the surgical options) are preferred over combination hor-
contraceptive choices for these patients more difficult. monal contraceptives. Approximately 25% of pregnant
Twelve months of combination hormonal contracep- women with SLE ultimately terminate their pregnancies,
tive use by women with mild SLE but without antiphos- which emphasizes the importance of effective birth con-
pholipid antibodies has been shown to carry a risk no trol for this patient population.
different than use of a progestin-only oral contraceptive
or the copper IUD in terms of disease progression, flares, Culwell KR, Curtis KM, del Carmen Cravioto M. Safety of contracep-
tive method use among women with systemic lupus erythematosus: a
thromboses, or infections. Experts agree that women with systematic review. Obstet Gynecol 2009;114:341–53.
SLE and antiphospholipid antibodies carry a baseline Petri M, Kim MY, Kalunian KC, Grossman J, Hahn BH, Sammaritano
increased risk of thromboembolism and, thus, are not LR, et al. Combined oral contraceptives in women with systemic lupus
good candidates for combination hormonal contracep- erythematosus. OC-SELENA Trial. N Engl J Med 2005;353:2550–8.
tives. Additionally, combination hormonal contraceptive Sanchez-Guerrero J, Uribe AG, Jimenez-Santana L, Mestanza-Peralta M,
Lara-Reyes P, Seuc AH, et al. A trial of contraceptive methods in women
use and obesity represent independent risk factors for with systemic lupus erythematosus. N Engl J Med 2005;353:2539–49.
venous thromboembolism. Theoretical concerns have Use of hormonal contraception in women with coexisting medical con-
been raised regarding the risk of infection with IUD ditions. ACOG Practice Bulletin. No. 73. American College of Obstet-
insertion in immunosuppressed patients with autoimmune ricians and Gynecologists. Obstet Gynecol 2006;107:1453–72.
disease. Limited evidence exists to address this issue, but
IUD insertion does not appear to be associated with wors-
ening disease activity or infection in women with SLE.
Reproductive Endocrinology and Infertility 95
67
Contraception for a patient with diabetes mellitus
A 36-year-old primigravid woman with type 1 diabetes mellitus is currently at 35 weeks of gesta-
tion. She is considering different contraceptive methods postpartum. In the absence of appropriate
counseling, the contraceptive method that this patient is most likely to choose is
Women with diabetes mellitus have many misperceptions with diabetes. Some of the use of highly effective over
about appropriate contraceptive choices. The mispercep- moderately effective contraceptive methods may be
tions vary depending on the patient’s age. In a study of because of health care provider counseling in this age
contraceptive methods practiced by women in a large group, but even among the women who chose the highly
managed care organization where contraceptive pre- effective methods, the 19% use of irreversible methods
scriptions and services were covered benefits, 11.6% (tubal sterilization, hysterectomy) in the diabetes group
of women ages 35–44 years with diabetes chose tubal exceeds the 8% use in the women with no chronic condi-
sterilization compared with 6.9% of women with no tion. Similarly, within the group of women with diabetes
chronic condition (Table 67-1). Women ages 35–44 years aged 35–44 years, the 19% use of irreversible methods
with diabetes were also more likely to choose highly is greater than the 5.8% use of reversible methods (IUD
effective methods (sterilization, hysterectomy, IUD, and or subdermal implant). For the group of women aged
subdermal progestin implant) versus women with no 35–44 years with no chronic disease, the 7.6% use of
chronic condition at a rate of 24.8% versus 15.6%. In reversible, highly effective methods is just slightly below
comparison, 18.7% of women ages 35–44 years with the 8% use of irreversible, highly effective methods. In
no chronic condition were more likely to use moder- summary, patients aged 35–44 years with diabetes tend
ately effective contraceptive methods, ie, combination to use more irreversible, highly effective methods, such
oral contraceptives, patch, or ring, versus 9% of women as tubal sterilization and hysterectomy, than reversible,
TABLE 67-1. Proportion of Nonpregnant Women Aged 35–44 Years Who Received
Contraceptive Services by Disease History (%)
highly effective methods, such as the IUD and subdermal organization revealed that 61.3% of the women did not
implant. receive contraceptive counseling or practice a contracep-
Some of these preferences expressed by patients aged tive method compared with 39.7% of the women without
35–44 years with diabetes may have their origins in prior a chronic condition. Clinicians appear to provide less
counseling sessions with health care providers when the preconception counseling and contraceptive therapy to
patient was much younger or be based upon the patient’s women with diabetes than women without a chronic
perception of her diabetes. In the United States, approxi- medical condition.
mately 50% of all pregnancies are unintended. When an Similarly, adolescents with type 1 diabetes have fur-
unintended pregnancy occurs, a patient with diabetes ther misconceptions and have access to less reliable infor-
is more likely than a patient without diabetes to be in mation about contraceptive methods. In a recent study,
less-than-optimal glycemic control. As a consequence, 43% of adolescents with type 1 diabetes felt that all birth
the birth defect rate among women with diabetes is esti- control methods were less effective in women with dia-
mated to be 9%, 2–3 times greater than the risk for the betes versus women without diabetes and only 69% felt
general population. These factors could be responsible comfortable about asking a health care professional about
for a health care provider bias to counsel women to use birth control. Clearly, an increased education effort is
more effective contraceptive methods and may account needed to inform adolescent women with diabetes about
for the observed differences in the contraceptive choices the necessity, efficacy, and safety of current reversible
of patients aged 35–44 years with diabetes. Additional forms of contraception. This educational effort needs to
data suggest that the increased prevalence of tubal ster- be continued for the group of women in the 35–44-year
ilization and hysterectomy procedures in patients with age range, where reversible forms of highly effective
diabetes over highly effective reversible contraceptive contraceptive methods need to be emphasized as suitable
methods might reflect a knowledge gap among health options to the more prevalent current use of irreversible
care providers who are unfamiliar with these less inva- methods, such as tubal sterilization and hysterectomy.
sive, less expensive, long-acting reversible contraceptive This education effort needs to be further extended to all
methods. Short-term utilization of long-acting reversible health care providers who counsel women with diabetes
contraceptive methods may avoid patient regret over an about their contraceptive options.
irreversible hysterectomy or tubal sterilization. Even the
use of moderately effective methods, such as low-dose Schwarz EB, Postlethwaite D, Hung YY, Lantzman E, Armstrong MA,
combination oral contraceptives, may be preferable to Horberg MA. Provision of contraceptive services to women with dia-
betes mellitus. J Gen Intern Med 2012;27:196–201.
an irreversible method, and their use is safe among
Schwarz EB, Sobota M, Charron-Prochownik D. Perceived access to
women with well-controlled diabetes. Further data from contraception among adolescents with diabetes: barriers to preventing
the aforementioned research study in which women pregnancy complications. Diabetes Educ 2010;36:489–94.
with diabetes were monitored in a large managed care
Reproductive Endocrinology and Infertility 97
68
Maternal virilization in hyperreactio luteinalis
Virilization of the mother during pregnancy, although Hyperreactio luteinalis is defined as the presence
uncommon, is most often due to benign ovarian condi- of multiple theca-lutein cysts bilaterally due to hCG
tions and rarely due to neoplasia, placental enzyme dis- stimulation. The condition is more common with mul-
order, or iatrogenic causes. Normally, circulating levels tiple gestations, molar pregnancy, and choriocarcinoma
of the androgen testosterone and the androgen precursor but may be seen in apparently normal pregnancies. In
androstenedione increase in pregnancy because of human approximately 70% of cases, the presentation occurs in
chorionic gonadotropin (hCG) stimulation of ovarian the- the third trimester or after delivery in cases of persistently
cal cells. In contrast, circulating dehydroepiandrosterone high hCG. Approximately 30% of women experience
and dehydroepiandrosterone sulfate (DHEAS) levels virilization. The condition resembles ovarian hyper-
decrease during pregnancy despite increased produc- stimulation syndrome observed in women with multiple
tion by the adrenal gland. This is because of the primary theca-lutein cysts, which are caused by drugs taken to
conversion of dehydroepiandrosterone and DHEAS to induce ovulation. Occasionally, hyperreactio luteinalis
estrogen in the placenta. will mimic ovarian hyperstimulation syndrome, with
Two principal mechanisms prevent androgen manifes- the loss of ovarian vascular integrity leading to ascites
tations during pregnancy: 1) an increase in sex hormone- and pleural effusion. As long as there is not a condition
binding globulin (SHBG) and 2) high aromatase activity leading to persistence of hCG after delivery, hyperreactio
in the placenta. Levels of SHBG increase dramatically luteinalis resolves postpartum. As with any mass effect,
during pregnancy because estrogen stimulates the liver; an increased risk exists of ovarian torsion, cyst rupture,
SHBG binds a high proportion of dihydrotestosterone and bleeding.
(DHT) and testosterone, thus decreasing the availability Although not typical, women with significant polycys-
of free androgens. Meanwhile, the high aromatase con- tic ovary syndrome-associated hyperandrogenism may
tent of the placenta converts maternal androstenedione to experience worsening of hirsutism during pregnancy.
estrone and testosterone to estradiol. It should be noted This rarely causes virilization. Symptoms improve after
that DHT is not a substrate for aromatase. Thus, viriliza- pregnancy but tend to have a high recurrence rate in sub-
tion of the pregnant woman occurs when the level of sequent pregnancies.
androgens exceeds the capability of these mechanisms or Ovarian tumors that may coincidentally develop
there is a defect in placental aromatization. during pregnancy include Sertoli–Leydig cell, granu-
Ovarian luteoma and hyperreactio luteinalis are benign losa–theca cell, thecoma, cystadenoma, Brenner, cystad-
conditions of the ovary that are unique to pregnancy enocarcinoma, and Krukenberg tumors. These entities
and lead to hirsutism and virilization. Luteoma typically cause hyperandrogenism either by direct production of
presents as a solid ovarian mass and is bilateral 30–50% androgens or by inducing a surrounding theca hyper-
of the time. Luteomas vary in size, but typically are plasia responsible for androgen production. Maternal
6–10 cm. Approximately one third of luteomas will virilization is seen most of the time. Adenocarcinoma of
produce adequate androgens to result in maternal viril- the adrenal gland and Cushing disease are rare causes of
ization. In addition to hormonal manifestations, the solid maternal virilization.
mass may increase the risk of ovarian torsion. Cut surface Placental aromatase deficiency is an autosomal
of the mass appears yellow because of the high choles- recessive mutation of the CYP19 gene in the fetus. The
terol content. inability of the placenta to convert androstenedione to
98 PROLOG
estrone and testosterone to estradiol results in maternal evaluation to show normalization of hCG and androgen
virilization. A female fetus also will be androgenized levels. There is no indication for oophorectomy, biopsy,
with ambiguous genitalia. The diagnosis is suspected or fine-needle aspiration. With the appearance of multiple
when low levels of estriol are observed in the maternal ovarian cysts and no other lesions in the pelvis, CA 125
circulation. and carcinoembryonic antigen tumor markers are not
Rarely, maternal virilization is caused by exposure to indicated.
synthetic androgen. Androgenization of a female fetus A solid mass in the ovary with maternal virilization is
in disorders leading to maternal virilization is dependent a more difficult problem. Luteomas and ovarian tumors
on the level and type of androgen present. In women associated with maternal virilization are very rare. The
who exhibit virilization from a luteoma, approximately decision to remove a solid mass must take into account
60–70% will have some degree of masculinization of a the gestational age and other findings that may indicate
female infant. In contrast, masculinization of a female malignancy, such as ascites, weight loss, elevated tumor
infant is essentially never seen with hyperreactio lutei- markers, or findings suggestive of metastatic disease.
nalis or pregnancy exacerbation of polycystic ovary syn- Luteomas regress rapidly with delivery, whereas other
drome. In the case of fetal aromatase deficiency, female ovarian tumors will not be substantially affected.
infants will always show some degree of masculinization.
Evaluation of maternal virilization includes a detailed Kanova N, Bicikova M. Hyperandrogenic states in pregnancy. Physiol
Res 2011;60:243–52.
history in regard to all drugs used and onset of symp-
Morishima A, Grumbach MM, Simpson ER, Fisher C, Qin K.
toms along with laboratory testing, including ovarian Aromatase deficiency in male and female siblings caused by a novel
ultrasonography and laboratory testing for testosterone, mutation and the physiological role of estrogens. J Clin Endocrinol
DHEAS, DHT, hCG, and estriol levels. The finding Metab 1995;80:3689 – 98.
of elevated hCG and bilateral multiple ovarian cysts is Sir-Petermann T, Maliqueo M, Angel B, Lara HE, Perez-Bravo F,
Recabarren SE. Maternal serum androgens in pregnant women with
consistent with hyperreactio luteinalis and requires post- polycystic ovarian syndrome: possible implications in prenatal andro-
partum follow-up with ultrasonography and laboratory genization. Hum Reprod 2002;17:2573 – 9.
69
Recurrent pregnancy loss
A 28-year-old woman, gravida 2, comes to your office for evaluation after three first-trimester
pregnancy losses. She and her husband are in good health. Her thyroid-stimulating hormone and
prolactin levels, pelvic ultrasonography, and hysterosalpingography are all normal. To evaluate the
couple’s recurrent pregnancy loss, you should order
Recurrent pregnancy loss is defined as two or more failed three losses, depending on family history. Reciprocal
clinical pregnancies. Evaluation of pregnancy loss should and robertsonian translocations can be identified in
be initiated after two first-trimester pregnancy losses. The couples with recurrent pregnancy loss. Translocations
most common contributors to recurrent pregnancy loss are are an exchange of material between nonhomologous
maternal uterine malformations, maternal antiphospho- chromosomes. If there is no loss of genetic material, the
lipid syndrome, and cytogenetic abnormalities in either translocation is considered balanced; however, if genetic
partner. material is lost, it is considered unbalanced. Robertsonian
Cytogenetic abnormalities account for approximately translocations are similar to reciprocal translocations
5% of recurrent pregnancy loss etiologies. Peripheral but are specific to the exchange of material between
blood for karyotyping should be collected after two or two acrocentric chromosomes (chromosomes 13, 14, 15,
Reproductive Endocrinology and Infertility 99
21, and 22) with subsequent loss of their short arms. An endometrial biopsy to define luteal phase defi-
Translocations are more common in women. Therefore, ciency is not warranted because the diagnosis of luteal
karyotyping is more likely to detect a translocation in phase deficiency is difficult and its treatment remains
the female partner if a chromosomal abnormality exists. controversial. Use of progesterone in unexplained recur-
However, this should not be a reason for excluding the rent pregnancy loss has not been shown to improve
male partner from evaluation. Treatment for couples pregnancy outcome. Fragile X testing will not be helpful
with parental cytogenetic abnormalities should start with in providing information regarding this couple’s etiology
genetic counseling. Couples also may opt for preim- for recurrent pregnancy loss. Fragile X carrier testing is
plantation genetic diagnosis with transfer of unaffected more beneficial in the evaluation of the patient with pre-
embryos. mature ovarian failure or with diminished ovarian reserve
Up to 20% of recurrent pregnancy loss patients will because premutations may predispose the female partner
test positive for antiphospholipid antibodies. The most to ovarian deficiency. Antithrombin III would not be
widely accepted tests for this condition include tests for helpful; patients with antithrombin III deficiency usually
lupus anticoagulant, anticardiolipin antibodies, and anti- have thrombotic events before pregnancy loss, which is
β2-glycoprotein. Abnormalities detected with the use of not the case for the described patient.
any of these tests require repeat testing in 12 weeks for
confirmation. Prophylactic therapies for patients with a Antiphospholipid syndrome. Practice Bulletin No. 132. American
College of Obstetricians and Gynecologists. Obstet Gynecol 2012;
finding of antiphospholipid antibodies include daily baby 120:1514–21.
aspirin and heparin or low-molecular weight heparin. Evaluation and treatment of recurrent pregnancy loss: a committee
Although congenital müllerian anomalies usually affect opinion. Practice Committee of the American Society for Reproductive
pregnancies in the second trimester, they also may have Medicine. Fertil Steril 2012;98:1103–11.
a role in first-trimester losses. It is therefore wise to Hirshfeld-Cytron J, Sugiura-Ogasawara M, Stephenson MD. Man-
agement of recurrent pregnancy loss associated with a parental carrier
evaluate the endometrial cavity, often starting with pelvic of a reciprocal translocation: a systematic review. Semin Reprod Med
ultrasonography. Saline-infusion ultrasonography, hys- 2011;29:470–81.
terosalpingography, and hysteroscopy can be used to fur- Miyakis S, Lockshin MD, Atsumi T, Branch DW, Brey RL, Cervera R,
ther evaluate the endometrial cavity. Treatment typically et al. International consensus statement on an update of the classifica-
involves repair of the anomaly or use of a gestational car- tion criteria for definite antiphospholipid syndrome (APS). J Thromb
Haemost 2006;4:295–306.
rier if repair is not possible. In patients with no personal
Rai R, Regan L. Recurrent miscarriage. Lancet 2006;368:601–11.
or close family history of thrombotic events, screening
Stephenson MD, Awartani KA, Robinson WP. Cytogenetic analysis of
for thrombophilia (factor V Leiden mutations and protein miscarriages from couples with recurrent miscarriage: a case-control
C and S deficiency) is not warranted. study. Hum Reprod 2002;17:446–51.
100 PROLOG
70
Sheehan syndrome
A 29-year-old woman, gravida 2, para 2, comes to your office with secondary amenorrhea. She is
otherwise healthy. She stopped having menstrual cycles after her last delivery 2 years ago, which
was complicated by severe postpartum hemorrhage. Magnetic resonance imaging of the patient’s
brain is as shown (Fig. 70-1). The test that would best identify the most life-threatening manifesta-
tion of this condition is a test for the level of
Pituitary Stalk
Pituitary Stalk
CSF
CSF
A B
FIG. 70-1
Postpartum infarction of the pituitary, also known as care, postpartum hemorrhage and subsequent Sheehan
Sheehan syndrome, usually occurs after a significant syndrome is a common cause of hypopituitarism.
postpartum hemorrhage involving hypotension and trans- Patients with Sheehan syndrome can have abnormal
fusion of multiple units of blood. For severe cases, levels of some or all hormones that originate from the
patients may have immediate severe hypopituitarism with anterior and posterior pituitary. In addition, magnetic
lethargy, anorexia, and weight loss. In mild-to-moderate resonance imaging of the brain can show a partially or
cases, the initial symptom may be absent lactation due completely empty sella as seen in Figure 70-1. Clinical
to diminished prolactin, although many patients have a manifestations of Sheehan syndrome can be quite hetero-
delayed presentation with secondary amenorrhea. Some geneous. Although the most common hormonal altera-
patients may consult their physician weeks, months, or tions include changes in levels of growth hormone,
years after the precipitating event. In developed countries, prolactin, and gonadotropins, many patients also have
the incidence of Sheehan syndrome is decreasing because defects in adrenocorticotropic hormone (ACTH) and
of improvements in obstetric care. However, for patients thyroid-stimulating hormone (TSH) secretion. Growth
from countries without access to high-quality medical hormone deficiency in adults tends to have signs and
Reproductive Endocrinology and Infertility 101
symptoms associated with body composition changes, TSH can cause secondary hypothyroidism. Serum levels
including a decrease in lean body mass and an increase in of free T4 can be low to normal, and TSH levels may be
fat mass. Growth hormone deficiency in adults is not life low or normal, demonstrating inability of the pituitary
threatening and replacement of growth hormone is con- to compensate for the low levels of thyroid hormone.
troversial outside of the pediatric population. Prolactin Therefore, practitioners must always check for TSH and
levels may be chronically low but have no clinical sig- free T4 when secondary hypothyroidism is suspected.
nificance in a premenopausal woman with secondary Common symptoms of hypothyroidism include fatigue,
amenorrhea, such as the described patient. cold intolerance, weight gain, and constipation. Most
Deficiency in ACTH secretion is a serious condition cases of hypothyroidism are not life threatening.
associated with Sheehan syndrome. Chronic absence of Gonadotropin abnormalities are noted frequently in
ACTH can cause adrenal atrophy and inability to mount women with pituitary insufficiency. Such abnormalities
an adequate physiologic response to a crisis such as often present with secondary amenorrhea. Testing for
a serious infection or other acute medical problem. FSH, luteinizing hormone (LH), and estradiol often can
Because adrenal insufficiency can result in a life-threat- help determine if amenorrhea is due to a central cause
ening adrenal crisis, making it the most serious condition (low FSH, LH, and estradiol levels) or an ovarian cause
for which to screen, testing for adrenal insufficiency with (high FSH and LH levels and low estradiol levels). If
early-morning serum cortisol levels is essential. When fertility is desired, medical therapy with exogenous
testing for adrenal insufficiency, it is important to wait gonadotropins can be helpful. Otherwise, low levels of
at least 6 weeks after the pituitary insult because adrenal gonadotropins and, subsequently, estrogen, can precipi-
atrophy can take time to develop and premature testing tate bone loss and osteoporosis, necessitating supplemen-
can yield falsely normal results. After the diagnosis of tal estrogen therapy.
adrenal insufficiency is confirmed, replacement of cor-
ticosteroids is essential as the first step in treatment of Sheehan HL. The recognition of chronic hypopituitarism resulting from
postpartum pituitary necrosis. Am J Obstet Gynecol 1971;111:852–4.
Sheehan syndrome. Replacement of thyroid hormone
Speroff L, Fritz MA. Amenorrhea. In: Speroff L, Fritz MA, editors.
in cases of untreated adrenal insufficiency can cause an Clinical gynecologic endocrinology and infertility. 8th ed. Philadelphia
even more profound drop in cortisol levels. This may be (PA): Lippincott Williams and Wilkins; 2011. p. 483–4.
due to increased metabolic activity that cannot be handled Tessnow AH, Wilson JD. The changing face of Sheehan’s syndrome.
by the low levels of cortisol. Am J Med Sci 2010;340:402–6.
Thyroid hormone levels can be abnormal in patients
with Sheehan syndrome because relative deficiencies in
102 PROLOG
71
Preimplantation genetic screening
A 41-year-old woman, gravida 2, aborta 2, with a history of recurrent first-trimester pregnancy loss
would like to undergo in vitro fertilization (IVF) to screen embryos for chromosomal abnormalities.
The technique associated with IVF most likely to identify a genetically normal ongoing pregnancy is
Preimplantation genetic diagnosis and screening were to show a benefit and transfer of abnormal embryos
established more than 2 decades ago in order to select believed to be free of disease, or else free of structural
genetically normal embryos from among others cre- chromosomal abnormalities. This technology hinges on
ated through IVF to avoid inherited diseases and give the assessment of a single cell for analysis. Some stud-
the highest potential to achieve pregnancies. Couples ies have suggested that aneuploidy screening by FISH
with a family history of genetic disorders or structural analysis may be inaccurate up to 60% of the time. In
chromosomal abnormalities had few options before the contrast, blastocyst trophectoderm biopsy is the analysis
implementation of these technologies. of hundreds of cells. Therefore, it is more representative
Various methodologies have been implemented to of the embryo itself. The resulting larger DNA yield may
screen embryos for either aneuploidy or single-gene improve the rates of diagnosis when compared with other
disorders to help assess “normal” embryos for embryo methods. A number of laboratory strategies are avail-
transfer in an IVF cycle. These methodologies include able for analysis of trophectoderm-biopsied cells, such
polar body biopsy, blastomere biopsy from an embryo at as microarray, polymerase chain reaction, isothermal
the multicellular stage (day 2 or 3), blastocyst trophec- rolling-circle-based amplification, and complete genomic
toderm biopsy (day 5), and various DNA analysis tech- hybridization. To date, there have been no randomized
niques which include but are not limited to fluorescence in controlled trials to assess their efficacy in preimplanta-
situ hybridization (FISH) analysis, comparative genomic tion genetic screening for advanced maternal age or the
hybridization analysis, and microarray technologies. advantages of one technique over another in the analysis
The initial preimplantation genetic diagnosis (PGD)– of trophectoderm-biopsied cells. However, promising
preimplantation genetic screening standard technique data suggests that identifying euploid embryos may
for detecting chromosomes was FISH analysis. It used improve implantation and the likelihood of live birth.
specific probes for the chromosomes most commonly To date, it has not been demonstrated that aneu-
involved in aneuploidy. The presence or absence of a ploidy screening for women of advanced maternal age
normal pair of chromosomes could be identified, thereby with IVF increases their likelihood of pregnancy. In
allowing the selection of only chromosomally normal fact, preimplantation genetic screening data for day-3
embryos for transfer. Choosing embryos with normal embryo biopsy demonstrates that the preimplantation
chromosomes should increase implantation rates and genetic screening process may do more harm than good.
live-birth rates and reduce miscarriage rates. Prospective Additionally, polar body biopsy on either day 1 or
analysis of this technology did not justify its use and in day 3 is considered inferior to blastomere biopsy. The
many instances suggested a decrease in overall preg- trend in preimplantation genetic screening and PGD is
nancy rates. toward blastocyst trophectoderm biopsy, given the pleth-
Until recently, cleavage-stage embryo biopsy was ora of cells available for analysis. However, embryos
the prevailing method of embryo assessment for PGD– often need to be frozen before transfer to allow time for
preimplantation genetic screening. However, the well- the analysis of the embryo itself. Data are limited as to
documented chromosome mosaicism that exists in early whether preimplantation genetic screening for advanced
human embryos suggests that a biopsied cell may not be maternal age is worth the effort. Studies have shown
truly representative of the rest of the embryo. Mosaicism excellent pregnancy rates with 24-chromosome screening
could complicate the accuracy of diagnosis, thus contrib- of trophectoderm-biopsied embryos. This demonstrates
uting to the failure of preimplantation genetic screening that if preimplantation genetic screening is performed in
Reproductive Endocrinology and Infertility 103
this patient population, day-5 embryo biopsy is the best Al-Asmar N, Peinado V, Vera M, Remohi J, Pellicer A, Simon C, et al.
Chromosomal abnormalities in embryos from couples with a previous
way to assess the embryo. aneuploid miscarriage. Fertil Steril 2012;98:145–50.
Patients are encouraged to work with their obstetrician– Al-Asmar N, Peinado V, Vera M, Remohi J, Pellicer A, Simon C, et al.
gynecologist to assess the need for aneuploidy screening Oxidative stress in an assisted reproductive techniques setting. Fertil
once they become pregnant. Obstetrician–gynecologists Steril 2006;86:503–12.
are encouraged to offer fetal screening once patients Cohen-Bacrie P, Belloc S, Menezo YJ, Clement P, Hamidi J, Benkhalifa
M. Correlation between DNA damage and sperm parameters: a pro-
conceive. Options for screening include but are not lim- spective study of 1,633 patients. Fertil Steril 2009;91:1801–5.
ited to chromosome-selective sequencing of maternal Hardarson T, Hanson C, Lundin K, Hillensjo T, Nilsson L, Stevic J, et
plasma cell-free DNA, chorionic villi sampling, and al. Preimplantation genetic screening in women of advanced maternal
amniocentesis. age caused a decrease in clinical pregnancy rate: a randomized con-
trolled trial. Hum Reprod 2008;23:2806–12.
DNA fragmentation testing is a measure of sperm oxi-
Harton GL, Magli MC, Lundin K, Montag M, Lemmen J, Harper JC.
dative DNA damage within a semen specimen. It is theo- ESHRE PGD Consortium/Embryology Special Interest Group--best
rized that subfertile men carry an increased likelihood of practice guidelines for polar body and embryo biopsy for preimplanta-
having single- or double-stranded DNA breaks packaged tion genetic diagnosis/screening (PGD/PGS). European Society for
Human Reproduction and Embryology (ESHRE) PGD Consortium/
within the head of the sperm. Additionally, oxidative Embryology Special Interest Group. Hum Reprod 2011;26:41–6.
damage can decrease sperm motility, leading to prema- Scott RT Jr, Ferry K, Su J, Tao X, Scott K, Treff NR. Comprehensive
ture acrosome reaction, lipid peroxidation, apoptosis, and chromosome screening is highly predictive of the reproductive potential
reduction in mitochondria membrane potential. These of human embryos: a prospective, blinded, nonselection study. Fertil
Steril 2012;97:870–5.
effects on sperm are thought to contribute to subfertility
in male patients.
72
Low-dose estrogen therapy
A 62-year-old woman underwent natural menopause at age 52 years. She tells you she has constant
vaginal dryness with irritation. She tried combination hormone therapy in the past but was intoler-
ant to the progestin because of constipation. Her physical examination shows vaginal atrophy. She
does not wish to wear a device in her vagina, such as an estradiol ring. The best treatment option
for her is
The Women’s Health Initiative trial and other randomized to a dramatic shift in the philosophy of HT. Currently,
clinical trials have dramatically changed the approach the prevailing attitude toward HT is to use the lowest
to postmenopausal hormone therapy (HT). Before these effective dose for the shortest period of time to relieve
trials, observational studies suggested a benefit to post- menopausal symptoms, including hot flushes and vaginal
menopausal HT in the prevention of coronary artery atrophy.
disease and osteoporosis. These benefits were thought to Hormone therapy consists of estrogen replacement
outweigh the known risks of breast cancer and venous therapy with the addition of progesterone or a progestin to
thrombotic events. Instead of heart protection, the trials prevent endometrial hyperplasia. This is the only proven
found an increased risk of myocardial infarction, stroke, benefit of progesterone–progestin. As a consequence,
and dementia, and they confirmed the risks of breast can- HT is used in women with a uterus, whereas estrogen
cer and venous thrombotic events. Although the param- replacement therapy is used in women with a previous
eters of such studies, such as the age of subjects and the hysterectomy. Many of the risks of estrogen replace-
type of HT used, continue to be debated, the results led ment therapy are dose dependent, including breast cancer
104 PROLOG
and venous thrombotic events. For this reason, greater Common risks of estrogen replacement therapy include
attention has been paid to the use of low-dose estrogen endometrial hyperplasia and cancer, venous thrombotic
to minimize risks. There is no well-established defini- events, and gallbladder disease. The risk of endometrial
tion of low-dose estrogen, but common choices include hyperplasia is estrogen-dose dependent and may be seen
0.3–0.45 mg conjugated equine estrogen, 0.5 mg oral with low-dose systemic administration and standard dose
micronized estradiol, and 0.05 mg transdermal estradiol. vaginal administration. For this reason, these preparations
Low-dose vaginal estrogen has been obtained by either require the use of a continuous or sequential progesterone
reducing the amount of standard preparations or by using or progestin for endometrial protection. The dose of pro-
specially designed products. Two products designed to gesterone–progestin is dependent on whether it is given
give low-dose vaginal estrogen include coated estradiol continuously or sequentially and on the dose of estrogen
tablets and an estradiol-impregnated vaginal ring that being used. The only estrogen preparations that do not
releases estradiol for 3 months. require progesterone or a progestin are estradiol tablets
The U.S. Food and Drug Administration-approved and the estradiol-impregnated vaginal ring. Estradiol
indications for estrogen therapy in postmenopausal tablets are used daily for 2 weeks, then twice weekly.
women include treatment of vasomotor symptoms (hot The inert synthetic estradiol-impregnated vaginal ring
flushes), vaginal symptoms from atrophy, and prevention is changed after 3 months. Circulating estradiol levels
of postmenopausal osteoporosis. This last indication now for the products are in the ranges of 5–11 pg/mL and
conflicts with the recommendation of short-term therapy 7–8 pg/mL, respectively. These levels are within the
for symptoms. range of a normal postmenopausal female. Studies for up
Treatment of vasomotor symptoms requires systemic to a year have shown no evidence of endometrial hyper-
therapy. The dose of estrogen correlates with the relief of plasia. Given that this patient does not want a vaginal ring
hot flushes. All of the aforementioned low-dose medica- and has an adverse reaction to progesterone–progestin,
tions decrease hot flushes when compared with placebo. vaginal estradiol tablets twice weekly would be the best
The addition of progesterone or a progestin to low-dose regimen for her.
estrogen replacement therapy does not detract from the The risk of venous thrombotic events and gallblad-
effect on hot flushes. High-dose oral progesterone or syn- der disease is dependent upon the dose and the route of
thetic progestins also decrease hot flushes compared with estrogen administration. Higher odds of both are seen
placebo, but not nearly to the extent that estrogen does. with oral versus nonoral therapy. The initial pass (first-
The dose of estrogen also correlates with increased pass effect) of estrogen through the liver causes a greater
bone density. The aforementioned low-dose prepara- increase in coagulation factors, whereas the same first
tions have been shown to increase bone mineral density pass inhibits bile acid synthesis, resulting in higher biliary
(BMD) at the hip or spine compared with placebo. The cholesterol. Low-dose vaginal estrogen preparations are
addition of progesterone or a progestin to estrogen does not known to change the risk of venous thrombotic events
not detract from the increase in BMD. Alone, progester- or gallbladder disease. The use of a daily vaginal lubri-
one, medroxyprogesterone acetate, and megestrol have cant would not correct this patient’s atrophic changes.
no significant impact on BMD. In contrast, the proges-
tin norethindrone acetate does offer bone protection, Furness S, Roberts H, Marjoribanks J, Lethaby A. Hormone therapy in
possibly via aromatization to small amounts of ethinyl postmenopausal women and risk of endometrial hyperplasia. Cochrane
estradiol. Vaginal administration of estrogen has not been Database of Systematic Reviews 2012, Issue 8. Art. No.: CD000402.
DOI: 10.1002/14651858.CD000402.pub4.
shown to increase BMD.
Systemic and vaginal therapies alleviate the symptoms Peeyananjarassri K, Baber R. Effects of low-dose hormone therapy
on menopausal symptoms, bone mineral density, endometrium, and
of vaginal atrophy, including vaginal dryness, burning, the cardiovascular system: a review of randomized clinical trials.
and pain during intercourse. Climacteric 2005;8:13–23.
Reproductive Endocrinology and Infertility 105
73
Polycystic ovary syndrome and estrogen breakthrough bleeding
A 37-year-old hirsute woman, gravida 2, para 2, has a body mass index of 37 (calculated as weight
in kilograms divided by height in meters squared). She comes to your office with irregular, heavy
menses that recently required her to be seen in the emergency department, where she received
a transfusion because of anemia resulting from her menses. Thyroid-stimulating hormone and
prolactin levels are normal. Transvaginal ultrasonography reveals bilateral polycystic-appearing
ovaries and a 15-mm endometrium. The finding that an endometrial biopsy is most likely to reveal is
Polycystic ovary syndrome (PCOS) is the most common secretory endometrium would be observed. Amenorrheic
reproductive endocrine disorder in women, with a preva- women with PCOS usually have the most severe hyper-
lence of approximately 6–15%, depending on the criteria androgenism and high antral follicle counts compared
used to diagnose it. The clinical expression of PCOS var- with women who have oligomenorrhea or regular men-
ies but commonly includes oligoanovulation or anovula- strual cycles. In addition, women with PCOS are more
tion, hyperandrogenism (either clinical or biochemical), likely to have upper body fat distribution compared with
and the presence of polycystic ovaries. The condition is weight-matched controls. Greater abdominal or visceral
heterogeneous, and the etiology of the syndrome remains adiposity is associated with greater insulin resistance,
obscure. which could exacerbate the reproductive and metabolic
Biopsy of the endometrium should be considered in abnormalities. It is known that obesity is associated with
patients older than 35 years with unopposed estrogen PCOS, but its causal role in this condition has yet to be
production and amenorrhea that exceeds 12 months. determined.
Anovulatory bleeding from unopposed estrogen created Disordered proliferative endometrium is the most com-
in a stagnant follicular phase can cause uncontrolled mon finding, but some data indicate that women with
proliferation of the endometrium. Thus, the finding in PCOS have an approximately 2.7-fold increased risk of
the described patient would not be scant endometrium endometrial cancer, which can be preceded by hyperpla-
inadequate for evaluation. This endocrine abnormality sia with atypia. Most cases of endometrial cancer are well
does not predispose a patient to infection, such as chronic differentiated and have a good prognosis. Scant endome-
endometritis, and if biopsied, disordered proliferative trium would be more common in a hypoestrogenic state
endometrium often is identified. Although progression such as menopause or hypothalamic amenorrhea.
to endometrial hyperplasia with atypia may occur, dis-
Consensus on women’s health aspects of polycystic ovary syndrome
ordered proliferative endometrium is the more common (PCOS). Amsterdam ESHRE/ASRM-Sponsored 3rd PCOS Consensus
finding. Secretory endometrium is seen after ovulation Workshop Group. Hum Reprod 2012;27:14–24.
and exposure of the endometrium to progesterone. This Giudice LC. Endometrium in PCOS: Implantation and predisposition to
would not been seen in the anovulatory patient. endocrine CA. Best Pract Res Clin Endocrinol Metab 2006;20:235–44.
Although menstrual cycle abnormalities are common Holm NS, Glintborg D, Andersen MS, Schledermann D, Ravn P.
The prevalence of endometrial hyperplasia and endometrial cancer in
during the reproductive years, women with PCOS may women with polycystic ovary syndrome or hyperandrogenism. Acta
ovulate spontaneously. In this minority of PCOS patients, Obstet Gynecol Scand 2012;91:1173–6.
106 PROLOG
74
Hyperthyroidism
A 32-year-old multiparous patient comes to your office for her annual gynecologic examination.
She has been experiencing palpitations and slight tremors in her hands. On physical examination,
you find that she has a symmetrically enlarged, smooth, and painless thyroid gland. She has a
thyroid-stimulating hormone (TSH) level of 0.05 mIU/mL and free thyroxin (T4) level of 8 ng/mL,
results that confirm an overactive thyroid gland. The next step in her evaluation is
Bahn (Chair) RS, Burch HB, Cooper DS, Garber JR, Greenlee MC, Endocrinologists [published errata appear in Thyroid 2011;21:1169.
Klein I, et al. Hyperthyroidism and other causes of thyrotoxico- Thyroid 2012;22:1195]. Thyroid 2011;21:593–646.
sis: management guidelines of the American Thyroid Association
and American Association of Clinical Endocrinologists. American McDermott MT. Hyperthyroidism. Ann Intern Med 2012;157:ITC1–16.
Thyroid Association and the American Association of Clinical
75
Fertility preservation techniques
A 21-year-old nulligravid woman comes to your office after she has received a diagnosis of can-
cer. She is single and interested in future fertility. She would like to explore her options ahead of
undergoing gonadotoxic chemotherapy. The fertility preservation technique that you recommend
as most appropriate for her is
Use of fertility preservation has increased in recent oocyte microtubules to temperature changes leads to poor
years, especially with improvement of techniques such viability or damage after cryopreservation. Early studies
as oocyte cryopreservation. Fertility preservation offers of oocyte cryopreservation yielded disappointing results
hope for a male or female patient who has received the in which the survival of an oocyte after thawing was
diagnosis of cancer, given that cancer treatments may be reported to be as low as 25% and as high as 58% with
gonadotoxic, limit future fertility, or in some cases render poor pregnancy rates.
the patient sterile. The level of loss of ovarian function is Advancements in freezing technology have led to an
dependent on the specific chemotherapeutic agent, length improvement in oocyte survival. The two main techniques
of use, and dose. Chemotherapy agents that may cause used in oocyte cryopreservation are slow cooling and
loss of ovarian function include alkylating agents, such as vitrification. In the slow-cooling technique, freezing takes
cyclophosphamide. Patients should be advised that online place at a rate of 0.3–2°C/min (32.5–35.6°F/min). When
chemotherapy fertility risk calculators are available. the cooling is slow, water is removed from the cells by the
Cryopreservation now plays an integral role in infertil- action of the cryoprotectant, which enables avoidance of
ity services. Men can have sperm cryopreserved before damaging intracellular ice crystallization. The oocytes are
anticipated chemotherapy, radiation, or surgery. The then stored in liquid nitrogen at –196°C (–320.8°F). To
relatively lower cytoplasmic content found in sperm com- avoid water recrystallization during thawing, the oocytes
pared with other cells in the body potentially provides an are thawed quickly. Vitrification, a procedure of ultrarapid
advantage that allows for successful cryopreservation. cooling of samples, has gained popularity in recent years.
Sperm cryopreservation has permitted the use of cryo- Vitrification is the solidification of a solution in a glassy
preserved sperm decades after freezing. Cryopreservation state, which avoids damaging water crystal formation in
also has been used successfully to store for future use the oocyte and the surrounding solution. Vitrification also
excess embryos produced during in vitro fertilization minimizes the exposure time of the oocyte to the toxic
cycles ahead of chemotherapy and radiation therapy. cryoprotectant because freezing is done very rapidly.
Thawed embryos have provided pregnancy success rates Some researchers have reported an oocyte survival rate
similar to fresh embryo transfer cycles. as high as 99% and a pregnancy rate of 32.5%. With the
Oocyte cryopreservation is no longer considered exper- advancement of technology, improved oocyte survival
imental, but it is more technically difficult than sperm or rates and improved pregnancy rates have been achieved.
embryo cryopreservation. The large cytoplasmic content Because of the relative newness of oocyte cryopreser-
of oocytes increases the risk of water crystal damage vation technology, the long-term outcomes of individuals
upon freezing and thawing. Also, the high sensitivity of conceived from cryopreserved oocytes are not known.
108 PROLOG
A small study has shown that vitrification of oocytes after are exposed to gonadotoxic chemotherapeutics compared
ovarian stimulation does not appear to be associated with with postpubertal women. However, the use of GnRH
adverse pregnancy outcomes. The major limitation of agonists during gonadotoxic treatment has been shown to
oocyte cryopreservation is the lack of long-term studies provide limited benefits in terms of fertility preservation.
to evaluate offspring born by means of oocyte cryopreser- Indeed, in many cases, the use of GnRH agonists has
vation technology. not been shown to preserve fertility in women who are
Although the survival rate for cryopreserved oocytes undergoing gonadotoxic treatment.
is low, it has improved via vitrification. Pregnancy rates
per oocyte also have improved via vitrification. Genetic Ayensu-Coker L, Bauman D, Lindheim SR, Breech L. Fertility preser-
vation in pediatric, adolescent and young adult female cancer patients.
abnormalities in offspring from cryopreserved oocytes Pediatr Endocrinol Rev 2012;10:174–87.
have not been shown. However, the cost of oocyte cryo- Cobo A, Garcia-Velasco JA, Domingo J, Remohi J, Pellicer A. Is
preservation may prove prohibitive for many patients. vitrification of oocytes useful for fertility preservation for age-related
Tissue cryopreservation with later autotransplantation fertility decline and in cancer patients? Fertil Steril 2013;99:1485–95.
does hold promise, but improved freezing techniques are Elgindy EA, El-Haieg DO, Khorshid OM, Ismail EI, Abdelgawad M,
Sallam HN, et al. Gonadatrophin suppression to prevent chemotherapy-
needed before offering this as a treatment option. This induced ovarian damage: a randomized controlled trial. Obstet Gynecol
process remains experimental and carries the risk of 2013;121:78–86.
reintroducing cancer cells via the autotransplanted tissue. Kim SS, Donnez J, Barri P, Pellicer A, Patrizio P, Rosenwaks Z, et al.
Embryo cryopreservation is probably the best way to Recommendations for fertility preservation in patients with lymphoma,
preserve future fertility but requires a partner or the use leukemia, and breast cancer. ISFP Practice Committee [published erra-
tum appears in J Assist Reprod Genet 2012;29:1155]. J Assist Reprod
of donor sperm. This may not be an option for women Genet 2012;29:465–8.
without a partner who decline to use a donor specimen Loren AW, Mangu PB, Beck LN, Brennan L, Magdalinski AJ,
for oocyte fertilization. In addition, there are many ethical Partridge AH, et al. Fertility preservation for patients with can-
and legal implications to consider for embryos that will cer: American Society of Clinical Oncology clinical practice guide-
line update. American Society of Clinical Oncology. J Clin Oncol
not be used. Oocyte cryopreservation has the advantage 2013;31:2500–10.
of minimizing these issues when considering disposition Mature oocyte cryopreservation: a guideline. Practice Committees of
of stored oocytes. the American Society for Reproductive Medicine and the Society for
One reason for the use of GnRH agonist treatment is Assisted Reproductive Technology. Fertil Steril 2013;99:37–43.
that less oocyte damage is seen in prepubertal girls who
Reproductive Endocrinology and Infertility 109
76
Müllerian anomaly
A 14-year-old nulligravid girl comes to your office with cyclic pelvic pain that increases with
menses. Her medical history is notable for placement of a Harrington rod for treatment of scoliosis.
You find a palpable pelvic mass on bimanual rectal examination and suspect a müllerian anomaly.
The best next step in the evaluation of this patient is
Although the true prevalence of müllerian anomalies is teristics completely, thus alleviating the need for invasive
unknown because of the high percentage of asymptom- tests. It allows for accurate depiction of the endometrium
atic patients, estimates of prevalence have ranged from and serosal superior area of the uterus in the midcoronal
7% in the general population to 8–10% in women with plane and a precise measurement of the distance between
recurrent pregnancy loss. Accurate diagnosis is essential the midfundus and a line connecting the two internal
to minimize complications, such as endometriosis and tubal ostia. With coronal reconstruction, it provides com-
infertility, as well as to prevent unnecessary surgery. plete information about the nature and extent of uterine
Precise classification of a uterine anomaly is of clinical masses and congenital anomalies. This information, in
importance because the type of intervention depends on addition to lower overall procedure-related risks and
its distinction. Furthermore, accurate classification of patient discomfort, makes it the best diagnostic modality
uterine anomalies has prognostic importance with respect for the described patient.
to obstetric and gynecologic complications. Renal anomalies are commonly seen in association
Historically, conventional two-dimensional transvagi- with müllerian anomalies, such as renal agenesis, ectopic
nal ultrasonography was considered a good screening pelvic kidneys, horseshoe kidneys, malrotated kidneys, a
tool for the detection of uterine anomalies with high duplicated renal pelvis, and unilateral medullary sponge
sensitivity (approximately 90–92%). However, the tech- kidney. The renal tract in such patients can be evaluated
nique is operator dependent and does not reconstruct the by means of ultrasonography or magnetic resonance
coronal plane. Traditionally, patients have been screened imaging. Although magnetic resonance imaging usually
with hysterosalpingography, but imaging of the uterine would be considered the best approach to evaluate uterine
cavity alone without fundal contour is inadequate in anomalies, it is contradicted in this patient because of the
many clinical scenarios and in guiding surgical interven- placement of a Harrington rod.
tion (uterine septum versus bicornuate uterus). This pre-
operative uncertainty led to laparoscopy with concurrent Acien P, Acien M, Sanchez-Ferrer M. Complex malformations of the
hysteroscopy as the reference standard for evaluation female genital tract. New types and revision of classification. Hum
Reprod 2004;19:2377–84.
of müllerian anomalies. In young patients, it is best to
Bocca SM, Abuhamad AZ. Use of 3-dimensional sonography to assess
avoid invasive procedures such as hysterosalpingography
uterine anomalies. J Ultrasound Med 2013;32:1–6.
and sonohysterography. Brief fluoroscopy is needed for
Raga F, Bauset C, Remohi J, Bonilla-Musoles F, Simon C, Pellicer A.
hysterosalpingography and is contraindicated in patients Reproductive impact of congenital Mullerian anomalies. Hum Reprod
with iodine contrast allergy. 1997;12:2277–81.
Three-dimensional transvaginal ultrasonography ena- Rock JA, Schlaff WD. The obstetric consequences of uterovaginal
bles the clinician to assess uterine morphologic charac- anomalies. Fertil Steril 1985;43:681–92.
110 PROLOG
77
Oligospermia
A 28-year-old nulligravid woman comes to your office with primary infertility. She has regular
menstrual cycles. Hysterosalpingography is normal. Her partner’s repeat semen analysis confirms
oligospermia. Physical examination demonstrates palpable varicoceles when standing and with
Valsalva maneuver. The best next step in management for the male partner is
(A) observation
(B) testosterone therapy
* (C) varicocelectomy
(D) clomiphene citrate therapy
Varicoceles, or abnormally dilated veins in the pampini- varicocele, waiting on surgical treatment may increase the
form plexus, have long been associated with male factor length of time to pregnancy. Repeat semen analysis may
infertility. The link was made because of the increased enable the patient and physician to assess if the abnormal
incidence of varicoceles observed among infertile men semen parameter is consistent. However, given that the
and associated abnormalities found in semen analyses. patient is infertile, he is a candidate for surgical repair.
Varicoceles have an incidence of approximately 4–23% Whether or not the patient is in pain, surgical repair is
in the general population, 21– 41% in men with primary indicated.
infertility, and 75 – 81% in men with secondary infertility. A recent meta-analysis done by the American Society
Untreated varicoceles may worsen over time. of Andrology and the European Academy of Andrology
Varicoceles can be diagnosed by several means; physi- demonstrated that men with idiopathic male infertility
cal examination and scrotal ultrasonography are the most may benefit from therapy with clomiphene citrate, a selec-
commonly used methods. The condition is graded at the tive estrogen receptor modulator. Clomiphene may
time of the initial physical examination from 1 to 3 by increase spontaneous pregnancy rate and improve sperm
means of the Dubin grading system: grade 3 is visible concentration and motility. If clomiphene therapy is
while the patient is standing; grade 2 is palpable without chosen, it is advisable to prescribe such therapy under
Valsalva maneuver; and grade 1 cannot be visualized and the care of a urologist. Treatment for male infertility is
is only palpable with Valsalva maneuver. Clinical varico- based on identifying reversible causes of infertility and
cele refers to those detectable by physical examination, addressing them with appropriate intervention to achieve
either by palpation or visual inspection. a pregnancy. The described male patient has a structural
Most men with varicoceles are able to father children. condition that may be significantly contributing to his
Evidence shows that varicoceles are detrimental to male abnormal semen parameters. Therefore, clomiphene is not
fertility and surgical correction offers an improvement in the best treatment option for him. Testosterone therapy
a couple’s chances of obtaining a pregnancy, either spon- does not provide efficacious treatment for varicocele.
taneously or through assisted reproductive technology.
Jarow JP, Sharlip ID, Belker AM, Lipshultz LI, Sigman M, Thomas
To date, little research has been done to assess the AJ, et al. Best practice policies for male infertility. Male Infertility Best
utility of varicocele repair in the treatment of male fac- Practice Policy Committee of the American Urological Association Inc.
tor infertility. The American Urological Association and J Urol 2002;167:2138–44.
the American Society for Reproductive Medicine have Kroese AC, de Lange NM, Collins J, Evers JLH. Surgery or emboliza-
tion for varicoceles in subfertile men. Cochrane Database of Systematic
issued practice guidelines on varicocele and infertility. Reviews 2012, Issue 10. Art. No.: CD000479. DOI: 10.1002/14651858.
These organizations recommend that varicocele repair CD000479.pub5.
should be offered to infertile men with palpable lesions Report on varicocele and infertility. Male Infertility Best Practice Policy
and one or more abnormal semen parameters. Therefore, Committee of the American Urological Association and the Practice
Committee of the American Society for Reproductive Medicine. Fertil
the described patient should be offered varicocelectomy Steril 2004;82(suppl):S142–5.
and not conservative management. Although pregnancies Report on varicocele and infertility. Practice Committee of the American
have been reported in women impregnated by men with Society for Reproductive Medicine. Fertil Steril 2006;86(suppl):S93–5.
Reproductive Endocrinology and Infertility 111
78
Obesity and insulin resistance
A 27-year-old nulliparous woman is referred to you for evaluation of polycystic ovary syndrome
(PCOS). She has oligomenorrhea and has a body mass index (BMI) of 38 (calculated as weight in
kilograms divided by height in meters squared). Her hirsutism is minimal and is managed with laser
treatment. Several of her relatives have type 2 diabetes mellitus. She wants to become pregnant in
the near future. The most appropriate next step for this patient is
Polycystic ovary syndrome is the most common endocrine should be encouraged by their obstetricians to initiate a
disorder in women, with prevalence averaging 6–15%. weight-reduction and exercise program before attempting
Clinical manifestations include polycystic ovaries, pregnancy. A consultation with a nutritionist will provide
hyperandrogenism (either clinical or biochemical), and the framework and knowledge for healthier eating habits.
oligo-ovulation or anovulation. More than 60% of PCOS Substituting healthy foods for unhealthy ones and learn-
patients are obese. Increases in BMI and abdominal ing which snacks to incorporate into a dietary plan all
(visceral) adiposity are associated with PCOS and wors- contribute to weight loss. In addition, 150 minutes a week
ening insulin resistance. Approximately 35% of women of moderate physical activity (ie, five 30-minute sessions)
with PCOS will develop impaired glucose tolerance and will produce a beneficial effect on glycemic control. Loss
7–10% will develop type 2 diabetes mellitus. Increased of 5% of body weight can lower circulating androgens and
hepatic gluconeogenesis and decreased peripheral utiliza- induce regular ovulatory cycles while allowing patients to
tion of glucose cause compensatory increases in insulin attempt pregnancy without ovulation induction agents. It
secretion. Elevated insulin levels contribute to the risk of is important as a first step to determine whether the patient
hypertension, cardiovascular disease, and type 2 diabetes. has diabetes because the presence or absence of the dis-
In addition, hyperinsulinemia is one of the main promot- order will influence nutritional counseling and treatment.
ers of hyperandrogenism seen in PCOS patients. Metformin hydrochloride is a biguanide used in the
This patient has PCOS because she is oligomenorrheic, treatment and prevention of type 2 diabetes mellitus. It is
hyperandrogenic, and obese. Sixty percent of PCOS an insulin-sensitizing agent that reduces peripheral insulin
women are hirsute. The diagnosis of PCOS necessitates levels. In a large meta-analysis of patients at risk of dia-
screening for impaired glucose tolerance and type 2 betes, metformin reduced BMI by 5.3%, increased high-
diabetes with a 2-hour glucose level after a 75-g glucose density lipoprotein, decreased low-density lipoprotein,
load (Box 78-1). This is the most appropriate next step decreased fasting insulin and glucose, and reduced new-
in this patient. A hemoglobin A1c test may be a potential onset diabetes by 40%. Metformin typically is reserved
substitute for the 2-hour glucose test; however, to date, the for patients who are diagnosed with impaired glucose
efficacy of using a hemoglobin A1c test in patients with tolerance or type 2 diabetes when lifestyle interventions
PCOS has not been completely proven. have failed.
The described patient has primary infertility but more Bariatric surgery is indicated when lifestyle modifica-
importantly is obese with PCOS. Obese pregnant women tions and pharmacotherapy are unsuccessful. Comorbidi-
are at increased risk of maternal and fetal complica- ties are reduced or eliminated with an improvement in
tions. Maternal complications include gestational diabetes, quality of life. The most common bariatric procedures are
hypertension, preeclampsia, and fetal macrosomia. Fetal Roux-en-Y (restrictive and malabsorptive) and adjustable
complications include prematurity, stillbirth, spontaneous gastric banding (restrictive). Most experts reserve surgery
abortion, congenital anomalies, macrosomia, and child- for patients with a BMI greater than 40 or patients with
hood and adolescent obesity. Based on these complications, a BMI greater than 35 with comorbidities. When a bar-
weight loss is recommended before treating her infertility. iatric patient becomes pregnant, it is prudent to consult
Because obese women are at increased risk of mater- her nutritionist and surgeon and incorporate them into the
nal and fetal complications of pregnancy, obese women obstetric team.
112 PROLOG
BOX 78-1
Physical
• Blood pressure
• BMI (weight in kg divided by height in m2)
— 25–30 = overweight, greater than 30 = obese
• Waist circumference to determine body fat distribution
— Value greater than 35 inches = abnormal
• Presence of stigmata of hyperandrogenism and insulin resistance
— Acne, hirsutism, androgenic alopecia, acanthosis nigricans
Laboratory
• Documentation of biochemical hyperandrogenemia
— Total testosterone and sex hormone-binding globulin or bioavailable and free
testosterone
• Exclusion of other causes of hyperandrogenism
— Thyroid-stimulating hormone levels (thyroid dysfunction)
— Prolactin (hyperprolactinemia)
— 17-hydroxyprogesterone (nonclassical congenital adrenal hyperplasia due
to 21-hydroxylase deficiency)
Random normal level less than 4 ng/mL or morning fasting level less than
2 ng/mL
— Consider screening for Cushing syndrome and other rare disorders such
as acromegaly
• Evaluation for metabolic abnormalities
— 2-hour oral glucose tolerance test (fasting glucose less than 110 mg/dL
= normal, 110–125 mg/dL = impaired, greater than 126 mg/dL = type 2
diabetes) followed by 75-g oral glucose ingestion and then 2-hour glucose
level (less than 140 mg/dL = normal glucose tolerance, 140–199 mg/dL =
impaired glucose tolerance, greater than 200 mg/dL = type 2 diabetes)
• Fasting lipid and lipoprotein level (total cholesterol, high-density lipoproteins
less than 50 mg/dL abnormal, triglycerides greater than 150 mg/dL abnormal
[low-density lipoproteins usually calculated by Friedewald equation])
Ultrasonographic Examination
• Determination of polycystic ovaries: in one or both ovaries, either 12 or more
follicles measuring 2–9 mm in diameter or increased ovarian volume (greater
than 10 cm3). If there is a follicle greater than 10 mm in diameter, the scan
should be repeated at a time of ovarian quiescence in order to calculate vol-
ume and area. The presence of one polycystic ovary is sufficient to provide
the diagnosis.
• Identification of endometrial abnormalities
Optional Tests to Consider
• Gonadotropin determinations to determine cause of amenorrhea
• Fasting insulin levels in younger women, those with severe stigmata of insulin
resistance and hyperandrogenism, or those undergoing ovulation induction
• 24-hour urinary free-cortisol excretion test or a low-dose dexamethasone
suppression test in women with late onset of polycystic ovary syndrome
symptoms or stigmata of Cushing syndrome
Diamanti-Kandarakis E, Dunaif A. Insulin resistance and the polycystic Obesity in pregnancy. Committee Opinion No. 549. American College
ovary syndrome revisited: an update on mechanisms and implications. of Obstetricians and Gynecologists. Obstet Gynecol 2013;121:213–7.
Endocr Rev 2012;33:981–1030. Polycystic ovary syndrome. ACOG Practice Bulletin No. 108. American
Knowler WC, Barrett-Connor E, Fowler SE, Hamman RF, Lachin College of Obstetricians and Gynecologists. Obstet Gynecol 2009;
JM, Walker EA, et al. Reduction in the incidence of type 2 diabetes 114:936–49.
with lifestyle intervention or metformin. Diabetes Prevention Program Salpeter SR, Buckley NS, Kahn JA, Salpeter EE. Meta-analysis: met-
Research Group. N Engl J Med 2002;346:393–403. formin treatment in persons at risk for diabetes mellitus. Am J Med
Lerchbaum E, Schwetz V, Giuliani A, Obermayer-Pietsch B. 2008;121:149–57.
Assessment of glucose metabolism in polycystic ovary syndrome:
HbA1c or fasting glucose compared with the oral glucose tolerance test
as a screening method. Hum Reprod 2013;28:2537–44.
Reproductive Endocrinology and Infertility 113
79
Salpingitis isthmica nodosa
A 30-year-old nulligravid woman and her husband visit your clinic for primary infertility.
Your diagnostic evaluation reveals a normal semen analysis and evidence of ovulatory cycles.
Hysterosalpingography demonstrates bilateral proximal tubal occlusion. The situation in which
microsurgical segmental tubal resection and reanastomosis is less successful is
Hysterosalpingography is considered a diagnostic test True proximal tubal occlusion is most often the result
to assess the uterine cavity and fallopian tube patency of luminal tubal fibrosis. Chronic inflammation and
in infertile patients. Proximal fallopian tube occlusion is intratubal endometriosis also can cause blockage of the
demonstrated by failure of the contrast agent to enter the proximal fallopian tube. Intraluminar fibrosis, intratubal
fallopian tubes. The failure of the dye to enter the fal- endometriosis, and chronic inflammation may be ame-
lopian tubes could be a result of a tubal spasm that has nable to microsurgical excision and repair. Salpingitis
prevented the dye from entering the tubes. Alternatively, isthmica nodosa is not amenable to surgical correction
the proximal tubal occlusion may be diagnostic of a true given that the fallopian tube has inherent abnormalities.
fallopian tube abnormality. The true cause for the diverticular appearance of the tube
In vitro fertilization (IVF) is an infertility treatment is unknown. However, repair of the tube is fruitless given
that can bypass the fallopian tubes. However, treatment that tubal damage is multifocal and that women with
with IVF may not always be necessary or accessible to salpingitis isthmica nodosa are at high risk for ectopic
the patient. Therefore, surgical treatment of proximal pregnancy. Such patients are good candidates for IVF and
tubal occlusion is a reasonable alternative. There are two bypassing of the abnormal fallopian tubes. Neither modi-
methodologies associated with proximal tubal repair: fied Pomeroy tubal ligation nor tubal ligation by means
1) hysteroscopic proximal tubal catheterization and of the clip have significant bearing on microsurgical seg-
2) microsurgical tubal resection. Both techniques require mental tubal resection and reanastomosis.
specialized skill for microsurgery and advanced hystero-
Das S, Nardo LG, Seif MW. Proximal tubal disease: the place for tubal
scopic techniques. cannulation. Reprod Biomed Online 2007;15:383–8.
Salpingitis isthmica nodosa is a fallopian tube abnor- Renbaum L, Ufberg D, Sammel M, Zhou L, Jabara S, Barnhart K.
mality typified by diverticula (outpouchings) of the Reliability of clinicians versus radiologists for detecting abnormalities
fallopian tube. The tubal mucosa penetrates in the myosal- on hysterosalpingogram films. Fertil Steril 2002;78:614–8.
pinx and leads to a diverticular appearance of the fallopian Schippert C, Soergel P, Staboulidou I, Bassler C, Gagalick S, Hillemanns
P, et al. The risk of ectopic pregnancy following tubal reconstructive
tubes on hysterosalpingography. This condition is largely microsurgery and assisted reproductive technology procedures. Arch
associated with infertility. Gynecol Obstet 2012;285:863–71.
114 PROLOG
80
Heavy menstrual bleeding
After she has undergone tubal ligation, a 29-year-old woman comes to your office with a history
of monthly heavy menstrual bleeding. Ultrasonographic evaluation of her uterus and thyroid-
stimulating hormone level are normal. The best nonsurgical therapy for her is
Heavy menstrual bleeding, defined as menstrual blood provides treatment for at least 5 years, is as effective as
loss greater than 80 mL per cycle, affects up to 30% of endometrial ablation, and is reversible.
reproductive-aged women and negatively affects their A recent study was performed among 571 women with
physical activities and quality of life. Idiopathic heavy AUB who were randomly assigned to the levonorgestrel
menstrual bleeding is regular bleeding in the absence IUD or usual medical treatment (NSAIDs, tranexamic
of recognizable pelvic pathology or a bleeding disorder. acid, continuous combination oral contraceptives, and a
Leiomyomas have been found in up to 40% of women progestin-only oral contraceptive). Over a 2-year period,
with heavy menstrual bleeding; however, approximately the levonorgestrel IUD was more effective than usual
50% of women who undergo hysterectomy for this con- medical treatment in reducing the effect of AUB on
dition are found to have a normal uterus. Appendix C quality of life. Therefore, the levonorgestrel IUD would
shows the International Federation of Gynecology and provide the best therapy for the described patient.
Obstetrics classification system for causes of abnormal
uterine bleeding (AUB). Abu Hashim H. Medical treatment of idiopathic heavy menstrual bleed-
ing. What is new? An evidence based approach. Arch Gynecol Obstet
Data from some studies suggest that unrestrained local 2013;287:251–60.
inflammatory events with or without deficient repair pro- Endrikat J, Vilos G, Muysers C, Fortier M, Solomayer E, Lukkari-Lax
cesses in the endometrium may contribute to the onset of E. The levonorgestrel-releasing intrauterine system provides a reliable,
AUB. Medical therapy such as NSAIDs, antifibrinolytics, long-term treatment option for women with idiopathic menorrhagia.
Arch Gynecol Obstet 2012;285:117–21.
continuous combination hormonal contraceptives, pro-
Gupta J, Kai J, Middleton L, Pattison H, Gray R, Daniels J. Levonorges-
gestins, and the levonorgestrel IUD represent attractive trel intrauterine system versus medical therapy for menorrhagia.
options to avoid unnecessary surgery, such as endome- ECLIPSE Trial Collaborative Group. N Engl J Med 2013;368:
trial ablation and hysterectomy. The levonorgestrel IUD 128–37.
Reproductive Endocrinology and Infertility 115
81
Late-onset congenital adrenal hyperplasia
An Ashkenazi Jewish 23-year-old nulligravid woman visits your office with irregular menstrual
cycles. She is interested in becoming pregnant. She has seen her dermatologist for acne and hirsut-
ism and has been shaving her chin weekly for the past 10 years. On physical examination, she has
acne on her face and mild hair growth on her upper lip and chin. She shows no signs or symptoms
of virilization. Her thyroid-stimulating hormone level is normal. The best next diagnostic step is
to test for the level of
Late-onset congenital adrenal hyperplasia or nonclas- androgen-secreting tumors and nonclassic CAH. Labor-
sic adrenal hyperplasia (CAH) affects approximately atory evaluation with basal 17α-hydroxyprogesterone
1 in 1,000 non-Jewish whites and 1 in 30 Ashkenazi level helps to differentiate between nonclassic CAH
Jews. The disease is also prevalent in Alaska Native and polycystic ovary syndrome. Most patients with non-
American Inuits, Hispanics, Italians, and Yugoslavs. classic CAH will have a basal 17α-hydroxyprogesterone
The condition predominates in women who have a fam- level greater than 200 ng/dL.
ily history of nonclassic CAH or who have onset of The best diagnostic test to assess for nonclassic CAH
hirsutism around menarche. It is an autosomal recessive is a morning follicular serum 17α-hydroxyprogesterone
condition that results in 50–80% loss of 21-hydroxy- level. Levels peak in the morning and reach a low later
lase activity (P450c21 enzyme) due to mutations in the in the day. Morning follicular 17α-hydroxyprogesterone
CYP21A2 gene. This results in excessive adrenal andro- levels are higher in women with nonclassic CAH than
gen production. Enzyme defects in 11β-hydroxylase in controls. By obtaining levels in the follicular phase
and 3β-hydroxysteroid dehydrogenase are rare causes and not in the luteal phase, false-positive levels are
of adrenal androgen overproduction. In patients with greatly reduced. Levels less than 200 ng/dL exclude
nonclassic CAH, the pathophysiology is characterized nonclassic CAH, whereas levels greater than 800 ng/dL
by inadequate cortisol synthesis, which stimulates pitu- are virtually diagnostic for nonclassic CAH. The criter-
itary adrenocorticotropic hormone (ACTH) secretion. ion standard for confirming the diagnosis of nonclas-
Androgens proximal to the enzymatic blockade increase, sic CAH is the ACTH (cosyntropin) stimulation test.
causing hyperandrogenism. Baseline 17α-hydroxyprogesterone is drawn in the morn-
The described patient has hirsutism, acne, and oligo- ing at any time during the menstrual cycle; ACTH
menorrhea. These are the three most common symptoms (cosyntropin) 250 micrograms is administered intrave-
in women with nonclassic CAH. Patients with nonclassic nously as a bolus over a minute. A blood sample for
CAH have normal external genitalia, experience preco- 17α-hydroxyprogesterone is drawn 60 minutes after
cious pubarche, or may present as adolescents or adults ACTH is given. In most patients with nonclassic CAH,
with hyperandrogenism. This is in contrast to classic stimulated 17α-hydroxyprogesterone levels will be in
CAH (salt-wasting and virilizing forms), which manifests excess of 1,500 ng/dL at 1 hour. In clinical practice, it is
at birth with ambiguous genitalia. not realistic to perform an ACTH stimulation test on all
Approximately 10% of all women are affected by hyperandrogenic (hirsute) women who may have nonclas-
androgen excess. The differential diagnosis includes poly- sic CAH, given that it is a rare condition.
cystic ovary syndrome, nonclassic CAH, and androgen- Total testosterone level would not be the best next
secreting tumors. Differentiating between these disorders diagnostic test. The described patient has risk factors for
can present a diagnostic challenge. Polycystic ovary nonclassic CAH that include her ethnicity, perimenarchal
syndrome patients have a higher frequency of oligomen- onset of hirsutism, irregular cycles, and no virilization.
orrhea and amenorrhea with ovaries that are polycystic in If she was virilized, had rapid onset, or exhibited severe
appearance compared with the ovaries of patients with hirsutism, a total testosterone level would be indicated.
116 PROLOG
A total testosterone level greater than 200 ng/dL will addition, hyperprolactinemia can cause menstrual irregu-
identify most androgen-producing tumors. If the level is larity and galactorrhea in some women. For the described
less than 200 ng/dL and the patient has rapid virilization patient, testing for the prolactin level would not be the
or progressive hirsutism, an androgen-secreting tumor next best diagnostic test.
still should be assumed and further evaluation would be
Auchus RJ. Congenital adrenal hyperplasia in adults. Curr Opin
warranted. Endocrinol Diabetes Obes 2010;17:210–6.
Dehydroepiandrosterone sulfate is secreted primarily Fritz MA, Speroff L. Hirsutism. In: Fritz MA, Speroff L, edi-
from the adrenal gland. It provides substrate for testoster- tors. Clinical gynecologic and infertility. 8th ed. Philadelphia (PA):
Lippincott Williams & Wilkins; 2011. p. 533–65.
one and dihydrotestosterone conversion; DHEAS levels
Pall M, Azziz R, Beires J, Pignatelli D. The phenotype of hirsute
are not elevated in nonclassic CAH. Women with andro- women: a comparison of polycystic ovary syndrome and 21-hydrox-
gen-secreting adrenal tumors can have elevated DHEAS ylase-deficient nonclassic adrenal hyperplasia. Fertil Steril 2010;94:
levels in addition to elevated testosterone. However, a 684–9.
DHEAS level has little clinical value in a hirsute patient Trapp CM, Oberfield SE. Recommendations for treatment of nonclas-
sic congenital adrenal hyperplasia (NCCAH): an update. Steroids
who is not virilized. 2012;77:34–6.
Hyperprolactinemia does not cause mild hirsutism by Witchel SF. Nonclassic congenital adrenal hyperplasia. Curr Opin
stimulating adrenal or ovarian androgen production. In Endocrinol Diabetes Obes 2012;19:151–8.
82
Precocious puberty
You examine a 5-year-old Asian girl with a 9-month history of breast development and two epi-
sodes of vaginal bleeding. Physical examination demonstrates Tanner stage 3 breast development
and Tanner stage 2 pubic hair development. Laboratory studies show a follicle-stimulating hor-
mone (FSH) level of 4.3 mU/L, luteinizing hormone (LH) level of 3.6 mU/L, thyroid-stimulating
hormone (TSH) level of 0.98 mU/L, prolactin level of 7.0 ng/mL, and estradiol level of 75 pg/
mL. Bone age is advanced by 3 years with a predicted height of less than 1.52 m (60 in.). Cranial
magnetic resonance imaging is normal. The most appropriate medical therapy is
Precocious puberty is defined as early breast develop- child, the more likely she will have a pathologic cause for
ment, early pubic hair development, or both with menses premature sexual development. The etiologies of preco-
before age 8 years. Because of earlier pubertal milestones cious puberty are commonly defined as central (GnRH-
among different ethnicities, some experts have recom- dependent) causes and peripheral (GnRH-independent)
mended that these guidelines be changed to thelarche causes (Table 82-1).
before age 7 years in white girls and before age 6 years in Central precocious puberty consists of early activa-
African American girls. Observations that suggest pathol- tion of the hypothalamic–pituitary–ovarian axis and is
ogy and that indicate the need for evaluation at a border- idiopathic in greater than 50% of girls with central preco-
line age are rapid development with bone age greater than cious puberty. Other cases involve intracranial processes
2 years and a predicted height of less than 1.5 m (59 in.) that activate GnRH secretion through mostly unknown
or 2 standard deviations below the target height, central mechanisms. The most common tumor that causes cen-
nervous system symptoms, and symptoms that affect the tral precocious puberty is a hypothalamic hamartoma, in
emotional health of the girl. which astroglial cells within the tumor release transform-
Precocious puberty is early sexual development in ing growth factor-beta protein to induce GnRH release
which the normal sequence of events usually is preserved from the hypothalamus.
with shortened time intervals. Estrogen-induced acceler- Adoption of children from less developed countries
ated growth, advanced bone age, and early closure of bone may be associated with central precocious puberty, most
epiphyseal plates result in short stature. The younger the likely due to weight gain from rapid improvement in
Reproductive Endocrinology and Infertility 117
TABLE 82-1. Causes of Isosexual Female Precocious McCune–Albright syndrome typically has a very early
Puberty onset in infancy.
The evaluation of isosexual precocious puberty includes
Central (GnRH- Peripheral (GnRH-
a history with close attention to possible estrogen expo-
Dependent) Causes Independent) Causes
sure, review of growth records looking for an increase in
Idiopathic Estrogen-producing height percentile, and a physical examination to determine
estrogen tumor Tanner stages and presence of skin lesions. Initial testing
International adoption Simple ovarian cyst includes a bone age test and blood analysis for estradiol,
Hypothalamic hamartoma Hypothyroidism FSH, LH, and TSH. Bone age is increased in all cases
Brain tumors Estrogen-producing of precocious puberty except when caused by primary
adrenal tumor hypothyroidism. A high estradiol level with a low gonad-
Hydrocephalus McCune–Albright otropin level suggests an ovarian tumor with the next
syndrome test being pelvic ultrasonography. A high gonadotropin
Previous encephalitis, Exogenous estrogen level for age is consistent with central precocious puberty
meningitis, head trauma, and the next step is magnetic resonance imaging. Cases
granulomatous disease without obvious diagnosis after initial testing may require
Subarachnoid cyst Peutz–Jeghers syndrome GnRH stimulation testing. Administration of GnRH (or a
Neurofibromatosis 1 Silver–Russell syndrome GnRH agonist) results in a specific pattern of FSH and LH
response depending on the diagnosis. In normal children,
Sturge–Weber syndrome
no significant increase in FSH or LH is seen. Children
Abbreviation: GnRH, gonadotropin-releasing hormone. with premature thelarche have an FSH-dominant response
whereas an LH-dominant response is diagnostic of central
precocious puberty.
nutrition. Primary hypothyroidism sometimes is consid- Treatment of precocious puberty depends on the etiol-
ered a central cause but does not initially involve GnRH ogy and the goals of therapy. Obviously, sources such as
activation. Most likely, high TSH levels cross-react with hypothyroidism and ovarian tumors require correction.
FSH receptors on the pituitary. Hypothyroidism is the Simple ovarian cysts often will resolve with time and
only cause of precocious puberty that does not result in observation. The decision to treat idiopathic or noncor-
an advanced bone age. rectable causes of central precocious puberty is dependent
Peripheral precocious puberty is due to independent on the age of the child, the predicted final height, and the
secretion of estrogen from the ovary or to an iatrogenic severity of symptoms. The treatment of choice is a GnRH
etiology from exposure to an estrogen or estrogen-like agonist, with the most convenient being a 1-year implant
compound. Ovarian tumors, such as granulosa–theca of histrelin acetate. Before the discovery of GnRH ago-
cell, teratomas, or dysgerminoma, may directly secrete nist, precocious puberty was commonly treated with
estrogen. The mechanism with other tumors appears to depot medroxyprogesterone acetate. This drug would
be induction of surrounding theca cell proliferation lead- decrease breast development and cease menses but would
ing to increased hormone production. Cases of unilateral have little effect on bone maturation. Treatment for
ovarian cysts causing peripheral precocious puberty have McCune–Albright syndrome requires the use of enzyme
been reported, most likely with a similar mechanism. This inhibitors given the autonomous action of the gonado-
is not to be confused with multiple ovarian cysts, which tropin receptors independent of ligand. Options include
are the consequence of gonadotropin stimulation of the the use of ketoconazole, which blocks production of sex
ovaries. steroids primarily through inhibition of 17α-hydroxylase
McCune–Albright syndrome is a unique cause of activity; letrozole through inhibition of aromatase; and
peripheral precocious puberty involving autonomous fulvestrant through antagonism of the estrogen receptor.
activation of gonadotropin receptors. Two G protein com- The most appropriate medical therapy for the described
plexes, one stimulatory (Gsα) and one inhibitory (Giα), patient is injectable GnRH agonist. The long-term prog-
are responsible for cyclic-adenosine monophosphate acti- nosis is positive for girls treated with a GnRH agonist for
vation after binding of a ligand (such as FSH and LH) idiopathic central precocious puberty. Compared with
to a G protein-coupled receptor. McCune–Albright syn- controls, girls treated with GnRH agonist will have no
drome is caused by a postfertilization mutation of the difference in menstrual cycles or fertility.
Gsα complex resulting in ovarian production of estrogen
in the absence of gonadotropins. Although McCune– Carel JC, Leger J. Clinical practice. Precocious puberty. N Engl J Med
2008;358:2366–77.
Albright syndrome can result in numerous abnormalities,
Fuqua JS. Treatment and outcomes of precocious puberty: an update.
the hallmarks include the triad of a unique bone lesion J Clin Endocrinol Metab 2013;98:2198–207.
called polyostotic fibrous dysplasia, large pigmented skin Mason P, Narad C. Long-term growth and puberty concerns in interna-
lesions (café au lait patches), and peripheral precocious tional adoptees. Pediatr Clin North Am 2005;52:1351–8, vii.
puberty. The peripheral precocious puberty seen with
118 PROLOG
83
Premature ovarian failure
A 28-year-old healthy nulligravid woman stopped taking oral contraceptives 1 year ago with the
goal of pregnancy. However, she did not resume spontaneous menstrual cycles. Her primary care
provider ordered tests that showed a follicle-stimulating hormone (FSH) level of 45 mIU/mL, estra-
diol level of 12 pg/mL, thyroid-stimulating hormone level of 1.37 mIU/mL, and a prolactin level of
9 ng/mL. The most appropriate next test or screen in her evaluation is
Primary ovarian insufficiency, also known as prema- regions of the X chromosome, trisomy X, and mutations
ture ovarian failure or hypergonadotropic hypogonad- that have a 46,XY karyotype (Swyer syndrome).
ism, is defined as ovarian failure before age 40 years. As a first step in the evaluation of women with primary
Approximately 1% of women will experience primary ovarian insufficiency, such as the described patient, it is
ovarian insufficiency. The diagnosis generally includes important to obtain a karyotype to evaluate for a missing
amenorrhea, although some women with primary ovarian or abnormal X chromosome and for the presence of any
insufficiency may experience an occasional ovulatory portion of a Y chromosome. If there is evidence of Turner
cycle. Approximately 5–10% of women with primary syndrome, even in the mosaic form, the patient may need
ovarian insufficiency may conceive without assistance evaluation for serious sequelae of this syndrome, such
and have an uncomplicated pregnancy. Generally, patients as aortic rupture. If any portion of a Y chromosome is
have FSH levels above 30 mIU/mL. Some women with found, the patient is at risk of malignancy and should
primary ovarian insufficiency will experience bleeding consider oophorectomy.
after a progestin withdrawal test. As a result, caution Evidence exists of an association between carriers
should be employed when using this test as part of the of fragile X mutation and women with primary ovarian
diagnostic evaluation of amenorrhea. insufficiency. Fragile X mutations involve a dynamic
Premature ovarian failure is caused by a heterogenous trinucleotide repeat (CGG) sequence mutation in the
group of disorders, although for most women no cause X-linked FMR1 gene at the terminal end of the long
is identified. Women who were previously treated with arm of the X chromosome. The normal FMR1 gene has
gonadotoxic chemotherapy for a malignancy or autoim- approximately 30 CGG repeats. The fully expanded form
mune disease, such as lupus nephritis, may have transient of the condition is called fragile X syndrome and has
or permanent damage to the ovarian function. Some of more than 200 repeats. Fragile X syndrome is the most
these women have immediate ovarian failure, whereas common known genetic cause of mental retardation and
others will continue to menstruate but undergo early autism in males. Fragile X premutation carriers have
menopause. An autoimmune cause for primary ovarian between 55 repeats and 200 repeats and may present
insufficiency may be found in some women, and such with no symptoms, premature ovarian failure (approxi-
individuals often have antiadrenal antibodies. mately 15% of women with the premutation), or fragile
A number of genetic causes for primary ovarian insuf- X-associated tremor–ataxia syndrome.
ficiency have been identified. Karyotype abnormalities, In women with primary ovarian insufficiency, testing
single-gene defects, and multifactorial polygenic condi- will reveal a fragile X premutation in 12% of women with
tions all can result in primary ovarian insufficiency. a family history of primary ovarian insufficiency and 3%
Turner syndrome (45,X) usually presents as primary of women without such a family history. Male offspring
amenorrhea and has a classic phenotype that includes born to women who are carriers of fragile X mutations
streak ovaries. Other structural abnormalities of the are at significant risk of having mental retardation. It is
X chromosome can result in primary ovarian insuffi- important to offer screening for fragile X mutations in all
ciency, such as Turner syndrome mosaic, mutations in women with unexplained primary ovarian insufficiency
Reproductive Endocrinology and Infertility 119
after testing for karyotype abnormalities has been per- ian insufficiency, and an elevated FSH level and amenor-
formed. All carriers should be offered formal genetic rhea are adequate to make the diagnosis. Similarly, serum
counseling. antiovarian antibody levels have not been shown to be
Women with primary ovarian insufficiency are at useful in the diagnosis of primary ovarian insufficiency.
increased risk of osteopenia and osteoporosis. Once the Moreover, serum antiovarian antibody level testing is not
diagnosis of primary ovarian insufficiency is made, the currently available in many clinical settings.
clinician should discuss strategies for bone protection,
Carrier screening for fragile X syndrome. Committee Opinion No. 469.
such as hormone therapy and dietary supplementation with American College of Obstetricians and Gynecologists. Obstet Gynecol
calcium and vitamin D. Bone density evaluation, with a 2010;116:1008–10.
technique such as a dual energy X-ray absorptiometry Rebar RW. Premature ovarian failure. Obstet Gynecol 2009;113:
scan, is important in women with primary ovarian insuffi- 1355–63.
ciency. However, bone density testing is not an urgent first Rebar RW, Connolly HV. Clinical features of young women with
hypergonadotropic amenorrhea. Fertil Steril 1990;53:804–10.
step in the evaluation of primary ovarian insufficiency.
Speroff L, Fritz MA. Amenorrhea. In: Speroff L, Fritz MA, editors.
No evidence exists that testing for the antimüllerian Clinical gynecologic endocrinology and infertility. 8th ed. Philadelphia
hormone level will help in the diagnosis of primary ovar- (PA): Lippincott Williams and Wilkins; 2011. p. 464–73.
84
Acne in an adolescent patient
A 17-year-old adolescent woman with irregular menses and acne comes to your office. She is
being treated by her dermatologist with topical antibiotics and topical tretinoin. Her evaluation is
consistent with polycystic ovary syndrome (PCOS). Her best treatment option is
Acne is a multifactorial disease consisting of skin inflam- will meet the clinical diagnosis of PCOS. The severity of
mation with exacerbation by systemic androgens. The acne does not correlate with androgen levels. The pubertal
major factors in acne include increase in insulin-like growth factor 1 (IGF-1) also plays a
• inflammation role in acne. The action of IGF-1 appears to increase active
androgen receptors. The roles of the androgen receptor and
• colonization of the pilosebaceous unit with the bacte-
IGF-1 in acne are made evident by clinical syndromes.
rium Propionibacterium acnes
Patients with complete androgen insensitivity syndrome or
• altered follicular growth and differentiation who lack IGF-1 (Laron dwarfism) do not have acne.
• sebaceous gland hyperplasia with seborrhea Females with obesity or a finding of acanthosis nigri-
The main action of androgens is the increased production cans should have testing for glucose tolerance to evaluate
of sebum by the sebaceous gland. Sebum acts as a nutrient for impaired glucose tolerance or type 2 diabetes mellitus.
source for P acnes and enhances growth. Ideally, this is done with a 2-hour oral glucose tolerance
Acne is not normal in females and may require evalu- test. All women with acne need a referral to a dermatolo-
ation for syndromes of androgen excess such as PCOS, gist to help them manage the acne.
congenital adrenal hyperplasia, and very rarely Cushing Treatment of acne is directed at decreasing inflam-
syndrome. Most females with acne will have biochemical mation and sebum production with topical and systemic
evidence of elevated dehydroepiandrosterone sulfate with therapy. Combination topical therapy is superior to mono-
or without elevated free testosterone and decreased levels therapy. Benzoyl peroxide is a mainstay of acne therapy
of sex hormone-binding globulin (SHBG). Although num- because it has antiinflammatory, bactericidal, and anti-
bers vary, approximately one in three women with acne keratinization properties. Topical antibiotics may offer
120 PROLOG
benefits, but overuse has led to increased bacterial resis- Type 2 5α-reductase inhibitor finasteride has been
tance. Retinoids are vitamin A derivatives that normalize studied for treatment of acne and yields less response
abnormal growth of keratinocytes and decrease inflam- compared with combination OCs, spironolactone, and
mation. Topical azelaic acid is a natural substance pro- flutamide. This is most likely because of the lack of
duced by a ubiquitous yeast and is bactericidal. Multiple inhibition of type 1 5α-reductase that is found in the
combinations of these products are available. pilosebaceous unit. Dutasteride, an inhibitor of type 1 and
Systemic treatment of acne incudes antibiotics, retinoic type 2 5α-reductase, has yet to be studied.
acid, and drugs to decrease androgens or block androgen Spironolactone has dual actions: blocking the andro-
action. The most commonly used antibiotic is tetracycline gen receptor and inhibiting 5α-reductase. The drug is
or minocycline. Oral isotretinoin is related to vitamin A used most commonly with a combination OC for several
and is highly effective for severe acne or cases refrac- reasons. Spironolactone is potentially teratogenic and
tory to standard therapy. However, it has the potential requires effective birth control. Spironolactone also may
for severe adverse effects, including onset of psychiatric cause increased menstrual bleeding, which is prevented
disorders, pseudotumor cerebri, severe hypertriglyceride- by a combination OC. The joint use of a combination OC
mia with associated pancreatitis, hepatotoxicity, inflam- and spironolactone is more effective than single therapy.
matory bowel disease, vision changes, hearing loss, and Nonsteroidal pure antiandrogens, such as bicalutamide,
teratogenicity. Use of the drug requires use of efficient flutamide, and hydroxyflutamide, also are effective for
birth control and monitoring of lipid levels and liver the treatment of acne. These drugs are not used frequently
function tests. because of possible hepatotoxicity and the need for ini-
Hormonal therapy for acne seeks to decrease free tial routine monitoring of liver function. They are also
androgen levels, decrease conversion of testosterone to teratogenic.
5α-dihydrotestosterone, and block the androgen recep- Progestin-only contraceptives, such as oral noreth-
tor. The most common therapy is a combination OC. indrone, implantable etonogestrel, injectable depot
Combination OCs decrease free androgens by increasing medroxyprogesterone acetate, and the levonorgestrel
levels of SHBG and decrease ovarian androgen produc- intrauterine device, do not increase SHBG and are not
tion. The magnitude of SHBG increase is dependent on effective for the treatment of acne.
the dose of ethinyl estradiol and the type of progestin in Metformin hydrochloride has a beneficial effect on
the pill. Third-generation progestins, including desoges- acne in women diagnosed with PCOS. Because glucose
trel and norgestimate, result in much higher levels of utilization increases, insulin levels and circulating andro-
SHBG compared with first-generation norethindrone and gen levels decrease and SHBG levels increase. To date, no
second-generation levonorgestrel. Drospirenone, which studies have been done to compare the efficacy of metfor-
is chemically related to spironolactone, yields an increase min with other treatment options for acne, and metformin
in SHBG similar to the third-generation progestins. The is not approved by the FDA for the treatment of acne.
U.S. Food and Drug Administration (FDA) has approved Oral corticosteroids are used only rarely for long-term
three combination OCs for the treatment of acne: a treatment of acne because of the drugs’ adverse effects.
20-microgram ethinyl estradiol–drospirenone prepara- Oral corticosteroids work by decreasing inflammation
tion, a 35-microgram ethinyl estradiol–norgestimate and adrenal androgen production. Chronic therapy may
preparation in which the norgestimate dose increases be needed in women with adult-onset congenital adrenal
from 0.18 mg to 0.215 mg to 0.25 mg, and an ethinyl hyperplasia who fail to respond to nonsteroidal therapies.
estradiol–norethindrone combination in which the ethinyl Steroid adverse effects are dose dependent and numerous,
estradiol dose increases from 20 micrograms to 30 micro- the most significant adverse effects being diabetes mel-
grams to 35 micrograms. Note that FDA approval should litus, osteoporosis, and aseptic necrosis of the hip.
not suggest that other combination OCs are not effective
for acne treatment but more likely that the pharmaceutical Dreno B, Layton A, Zouboulis CC, Lopez-Estebaranz JL, Zalewska-
company has chosen not to perform the necessary trials Janowska A, Bagatin E, et al. Adult female acne: a new paradigm. J Eur
Acad Dermatol Venereol 2013;27:1063–70.
for this indication. Oral finasteride, oral spironolactone,
Katsambas AD, Dessinioti C. Hormonal therapy for acne: why not
and the progestin-only OC have not been approved by the as first line therapy? facts and controversies. Clin Dermatol 2010;28:
FDA for the treatment of acne. 17–23.
Reproductive Endocrinology and Infertility 121
85
Psychologic effect of infertility
A 33-year-old nulligravid patient with a 2-year history of infertility has just completed her second
cycle of in vitro fertilization (IVF) for severe male factor infertility. She had a negative serum
β-hCG test result. After several weeks, her concerned husband calls your office and tells you that
she has decreased energy, is tearful and anxious, and spends nearly all day in bed. The best next
treatment option for the patient is
In 2005, the number of infertile individuals worldwide vide the patient or the couple with the support to endure
was estimated to be 60–80 million with a growth of additional treatment that is likely to result in pregnancy.
approximately 2 million new cases per year. In devel- Mental health practitioners experienced with this patient
oped countries, infertility is diagnosed in approximately population would be preferable because they are sensi-
17–26% of reproductive-aged couples. The prevalence of tive to the nuances and complexities of infertile couples.
infertility increases with age from 20% among women Acupuncture is considered an alternative treatment to
aged 35–39 years to 25–30% among women 40 years psychologic counseling, but its utility in this patient
and older. Infertility treatment often is accompanied by population remains controversial. There is no evidence
psychologic disturbances that may significantly affect the that another course of infertility therapy would be benefi-
relationship between the patient and her partner. cial for this couple. Many infertile couples who reinitiate
Numerous studies have shown that more than 50% treatment before their psychologic issues are addressed
of infertile couples consider infertility to be the most are likely to experience worsening of their effects and
disappointing experience of their lives. Approximately subsequent decline in their quality-of-life measures.
80% of infertile couples report infertility to be a stressful Alternative treatments, such as antidepressants, may
experience. Among infertile couples screened with the be warranted. However, the patient must be evalu-
self-reported Beck Depression Inventory, symptoms of ated thoroughly to assess the best treatment modality.
depression were observed in 37% of infertile women, Antidepressant medication may not provide the couple
and among those women, 19.4% had moderate or severe with the support required to proceed with additional treat-
symptoms of depression. Depression among infertile ments. Moreover, a number of anxiolytic medications are
women is the result of prolonged and severe normal of the benzodiazepine class, which is contraindicated in
psychologic response to the diagnosis, which comprises pregnancy. Most patients would benefit from psycho-
grief and mourning. Depression is highest in women who logic therapy to address the underlying problem and also
have been undergoing infertility treatment for the longest cognitive–behavioral strategies to help address their
period. Among couples who are undergoing IVF, studies stress.
have demonstrated depression prevalence before IVF
Malcolm CE, Cumming DC. Follow-up of infertile couples who
in 33% of women and 3.5% of men. After IVF failure, dropped out of a specialist fertility clinic. Fertil Steril 2004;81:269–70.
depression prevalence increased to 43% and 8% among Pinborg A, Hougaard CO, Nyboe Andersen A, Molbo D, Schmidt L.
women and men, respectively. It is believed that the Prospective longitudinal cohort study on cumulative 5-year delivery
incidence of depression in women is higher than in men and adoption rates among 1338 couples initiating infertility treatment.
Hum Reprod 2009;24:991–9.
because of a potentially higher propensity for women to
Schmidt L. Psychosocial burden of infertility and assisted reproduction.
express their feelings compared with men. Lancet 2006;367:379–80.
The best next treatment in the management of this Verberg MF, Eijkemans MJ, Heijnen EM, Broekmans FJ, de Klerk C,
patient is psychologic counseling. It may help to address Fauser BC, et al. Why do couples drop-out from IVF treatment? A
the severity of her depression. Additionally, it may pro- prospective cohort study. Hum Reprod 2008;23:2050–5.
122 PROLOG
86
Primary amenorrhea due to gonadal dysgenesis
A 19-year-old nulligravid woman comes to your office with primary amenorrhea. She is 1.75 m
(69 in.) tall with Tanner stage 1 breast development and Tanner stage 3 pubic hair. Her follicle-
stimulating hormone level is 52 mIU/mL. Her endocrine screening is otherwise normal. Her
karyotype is 46,XY. In addition to psychologic counseling, the next step in management should be
* (A) gonadectomy
(B) Y-sequence analysis identification
(C) combination oral contraceptives
(D) growth hormone therapy
(E) observation
The described patient has 46,XY gonadal dysgenesis, After gonadectomy, exogenous hormone replacement
also known as Swyer syndrome. The classic presentation is recommended to promote breast and uterine develop-
is a prepubertal phenotypic female with primary amenor- ment. Estrogen alone should be given initially in a gradu-
rhea. Typically, there are no somatic abnormalities and ated fashion and cyclic progesterone or progestin should
patients will exhibit infantile uterus, cervix, and fallopian be added after the first episode of menstrual bleeding
tubes as well as streak gonads. At the time of diagnosis, or typically after 2–3 years if no bleeding has occurred.
gonadectomy is recommended because of the significant Successful pregnancies have been achieved with oocyte
risk (approximately 30%) of malignant transformation donation in patients with 46,XY gonadal dysgenesis. The
in the streak gonads. However, gonadectomy should vast majority of reported pregnancies in patients with
be delayed until after puberty in women with androgen Swyer syndrome have required cesarean delivery for a
insensitivity syndrome. Gonadoblastoma is the most variety of indications. The etiology of the high rate of
common tumor, followed by malignant transforma- cesarean delivery remains unclear but is probably multi-
tion to a dysgerminoma. Choriocarcinoma may be seen factorial. The android shape of the pelvis in patients with
rarely. Approximately 10–15% of cases of XY gonadal a 46,XY karyotype may predispose these patients to labor
dysgenesis result from SRY mutations, another 10–15% abnormalities.
arise from SRY deletions, and approximately 70% are of In the described patient, gonadectomy should be
unknown genetic origin. the next step in management. Because the karyotype
In diagnosing Swyer syndrome, the possibility of clearly identifies a Y chromosome, Y-sequence analysis
45,X/46,XY mosaicism must be excluded. Patients with a identification is not necessary. Testing for Y chromo-
45,X cell line are at increased risk of the spectrum of car- some material would be necessary in a patient with
diovascular issues associated with classic 45,X gonadal gonadal dysgenesis if the karyotype did not identify
dysgenesis (Turner syndrome). Patients with a 45,X cell Y chromosome material but perhaps showed a chromo-
line may exhibit aortic dilation, coarctation, or a bicuspid somal fragment of unknown origin. Patients with cryptic
aortic valve and are at increased risk of aortic dissection Y chromosome sequences still should undergo gonad-
and rupture and possible death during pregnancy. Patients ectomy, so the identification of even small amounts of
with nonmosaic pure 46,XY gonadal dysgenesis do not Y chromosome material is important. Combination oral
appear to be at increased risk of cardiovascular abnor- contraceptives are not appropriate for this patient at this
malities or adverse obstetric outcomes. time. Low-dose estrogen alone should be given initially
Height may serve as one clue in distinguishing patients for an extended period of months in order to try to mimic
with pure 46,XY gonadal dysgenesis from patients with natural puberty and to facilitate normal breast and uterine
45,X/46,XY mosaicism if classic features of Turner syn- development. Growth hormone therapy is not neces-
drome are not present. Patients with Swyer syndrome are sary in this patient because she is above-average height,
typically of average or above-average height, whereas although such therapy is prescribed frequently for girls
many patients with a 45,X cell line will exhibit short with Turner syndrome. Observation is not appropriate
stature. A karyotype is essential in establishing the cor- given the need for gonadectomy and hormone therapy.
rect diagnosis.
Reproductive Endocrinology and Infertility 123
Plante BJ, Fritz MA. A case report of successful pregnancy in a patient Stachowicz-Stencel T, Synakiewicz A, Izycka-Swieszewska E,
with pure 46,XY gonadal dysgenesis. Fertil Steril 2008;90:2015.e1–2. Kobierska-Gulida G, Balcerska A. Malignant germ cell tumors associ-
Scherer G, Held M, Erdel M, Meschede D, Horst J, Lesniewicz R, et al. ated with Swyer syndrome. Pediatr Blood Cancer 2011;56:482–3.
Three novel SRY mutations in XY gonadal dysgenesis and the enigma
of XY gonadal dysgenesis cases without SRY mutations. Cytogenet
Cell Genet 1998;80:188–92.
87
Intrauterine microinsert follow-up
A 35-year-old woman undergoes a hysteroscopic microinsert placement in her fallopian tubes with-
out complication. After 3 months, follow-up hysterosalpingography (HSG) demonstrates unilateral
tubal patency. The frequency with which patients will have at least one patent fallopian tube at
3 months after placement is
The hysteroscopically placed permanent contraceptive In a retrospective report covering 7 years of experi-
microinsert was approved by the U.S. Food and Drug ence of placement of the microinsert in 4,306 women in a
Administration for female sterilization in 2002. The single center, 4,108 (96.8%) women completed the stan-
microinsert is placed within the tubal lumen with the dard 3-month follow-up protocol. In the 4,108 women,
assistance of a disposable delivery system in a wound- the rate of successful bilateral obstruction (including
down state. After release from the delivery system, the unilateral occlusion in women with only one tube) was
insert expands and anchors itself within the tubal lumen, 99.7% (4,095/4,108). Only 13 out of 4,108 women (0.3%)
spanning the uterotubal junction. The microinsert has had one or both tubes patent at the 3-month follow-up
three components—a flexible stainless steel inner coil, visit. In support of this low tubal patency rate, only seven
a dynamic nickel–titanium expanding outer coil, and women (0.17%), three before the 3-month follow-up and
polyethylene terephthalate fibers. The polyethylene tere- four after the 3-month follow-up, became pregnant after
phthalate fibers are wound in and around the inner coil. the microinsert placement. One limitation of applying the
In its wound-down state, the insert is 4 cm long and aforementioned findings to others using the microinsert
0.8 mm in diameter. When expanded, the outer coil is system may be the presence of a publication bias. All the
1.5–2 mm in diameter and secures the insert within the procedures were performed (59%) or supervised (41%)
proximal isthmic portion of the tube. by four gynecologists skilled in performing outpatient
After placement of the microinsert, there is a local tis- hysteroscopic procedures and very familiar with the ster-
sue response that results in a chronic inflammatory and ilization technique. The aforementioned 3-month patency
fibrotic response to the polyethylene terephthalate fibers. rate of 0.3% may not be achieved in all institutions. A
The benign local tissue ingrowth against the polyethylene recent review of the 3-month tubal patency rate after
terephthalate fibers aids in device retention and preven- successful microinsert placement reported a rate of 3.5%
tion of pregnancy. After placement, the manufacturer tubal patency confirmed by HSG and subsequently a 0%
recommends that 3–8 trailing outer coils be visible in the patency rate at 6 months. The range of the tubal patency
uterine cavity to ensure that the insert is placed distally rates after successful microinsert placement appears to be
enough to avoid expulsion but proximally enough to less than 5% at 3 months after the initial procedure.
decrease the likelihood of insert perforation outside the
tubal lumen. After the procedure, the patient is instructed Benefits and risks of sterilization. Practice Bulletin No. 133.
American College of Obstetricians and Gynecologists. Obstet Gynecol
that she is not protected from conception until follow-up 2013;121:392–404.
HSG documents tubal occlusion. The documentation by
Povedano B, Arjona JE, Velasco E, Monserrat JA, Lorente J, Castelo-
HSG should be performed no sooner than 3 months after Branco C. Complications of hysteroscopic Essure ® sterilisation: report
the placement of the microinsert. on 4306 procedures performed in a single centre. BJOG 2012;119:795–9.
124 PROLOG
88
Primary hypogonadotropic hypogonadism
A male partner of your fertility patient is found to be azoospermic on two consecutive semen
analyses. Volume is normal but no sperm is identified. Postejaculatory urine also does not reveal
any sperm. Laboratory results reveal a follicle-stimulating hormone (FSH) level of 0.2 mIU/mL,
a luteinizing hormone (LH) level of 0.4 mIU/mL, a prolactin level of 3 micrograms/L, a thyroid-
stimulating hormone level of 1.24 mIU/mL, and a total testosterone level of 162 ng/dL. The best
next step is
Primary hypogonadotropic hypogonadism is caused either Men with cystic fibrosis are azoospermic because of con-
by hypothalamic or pituitary disease and is characterized genital bilateral absence of the vas deferens. Men with
by low gonadotropin levels. By contrast, early andro- isolated congenital bilateral absence of the vas deferens
pause is accompanied by markedly elevated FSH and LH often carry mutations of the cystic fibrosis transmem-
levels not seen in this clinical scenario. Pituitary imaging, brane conductance regulator gene. This gene is located in
preferably with MRI and gadolinium contrast, is neces- the short arm of chromosome 7 and encodes a membrane
sary to evaluate a mass-occupying lesion in primary hypo- protein that functions as an ion channel and also influ-
gonadotropic hypogonadism. A mass-occupying lesion, ences formation of the ejaculatory duct, seminal vesicle,
such as a hamartoma, craniopharyngioma, lymphoma, or vas deferens, and distal two thirds of the epididymis.
pituitary tumor, would interrupt pulsatile gonadotropin- When a man has congenital bilateral absence of the vas
releasing hormone secretion. The failure of hormonal deferens, it is important to test him and his partner for
regulation can be determined easily. Endocrine deficiency cystic fibrosis mutations. If the female partner is found to
leads to a lack of spermatogenesis and testosterone secre- be negative for known mutations, her chance of being a
tion due to decreased secretion patterns of LH and FSH. carrier of unknown mutations is approximately 0.4%. The
Standard treatment is human chorionic gonadotropin with condition can be transmitted to offspring, and therefore
later addition of human menopausal gonadotropin. These genetic counseling and possibly preimplantation genetic
strategies can be effective in stimulating spermatogenesis. diagnosis are indicated. Gonadotropin levels are typically
Only once pregnancy has been established should the normal with this condition. Low gonadotropin levels as
patient return to testosterone substitution. described in this patient would not be seen in cases of
Secondary hypogonadotropic hypogonadism can be varicocele, andropause, congenital bilateral absence of
caused by obesity and anabolic steroids. As direct-to- the vas deferens, or chromosomal abnormality.
consumer marketing becomes more common, many Klinefelter syndrome and Y chromosome microdele-
young men are developing secondary hypogonadotropic tion often are associated with azoospermia and elevated
hypogonadism, azoospermia, and resultant infertility gonadotropin levels. These conditions may be detected
due to topical or injectable testosterone replacement. on peripheral blood testing. The best next step in diag-
Varicocele is a physical abnormality present in 2–22% nosis and evaluation of the described patient is MRI to
of the adult male population and is more common in evaluate the anterior pituitary. Testicular biopsy, at times
infertile men (25–40%). The exact effect of varicocele used to confirm the diagnosis of primary testicular failure
on fertility is unknown, but researchers suggest a relation and nonobstructive azoospermia, is not indicated in this
to decreased testicular volume and decline in Leydig cell patient with low gonadotropin levels.
function. Treatment of varicocele to achieve pregnancy
in infertile couples has been subject to criticism but Dohle GR, Colpi GM, Hargreave TB, Papp GK, Jungwirth A, Weidner
may be considered, particularly if the defect is clinically W. EAU guidelines on male infertility. EAU Working Group on Male
Infertility. Eur Urol 2005;48:703–11.
palpable.
Giannetta E, Gianfrilli D, Barbagallo F, Isidori AM, Lenzi A.
Cystic fibrosis is a fatal autosomal recessive disorder Subclinical male hypogonadism. Best Pract Res Clin Endocrinol Metab
and is the most common genetic disease in white people. 2012;26:539–50.
Reproductive Endocrinology and Infertility 125
Nachtigall LB, Boepple PA, Pralong FP, Crowley WF Jr. Adult-onset Oates RD, Amos JA. The genetic basis of congenital bilateral absence
idiopathic hypogonadotropic hypogonadism--a treatable form of male of the vas deferens and cystic fibrosis. J Androl 1994;15:1–8.
infertility. N Engl J Med 1997;336:410–5.
89
Hysterosalpingography complications
A 31-year-old woman, gravida 2, para 0, has been attempting conception without success for the
past 5 years. She has a history of two ectopic pregnancies, 6 and 8 years ago. Both ectopic preg-
nancies were treated with methotrexate. She has regular menstrual cycles with significantly painful
periods. Her husband has two children from a prior relationship. Among other tests, you perform
hysterosalpingography (HSG) with the findings shown in Appendix B. The most appropriate imme-
diate next step is
Hysterosalpingography is a radiologic procedure designed HSG demonstrates dilated, obstructed fallopian tubes,
to evaluate the uterine cavity and the patency of the fallo- the patient should receive a postprocedure antibiotic, ie,
pian tubes. It involves inserting water-soluble radiopaque doxycycline twice daily for 5 days. Treatment with anti-
dye through a transcervical catheter and often is per- biotics is the most appropriate next step in this case.
formed during evaluation for infertility or to confirm tubal Leuprolide is a gonadotropin-releasing hormone ago-
obstruction after placement of a birth control implant for nist that often is used for treatment of endometriosis or
sterilization. Generally, HSG testing is performed within leiomyomas as well as for IVF. The described patient
5 days after menstrual bleeding ceases to avoid the pos- has obstructed fallopian tubes, although the cause of the
sibility that the procedure could be done during the luteal blockage is unknown. Common causes of tubal obstruc-
phase after conception has occurred. To be conservative, tion include prior pelvic infection or adhesions related to
a pregnancy test may be performed before the procedure. endometriosis or prior surgeries. Leuprolide would not be
The technique for HSG involves using image-intensifica- useful for the described patient.
tion fluoroscopy and usually requires only three to five In some rare instances, patients have a systemic reaction
basic films (a scout, an image evaluating the uterus and to the contrast dye used for HSG, with symptoms rang-
tubal patency, and an image to assess for pelvic areas of ing from urticaria to bronchospasm and laryngeal edema.
loculation). The total radiation exposure time is usually In patients with a history of prior sensitivity to iodinated
only 20–30 seconds. Many patients feel discomfort and contrast agents, it is recommended to pretreat with antihis-
uterine cramping during the procedure; pretreatment with tamines and corticosteroids to prevent an allergic reaction.
a nonsteroidal antiinflammatory drug may mitigate those In the described patient, there is no evidence of an allergic
symptoms. reaction, so this treatment is not indicated. Another rare
The described patient is at risk of tubal obstruc- complication of HSG is uterine perforation. This may
tion, given her history of two prior ectopic pregnan- result in acute bleeding and require urgent surgery. Even
cies. Although post-HSG pelvic inflammatory disease in the extremely rare scenario of uterine perforation that
is uncommon (1.4–3.4%), it is a serious complication, requires surgery, it would be highly unlikely to require
especially in a patient population that strongly desires immediate bilateral salpingectomy.
conception. If the tubes are dilated at the time of HSG, This patient probably has significant tubal obstruction
the risk of pelvic inflammatory disease after the procedure that is causing her infertility, and she will either require
is as high as 10%. If there is any concern about active surgery or IVF to help her conceive. Distal tubal obstruc-
cervical or uterine infection on the day of the scheduled tion seen on HSG may require follow-up laparoscopy to
HSG, the test should be canceled and the patient should better characterize the extent of disease and possibly repair
undergo evaluation and treatment for the infection. If the the defect. In younger patients with mild pelvic disease,
126 PROLOG
neosalpingotomy with fimbrioplasty can be attempted as treatment for the described patient is antibiotics to pre-
an alternative to IVF. However, if pregnancy does not vent infection.
occur within 1 year after corrective surgery, IVF will
most likely be necessary. Older women with distal tubal Antibiotic prophylaxis for gynecologic procedures. ACOG Practice
Bulletin No. 104. American College of Obstetricians and Gynecologists.
disease, irrespective of the severity, should consider IVF Obstet Gynecol 2009;113:1180–9.
as the first option, given that their reproductive window Pittaway DE, Winfield AC, Maxson W, Daniell J, Herbert C, Wentz
is narrow. For the described patient, IVF may be an AC. Prevention of acute pelvic inflammatory disease after hystero-
appropriate treatment option in the future. If bilateral salpingography: efficacy of doxycycline prophylaxis. Am J Obstet
Gynecol 1983;147:623–6.
hydrosalpinges are confirmed at the time of laparoscopy,
Speroff L, Fritz MA. Female infertility. In: Speroff L, Fritz MA,
consideration should be given to bilateral salpingectomy editors. Clinical gynecologic endocrinology and infertility. 8th ed.
before IVF. However, at this time, the most important Philadelphia (PA): Lippincott Williams and Wilkins; 2011. p. 1185 –90.
90
Ovulation induction treatment options
A 30-year-old nulligravid woman requests counseling about how to become pregnant. She is cur-
rently taking a combination oral contraceptive as hormone therapy for Kallmann syndrome. Fifteen
years ago, she was treated medically to develop secondary sexual characteristics. She was placed
on subcutaneous recombinant follicle-stimulating hormone (FSH) therapy but did not respond to
ovulation induction. The most appropriate next step in her management is
Kallmann syndrome is a well-characterized disorder that using lower doses of FSH with the addition of low-dose
features hypogonadotropic hypogonadism and anosmia. human chorionic gonadotropin or LH. Given the current
The syndrome affects 1 in 10,000 men and 1 in 50,000 understanding of folliculogenesis based on the two-cell
women. Patients with this syndrome present with primary theory, LH stimulates thecal cell androgen production,
amenorrhea without development of secondary sexual which serves as the substrate for aromatization in the
characteristics. The failure of proper neuronal migration follicle and release of estradiol, which is required for
of the GnRH-secreting and olfactory neurons results in optimal follicular growth. The most appropriate next step
inadequate release of GnRH with a lack of production in the management of the described patient would be to
of FSH and luteinizing hormone (LH). This leads to a prescribe human menopausal gonadotropins.
lack of folliculogenesis and steroid hormone production, In numerous case reports, pregnancy has been achieved
which renders the patient infertile. using either a GnRH pump to mimic hypothalamic secre-
Initial treatment of Kallmann syndrome is aimed at tion or gonadotropin injections. Gonadotropin-releasing
restoring breast development and initiation of menstrua- hormone agonists and GnRH antagonists can cause ovar-
tion using estrogens and progestins. Pubic and axillary ian suppression and, thus, are ineffective for inducing
hair develops in response to adrenal androgens. Many folliculogenesis.
case reports have observed the need for the use of meno- Clomiphene citrate is a selective estrogen receptor
tropins (which carry LH and FSH activity) and a long antagonist that is commonly used for ovulation induction
ovarian stimulation for the development of a mature in patients with an intact hypothalamic–gonadal–pituitary
follicle. Although prolonged high doses of FSH alone axis. Clomiphene has no utility in patients with Kallmann
can eventually induce folliculogenesis, there is always syndrome.
the risk of a multiple follicular response that requires Aromatase inhibitors are a class of drugs that block the
cancellation of the cycle or of ovarian hyperstimulation activity of aromatase, the enzyme responsible for the con-
syndrome. Reports have described a faster stimulation version of androgens into estradiol. Aromatase inhibitors
Reproductive Endocrinology and Infertility 127
cause a relative estrogen deficiency, which induces an Meczekalski B, Podfigurna-Stopa A, Smolarczyk R, Katulski K,
Genazzani AR. Kallmann syndrome in women: from genes to diagnosis
increase of gonadotropin release in patients with a normal and treatment. Gynecol Endocrinol 2013;29:296–300.
hypothalamic–gonadal–pituitary axis. However, they are Sipe CS, Van Voorhis BJ. Testosterone patch improves ovarian follicu-
not effective in patients with Kallmann syndrome. lar response to gonadotrophins in a patient with Kallmann’s syndrome:
a case report. Hum Reprod 2007;22:1380–3.
Fechner A, Fong S, McGovern P. A review of Kallmann syndrome:
genetics, pathophysiology, and clinical management. Obstet Gynecol
Surv 2008;63:189–94.
91
Testing for insulin resistance in polycystic ovary syndrome
A 24-year-old nulligravid woman with periods of amenorrhea greater than 6 months may have
polycystic ovary syndrome (PCOS). You notice darkening on the back of her neck that is velvety
in appearance (Fig. 91-1; see color plate). The most accurate way to evaluate insulin resistance in
patients with PCOS is
Insulin resistance is a condition in which endogenous or Euglycemic clamp studies are impractical in the busy
exogenously administered insulin has less-than-normal clinical setting. Therefore, other surrogate markers are
effects on fat, muscle, and the liver. This insulin level utilized to try to assess insulin resistance.
results in increased hydrolysis of stored triglycerides and The standard glucose tolerance test is the most com-
elevated circulating free fatty acid levels. In women with monly utilized method to assess impaired glucose toler-
insulin resistance, there is increased hepatic gluconeogen- ance (IGT) and diabetes mellitus in patients with PCOS.
esis resulting in an increased blood glucose concentration The terms insulin resistance and glucose intolerance are
and a compensatory hyperinsulinemia. Increased circu- erroneously used interchangeably. It is important to under-
lating insulin levels contribute to hyperandrogenism by stand the difference between the information obtained
stimulating ovarian androgen production and inhibiting from a euglycemic clamp and a 2-hour glucose tolerance
hepatic sex hormone-binding globulin production. test. A 2-hour glucose tolerance test involves ingestion
Women with insulin resistance often exhibit acanthosis of 75 g of glucose after fasting and assessment of the
nigricans (Fig. 91-1; see color plate), a hyperpigmented patient for fasting glucose and glucose level 2 hours after
and velvety appearance of the skin commonly observed the load of glucose. Up to 35% of the PCOS population
at the base of the neck, in the groin or axilla, or beneath may exhibit IGT and 10% may exhibit obvious diabetes
the breasts. It strongly suggests insulin resistance and mellitus.
the possibility of diabetes mellitus. Insulin resistance The prevalence of IGT and type 2 diabetes mellitus in
and hyperinsulinemia are common among women with U.S. women has been assessed in three large cross-sec-
PCOS. tional studies. The prevalence was 23–35% for IGT and
The criterion standard for assessment of insulin resis- 4–10% for type 2 diabetes mellitus. The prevalence rate
tance is the hyperinsulinemic euglycemic clamp, which of IGT in PCOS was three times higher than age-matched
involves an intravenous infusion of insulin and glucose. controls.
The glucose infusion is increased until the infusion rate A fasting glucose level does not assess glucose control
matches the rate of glucose uptake in tissues at a set when challenged by food intake. It is considered inferior to
insulin concentration. This rate is inversely proportional test for glucose control after food intake. Fasting insulin
to the degree of insulin resistance. The lower the glu- levels can suggest insulin resistance. Fasting insulin levels
cose concentration infused to maintain a normal fasting greater than 20–30 mU/mL suggest insulin resistance but
glucose level, the higher the degree of insulin resistance. do not diagnose diabetes mellitus.
128 PROLOG
Historically, the diagnosis for diabetes mellitus was Ehrmann DA, Barnes RB, Rosenfield RL, Cavaghan MK, Imperial J.
Prevalence of impaired glucose tolerance and diabetes in women with
based on plasma glucose criteria and the 2-hour value on polycystic ovary syndrome. Diabetes Care 1999;22:141–6.
the 2-hour glucose tolerance test. In 2010, the American Ehrmann DA, Kasza K, Azziz R, Legro RS, Ghazzi MN. Effects of race
Diabetes Association and the Endocrine Society jointly and family history of type 2 diabetes on metabolic status of women with
recommended the use of the hemoglobin A1c (HbA1c) test polycystic ovary syndrome. PCOS/Troglitazone Study Group. J Clin
Endocrinol Metab 2005;90:66–71.
to diagnose diabetes mellitus with a threshold of 6.5% or
Legro RS, Kunselman AR, Dodson WC, Dunaif A. Prevalence and
greater. The HbA1c test has advantages over the 2-hour predictors of risk for type 2 diabetes mellitus and impaired glucose
glucose tolerance test and fasting plasma glucose level tolerance in polycystic ovary syndrome: a prospective, controlled study
criterion. The HbA1c test is fast and does not require in 254 affected women. J Clin Endocrinol Metab 1999;84:165–9.
2 hours of the patient’s time. The convenience of the
HbA1c test may help health care providers to establish
more diagnoses. Additionally, the HbA1c test has little
variability at times of stress or illness.
92
Abnormal uterine bleeding in an adolescent
A 14-year-old virginal girl comes to your office. She has had irregular heavy menses since men-
arche 1 year ago. She has menses that occur on an irregular basis every 2–5 months, and her bleed-
ing lasts for 10–15 days, sometimes with spotting and other times with the need to change her pad
every 2–3 hours. She has felt no lightheadedness or dizziness and has no other medical problems.
The most likely explanation for her bleeding pattern is
(A) endometriosis
* (B) oligo-ovulation
(C) pregnancy
(D) coagulopathy
Heavy menstrual bleeding is classically defined as bleed- (Appendix C). The etiologies of abnormal uterine bleed-
ing that lasts longer than 7 days or blood loss in excess of ing fall into two categories:
80 mL per cycle. Research has shown, however, that the 1. Structural causes, ie, polyp, adenomyosis, leiomyo-
patient’s perception of light, moderate, or heavy bleed- mata, malignancy (PALM)
ing corresponds poorly with her actual blood loss. This
2. Nonstructural causes, ie, coagulopathy, ovulatory
was clearly demonstrated in a classic Swedish study of
dysfunction, endometrial, iatrogenic, and “not yet clas-
menstrual blood loss. The investigators reported that 30%
sified” (COEIN)
of women with light-to-average menstrual bleeding self-
reported their menstrual periods to be heavy, whereas The abnormal bleeding in the described patient is likely to
40% of women with blood loss greater than 80 mL be related to anovulation or oligo-ovulation.
per cycle considered their menstrual periods to be only With anovulatory bleeding patterns, the history is often
of moderate intensity. Consequently, heavy menstrual adequate to establish the diagnosis, and further testing
bleeding or “menorrhagia” is a relative term that cannot can confirm the suspicion. In this case, the patient has
be defined solely by quantitative blood volume loss or infrequent, unpredictable bleeding that varies in intensity
days of bleeding. The physician’s assessment of heavy and has been occurring in this pattern since menarche.
menstrual bleeding must take into account the patient’s Such infrequent, unpredictable bleeding occurs most
perception of blood loss and the degree to which it affects frequently in adolescents, perimenopausal women, obese
her quality of life. women, and women with polycystic ovary syndrome. In
The PALM–COEIN classification system for causes of adolescents, the condition can be related to immaturity of
abnormal uterine bleeding was developed in 2011 by the the hypothalamic–pituitary–ovarian axis, and it is com-
International Federation of Gynecology and Obstetrics mon for the first few years after menarche. Although this
Reproductive Endocrinology and Infertility 129
is the most likely scenario in this case, other more serious It is important to rule out a coagulopathy in adolescents
pathologies must be ruled out. with abnormal bleeding, especially in adolescents who
Endometriosis is a common, benign condition that have excessive bleeding. Although the most common
often presents with chronic pelvic pain, dysmenorrhea, cause of abnormal bleeding in this age group is anovula-
dyspareunia, and infertility. Although this condition can tion, coagulopathies can occur up to one third of the time.
occur in adolescents, it is more common in women aged Coagulopathies may have significant consequences if
25–35 years. Usually women with endometriosis have they remain untreated. The most common coagulopathy
regular menstrual cycles, making this an unlikely cause is von Willebrand disease, but other coagulopathies, such
of the symptoms in this case. as idiopathic thrombocytopenic purpura, platelet dys-
Anatomic causes of menstrual bleeding are common, function, and thrombocytopenia due to malignancy, are
eg, benign lesions, such as polyps or leiomyomas, and possible. Most patients with coagulopathy exhibit regular
malignancies (cervical or uterine). Uterine fibroids are bleeding patterns but have excessive bleeding that can
the most common pelvic tumor in women with present- necessitate changing a pad more frequently than every
ing symptoms of abnormal bleeding, pelvic pain or pres- hour. For the described patient, testing for coagulopathy
sure, infertility, or recurrent pregnancy loss. Leiomyomas would be important, but coagulopathy is not the most
are common in reproductive-aged women and are found likely diagnosis.
in approximately 80% of surgically removed uteri.
Deligeoroglou E, Karountzos V, Creatsas G. Abnormal uterine bleeding
Leiomyomas and polyps are uncommon in adolescents. and dysfunctional uterine bleeding in pediatric and adolescent gynecol-
Pregnancy should always be considered as a cause of ogy. Gynecol Endocrinol 2013;29:74–8.
abnormal uterine bleeding, especially in a case where the Diagnosis of abnormal uterine bleeding in reproductive-aged
current bleeding pattern is a relatively sudden departure women. Practice Bulletin No. 128. American College of Obstetricians
and Gynecologists. Obstet Gynecol 2012;120:197–206.
from the prior pattern. Adolescents may be unwilling to Hallberg L, Hogdahl AM, Nilsson L, Rybo G. Menstrual blood loss--a
reveal their sexual history, so a pregnancy test should population study. Variation at different ages and attempts to define
always be performed, even in girls who deny sexual normality. Acta Obstet Gynecol Scand 1966;45:320–51.
activity. However, given that the timeframe for bleeding Munro MG, Critchley HO, Broder MS, Fraser IS. FIGO classifica-
tion system (PALM-COEIN) for causes of abnormal uterine bleeding
in this case is 1 year, it is unlikely that pregnancy-related in nongravid women of reproductive age. FIGO Working Group on
complications are the cause of her symptoms. Menstrual Disorders. Int J Gynaecol Obstet 2011;113:3–13.
93
Elevated dehydroepiandrosterone sulfate
A 23-year-old nulligravid woman has had a history of irregular menses and hirsutism since she
underwent menarche. She is suspected to have polycystic ovary syndrome. During the evaluation,
her dehydroepiandrosterone sulfate (DHEAS) level is found to be 760 micrograms/dL. In order to
further evaluate her abnormally elevated DHEAS level, you order imaging of the
(A) pelvis
(B) brain
* (C) abdomen
(D) chest
Compared with other steroids, DHEAS circulates in synthesis in the hair follicle. Elevated levels of DHEAS
higher concentrations and is almost exclusively derived also have been noted in Cushing syndrome, adrenal
from the adrenal glands. The upper limit of normal is adenomas, adrenal carcinomas, and congenital adre-
350 micrograms/dL but can vary between laboratories. nal hyperplasia resulting from CYP11B, CYP21, and
Mild elevations in DHEAS levels (350–700 micrograms/ 3β-HSD gene deficiencies. Levels of DHEAS are low
dL) have been noted in women with polycystic ovary syn- in CYP17 gene deficiency.
drome and hirsutism. Elevation in DHEAS contributes to Elevated testosterone levels may originate from either
hirsutism because it acts as a prehormone for androgen the adrenal glands or the ovaries; however, a DHEAS
130 PROLOG
level of 700 micrograms/dL or higher may indicate an ovarian tumor is suspected. Brain magnetic resonance
excessive adrenal secretion of this hormone and the imaging may be appropriate for pituitary or hypothalamic
possibility of an adrenal tumor. Malignancy cannot be abnormalities. A chest X-ray would not be useful in the
diagnosed based on an elevated DHEAS level because care of this patient at this time.
approximately one half of patients with adrenal tumors
and an elevated DHEAS level have a benign adrenal Azziz R, Sanchez LA, Knochenhauer ES, Moran C, Lazenby J, Stephens
KC, et al. Androgen excess in women: experience with over 1000 con-
tumor. Adrenal malignant tumors are rare tumors with a secutive patients. J Clin Endocrinol Metab 2004;89:453–62.
higher occurrence in females than in males. The evalua- Derksen J, Nagesser SK, Meinders AE, Haak HR, van de Velde CJ.
tion should include an abdominal computed tomography Identification of virilizing adrenal tumors in hirsute women. N Engl J
scan with specific evaluation of the adrenal glands for Med 1994;331:968–73.
tumors. Abdominal magnetic resonance imaging provides McKenna TJ. Screening for sinister causes of hirsutism. N Engl J Med
1994;331:1015–6.
an alternative method to evaluate elevated DHEAS lev-
Stewart PM. The adrenal cortex. In: Williams textbook of endocrinol-
els. It may be preferable in patients who cannot tolerate ogy. In: Kronenberg HM, Melmed S, Polonsky KS, Larsen PR, eds.
intravenous iodinated contrast dye or if hemorrhage is Williams textbook of endocrinology. 11th ed. Philadelphia (PA): WW
suspected. Pelvic ultrasonography may be appropriate if Saunders; 2008. p. 445–58.
94
Low bone mass and bone physiology
A 62-year-old woman experienced a Colles (wrist) fracture in the past year. She has not used
hormone therapy since she went through menopause 10 years ago. She currently takes dietary
supplements of 400 international units/d of vitamin D and 1,500 mg/d of calcium. She is 1.52 m
(5 ft) tall and weighs 68 kg (150 lb). Dual-energy X-ray absorptiometry (DXA) scans of the lumbar
spine and hip were done (Fig. 94-1 and Fig. 94-2). The most appropriate next step in management
is to prescribe
* (A) bisphosphonates
(B) increased vitamin D and calcium supplements
(C) teriparatide
(D) estrogen therapy
(E) calcitonin
Most postmenopausal women have low bone mass com- occur in the years after menopause. One meta-analysis of
pared with the young adult reference population. The more than 40,000 women found that a prior fracture was
diagnostic thresholds for osteoporosis for any specific indi- associated with a significant risk for another fracture (rela-
vidual are based on the patient’s bone mineral density test tive risk = 1.86, 95% confidence interval, 1.75–1.98).
results compared with individuals of comparable age and Screening for low bone mass should occur at age
sex. However, osteoporosis generally has no clinical symp- 65 years for all women and earlier for women with one or
toms until a fracture occurs, and ideally the disease should more risk factors. Risk factors include tobacco use, body
be identified and treated before a devastating fracture mass index greater than 21 (calculated as weight in kilo-
occurs. The most common type of fracture with osteopo- grams divided by height in meters squared), alcohol use,
rosis is vertebral fracture; such fractures are asymptomatic and parental fracture history.
two thirds of the time and may be diagnosed as an inciden- Screening for low bone mass involves evaluation with
tal finding on chest or abdominal X-rays. Hip fractures also a DXA scan of the spine and hip. In addition, in 2008, the
are common and can occur suddenly with a fall or silently World Health Organization task force developed a Frac-
with subsequent destructive secondary osteoarthritis of the ture Risk Assessment Tool, which estimates the 10-year
hip. Distal radial fractures (Colles fractures) frequently probability of a hip or other major fracture (Fig. 94-3).
Reproductive Endocrinology and Infertility 131
Total
1.6
1.4
1.2
1.0
BMD
0.8
0.6
0.4
0.2
20 25 30 35 40 45 50 55 60 65 70 75 80 85
Age
FIG. 94-1. Dual energy X-ray absorptiometry (DXA) scan of the spine. (Courtesy of Janet Rubin, MD, University of North
Carolina at Chapel Hill School of Medicine.)
132 PROLOG
Neck
1.4
1.2
1.0
0.8
BMD
0.6
0.4
0.2
0.0
20 25 30 35 40 45 50 55 60 65 70 75 80 85
Age
FIG. 94-2. Dual energy X-ray absorptiometry (DXA) scan of the hip. (Courtesy of Janet Rubin, MD, University of North
Carolina at Chapel Hill School of Medicine.)
Reproductive Endocrinology and Infertility 133
FIG. 94-3. FRAX® World Health Organization Fracture Risk Assessment Tool. Available at: http://www.shef.ac.uk/
FRAX/. Retrieved June 21, 2013.
The National Osteoporosis Foundation recommends treat- phosphate of calcium that is the chief structural element of
ment if the patient has either vertebrate bone. Commonly used bisphosphonates in the
• a personal history of hip or vertebral fracture, United States include alendronate sodium, ibandronate
sodium, risedronate sodium, and zoledronic acid. To help
• a DXA scan with a T-score less than –2.5, or
reduce the adverse effect of gastric irritation, these oral
• a bone mass in the osteopenic range (T-score of –1.0 formulations must be taken on an empty stomach with
to –2.5) and a 10-year probability of a hip fracture of a glass of water (no other liquid), and the patient must
3% or more or a 10-year probability of any fracture remain upright for 30–60 minutes. Case reports show that
of 20% or more based on Fracture Risk Assessment osteonecrosis of the jaw is associated with bisphospho-
Tool estimates. nate use. In general, most of these cases occur in associa-
Traditionally, vitamin D and calcium supplementation tion with dental procedures in patients with poor dentition
have been standard treatments to reduce bone loss dur- and established malignancies.
ing menopause. However, a 2013 report from the U.S. Other approved treatments to prevent fracture include
Preventive Services Task Force concluded that there was estrogen, calcitonin, parathyroid hormone, and raloxifene
insufficient evidence to confirm that supplementation hydrochloride. Teriparatide is a parathyroid hormone that
with 1,000 mg or more of calcium or 400 international is injected subcutaneously each day. It has been shown to
units of vitamin D has benefits in terms of primary pre- increase bone density by stimulating new bone formation.
vention of fractures in postmenopausal women. However, given the cost, route of administration, and
Bisphosphonates are a class of compounds that inhibit availability of other options, parathyroid hormone is not
osteoclastic action and therefore reduce bone resorption. considered first-line therapy for treatment or prevention
Most bisphosphonates available in the United States have of osteoporosis.
been shown to reduce the incidence of spine and hip Estrogen is a very effective antiresorptive agent for the
fractures and are considered the first-line treatment for treatment of osteoporosis. In the Women’s Health Initia-
postmenopausal women at risk for fracture. All bisphos- tive study, estrogen was observed to reduce fracture risk
phonates share a common P–C–P backbone structure, by 40% in a low-risk population of menopausal women
where C is carbon and each P is a phosphonate group; compared with placebo. However, the risks associated
the two phosphonate groups act as a “bone hook” and with estrogen use (eg, risk of breast cancer and cardiovas-
are essential for binding to hydroxyapatite, a complex cular disease) generally are considered significant enough
134 PROLOG
to warrant use of other treatments, such as bisphospho- Moyer VA; U.S. Preventive Services Task Force. Vitamin D and cal-
cium supplementation to prevent fractures in adults: U.S. Preventive
nates, as first-line options for fracture prevention. Services Task Force recommendation statement. Ann Intern Med
Calcitonin is administered in a daily nasal spray and 2013;158:691–6.
has a relatively modest effect on bone mineral density National Osteoporosis Foundation. Clinician’s guide to prevention and
compared with other treatment options. For this rea- treatment of osteoporosis. Washington, DC: National Osteoporosis
Foundation; 2010. Available at: http://nof.org/files/nof/public/content/
son, it is rarely used as first-line treatment and instead file/344/upload/159.pdf. Retrieved July 12, 2013.
is reserved for patients who cannot tolerate other more World Health Organization Collaborating Centre for Metabolic Bone
efficacious options. Diseases. WHO Fracture Risk Assessment Tool (FRAX). Sheffield,
United Kingdom: University of Sheffield; 2013. Available at: http://
Kanis JA, Johnell O, De Laet C, Johansson H, Oden A, Delmas P, www.shef.ac.uk/FRAX/. Retrieved June 21, 2013.
et al. A meta-analysis of previous fracture and subsequent fracture risk.
Bone 2004;35:375–82.
95
Testing for ovarian reserve
A 30-year-old nulligravid woman comes to your office with a 1-year history of infertility. On
evaluation, her partner has a normal semen analysis. Hysterosalpingography is normal. The earliest
test that will identify a diminished ovarian reserve is
Ovarian reserve testing is an important aspect in the eval- follicular phase may be related to the advanced follicular
uation of fertility. It enables identification of women who recruitment seen in ovarian aging.
may need more aggressive treatment. Ovarian reserve If, on cycle days 2–4, the level of inhibin B (a product
testing should be performed in all women older than of granulosa cell secretion in the follicular phase) is low
35 years, women with unexplained infertility, women (below 45 pg/mL), there will be a poorer response to ovu-
with a prior history of ovarian surgery, women with a lation stimulation. Women with low inhibin B levels will
prior history of gonadotoxic treatment (chemotherapy or be less likely to conceive than women with high inhibin
radiotherapy), and women who smoke. B levels. However, because of the low sensitivity of this
It is important to realize that ovarian reserve test- test, it has been largely abandoned.
ing only allows for quantitative evaluation of oocytes. The clomiphene citrate challenge test involves the
Qualitative assessment of oocytes is unavailable and administration of 100 mg clomiphene on days 5–9 of
depends upon the age of the female partner. In addi- the menstrual cycle with FSH levels obtained on day 3
tion, ovarian reserve testing does not predict pregnancy. (before the administration of clomiphene) and on day 10.
Rather, it predicts the response of the patient to gonado- In patients with normal ovarian reserve, the clomiphene-
tropin therapy. dependent rise in FSH will be suppressed by inhibin B
The gonadotropin level will increase as the patient produced by the follicles. An abnormal test is defined by
approaches menopause. The FSH level will typically an abnormally high FSH on days 2–4 or day 10 (usually
increase first. The luteinizing hormone level may remain greater than 10 mIU/mL). Because of the low sensitiv-
normal and increase later. An increase in FSH level is ity of the clomiphene challenge test, it has been largely
secondary to quantitative loss of follicles from the ova- replaced by antimüllerian hormone level.
ries. With the loss of granulosa cells, inhibin production In women, antimüllerian hormone is produced by
drops, and FSH is no longer suppressed because inhibin granulosa cells of preantral and small antral follicles. It
B inhibits FSH. This test can be performed at any time is believed that antimüllerian hormone plays a role in the
during cycle days 2–4, typically in conjunction with transition from resting primordial to growing follicles
testing for estradiol level. Estradiol elevation in the early and in the recruitment of FSH-sensitive follicles. Because
Reproductive Endocrinology and Infertility 135
antimüllerian hormone declines with advancing age, it can Grynnerup AG, Lindhard A, Sorensen S. The role of anti-Müllerian
hormone in female fertility and infertility—an overview. Acta Obstet
be used as a marker of ovarian reserve. This decline takes Gynecol Scand 2012;91:1252–60.
place ahead of changes in FSH and luteinizing hormone La Marca A, Sighinolfi G, Radi D, Argento C, Baraldi E, Artenisio AC,
levels. Normal antimüllerian hormone, which will vary et al. Anti-Müllerian hormone (AMH) as a predictive marker in assisted
between assays, is greater than 1.5 ng/mL. Thus, in test- reproductive technology (ART). Hum Reprod Update 2010;16:113–30.
ing diminished ovarian reserve, the earliest test to show La Marca A, Volpe A. Anti-Müllerian hormone (AMH) in female
reproduction: is measurement of circulating AMH a useful tool? Clin
an abnormal result is antimüllerian hormone level rather Endocrinol (Oxf) 2006;64:603–10.
than the clomiphene challenge test, FSH level, or inhibin Toner JP, Seifer DB. Why we may abandon basal follicle-stimulating
B level. Note that women with polycystic ovary syndrome hormone testing: a sea change in determining ovarian reserve using
may have a higher antimüllerian hormone level as a result antimüllerian hormone. Fertil Steril 2013;99:1825–30.
of the presence of many small antral follicles in their ova- Tremellen KP, Kolo M, Gilmore A, Lekamge DN. Anti-müllerian
hormone as a marker of ovarian reserve. Aust N Z J Obstet Gynaecol
ries (greater than 3.5 ng/mL). 2005;45:20–4.
96
Chronic pelvic pain
Endometriosis affects 10–20% of women of reproductive inhibit menses and do have long-term adverse effects.
age. Approximately 70–90% of women who experience Aromatase inhibitors are a newer option and to date have
chronic pelvic pain, dysmenorrhea, dyspareunia, infertil- not been proven to be efficacious in large randomized
ity, and menstrual disturbances have endometriosis. The trials.
most common drugs used for the conservative treatment A 2003 randomized controlled trial monitored women
of endometriosis include NSAIDs, progestins such as with Stage I–IV endometriosis postsurgically who were
depot medroxyprogesterone acetate, GnRH analogs either under observation or using the levonorgestrel IUD.
to induce pseudomenopause, androgen derivatives, and Bleeding and dysmenorrhea were reduced in the levo-
continuous combination OCs. Safety concerns may limit norgestrel IUD group with additional reduction in non-
the use of OCs in smokers, women older than 35 years, menstrual pain and dyspareunia. The reduction in relative
and patients with known coagulopathy. Depot medroxy- risk of recurrence of dysmenorrhea was 78%. A 2005
progesterone acetate and GnRH analogs require frequent randomized controlled trial compared women with Stage
visits for administration and may lead to hypoestrogenic I–IV endometriosis who used either the levonorgestrel
symptoms and, in the long term, decreased adherence IUD or a GnRH analog. Both treatments reduced bleed-
rates. Although GnRH analogs are effective for pain ing and improved pain; the quality of life assessment was
control, they should be limited to short-term use because similar in both groups.
of the risk of osteoporosis. Similarly, although NSAIDs The release rate of levonorgestrel from the levo-
might be beneficial for pain in the short term, they do not norgestrel IUD is 20 micrograms/d during the first year
136 PROLOG
and slowly decreases over the 5 years of use. As a resulting in an antiproliferative effect, glandular atrophy,
19-nortestosterone derivative, levonorgestrel exhibits and decidualization.
progestational activity, inducing profound effects on Although endometriosis often is associated with infer-
the endometrium, which becomes atrophic and inactive. tility, not all patients with endometriosis wish to conceive.
Therefore, based on the induced amenorrhea and endo- In patients who have completed childbearing or who wish
metrial atrophy, it is possible to speculate that the levo- to postpone pregnancy, the levonorgestrel IUD provides a
norgestrel IUD may be effective in controlling pain for good method for control of pain associated with endome-
the same period. Several hypotheses have been proposed triosis. The affordable cost and minimal adverse effects
to explain the mechanism of action of levonorgestrel in of the levonorgestrel IUD make it the best option for the
endometriosis. One hypothesis is that levonorgestrel acts described patient.
directly on the endometriotic lesions in the peritoneum;
Bahamondes L, Petta CA, Fernandes A, Monteiro I. Use of the
however, the concentration of levonorgestrel is high only levonorgestrel-releasing intrauterine system in women with endome-
within the endometrium. Another hypothesis is that the triosis, chronic pelvic pain and dysmenorrhea. Contraception 2007;75
level of levonorgestrel in peritoneal fluid is high and that (suppl):S134 – 9.
the steroid acts directly on the endometriotic lesions in Vercellini P, Frontino G, De Giorgi O, Aimi G, Zaina B, Crosignani
PG. Comparison of a levonorgestrel-releasing intrauterine device ver-
the peritoneum. Other theories suggest that endometrial sus expectant management after conservative surgery for symptomatic
production of estradiol-induced growth factors or growth endometriosis: a pilot study. Fertil Steril 2003;80:305–9.
factor-binding protein from the endometrium is inhibited,
97
Amenorrhea and galactorrhea
A 22-year-old woman who is not taking any medications comes to your office with a lack of menses
for 9 months. She desires contraception. Physical examination is remarkable for nonspontaneous
multiduct bilateral galactorrhea, atrophic-appearing vaginal mucosa, and absent cervical mucus.
Laboratory testing shows a normal thyroid-stimulating hormone level and a prolactin level of
80 ng/dL. Magnetic resonance imaging (MRI) indicates a possible 3-mm anterior pituitary ade-
noma. In addition to repeat MRI and prolactin level in 1 year, the most appropriate therapeutic
recommendation is
Management of hyperprolactinemia is dependent on the increases prolactin secretion by direct action of thyroid-
etiology of the condition and the prevention of associ- releasing hormone on the pituitary. Circumstances that
ated consequences of estrogen deficiency. The patho- result in increased estrogen levels, such as development
logic causes of hyperprolactinemia are varied and may of polycystic ovary syndrome or liver dysfunction (due
include pituitary tumors that secrete prolactin and other to decreased metabolism) and use of estrogen-containing
central nervous system (CNS) abnormalities that alter drugs, also increase prolactin level. Because prolactin
the inhibition by dopamine on pituitary lactotrophs. Such is renally excreted without metabolism, kidney failure
CNS abnormalities may include other pituitary lesions causes hyperprolactinemia. In rare instances, tumors,
such as acromegaly, Cushing syndrome, and empty such as ovarian teratomas and bronchogenic carcinomas,
sella syndrome. Additional CNS abnormalities include may secrete prolactin. Breast and chest wall abnormali-
tumors, infiltrating pituitary disorders such as sarcoidosis ties, such as mammoplasty, mastectomy, chest trauma,
and histiocytosis X, encephalitis, head trauma, hydro- thoracic burns, and thoracic herpes zoster, may cause
cephalus, Rathke pouch cyst, pinealoma, pseudotumor elevated prolactin levels by stimulating the serotonin-
cerebri, and metastatic disease. Primary hypothyroidism mediated neural reflex, mimicking suckling. Multiple
Reproductive Endocrinology and Infertility 137
drugs will increase prolactin, including dopamine antago- The presence of a pituitary macroadenoma (ie, size
nists (phenothiazines, metoclopramide, pimozide, sul- greater than 10 mm) requires additional evaluation with
piride), dopamine reuptake inhibitors (amphetamines, formal visual field testing and anterior pituitary func-
tricyclic antidepressants), and catecholamine-depleting tion testing consisting of testing for levels of insulin-like
drugs (reserpine, methyldopa). growth factor, morning and evening cortisol, follicle-
Generally, prolactin elevations result in suppression of stimulating hormone, luteinizing hormone, free thyrox-
GnRH amplitude and pulsatility and subsequent derange- ine, and free α-subunit. This testing serves to determine
ment of the hypothalamic–pituitary–gonadal axis. The anterior pituitary function and exclude production or
degree of estrogen suppression correlates with the level secretion of another hormone by the adenoma.
of prolactin and determines the presenting symptoms. Medical therapy with a dopamine agonist (bromocrip-
Mild elevations result in mild ovulatory dysfunction with tine, cabergoline) is indicated for women who do not
luteal phase insufficiency, moderate levels with irregular desire birth control and who have non-tumor-related
cycles and episodic anovulation, and high levels with hyperprolactinemia, microadenoma, or bothersome
amenorrhea. These symptoms correlate with increasing galactorrhea. Women with a macroadenoma require
levels of estrogen deficiency. Galactorrhea may be found immediate medical therapy and should use adequate
in up to 80% of women with hyperprolactinemia but is contraceptives until the tumor is smaller than 10 mm.
not specific to the disorder. Up to 40–50% of women Consultation with a neurosurgeon may be obtained for
with galactorrhea do not have elevated prolactin levels. women with substantial visual field defects or other CNS
Such women do not need further evaluation. Spontaneous symptoms.
resolution with avoidance of nipple stimulation is com- Patients with an initial negative MRI do not require
mon, and short-duration dopamine agonist may be help- repeat imaging unless new symptoms develop. Women
ful. Effects from pituitary tumors are correlated with size with a microadenoma who are not undergoing dopa-
and commonly include CNS findings such as headaches mine agonist therapy require repeat imaging because
and visual disturbances. approximately 7% will progress to a macroadenoma. No
Women with non-tumor-related hyperprolactinemia or consensus exists on the time interval of the first follow-
a microadenoma who lack bothersome galactorrhea or up scan, but 1 year seems prudent. In patients who are
have no desire for pregnancy may be treated with com- taking dopamine agonist therapy, normalization of the
bination hormonal contraceptives, which provide birth prolactin level correlates with absence of growth of a
control and prevent bone loss associated with estrogen microprolactinoma. However, because there is no way to
deficiency. A combination hormonal contraceptive, as prove a microadenoma is secreting prolactin (especially
opposed to a progestin-only oral contraceptive, should be with only mildly elevated blood prolactin), a repeat MRI
used for better bone protection. The best recommenda-
tion for the described patient is a combination hormonal
contraceptive with a repeat MRI and prolactin level test
in 1 year.
Because of the varied presentation of hyperprolac-
tinemia, prolactin level testing should be performed in
women with menstrual irregularity, galactorrhea, and
infertility. Blood should be drawn in the morning, fasting,
and during the presumed follicular phase in women who
have regular menses. The patient should avoid strenuous
exercise or sexual activity before testing. If the patient
has a borderline elevated prolactin level of 20–40 ng/mL,
the test should be repeated before further evaluation.
In cases without an obvious diagnosis from history or
physical examination, the next line of testing is a thyroid-
stimulating hormone level. Subsequent testing includes
head MRI to evaluate for a CNS lesion. Prolactin levels
correlate with the incidence of pituitary prolactinoma but
not with other lesions. There is no threshold prolactin
level for excluding a pituitary or CNS lesion, but lesions
are less likely with mildly elevated prolactin levels. For
this reason, other causes of mild hyperprolactinemia FIG. 97-1. Magnetic resonance imaging scan of a patient
with hyperprolactinemia showing a pituitary microaden-
should be excluded before considering MRI. oma. The top arrow shows the hypothalamus, the middle
Women with a pituitary microadenoma (ie, size less arrow shows the infundibulum, and the bottom arrow
than 10 mm) (Fig. 97-1) do not require additional testing. shows the microadenoma.
138 PROLOG
at 1 year often is performed to establish a correlation of Parkinson’s disease. Studies suggest no increase in clini-
prolactin normalization and tumor shrinkage. cally significant valvular disease in women treated with
The most common dopamine agonists used for treat- cabergoline for hyperprolactinemia.
ment are bromocriptine and cabergoline. Common
adverse effects include nausea, vomiting, postural hypo- Ben-Jonathan N, Hnasko R. Dopamine as a prolactin (PRL) inhibitor.
Endocrine Rev 2001;22:724–63.
tension, and headache. Cabergoline is better tolerated
Gillam MP, Molitch ME, Lombardi G, Colao A. Advances in the treat-
and more efficacious in regard to tumor shrinkage and ment of prolactinomas. Endocrine Rev 2006;27:485–534.
normalization of prolactin levels. Cabergoline does Steffensen C, Maegbaek ML, Laurberg P, Andersen M, Kistorp CM,
have one unique adverse effect: heart valve abnormali- Norrelund H, et al. Heart valve disease among patients with hyper-
ties leading to insufficiency. This has been seen only prolactinemia: a nationwide population-based cohort study. J Clin
Endocrinol Metab 2012;97:1629–34.
with high doses of cabergoline used for the treatment of
98
Ovarian androgen-secreting tumor
A nulliparous 45-year-old woman with amenorrhea visits your office. She reports a gradual
increase in hirsutism over the past 6 months. On clinical examination, excessive hair growth is
observed on her face, chest, lower back, thighs, and upper arms, with a Ferriman–Gallwey score
of 23. Her body mass index is 32 (calculated as weight in kilograms divided by height in meters
squared). She has a serum testosterone level of 350 pg/mL, sex hormone-binding globulin level of
35 pg/mL, and dehydroepiandrosterone sulfate (DHEAS) level of 85 micrograms/dL. Serum cor-
tisol and 17-hydroxyprogesterone levels are normal. Transvaginal ultrasonography reveals a 2-cm
solid mass in her left ovary. The best next step in management is
When a patient presents with features of atypical hirsut- quite often small. Any ovarian tumor has the ability to
ism or polycystic ovary syndrome (PCOS), an extensive stimulate the production of androgens indirectly by caus-
evaluation should be undertaken to exclude the 0.5% ing hyperplasia of the surrounding ovarian stroma, such as
of patients who have other causes of hyperandrogen- ovarian teratomas or Brenner tumors.
ism, such as congenital adrenal hyperplasia, androgen- Most androgen-secreting tumors are benign and in
secreting tumors of the ovary and adrenal gland, Cushing some rare instances require full staging. Less frequently,
syndrome, or hyperthecosis. Some of the unusual signs catheterization and sampling from adrenal and ovarian
or symptoms associated with these atypical presentations veins are required to elucidate the diagnosis or in cases
include the onset of symptoms before age 15 years or a in which no masses are identified by any type of imag-
new diagnosis of PCOS in the third decade of life or later, ing modality. Although this is a technically difficult
rapid progression of symptoms, and development of vir- procedure with potential complications, it may provide
ilization or a serum testosterone level in excess of twice definitive diagnostic information. Virilization or severe
the upper limit of the reference range (usually greater hyperandrogenemia are rarely associated with PCOS.
than 200 pg/mL). In women, the most common androgen- Clinical examination and a detailed history may be ade-
secreting tumors are ovarian, whereas in premenopausal quate to make the diagnosis of PCOS. However, measure-
women, the most common tumors are Sertoli–Leydig cell ment of serum testosterone, total testosterone, or an index
tumors, although lipoid and hilus cell tumors also may be of free testosterone is useful to screen for the rare patient
observed. In postmenopausal women, hilus cell tumors with severe hyperandrogenism, prompting evaluation for
are the most common androgen-secreting tumors and are diagnoses other than PCOS.
Reproductive Endocrinology and Infertility 139
Androgen-secreting ovarian tumors are frequently A computerized tomography scan of the adrenal gland
small, solid masses with nonspecific appearance. To is indicated as part of the evaluation of an androgen-
delineate the mass, it is best to perform a combination secreting adrenal tumor. This patient’s normal DHEAS
of transabdominal and transvaginal ultrasonography as level likely excludes this diagnosis because virtually all
part of the evaluation. An MRI scan also can be helpful; adrenal tumors associated with excess testosterone pro-
however, this modality is most useful in differentiating an duction have concurrent significant DHEAS elevation.
ovarian tumor from a uterine myoma or other pelvic mass It is also important to emphasize that doing unnecessary
and during pregnancy. For the described patient, laparo- adrenal gland imaging may result in unindicated inter-
scopic oophorectomy is the appropriate next step, given ventions. Nonfunctioning adrenal tumors are common,
her rapid-onset hirsutism and high testosterone level, and can be found in up to 2% of the general population, and
should prove curative in most cases. The final pathology do not secrete hormones.
will dictate if staging is necessary. If necessary, staging Ultrasonographic examination is the procedure of
can be performed as a separate procedure. choice when imaging ovaries in the evaluation of hyper-
An overnight dexamethasone suppression test is indi- androgenemia, whether in the diagnosis of polycystic
cated when there is suspicion for cortisol excess or as ovaries or tumors. The described patient underwent
part of an evaluation for hirsutism to exclude Cushing pelvic ultrasonography that showed a solid ovarian mass
syndrome from the differential diagnosis of PCOS. This that was clearly delineated; therefore, pelvic MRI will not
patient does exhibit some of the clinical signs of cortisol add information that is useful to the differential diagnosis.
excess, but the combination of a normal serum corti-
sol level and the very high testosterone level makes it Dennedy MC, Smith D, O’Shea D, McKenna TJ. Investigation of
unlikely that the correct diagnosis is Cushing syndrome. patients with atypical or severe hyperandrogenaemia including andro-
gen-secreting ovarian teratoma. Eur J Endocrinol 2010;162:213–20.
An adrenocorticotropic hormone stimulation test
Unluhizarci K, Kaltsas G, Kelestimur F. Non polycystic ovary syn-
would be indicated to confirm the diagnosis of adrenal drome-related endocrine disorders associated with hirsutism. Eur J Clin
hyperplasia if elevated serum 17-hydroxyprogesterone Invest 2012;42:86–94.
was identified as part of the evaluation for hirsutism or
PCOS. Given this patient’s normal 17-hydroxyprogester-
one level, further evaluation is unnecessary.
140 PROLOG
99
Ectopic pregnancy
A 24-year-old woman, gravida 2, para 0, comes to your office with an unknown last menstrual
period and a positive home pregnancy test. She reports postcoital spotting but no pelvic pain. Vital
signs obtained during her visit are within normal limits. Abdominal and pelvic examinations reveal
no tenderness, rebound, or guarding. The quantitative serum human chorionic gonadotropin (hCG)
level is 2,800 mIU/mL. Transvaginal ultrasonography confirms thickened endometrium without
evidence of an intrauterine gestational sac. A right paraovarian soft tissue mass is noted. Scant free
fluid is noted in the posterior cul-de-sac. On review of her electronic medical records, you note that
in an emergency department visit 2 nights before for nausea, her quantitative serum hCG level was
3,400 mIU/mL. The next step in management of this patient is
Early pregnancy failure is the most common complica- A dramatic change in management of ectopic preg-
tion of pregnancy. Approximately 25% of recognized nancy has occurred, with a shift from open surgical
pregnancies end in miscarriage and approximately 1–2% procedures transitioning through laparoscopic procedures
will be ectopic pregnancies. Undiagnosed ectopic preg- toward a current predominance of medical management.
nancy can lead to maternal morbidity and mortality, so Modern management should focus on ensuring that an
accurate and expeditious diagnosis is paramount. Ectopic intrauterine pregnancy is not interrupted as the result of
pregnancy can compromise a woman’s health and future diagnosis and treatment of a presumed ectopic pregnancy.
fertility and remains a leading cause of maternal morbid- The ultrasonographic results obtained for the described
ity and mortality, accounting for up to 6% of deaths of patient would suggest an ectopic pregnancy. However,
women during pregnancy. the findings must be put into clinical context with evalu-
Diagnosis of women who initially present with preg- ation of the serial quantitative serum hCG level with con-
nancy of unknown location requires multiple visits sideration given to the patient’s clinical circumstances.
for blood tests, ultrasonography, and possible lapa- The described patient’s serum hCG concentrations are
roscopy or D&C. During this diagnostic period, an falling and, therefore, close observation with serial serum
unidentified ectopic pregnancy could rupture, necessitat- hCG levels without immediate medical or surgical inter-
ing emergency intervention or contributing to maternal vention is warranted for this stable patient. Suction D&C
morbidity. Conversely, a small percentage of pregnan- and misoprostol would be appropriate only in the case of
cies of unknown location and ectopic pregnancies may a failed intrauterine pregnancy. Laparoscopy is not indi-
resolve spontaneously and not require medical or surgical cated at this time. Close follow-up is warranted.
intervention.
Modern management has resulted in a new set of pit- Autry AM. Medical treatment of ectopic pregnancy: is there something
new? Obstet Gynecol 2013;122:733–4.
falls, such as overinterpretation of a single ultrasound,
Barnhart KT. Early pregnancy failure: beware of the pitfalls of modern
misunderstanding of the utility of serial hCG values, and management. Fertil Steril 2012;98:1061–5.
inappropriate use of methotrexate that can result in iatro- Senapati S, Barnhart KT. Biomarkers for ectopic pregnancy and preg-
genic complications. An error can result in false reassur- nancy of unknown location. Fertil Steril 2013;99:1107–16.
ance that a woman does not have an ectopic pregnancy
or, conversely, interruption of a desired intrauterine preg-
nancy during diagnosis and management.
Reproductive Endocrinology and Infertility 141
100
Maternal virilization in hyperreactio luteinalis
A 28-year-old woman, gravida 1, comes to your office at 14 weeks of gestation with excessive
facial hair growth. To date, her pregnancy has been free of complications. Ultrasonography for
fetal organ evaluation shows adequate fetal growth and no fetal abnormalities. She has bilateral
ovarian enlargement 10 cm in diameter and a multicystic appearance to the ovaries. The most
likely diagnosis is
The incidence of hyperandrogenism in pregnancy is low. multiple ovulations and, when severe, can be associated
Affected women can have a variety of symptoms, includ- with ascites and pleural effusions. The condition typi-
ing hirsutism, acne, temporal balding, clitoromegaly, and cally regresses after the first trimester, and virilization is
deepening of the voice. The two most common causes of not seen with ovarian hyperstimulation syndrome. The
gestational hyperandrogenism are 1) luteomas of preg- degree of hirsutism in the hyperreactio luteinalis patient
nancy and 2) hyperreactio luteinalis. The most likely is measured by the Ferriman–Gallwey score through the
diagnosis in the described patient is hyperreactio lutei- use of representative images (Appendix D). This scoring
nalis, a condition characterized by moderate or marked system is used mainly for research purposes and is not a
cystic enlargement of the ovaries due to the formation of routinely used clinical tool.
multiple benign theca-lutein cysts. Hyperreactio luteina- Pregnancy luteomas are benign, solid, multinodular,
lis is a complication of pregnancy usually distinguished nonneoplastic lesions characterized by bilateral ovarian
by elevated serum human chorionic gonadotropin (hCG) enlargement during pregnancy that can simulate a tumor.
levels in conditions such as hydatidiform mole and cho- The normal ovarian parenchyma is replaced by solid pro-
riocarcinoma, although it may occur without markedly liferation of luteinized stromal cells under the influence
elevated hCG levels and has been reported in singleton of hCG. Because stromal cells produce androgen, viril-
pregnancies. ization of the mother can occur. Luteoma of pregnancy
Hyperreactio luteinalis is thought to represent an exag- is most often seen in African American multiparous
gerated ovarian response to hCG. It may occur at any women in the third or fourth decade of life. It is often
time in pregnancy, although most cases (54%) are noted asymptomatic and discovered incidentally during cesar-
in the third trimester. It is often bilateral but unilateral ean delivery.
cases have been described. Differentiation from malig- Granulosa cell tumors are rare tumors that arise from
nant conditions can be performed by radiologic studies, granulosa cells. Although granulosa cells normally occur
given that magnetic resonance imaging and ultrasonogra- only in the ovary, the peak age at which they occur is
phy show the ovaries to have a characteristic spoke-wheel 50–55 years.
appearance, representing compressed echogenic stroma Mature cystic teratomas (dermoid cysts) constitute the
surrounding multiple simple or minimally hemorrhagic most common tumor of the ovary to be identified in a
cysts without solid elements (Fig. 100-1; see color plate). pregnant patient. Dermoids often display a characteristic
As hCG levels regress after delivery of the pregnancy, the appearance as a uniformly solid and echogenic lesion or
ovaries slowly return to normal size without the need for as a complex cystic lesion with a focal echogenic nodule
surgical intervention. Fetal virilization during pregnancy that causes distal acoustic shadowing. They may be bilat-
does not occur because of placental and fetal aromatase eral in a minority of cases. They are of germ cell origin
activity. and do not produce androgens.
The morphologic appearance of hyperreactio luteinalis Ovarian hyperstimulation is a complication of ovarian
on ultrasonography is nearly identical to that of ovarian stimulation characterized by abdominal pain, bloating,
hyperstimulation syndrome, a condition that occurs in the and enlarged ovaries. If it presents in pregnancy, it occurs
first trimester, and is associated nearly exclusively with early in the first few weeks and typically resolves soon
ovulation stimulation with gonadotropins. This condi- thereafter. Hirsutism is not a presenting sign of ovarian
tion is associated with multiple corpus luteal cysts from hyperstimulation syndrome.
142 PROLOG
Angioni S, Portoghese E, Milano F, Melis GB, Fulghesu AM. Hirsutism Phelan N, Conway GS. Management of ovarian disease in pregnancy.
and hyperandrogenism associated with hyperreactio luteinalis in a sin- Best Pract Res Clin Endocrinol Metab 2011;25:985–92.
gleton pregnancy: a case report. Gynecol Endocrinol 2007;23:248–51. Yacobozzi M, Nguyen D, Rakita D. Adnexal masses in pregnancy.
Langer JE, Coleman BG. Case 1: Diagnosis: hyperreactio luteinalis Semin Ultrasound CT MR 2012;33:55–64.
complicating a normal pregnancy. Ultrasound Q 2007;23:63–6.
101
Osteoporosis and celiac disease
A postmenopausal woman reports to you with a diagnosis of osteoporosis for which she is taking
oral bisphosphonate. She reports a history of chronic diarrhea and celiac disease. In spite of her
bisphosphonate compliance, her bone mineral density (BMD) has worsened in comparison with her
previous screening. The best next step to optimize her BMD is
Celiac disease is a gluten-sensitive enteropathy charac- Prospective studies have shown significant improvement
terized by reversible small-bowel mucosal atrophy in a in BMD after introduction of a gluten-free diet. For the
genetically predisposed person resulting from an inappro- described patient, a gluten-free diet would decrease her
priate immune response to dietary gliadin, a component diarrhea, improve her gastrointestinal absorption, and
of wheat proteins. Celiac disease has long been associated have a greater effect on her BMD. It is paramount to insti-
with metabolic bone disease. It also has been associated tute a gluten-free diet first before initiating other medical
with infertility and pregnancy loss. Low BMD can be treatments. Thus, prescribing intravenous bisphosphonate
detected even in children and adolescents with newly would not be indicated as a first step. However, weight-
diagnosed celiac disease. The need to consider celiac bearing exercise along with supplemental calcium and
disease as a pathologic factor in individuals with osteopo- vitamin D should be encouraged. Calcitonin can be con-
rosis applies particularly to osteoporosis patients who fail sidered if bone loss continues in spite of adherence to the
to respond to standard treatment. gluten-free diet.
The American Gastroenterological Association recently
Bernstein CN, Leslie WD, Leboff MS. AGA technical review on
reviewed studies of osteoporosis in celiac disease accord- osteoporosis in gastrointestinal diseases. Gastroenterology 2003;124:
ing to standard levels of evidence. All such studies have 795–841.
shown low mean BMD around the time of diagnosis of Geraci A. Osteoporosis and celiac disease: is it useful to a new guideline
untreated celiac disease. A pooled analysis indicated very [editorial]? J Gastrointest Digest Syst 2011;1:e103.
low bone mass in approximately 40% of individuals in the Mazure R, Vazquez H, Gonzalez D, Mautalen C, Pedreira S, Boerr L,
et al. Bone mineral affection in asymptomatic adult patients with celiac
spine and 15% of individuals at the hip. disease. Am J Gastroenterol 1994;89:2130–4.
When patients with symptomatic celiac disease were Miller PD. Monitoring osteoporosis therapies. Curr Osteoporos Rep
compared with asymptomatic patients, symptomatic 2007;5:38–43.
patients were found to have significantly lower BMD.
Reproductive Endocrinology and Infertility 143
102
Intrauterine device complications with infection
A 35-year-old woman, gravida 2, para 2, visits your clinic for contraceptive counseling. She is a
healthy, monogamous woman who currently does not take any medications. She has had no prior
surgeries, abnormal Pap tests, pelvic infections, or other gynecologic problems. After discussing
several options, she expresses interest in an intrauterine device (IUD). When considering insertion
of an IUD in this patient, the most important way to prevent pelvic infection is
The IUD is an extremely safe and effective method of An approximate 10% risk of developing PID exists
contraception. In addition to its high efficacy for preg- if an IUD is inserted during an active cervical infection
nancy prevention, it is long acting and rapidly reversible, with Neisseria gonorrhoeae or Chlamydia trachomatis.
with minimal risks and adverse effects. It is a safe option For this reason, a patient’s medical history should be
for most women, and women who have contraindications evaluated before insertion of an IUD. If a patient is at
to the use of oral contraceptives should consider an IUD. high risk of infection or if cervicitis is noted on specu-
Although the IUD is the most commonly used method of lum examination, she should be screened for gonorrhea
reversible contraception worldwide, its usage is relatively and chlamydial infection, and IUD insertion should be
low in the United States, where approximately 2% of delayed. For women in whom the risk is unclear, screen-
women used IUD contraception in 2002 compared with ing for gonorrhea and chlamydial infection at the time
7.7% in 2009. This percentage, however, pales in com- of insertion with subsequent treatment if results are
parison to contraception use in other parts of the world positive is appropriate. However, in patients who are
(eg, it is estimated that in parts of Asia, 50% of women at low risk for cervicitis, such as the described patient,
use IUDs). A possible explanation of this low level of the risk of PID is relatively low. A study of 57,000 IUD
use in the United States is a history of negative publicity insertions between 2005 and 2009 found that only 0.5%
about IUDs, including inaccurate information about risks, of patients developed a significant pelvic infection. No
fears of litigation, and concerns about the mechanism difference in infection risk was observed for women who
of action to prevent pregnancy. A type of IUD from the were screened for gonorrhea and chlamydial infection
1970s was shown to be linked to an increased incidence before IUD insertion compared with those who were not
of pelvic inflammatory disease (PID). However, the screened. Therefore, it is not recommended to routinely
IUDs that are currently available have not been associ- screen low-risk women for subclinical cervical infection
ated with an increased risk of infection. before IUD insertion. Nonetheless, IUDs should always
An IUD generally is inserted in the office setting be inserted with the use of aseptic techniques, taking care
without sedation. The patient must be informed of the to adequately clean the cervix with povidone-iodine and
risks and benefits of the IUD and read the package bro- to maintain sterility for the procedure itself.
chure that comes with the device, then sign a written Antibiotic prophylaxis during IUD insertion is not
consent before insertion of the IUD. Reported complica- recommended for low-risk asymptomatic women. A
tions of insertion of an IUD include uterine perforation Cochrane review summarized data from four large ran-
and vasovagal response during insertion. If an IUD is domized controlled trials and concluded that there was no
inserted during an early, unrecognized pregnancy, it may significant decrease in the occurrence of PID in patients
result in a miscarriage. For this reason, an IUD is ideally who did or did not receive antibiotic prophylaxis (odds
inserted in the follicular phase, after cessation of menses, ratio = 0.89, 95% confidence interval, 0.53–1.51).
or in women who are using another reliable method of After insertion of the IUD, if there is any question
contraception, are abstinent, or who desire emergency about appropriate placement of the IUD, ultrasonography
contraception within 5 days of unprotected intercourse, can help confirm the location of the device. For example,
which is a U.S. Food and Drug Administration-approved if there is severe discomfort or bleeding during or after
indication for a copper-containing IUD. placement of an IUD, or if the IUD string is missing,
144 PROLOG
pelvic ultrasonography can help evaluate for misplace- Sufrin CB, Postlethwaite D, Armstrong MA, Merchant M, Wendt JM,
Steinauer JE. Neisseria gonorrhea and Chlamydia trachomatis screen-
ment or uterine perforation. Routine ultrasonography ing at intrauterine device insertion and pelvic inflammatory disease.
before insertion is not indicated. Obstet Gynecol 2012;120:1314–21.
103
Androgen disorders
A 19-year-old woman with primary amenorrhea and lack of breast development visits your clinic.
She is 1.73 m (68 in.) tall and weighs 63.5 kg (140 lb). On physical examination, she has Tanner
stage 1 breast development and Tanner stage 1 pubic hair. Genital examination reveals a cervix.
Her follicle-stimulating hormone and luteinizing hormone levels are 76 mIU/mL and 64 mIU/mL,
respectively. Her estradiol level is less than 20 pg/mL. The most likely diagnosis is
Primary amenorrhea is the lack of menstrual cycles after Therefore, these individuals will be born phenotypically
age 15 years and lack of development of secondary sexual female with female internal genitalia. In addition, these
characteristics. Amenorrhea can be secondary to several individuals do not have a functional gonad but rather have
etiologies, such as hypothalamic–pituitary causes, ovar- a streak of fibrous tissue called a streak gonad. Because
ian failure, anovulation secondary to polycystic ovary of the lack of functional gonads, the gonadotropin levels
syndrome, and anatomic abnormalities that affect the are elevated and there are undetectable ovarian steroid
outflow tract. hormone (estradiol) levels. The diagnosis is made when
Swyer syndrome is a form of gonadal dysgenesis. The a patient with this presentation has a 46,XY karyotype.
genetic sex of the affected individual is 46,XY, and at Treatment of these individuals starts with psychologic
birth the neonate is phenotypically female and does not counseling as well as pelvic ultrasonography to exclude
appear to have any sexual ambiguity. Swyer syndrome growing gonadal tumors. The presence of Y chromosome
patients are raised as females and typically are seen in material places the patient at risk of gonadal tumors such
puberty with primary amenorrhea and lack of develop- as gonadoblastomas and gonadal dysgerminomas. The
ment of secondary sexual characteristics. Approximately risk of tumor development can be as high as 75%. For this
10–20% of cases are a result of SRY gene mutations. reason, once diagnosis is made, the patient should undergo
Other genes, such as SF1, WT1, and DAX1, are possible gonadectomy. The gonads should be sent for pathology
candidates in the interference of sex determination. The in case tumors are noted. Secondary sexual development
remaining cases are of unknown origin. Because the SRY should be addressed using estrogen replacement with ini-
gene is responsible for development of the testes, and later tial low-dose estradiol or conjugated equine estrogen with
the Sertoli cells which produce antimüllerian hormone, interval increases in the dose until full breast development
the mutations in SRY lead to the continued development is achieved, at which time progesterone should be added or
of the müllerian ducts (uterus, fallopian tubes, cervix, and combination oral contraceptives can be used. Fertility still
upper vagina). The lack of testicular development and tes- can be achieved with the use of donor oocytes.
tosterone production does not allow for further wolffian The described patient most likely has Swyer syndrome.
duct development or virilization of the external genitalia. In androgen insensitivity syndrome, physical examination
Reproductive Endocrinology and Infertility 145
will reveal breast development, a blind-ending vagina, Jorgensen PB, Kjartansdottir KR, Fedder J. Care of women with XY
karyotype: a clinical practice guideline. Fertil Steril 2010;94:105–13.
and a lack of female internal genitalia. Individuals with
Lim HN, Freestone SH, Romero D, Kwok C, Hughes IA, Hawkins
Kallmann syndrome typically have low gonadotropin JR. Candidate genes in complete and partial XY sex reversal: mutation
levels. Turner syndrome patients are typically short and analysis of SRY, SRY-related genes and FTZ-F1. Mol Cell Endocrinol
may have other physical stigmata, such as a webbed neck, 1998;140:51–8.
a shield chest, and a wide-carrying angle. Some patients McElreavey K, Fellous M. Sex determination and the Y chromosome.
Am J Med Genet 1999;89:176–85.
with mosaic Turner syndrome may achieve normal
Zielinska D, Zajaczek S, Rzepka-Gorska I. Tumors of dysgenetic
height. Mosaic Turner syndrome could be considered in gonads in Swyer syndrome. J Pediatr Surg 2007;42:1721–4.
the differential diagnosis of the patient under discussion.
104
Metabolic syndrome
A 36-year-old woman, gravida 2, para 1, aborta 1, has gained a significant amount of weight
since she had a 10-week miscarriage 1 year ago; her weight gain was 17.2 kg (38 lb) over the past
15 months. She is 1.65 m (65 in.) tall and now weighs 88.4 kg (198 lb). Her body mass index is 32.9
(calculated as weight in kilograms divided by height in meters squared). She has recently noticed
malaise and the need to urinate more frequently. She has normal menstrual cycles with an interval
of 27–29 days and no gynecologic complaints. Her blood pressure is 142/92 mm Hg. Her waist
circumference is 99 cm (39 in.). She has acanthosis nigricans present in her axilla and at the nape
of her neck. She does not have abdominal striae or hyperpigmentation. Her fasting blood glucose
level is 116 mg/dL, her thyroid-stimulating hormone level is 1.1 mU/mL, her plasma triglyceride
level is 171 mg/dL, and her testosterone level is 30 ng/dL. The most likely diagnosis is
Metabolic syndrome, also known as syndrome X or metabolic syndrome still may be at very high risk of car-
insulin resistance syndrome, is associated with a cluster diovascular disease. The aberrations attributed to metabolic
of risk factors for atherosclerotic cardiovascular disease. syndrome are also risk factors for cardiovascular disease.
When first described, the syndrome constellation included “Insulin resistance syndrome” is also an unsatisfactory
obesity, type 2 diabetes mellitus, hypertension, and ath- term because insulin-mediated glucose uptake varies
erogenic dyslipidemia. Over the past 30 years, however, greatly. Individuals may have elevated insulin without
several definitions for metabolic syndrome have been hyperglycemia or else hyperglycemia without hyperin-
proposed (Table 104-1). sulinemia. Moreover, persons with normal insulin and
Although the American Diabetes Association has cho- glucose levels may be at significant risk of cardiovascular
sen to drop the term “metabolic syndrome,” the American disease. Not all insulin-resistant hyperinsulinemic indi-
College of Cardiology continues to use the term and viduals show the other features of metabolic syndrome.
believes that it helps frame and conceptualize patient man- The described patient has four of the five features
agement. The central question is whether there is a unique needed for diagnosis of metabolic syndrome according
pathophysiology for this “syndrome” and whether meta- to the National Cholesterol Education Program: 1) waist
bolic syndrome better estimates individual risk beyond circumference greater than 88 cm (35 in.) in women,
combining the individual components using a risk assess- 2) plasma triglycerides elevated over 150 mg/dL, 3) blood
ment tool, such as the Framingham Point Score for cardio- pressure over 130/85 mm Hg, and 4) fasting blood glu-
vascular disease. Moreover, individuals who do not have cose over 110 mg/dL. The fifth criterion (which does not
the exact combination of features to merit a diagnosis of apply to this case) is a high-density lipoprotein cholesterol
Reproductive
146 Endocrinology and Infertility 41
PROLOG
Blood
Pressure
Blood Central Diagnostic
Agency (BP)
Pressure Lipids Obesity Glucose Other Requirements
WHO On anti- Plasma Body mass index* IGT or as in Microalbuminuria Type 2 diabetes
hypertensive triglycerides over 30 or more and/ type 2 UAE 20 mellitus or IGT
therapy or BP 150 mg/dL and/ or waist–hip ratio diabetes micrograms/min and two
140/90 mg
mm HgHg or HDL-C less more than 0.9 or mellitus or more or additional
or more than 35 mg/dL higher in men or A:Cr 30 mg/g abnormalities;
in men or less greater than 0.85 or more if GT normal,
than 39 mg/dL in women three required
in women
NCEP BP 130/85 mg
mm Plasma Waist FBG Three or more of
ATP III Hg or
HG or greater
greater triglycerides over circumference 110 mg/dL criteria listed
150 mg/dL and/ greater than or more
or HDL-C less 102 cm (40 in.)
than 40 mg/dL in (men), greater
men or less than than 88 cm (35 in.)
50 mg/dL in (women)
women
AACE Hypertension Plasma Waist IFG or as in Insulin resistance, Not specified
triglycerides over circumference type 2 acanthosis
150 mg/dL and/or 102 cm (40 in.) or diabetes nigricans,
HDL-C less than greater in men, mellitus hyperuricemia.
35 mg/dL in men 88 cm (35 in.) or Minor criteria:
or less than greater in women hyper-
45 mg/dL in coagulability,
women CHD, PCOS,
vascular
endothelial
dysfunction,
microalbuminuria
IDF BP 130/85 mgmm Plasma Waist FBG
Hg or
HG or less
less triglycerides over circumference 110 mg/dL
150 mg/dL and/or greater than or greater
HDL-C less than 102 cm (40 in.)
40 mg/dL in men (men), greater
or less than than 88 cm (35 in.)
50 mg/dL in (women)
women
Abbreviations:
WHO, AACE,
World Health American Association
Organization; NCEP ATPofIII,Clinical Endocrinologists;
National A:Cr, albumin:creatine;
Cholesterol Education BP, bloodPanel
Program Adult Treatment pressure; CHD,American
III; AACE, coronary
heart disease;
Association FBG, fasting
of Clinical blood glucose;
Endocrinologists; IDF,HDL-C, high-density
International lipoprotein
Diabetes cholesterol;
Foundation; BP, blood IDF, International
pressure; HDL-C,Diabetes Foundation;
high density IFG,
lipoprotein
impaired fasting
cholesterol; glucose; glucose
IGT, impaired IGT, impaired glucose
tolerance; FBG,tolerance; NCEP
fasting blood ATP III,IFG,
glucose; National Cholesterol
impaired Education
fasting glucose; UAE,Program
urinaryAdult Treatment
albumin excre-
PanelA:Cr,
tion; III; PCOS, polycystic ovary
albumin:creatinine; CHD,syndrome;
coronaryUAE,
hearturinary albumin
disease; PCOS,excretion;
polycysticWHO,
ovaryWorld Health Organization.
syndrome.
*Body mass index, weight in kilograms divided by height in meters squared (kg/m2).
level of lesslisted.
conditions than 50 The mg/dL
mostinlikely
women. These criteria
diagnosis in the iden-
des- drogenism alsoofaretherecognized
native definitions features.thePatients
metabolic syndrome: are often
Insulin Resistance
tify a patient is with obesity,syndrome.
hypertension, diabetes mel- Atherosclerosis
obese, Study. Circulation
with acanthosis 2005;112:3713–21.
nigricans. Hirsutism is rarely as
cribed metabolic
litus, and dyslipidemia. This constellation of symptoms Kahn R,asBuse
severe thatJ,seenFerrannini E, Stern M,
in congenital American
adrenal Diabetes
hyperplasia.
Executive Summary of Thesyndrome.
Third Report of The National Cholesterol Association, European Association for the Study of Diabetes. The
comprises metabolic Because menstrual
metabolic syndrome: timeirregularities are common
for a critical appraisal: in PCOS
joint statement from
Education Program (NCEP) Expert Panel on Detection, Evaluation,
AndLate-onset congenital
Treatment of High adrenalInhyperplasia
Blood Cholesterol is due
Adults (Adult Treatment and the woman
the American described
Diabetes Associationhasandnormal cycles,
the European PCOS isfora
Association
to a III).
Panel defect
Expert and
Panel insufficiency in 21α-hydroxylase,
on Detection, Evaluation, and Treatment of the Study
less of Diabetes
likely diagnosis. [Review]. Diabetes Care 2005;28:2289–304.
High Blood Cholesterolor
11β-hydroxylase, in Adults. JAMA 2001;285:2486–97.
3β-hydroxysteroid (type II) dehy- LiuCushing syndromeW, is
J, Grundy SM, Wang theSCpersistent
Smith Jr, Vega GL,overproduction
Wu Z, et al. Ten-
Gabir MM, Hanson RL, Dabelea D, Imperatore G, Roumainandrogen
J, Bennett year risk of cardiovascular incidence related to diabetes, prediabetes,
drogenase enzymes with resultant excessive of cortisol, usually because of excess adrenocortico-
and the metabolic syndrome. Am Heart J 2007;153:552–8.
PH, et al. The 1997 American Diabetes Association and 1999 World
production.
Health Although
Organization criteriaobesity and fatigue
for hyperglycemia in themay be present,
diagnosis and pre- tropic hormone or adrenocorticotropic hormone-secreting
Mitka M. Does the metabolic syndrome really exist? Diabetes and
the hallmark
diction is Diabetes
of diabetes. severe Care
hirsutism and varying degrees of
2000;23:1108–12. tumors.
heart diseaseThe features
groups spar overof Cushing
issue. syndrome include
JAMA 2005;294:2010–3.
virilization.
Grundy SM, Cleeman JI, Daniels SR, Donato KA, Eckel RH, Franklin central
Orchard TJ,obesity
TemprosaandM,fat deposits
Goldberg in theS,cheeks,
R, Haffner Ratner R,dorsocer-
Marcovina
BA,Polycystic
et al. Diagnosis
ovary andsyndrome
managementisofseenmetabolic syndrome:
in women an
with vical
S, et al.area (buffalo
The effect hump),andand
of metformin supraclavicular
intensive regions.
lifestyle intervention on
American Heart Association/National Heart, Lung and Blood Institute
varying Statement.
degrees Circulation
of anovulation and androgen excess. the metabolic
The limbs aresyndrome:
thin andthe Diabetes
wasted,Prevention Program is
and hirsutism randomized
rarely a
Scientific 2005;112:2735–52.
trial. Ann Intern Med 2005;142:611–9.
Hyperinsulinemia
Hanley AJ, Karter AJ, or insulinK, resistance
Williams and hyperan-
Festa A, D’Agostino RB Jr, feature. Hyperpigmentation is confined to skin creases
Wagenknecht LE, et al. Prediction of type 2 diabetes mellitus with alter-
Reproductive Endocrinology and Infertility 147
and pressure points. The striae from Cushing syndrome an American Heart Association/National Heart, Lung, and Blood
Institute Scientific Statement. American Heart Association and the
are violaceous, whereas those from simple obesity are National Heart, Lung, and Blood Institute [published errata appear
silvery. in Circulation 2005;112:e298. Circulation 2005;112:e297]. Circulation
Autoimmune thyroiditis commonly presents with 2005;112:2735–52.
weight gain, fatigue, facial puffiness, and peripheral Hanley AJ, Karter AJ, Williams K, Festa A, D’Agostino RB Jr,
Wagenknecht LE, et al. Prediction of type 2 diabetes mellitus with alter-
edema. Hirsutism is rarely a feature, and indeed hair loss native definitions of the metabolic syndrome: the Insulin Resistance
often is reported with the condition. Atherosclerosis Study. Circulation 2005;112:3713–21.
Hyperlipidemia may be present with all of the condi- Kahn R, Buse J, Ferrannini E, Stern M. The metabolic syndrome:
tions listed. The most likely diagnosis in the described time for a critical appraisal: joint statement from the American
Diabetes Association and the European Association for the Study of
patient is metabolic syndrome. Diabetes. American Diabetes Association and the European Association
for the Study of Diabetes. Diabetes Care 2005;28:2289–304.
Executive Summary of The Third Report of The National Cholesterol
Liu J, Grundy SM, Wang W, Smith SC Jr, Vega GL, Wu Z, et al. Ten-
Education Program (NCEP) Expert Panel on Detection, Evaluation,
year risk of cardiovascular incidence related to diabetes, prediabetes,
And Treatment of High Blood Cholesterol In Adults (Adult Treatment
and the metabolic syndrome. Am Heart J 2007;153:552–8.
Panel III). Expert Panel on Detection, Evaluation, and Treatment of
High Blood Cholesterol in Adults. JAMA 2001;285:2486–97. Mitka M. Does the metabolic syndrome really exist? Diabetes and heart
disease groups spar over issue. JAMA 2005;294:2010–3.
Gabir MM, Hanson RL, Dabelea D, Imperatore G, Roumain J, Bennett
PH, et al. The 1997 American Diabetes Association and 1999 World Orchard TJ, Temprosa M, Goldberg R, Haffner S, Ratner R, Marcovina
Health Organization criteria for hyperglycemia in the diagnosis and S, et al. The effect of metformin and intensive lifestyle intervention on
prediction of diabetes. Diabetes Care 2000;23:1108–12. the metabolic syndrome: the Diabetes Prevention Program randomized
trial. Diabetes Prevention Program Research Group. Ann Intern Med
Grundy SM, Cleeman JI, Daniels SR, Donato KA, Eckel RH, Franklin
2005;142:611–9.
BA, et al. Diagnosis and management of the metabolic syndrome:
105
Complications of robotic-assisted surgery
A 45-year-old multiparous woman comes to your office for a preoperative visit in preparation for
a robotic-assisted total hysterectomy for symptomatic uterine fibroids. She has heard that hysterec-
tomy can be associated with small-bowel obstruction, bladder injury, increased blood loss, vaginal
cuff dehiscence, and brachial plexus injury. As part of informed consent, you inform her that of
all of the complications named, relative to laparoscopic-assisted surgery, robotic-assisted surgery
increases her risk of
Hysterectomy for benign gynecologic disease is per- The robotic platform was originally developed by
haps one of the most commonly performed gynecologic the Stanford Research Institute, the U.S. Department
surgical procedures; approximately one in nine women of Defense, and the National Aeronautics and Space
in the United States will ultimately have this procedure Administration in an attempt to facilitate telesurgery for
performed. Hysterectomies were originally performed via wounded soldiers. The intent was to position the operat-
laparotomy or vaginally. In the late 1980s, reports linking ing surgeon remote to the battlefield to perform surgery in
laparoscopic procedures with shorter hospital stays and the battlefield operating room through robotic assistance.
faster recoveries led to the inclusion of this surgical tech- Although telesurgery was found to be technically possible,
nique among the possible surgical options. More recently, its practical use was limited by several factors, including
robotic-assisted hysterectomy has been introduced as an telecommunication bandwidth requirements. Robotic-
alternative minimally invasive approach to traditional assisted laparoscopic surgery was further developed,
hysterectomy. commercialized, and approved by the U.S. Food and Drug
148 PROLOG
Administration in 2001 for urologic procedures, in 2002 of the patient on the surgical table. Intraoperative injuries
for thoracic surgery, and in 2005 for gynecologic proce- to the urinary system are equally common in laparoscopic
dures. Since Food and Drug Administration approval in and robotic procedures. A recent Cochrane review of
2005, thousands of gynecologic surgeons have received randomized controlled trials of robotic surgery for benign
training in this alternative technique. Supporters of the gynecologic conditions found no differences in intraop-
procedure argue that robotic assistance allows physicians erative complications between robotic and laparoscopic
to be more comfortable in applying a minimally invasive surgery. However, robotic surgery led to a higher inci-
technique, whereas in the past a laparotomy would have dence of postoperative complications (odds ratio =5.44,
been performed. 95% confidence interval, 1.57–18.82; P=.008), including
A recent cohort study of 264,758 women who under- urinary tract infection, small-bowel obstruction, wound
went hysterectomy for benign gynecologic disorders infection, erosion, abdominal wall pain necessitating trig-
at 441 hospitals across the United States in 2007–2010 ger point injection, and abscess. The authors concluded
showed that the use of robotic-assisted hysterectomy that robotic surgery was not associated with improved
increased from 0.5% to 9.5% of all hysterectomies in effectiveness or safety but that it was associated with
2010 with a concurrent decrease in the rate of abdomi- substantially increased costs.
nal hysterectomy. Although patients who underwent a
robotic-assisted hysterectomy had shorter stays, the trans- Kashani S, Gallo T, Sargent A, Elsahwi K, Silasi DA, Azodi M.
Vaginal cuff dehiscence in robotic-assisted total hysterectomy. JSLS
fusion requirements, death rates, and rates of discharge 2012;16:530–6.
to a nursing facility were similar. Total costs associated Liu H, Lu D, Wang L, Shi G, Song H, Clarke J. Robotic surgery for
with robotic-assisted hysterectomy were higher than for benign gynaecological disease. Cochrane Database of Systematic
laparoscopic hysterectomy. Reviews 2012, Issue 2. Art. No.: CD008978. DOI: 10.1002/14651858.
CD008978.pub2.
The rate of vaginal cuff dehiscence is comparable with
Wright JD, Ananth CV, Lewin SN, Burke WM, Lu YS, Neugut AI, et
other gynecologic techniques for hysterectomy. There are al. Robotically assisted vs laparoscopic hysterectomy among women
no reports of an increased risk of brachial plexus injury with benign gynecologic disease. JAMA 2013;20:689–98.
with robotic-assisted procedures despite the positioning
Reproductive Endocrinology and Infertility 149
106
Heavy menstrual bleeding
A 37-year-old woman, gravida 3, para 3, comes to your office with a 1-year history of heavy men-
strual bleeding. The most recent episode started 2 weeks ago and ended last night. She describes
the bleeding as heavy with passage of small clots. On examination, her uterus is normal in size
and no bleeding from the cervix is noted. Pelvic ultrasonography is normal and she has a negative
urine pregnancy test. Her thyroid-stimulating hormone level is normal and her hemoglobin level is
10 g/dL. She reports no orthostatic symptoms. The most effective nonsurgical management to
reduce the effects of vaginal bleeding on her quality of life is
Heavy menstrual bleeding is blood loss during menses in Although initially developed as a contraceptive
excess of 80 mL. It is a common problem that can sig- method, the levonorgestrel IUD can provide treatment
nificantly affect a woman’s quality of life. In the United for heavy menstrual bleeding. The levonorgestrel IUD
States, heavy menstrual flow accounts for approximately has been shown to cause greater reduction in menstrual
18% of visits to the gynecologist. The quantification of blood loss compared with other hormonal and nonhor-
blood loss during menses can be difficult to assess, given monal treatments and is more effective in reducing the
that it relies on the patient’s perception of the amount of effects of heavy menstrual bleeding on the patient’s qual-
blood lost and how she feels about its effects on her qual- ity of life. In addition to its effects on quality of life, the
ity of life. Appendix C shows the differential diagnosis levonorgestrel IUD is an effective reversible contracep-
of abnormal uterine bleeding. After an evaluation is com- tive that lacks any negative metabolic effects, does not
pleted, treatment of the underlying cause of excessive require daily use, and has effects that are not dependent
bleeding is performed. on patient adherence.
Heavy menstrual bleeding may be addressed with
either medical or surgical therapy. Surgery can be per- Endrikat J, Vilos G, Muysers C, Fortier M, Solomayer E, Lukkari-Lax
E. The levonorgestrel-releasing intrauterine system provides a reliable,
formed in cases in which removal of anatomic lesions is long-term treatment option for women with idiopathic menorrhagia.
necessary or, if the patient does not desire future fertility, Arch Gynecol Obstet 2012;285:117–21.
she may choose either endometrial ablation or hysterec- Gupta J, Kai J, Middleton L, Pattison H, Gray R, Daniels J; ECLIPSE
tomy. Medical treatment is less invasive and can provide Trial Collaborative Group. Levonorgestrel intrauterine system versus
medical therapy for menorrhagia. N Engl J Med 2013;368:128–37.
the patient with fertility-sparing treatment. Some patients
Higham JM, Shaw RW. Clinical associations with objective menstrual
are poor surgical candidates and may have to rely on blood volume. Eur J Obstet Gynecol Reprod Biol 1999;82:73–6.
medical interventions only. Munro MG, Critchley HO, Broder MS, Fraser IS; FIGO Working
Medical management can be hormonal or nonhor- Group on Menstrual Disorders. FIGO classification system (PALM–
monal. Hormonal therapies include combination hor- COEIN) for causes of abnormal uterine bleeding in nongravid women
of reproductive age. Int J Gynaecol Obstet 2011;113:3–13.
monal OCs, progestin-only OCs, and the levonorgestrel
Nicholson WK, Ellison SA, Grason H, Powe NR. Patterns of ambula-
IUD. The disadvantages of OCs are that they require daily tory care use for gynecologic conditions: a national study. Am J Obstet
administration, carry the risks of development of hyper- Gynecol 2001;184:523–30.
coagulability and deep vein thrombosis, and may not Stewart A, Cummins C, Gold L, Jordan R, Phillips W. The effective-
completely control the patient’s bleeding. Nonhormonal ness of the levonorgestrel-releasing intrauterine system in menorrhagia:
a systematic review. BJOG 2001;108:74–86.
methods include the use of tranexamic acid and nonste-
Whiteman MK, Kuklina E, Jamieson DJ, Hillis SD, Marchbanks PA.
roidal antiinflammatory drugs, eg, naproxen and ibupro- Inpatient hospitalization for gynecologic disorders in the United States.
fen. Tranexamic acid, although effective, is expensive Am J Obstet Gynecol 2010;202:541.e1–6.
and requires patient compliance for effectiveness.
150 PROLOG
107
Virilization due to Sertoli–Leydig cell tumor
A 28-year-old woman has a 3-month history of temporal scalp balding and amenorrhea.
Transvaginal ultrasonography reveals a 3-cm right solid ovarian mass, and a computed tomogra-
phy scan shows normal adrenal glands. The hormone that is most likely to be elevated is
Androgen-producing tumors that lead to signs of viriliza- of the associated hormone production of Sertoli–Leydig
tion can be isolated to an ovarian origin by a combination cell tumors (primarily androgen secretion and some-
of diagnostic imaging and an elevated serum testosterone times estrogen secretion), the presence of characteristic
level. The hallmark changes of virilization include tem- endocrine-mediated symptoms contributes to the diag-
poral scalp balding, deepening of the patient’s voice, and nosis of such tumors. Approximately two thirds of cases
the presence of clitoromegaly (abnormally enlarged clito- occur in women younger than 30 years, but these tumors
ris with a diameter greater than 10 mm). These changes can occur throughout a woman’s lifetime. The tumors
are in addition to facial and body hirsutism. The findings are typically unilateral with approximately 1.5% occur-
raise concern for an adrenal or ovarian tumor. Usually, ring bilaterally, and 90% are Stage 1 at diagnosis. The
the virilizing changes are sudden in onset and represent a overall prognosis of Sertoli–Leydig cell tumors is good
marked change noticed by the patient or family members. for Stage 1 well-differentiated neoplasms. Conservative
It often has been stated that one can diagnose the pres- surgery is an option for women with such tumors who
ence of an androgen-secreting tumor by patient history wish to preserve their fertility. Some Sertoli–Leydig
alone. cell tumors have primarily estrogenic symptoms such as
Imaging in combination with serum hormone testing postmenopausal or anovulatory bleeding; other types of
is helpful in localizing the source of excess androgen these tumors show no obvious endocrine manifestations.
secretion. Tumors within the adrenal glands are associ- The Sertoli–Leydig cell tumors with minimal hormonal
ated with an elevated serum DHEAS level of greater alterations are often larger tumors (greater than 10 cm
than 700 micrograms/dL. Because 95% of circulating in diameter), may be associated with tumor rupture, and
DHEAS is produced by the adrenal glands, DHEAS are more likely to be poorly differentiated. When a tumor
makes an ideal marker for adrenal function. Tumors exhibits no endocrine changes, this finding may indicate
within an ovary are associated with an elevated serum the presence of a tumor with a more aggressive biologic
testosterone level of greater than 200 ng/dL. The ovaries behavior.
produce approximately 50% of circulating testosterone, In the described patient, the presence of signs of
with the sources almost equally divided between direct virilization and an ovarian mass mean that testosterone
ovarian secretion and peripheral conversion of andro- will be the most elevated hormone. If the patient had an
stenedione to testosterone. Once the source of the andro- exclusive adrenal mass with normal-sized ovaries, the
gen excess is localized to either the adrenal glands or serum DHEAS level would be elevated. The presence of
ovaries, imaging can frequently narrow down the origin virilization (temporal scalp balding) makes other etiolo-
to a discrete mass. When the exact location of the tumor gies of general hirsutism such as adult-onset congenital
is in doubt, selective venous catheterization of the adrenal adrenal hyperplasia and Cushing syndrome less likely.
glands and the ovaries can be helpful in deciding which These two causes are associated with elevations in serum
organ should be explored further. This is especially 17-hydroxyprogesterone and cortisol, respectively. A
important in the case of an ovarian tumor when future fer- serum 17α-hydroxypregnenolone level would be elevated
tility is desired. In contrast, if the androgen excess occurs in comparison with a 17α-hydroxyprogesterone level in
in a postmenopausal woman without a discrete adrenal a patient with a 3β-hydroxysteroid dehydrogenase defi-
mass, bilateral oophorectomy would be preferable. ciency, which is a cause of adult-onset congenital adrenal
Ovarian Sertoli–Leydig cell tumors are rare, account- hyperplasia, but not in this setting of virilization with an
ing for approximately 0.5% of all ovarian tumors. Because ovarian mass.
Reproductive Endocrinology and Infertility 151
Gui T, Cao D, Shen K, Yang J, Zhang Y, Yu Q, et al. A clinico- Weng CS, Chen MY, Wang TY, Tsai HW, Hung YC, Yu KJ, et al.
pathological analysis of 40 cases of ovarian Sertoli-Leydig cell tumors. Sertoli-Leydig cell tumors of the ovary: a Taiwanese Gynecologic
Gynecol Oncol 2012;127:384–9. Oncology Group study. Taiwan J Obstet Gynecol 2013;52:66–70.
Petersons CJ, Burt MG. The utility of adrenal and ovarian venous sam-
pling in the investigation of androgen-secreting tumours. Intern Med J
2011;41:69–70.
108
Functional hypothalamic amenorrhea and osteoporosis
A 17-year-old nulligravid adolescent with secondary amenorrhea for 10 months comes to your
office with her mother. She exercises for 2 hours daily and has been actively working on weight
loss for the past 2 years, with a drop in weight from 54.4 kg to 43.5 kg (120 lb to 96 lb). Her height
is 1.57 m (62 in.) and her current body mass index is 17.6 (calculated as weight in kilograms
divided by height in meters squared). A dual-energy X-ray absorptiometry (DXA) scan shows that
her bone mineral density is 3 standard deviations below the mean for her age group. In addition
to nutritional and psychological counseling, the most appropriate medical option for this patient is
Personal, behavioral, and environmental factors that can women obtain 95% of their total lifetime bone mass by
disrupt the normal function of the hypothalamic–pitu- age 18 years. Young athletes may alter their bone density
itary–ovarian axis may cause functional amenorrhea. in an unevenly distributed fashion, with normal bone den-
Hypothalamic function typically is impaired, although sity in weight-bearing sites and decreased bone density
women with intact hypothalamic pathways may have in non-weight-bearing sites. These young women may
missed or absent menses. The degree of disruption never obtain an optimal bone mass, predisposing them to
depends on the inherent stability of the hypothalamic– significant osteoporosis later in life.
pituitary–ovarian axis and the severity of the imposed In premenopausal women, the z-score derived from
stress. Physical stress, emotional disturbances, illness, a DXA scan should be used to evaluate bone mineral
weight loss, and exercise have all been implicated as density at the lumbar spine and hip. An important clinical
causes of hypothalamic hypogonadism. The term “female advantage of the DXA scan compared with other types
athlete triad” refers to the combination of inadequate of bone mineral density measurement is that its ability to
caloric balance, amenorrhea, and osteoporosis and usually identify patients at risk of fracture has been assessed and
is seen in patients with self-imposed severe caloric restric- proven in a large number of epidemiologic studies. The
tion combined with extreme athletic output. Alterations in z-score represents the bone mass compared with individu-
neuropeptides, neurotransmitters, and neurosteroids can als of similar age and sex. A z-score is concerning when it
cause gonadotropin-releasing hormone pulsatile secretion is two or more standard deviations below the mean.
to cease. In the early phases of the female athlete triad, the The first step in treatment of young women with the
patient may maintain normal estrogen and gonadotropin female athlete triad should be to restore normal energy
levels, but in the later phases, the hormone levels drop to balance. Higher caloric intake is likely needed to restore
subphysiologic levels. bone mass and should include vitamin D, calcium,
Young women with the female athlete triad can have trace minerals, and adequate protein intake. Nutritional
low estrogen levels and subsequently low bone mass, counseling and often psychologic support and behavior
putting them at risk of pathologic stress fractures. In therapy are essential to reversing this challenging disease.
addition, young women with the female athlete triad have Oral contraceptives can provide estrogen to these
poor bone development at precisely the time they should women to help improve bone density. Although the best
be obtaining peak bone mass. It has been estimated that strategy to foster better bone health is through improved
152 PROLOG
nutritional status, this strategy can take time, and some women, these medications offer no advantage over
patients are extremely resistant to diet and exercise modi- exogenous estrogen or OCs. Parathyroid hormone has
fication. Hormonal therapy and OC use can stabilize bone been used in one trial to treat bone loss in women with
loss but do not reverse prior bone loss. Extensive counsel- endometriosis who take gonadotropin-releasing hormone
ing should occur when prescribing OCs to these patients. agonists. However, parathyroid hormone should be used
Women with the female athlete triad must understand with caution in young women who may have open
that OC use will result in cyclic bleeding, but that the epiphyses and would not be the first-line option for the
resumption of bleeding is iatrogenic in origin and is not described patient.
indicative of a return of their own hypothalamic function.
Rickenlund A, Carlstrom K, Ekblom B, Brismar TB, Von Schoultz B,
Bisphosphonates, such as alendronate sodium, should Hirschberg AL. Effects of oral contraceptives on body composition
be avoided in patients with the female athlete triad. These and physical performance in female athletes. J Clin Endocrinol Metab
medications have a long half-life and can be absorbed 2004;89:4364–70.
into bone and leach out later in life, possibly during a Silveira LF, Latronico AC. Approach to the patient with hypogonado-
tropic hypogonadism. J Clin Endocrinol Metab 2013;98:1781–8.
future pregnancy. These medications have demonstrated
Thein-Nissenbaum J. Long term consequences of the female athlete
adverse effects in animal models and should be used with triad. Maturitas 2013;75:107–12.
caution in reproductive-aged women. Young N, Formica C, Szmukler G, Seeman E. Bone density at weight-
The effects on bone density of selective estrogen bearing and nonweight-bearing sites in ballet dancers: the effects of
receptor modulators, such as raloxifene hydrochloride, exercise, hypogonadism, and body weight. J Clin Endocrinol Metab
1994;78:449–54.
are inferior to the effects of estrogen. For younger
109
Abnormal uterine bleeding
A 27-year-old nulligravid woman who has been diagnosed with polycystic ovary syndrome
(PCOS) comes to your office with irregular bleeding. She reports no vaginal bleeding for 6 months
followed by bleeding every 2 weeks. She has not experienced moliminal symptoms associated with
her menstrual periods. An office urine pregnancy test is negative. Transvaginal ultrasonography
identifies polycystic-appearing ovaries bilaterally and an endometrial stripe measuring 22 mm. Her
serum reproductive hormone levels today will most likely be
Polycystic ovary syndrome is a complex, multidimen- of LH, which has effects on ovarian androgen production
sional disorder resulting in defects in reproduction and and oocyte development. Patients may report false-posi-
metabolism. The prevalence of PCOS is estimated at tive urinary ovulation predictor tests due to chronic LH
4–12% of reproductive-aged women. Menstrual abnor- elevation. Persistently heightened GnRH pulsatility then
malities seen with chronic anovulation include second- alters the LH–follicle-stimulating hormone (FSH) ratio
ary amenorrhea, oligomenorrhea, and heavy menstrual and contributes to increased ovarian androgen secretion
bleeding. and aberrant follicle maturation. Changes in the LH–FSH
The pathophysiology of PCOS encompasses inher- ratio, although a contributor to hyperandrogenism, are not
ent ovarian dysfunction that is strongly influenced by used as criteria to diagnose PCOS.
external factors, such as disturbances of the hypothalamic– The irregular bleeding reported by the described patient
pituitary–ovarian axis. Exaggerated gonadotropin-releas- is most consistent with anovulatory bleeding and, there-
ing hormone (GnRH) pulsatility results in hypersecretion fore, progesterone values would be expected to be less
Reproductive Endocrinology and Infertility 153
than 3 ng/mL. Given her diagnosis of PCOS, we would reveal the hormonal level pattern of a prolonged follicular
expect altered GnRH pulsatility leading to LH hyperse- phase (option C).
cretion, absence of ovulation, progesterone level below
3–5 ng/mL, and some FSH secretion from the anterior Boutzios G, Karalaki M, Zapanti E. Common pathophysiological
mechanisms involved in luteal phase deficiency and polycystic ovary
pituitary, causing granulosa cell production of detectable syndrome. Impact on fertility. Endocrine 2013;43:314–7.
estradiol levels. This is emphasized in the clinical exam- Cheung AP. Polycystic ovary syndrome: a contemporary view. J Obstet
ple in the unopposed endometrial proliferation observed Gynaecol Can 2010;32:423–5, 426–8.
with measurement of a thickened endometrial stripe. An Johnston-MacAnanny EB, Park JK, Berga SL. Polycystic ovary syn-
estradiol level greater than 200 pg/mL would be consis- drome. In: Falcone T, Hurd WW, eds. Clinical reproductive medicine
and surgery: a practical guide. New York (NY): Springer-Verlag; 2013.
tent with a periovulatory follicle and would be unlikely in p. 113–23.
this patient. Bloodwork performed today most likely will
110
Cushing syndrome
A 28-year-old nulligravid woman is referred to you for secondary amenorrhea. She tells you that
she has been gaining weight and has fatigue and muscle weakness. On physical examination, her
blood pressure is 165/95 mm Hg and her weight is 99.8 kg (220 lb). She has round facies, facial
hirsutism, truncal obesity, and development of blue-tone abdominal striae. The best screening test
to confirm the diagnosis is
Cushing syndrome is defined as overexposure of the its etiology. Patients with Cushing syndrome are rela-
body’s tissues to cortisol. It is a rare diagnosis; approxi- tively insensitive to corticosteroid feedback and exhibit
mately one to five new cases occur per million people oversecretion of cortisol that is not circadian in nature. If
per year in the United States. Cushing syndrome should any of the screening test results are positive, an additional
be distinguished from Cushing disease and implies an screening test is performed to confirm the diagnosis. Once
ACTH-secreting tumor. Identification of the clinical the diagnosis is established with two positive screening
presentation of Cushing syndrome and its biochemical test results, the cause of Cushing syndrome is established
confirmation present a challenging clinical problem for in order to initiate treatment as soon as possible.
physicians. Early diagnosis of Cushing syndrome is cru- A recent meta-analysis commissioned by the Endocrine
cial because the natural history of the condition is marked Society Diagnosis of Cushing Syndrome Task Force
by significant excess of mortality and morbidity. Serum found similar accuracy for such tests as 24-hour urine
ACTH-dependent causes account for 85% of Cushing cortisol, 1-mg overnight dexamethasone suppression test,
syndrome cases. Patients with Cushing syndrome have serum or salivary midnight cortisol, and combined strate-
pituitary oversecretion or ectopic production of ACTH gies based on these tests. An overnight dexamethasone
or corticotrophin-releasing hormone; ACTH-independent suppression test offers the simplest way of screening
causes account for the remaining 15% of cases, which for the condition. Dexamethasone at a dose of 1 mg is
arise from adrenal adenoma or carcinoma. In addition, given orally at 11:00 pm and a plasma cortisol level is
some women develop the condition because of the exog- drawn at 8:00 am the next morning. A value of less than
enous use of corticosteroids. 5 micrograms/dL rules out Cushing syndrome, and the
The diagnostic process is divided into two sequential number of patients with abnormal suppression is less than
steps: first, the diagnosis and confirmation of hypersecre- 1%. Although historically obese patients have a false-
tion of cortisol, and, second, the differential diagnosis of positive rate that is higher than the general population, a
154 PROLOG
recent study of 86 obese patients where a cut-off for the decrease by at least 40%. Cushing disease is present
suppression of serum cortisol was reduced to 3 micro- when an ACTH and a chest X-ray are normal and an
grams/dL reduced the false-positive rate to 2.3%, mini- abnormal sella is found on computed tomography scan
mizing any extensive follow-up studies. A 24-hour urine of the pituitary gland. A low plasma ACTH level (less
collection for free cortisol is a useful measurement in the than 5 pg/mL) suggests an autonomous cortisol-secreting
basal state to detect Cushing syndrome; however, it is tumor, and the precise location of the abnormality would
cumbersome and difficult to collect. Although a midnight be identified with computed tomography or magnetic
plasma cortisol level of less than 15 micrograms/dL is resonance imaging of the adrenal gland. A high plasma
helpful to detect Cushing syndrome, it is difficult to use ACTH level (greater than 50 pg/mL) suggests ectopic
in clinical practice because of the time for collection. secretion and should be differentiated from patients
Likewise, morning serum cortisol is not a suitable screen- with Cushing disease. In cases where ACTH is elevated,
ing test for Cushing syndrome. intrapetrosal sinus catheterization is the only test able
The current guidelines published by the Endocrine to differentiate Cushing disease from ectopic secretion
Society recommend against the use of tests for random of cortisol. A 17-hydroxyprogesterone level is used as
serum cortisol or plasma ACTH levels and urinary a screening test for 21-hydroxylase deficiency and not
17-ketosteroids, pituitary and adrenal imaging, and the Cushing syndrome, so it would not be appropriate in this
8-mg dexamethasone suppression test as first-line tests case. Therefore, the best screening test to confirm the
for the diagnosis of Cushing syndrome. A high-dose dex- diagnosis is overnight dexamethasone suppression test.
amethasone suppression test is used to determine the
etiology of Cushing syndrome, particularly when it is Elamin MB, Murad MH, Mullan R, Erickson D, Harris K, Nadeem S,
et al. Accuracy of diagnostic tests for Cushing’s syndrome: a systematic
associated with an ACTH level. In the presence of a review and metaanalyses. J Clin Endocrinol Metab 2008;93:1553–62.
basal level ACTH less than 5 micrograms/mL, a high- Guignat L, Bertherat J. The diagnosis of Cushing’s syndrome: an
dose dexamethasone suppression (taking 2 mg every Endocrine Society Clinical Practice Guideline: commentary from a
6 hours for 2 consecutive days), and no decrease in uri- European perspective. Eur J Endocrinol 2010;163:9–13.
nary steroids (17-hydroxysteroid and cortisol) by at least Nieman LK, Biller BM, Findling JW, Newell-Price J, Savage
MO, Stewart PM, et al. The diagnosis of Cushing’s syndrome: an
40%, an adrenal tumor is likely. When ACTH is measur- Endocrine Society Clinical Practice Guideline. J Clin Endocrinol Metab
able in the blood (greater than 20 pg/mL), an ectopic 2008;93:1526–40.
ACTH-producing tumor is unlikely if the urinary steroids
Reproductive Endocrinology and Infertility 155
111
Retrograde ejaculation
A 35-year-old male with diabetes mellitus type 1 comes to your office with his 31-year-old partner
to be evaluated for infertility. His diabetes mellitus was poorly controlled for a number of years
but is now well controlled. He reports that his urine is cloudy after intercourse. His semen analysis
demonstrates low-volume oligospermia. A postejaculatory urinalysis demonstrates the presence of
sperm. A repeat semen analysis performed with the use of postejaculatory urine after alkalinization
is normal. The most appropriate initial treatment to help this couple achieve pregnancy is
Ejaculatory dysfunction is one the most common male urine should be obtained via voiding or catheteriza-
sexual disorders and includes premature ejaculation, tion. Sperm are isolated by centrifugation, washed,
retrograde ejaculation, delayed ejaculation, anorgasmia, and then prepared for use in intrauterine insemination
and aspermia (absence of sperm). Retrograde ejacula- or IVF if the semen parameters are unfavorable. In
tion can be categorized into two types: 1) functional and some patients, antihistamines or ephedrine can be used
2) anatomic. Functional disorders, such as multiple scle- to try to close the bladder neck to prevent retrograde
rosis, diabetic neuropathy, or spinal cord injury, may ejaculation at the time of orgasm. Potential adverse
result from medical conditions that predispose the patient effects include headache, hypertension, and urinary
to retrograde ejaculation. Anatomic causes include uro- retention.
logic bladder neck surgery such as transurethral resection In the couple described, intrauterine insemination
of the prostate. is the most appropriate initial treatment. Donor sperm
Retrograde ejaculation causes semen to reflux back insemination is not necessary for this couple given the
into the bladder rather than pass through the urethra. male partner’s normal semen analysis with the use of
Sperm exposed to acidic urine may affect normal semen postejaculatory urine after alkalinization. At this time,
parameters and contribute to infertility. Correct diagnosis IVF is not necessary given his normal semen analysis.
of retrograde ejaculation requires evaluation of postejacu- In patients with an abnormal semen analysis or patients
latory urine, usually in patients who have a history of low who have failed to conceive with intrauterine insemina-
semen volume (less than 1 mL). tion, IVF should be considered. This patient’s retrograde
In order to perform the postejaculatory urinalysis, ejaculation is probably due to diabetic neuropathy and,
the urine specimen should be centrifuged, followed thus, surgical correction is not indicated at this time.
by microscopic examination of the pellet. In men with Fedder J, Kaspersen MD, Brandslund I, Hojgaard A. Retrograde
azoospermia or aspermia, the presence of sperm on post- ejaculation and sexual dysfunction in men with diabetes mellitus: a
ejaculatory urinalysis is suggestive of retrograde ejacula- prospective, controlled study. Andrology 2013;1:602–6.
tion. Significant numbers of sperm observed in the urine Jefferys A, Siassakos D, Wardle P. The management of retrograde ejac-
ulation: a systematic review and update. Fertil Steril 2012;97:306–12.
confirm the diagnosis of retrograde ejaculation.
van der Linden PJ, Nan PM, te Velde ER, van Kooy RJ. Retrograde
Before ejaculation, the urine should be alkalinized ejaculation: successful treatment with artificial insemination. Obstet
using oral sodium bicarbonate. After ejaculation, the Gynecol 1992;79:126–8.
156 PROLOG
112
Hirsutism
A 27-year-old woman presents to your office with hirsutism and irregular menses related to poly-
cystic ovary syndrome (PCOS). She is otherwise healthy and is not trying to conceive at this time.
You counsel her that, in addition to lifestyle modifications, the best initial pharmacologic treatment
for her symptoms is
Hirsutism occurs when fine vellus hairs differentiate into Insulin-sensitizing agents, such as metformin, play an
dark, coarse terminal hairs. It is usually a clinical sign important role in addressing the metabolic sequelae of
of another underlying disease, such as, in this patient, PCOS, but they have not demonstrated a clear benefit in
PCOS. The subjective perception of the patient and not the treatment of hirsutism. However, insulin-sensitizing
the severity of the hirsutism should dictate the need for agents do decrease circulating androgen levels, improve
treatment. Management often will require a two-pronged ovulation rates, and improve glucose tolerance. The levo-
approach that comprises norgestrel intrauterine device would provide endometrial
1. medical therapy to decrease androgen activity and protection for this anovulatory patient but would not
improve her hirsutism. Topical eflornithine, an inhibi-
2. cosmetic therapy to remove terminal hairs that are
tor of the enzyme ornithine decarboxylase, is another
present.
approved treatment for facial hirsutism. It requires twice-
In the described patient, given her diagnosis of PCOS daily treatment and is effective while it is being used, but
together with irregular menses and hirsutism, a combina- symptoms tend to recur once the therapy is stopped.
tion estrogen–progestin OC should be recommended as Mechanical hair removal techniques, such as shaving,
the primary treatment. Combination OCs work through plucking, waxing, depilatory cream, electrolysis, and
several mechanisms. The estrogen component increases laser, are employed frequently by women with hirsut-
sex hormone-binding globulin production in the liver, ism. In the medical literature, laser treatment has been
thereby decreasing circulating testosterone levels. The studied to a greater extent than has electrolysis. Laser hair
progestin component inhibits luteinizing hormone pro- removal is effective in women with PCOS, particularly
duction, further decreasing testosterone levels. An addi- when utilized in conjunction with medical treatment to
tional benefit of OCs includes prevention of endometrial decrease circulating androgen levels. Classically, laser
neoplasia. hair removal works best on women with dark hair and
When combination OCs alone are not sufficient for light skin because the laser can selectively focus on the
management of a patient’s hirsutism, other modalities target hair without damaging or discoloring the sur-
can be used along with OCs. Antiandrogens, particularly rounding tissue. One randomized trial found that the
spironolactone, can be used to treat hyperandrogenism. combination of eflornithine and laser treatment was more
Antiandrogens have multiple mechanisms of action, effective than laser alone in treating hirsutism.
including competing with androgens to bind to the
androgen receptor. All antiandrogens are teratogenic and Escobar-Morreale HF, Carmina E, Dewailly D, Gambineri A, Kelestimur
F, Moghetti P, et al. Epidemiology, diagnosis and management of hir-
should be used only in conjunction with very reliable sutism: a consensus statement by the Androgen Excess and Polycystic
contraception. Spironolactone can cause hyperkalemia Ovary Syndrome Society. Hum Reprod Update 2012;18:146–70.
and probably should be avoided in women with renal Polycystic ovary syndrome. ACOG Practice Bulletin No. 108.
compromise. Flutamide and finasteride are other antian- American College of Obstetricians and Gynecologists. Obstet Gynecol
2009;114:936–49.
drogen alternatives, but they do not appear to be superior
Swiglo BA, Cosma M, Flynn DN, Kurtz DM, Labella ML, Mullan RJ,
to spironolactone. Additionally, flutamide carries the risk et al. Clinical review: Antiandrogens for the treatment of hirsutism: a
of hepatotoxicity. With all treatments for hyperandrogen- systematic review and metaanalyses of randomized controlled trials. J
ism, it may take 6 months or more before the patient’s Clin Endocrinol Metab 2008;93:1153–60.
symptoms improve.
Reproductive Endocrinology and Infertility 157
113
Bioidentical hormones
A 52-year-old woman, gravida 2, para 2, has been amenorrheic for 9 months and has been expe-
riencing frequent hot flushes. She has heard about bioidentical hormones and would like to learn
more. You inform her that one such U.S. Food and Drug Administration (FDA)-approved bioiden-
tical hormone prescribed for menopausal symptoms is
* (A) 17β-estradiol
(B) ethinyl estradiol
(C) biestrogen
(D) triestrogen
Hormone therapy (HT) is very effective for the treat- Frequently used compounded formulations include
ment of menopausal symptoms. Controversy about the biestrogen and triestrogen, which are both made up on
appropriate use of HT has developed in recent years. In a milligram-per-milligram basis. Biestrogen consists of
response to the Women’s Health Initiative (WHI) study, 20% estradiol and 80% estriol, and triestrogen is 10%
patients with menopausal symptoms have begun to seek estradiol, 10% estrone, and 80% estriol. Historically,
alternatives to traditional HT. Because of media cover- the intent of compounding was to provide a medication
age and direct-to-consumer advertising, compounded with different ingredients, preservatives, or routes of
bioidentical hormones have experienced a surge in administration than commercially available medications
popularity. Patients are told that a natural bioidentical (eg, to avoid a specific preservative for a patient with an
product can be created or compounded specifically for allergy). However, the practice of blending commercially
them. These compounded products are promoted as safe available products to create a new custom-made com-
and effective. Available evidence does not support claims pound for a specific patient is very different. A paucity
for the superiority of bioidentical products over conven- of data exists on the efficacy and safety of most of these
tional HT for the treatment of menopausal symptoms. new compounds. In contrast, all FDA-approved hor-
Moreover, some compounded bioidentical products may monal products must provide to consumers information
pose additional safety risks due to variable purity and on the efficacy, adverse effects, and complications of the
potency. product and display a black box warning to address the
As a result of the findings of the WHI study, HT is findings of the WHI study. Compounded hormones are
no longer prescribed to improve a woman’s long-term not approved or regulated by the FDA. As a result, such
health; however, HT is appropriate to treat menopausal products are also exempt from displaying warnings or
symptoms, particularly around the time of menopause. contraindications.
Such HT is available as commercial products or com- In general, HT should be used at the lowest pos-
pounded formulations, in which an individualized prepa- sible dose to manage a woman’s menopausal symp-
ration is created as a unique medication tailored to the toms. Because symptom control is the most important
needs of a specific patient. therapeutic endpoint, testing of hormone levels in any
The term “bioidentical hormones” refers to supple- fashion (serum, urine, or saliva) is typically unneces-
ments that are chemically and structurally similar if not sary. Salivary testing, which is recommended by many
identical to hormones naturally produced by the body. compounding advocates, is fraught with limitations and is
Commercially available bioidentical hormones that have not biologically meaningful for measuring sex hormones.
been approved by the FDA include estradiol and micron- For patients who are interested in treatment with natu-
ized progesterone. The term bioidentical hormones has rally occurring hormones, 17β-estradiol (transdermal or
become misleading because compounding pharmacies oral) and micronized progesterone (oral or vaginal) are
and the media use it to suggest that such preparations are FDA-approved products. Ethinyl estradiol is a synthetic
more natural and safer than HT prescribed by physicians. estrogen commonly used in oral contraceptives. The
Because of the confusion surrounding the term, the FDA American College of Obstetricians and Gynecologists
and the Endocrine Society now recognize it to be a mar- and the American Society for Reproductive Medicine
keting term and not a medical term. agree that compounded hormones are not superior to
158 PROLOG
conventional menopausal HT and that they pose potential Files JA, Ko MG, Pruthi S. Bioidentical hormone therapy. Mayo Clin
Proc 2011;86:673–80, quiz 680.
risks, including variable purity and potency as well as
Huntley AL. Compounded or confused? Bioidentical hormones and
possible underdosage or overdosage. menopausal health. Menopause Int 2011;17:16–8.
114
Hypothyroidism
During a primary infertility evaluation, a 35-year-old healthy woman has a normal physical exami-
nation with a thyroid-stimulating hormone (TSH) level of 4.5 international units/L and positive
thyroid microsomal antibodies. The best next test for this patient is
Primary subclinical hypothyroidism is defined as an in thyroid microsomal protein preparations, hence the
elevated TSH level; in comparison, clinical hypothyroid- term antimicrosomal antibodies. Subclinical hypothy-
ism is defined as an elevated TSH level accompanied by roidism with or without thyroid peroxidase antibodies
decreased free and total thyroxine levels. As the name increases the risk of hypothyroidism. The incidence of
suggests, patients with subclinical hypothyroidism are hypothyroidism in women with subclinical hypothyroid-
most commonly asymptomatic. Primary hypothyroidism ism and thyroid peroxidase antibodies is approximately
is caused by decreased thyroxine production in the thyroid 4.3% per year; incidence in women with subclinical
gland with a compensatory increase in TSH (thyrotropin). hypothyroidism alone is 2.6% per year and that in women
Secondary hypothyroidism is caused by a lack of produc- with thyroid peroxidase antibodies alone is 2.1% per year.
tion of TSH by the pituitary and is much less common. Approximately 16–20% of pregnant women found to
In developed countries, the most common cause of have thyroid peroxidase antibodies with normal TSH in
primary hypothyroidism is autoimmune destruction of the the first trimester will have an elevated TSH by the third
thyroid gland through Hashimoto thyroiditis, also known trimester. These women also have a much higher risk of
as autoimmune thyroiditis. Less common etiologies postpartum thyroiditis. The presence of thyroglobulin
include iodine deficiency, iatrogenic causes due to sur- antibodies does not substantially increase the risk of hypo-
gery, irradiation, and drugs used to treat hyperthyroidism thyroidism. Unlike the thyroid-stimulating immunoglobu-
(eg, lithium); the infiltrating diseases sarcoidosis and lins found in Graves disease, in the first trimester thyroid
amyloidosis; and TSH receptor mutations. Hashimoto peroxidase antibodies have no adverse effect on the fetus,
thyroiditis is 15–20 times more common in women and ultrasonographic monitoring of the fetal thyroid is not
compared with men, with peak incidence between ages necessary.
30 years and 50 years. The disease may present in several The upper limit of normal TSH levels in euthyroid
ways, including euthyroid goiter, subclinical hypothyroid- women remains controversial. Levels of TSH normally
ism with goiter, hypothyroidism without goiter, painless increase with obesity and age. For every 10 years of age
(silent) thyroiditis, and postpartum thyroiditis. after age 39 years, TSH increases 0.03 mU/L. In addition,
Thyroid disease in women includes subclinical hypo- TSH levels vary diurnally up to 50%, with the highest
thyroidism (approximately 7.5%), hypothyroidism (1.8%), level occurring at the onset of sleep. Thus, it is prudent
thyroid peroxidase antibodies (10%), and thyroglobulin to repeat the TSH level test in patients with borderline-
antibodies (3%). Thyroid peroxidase is responsible for elevated TSH levels. In nonpregnant women who are
converting iodide into iodine in the generation of thyrox- not trying to conceive, the National Academy of Clinical
ine. Antibodies to thyroid peroxidase were first identified Biochemistry reports an upper normal limit of TSH of
Reproductive Endocrinology and Infertility 159
2.5 mU/L. Data from the National Health and Nutrition found in seafood and many fruits and vegetables. Foods
Examination Survey III support an upper normal limit with the highest iodine content include haddock, yogurt,
of TSH of 4.12 mU/L. In addition, National Health and white bread, and milk. Iodine evaluation is not indicated
Nutrition Examination Survey III data support an upper in a patient with a normal TSH level and no goiter.
normal TSH level in women who are planning pregnancy Spontaneous normalization of elevated TSH levels is
or in pregnant women in the first trimester of 2.5 mU/L, not uncommon, especially in individuals with levels less
in the second trimester of 3.0 mU/L, and in the third tri- than 10 mU/L and absent antibodies. For patients who
mester of 3.5 mU/L. have started taking thyroxine in this situation, it is reason-
No universal agreement has been reached on screen- able to intermittently discontinue the drug and reassess
ing recommendations for hypothyroidism. The American the need for medication.
Thyroid Association supports screening all patients every Subclinical hypothyroidism is associated with health
5 years starting at age 35 years. The American College issues. Meta-analyses have shown increased coronary
of Physicians recommends screening women after age heart mortality in subjects younger than 65 years who
50 years if they have symptoms or findings suggestive of have a TSH level of 10 mU/L or higher. Contributing
thyroid disease. The American College of Obstetricians factors may include increased low-density lipoprotein
and Gynecologists does not recommend thyroid screen- cholesterol, increased insulin, impaired left ventricular
ing during pregnancy in asymptomatic women without a function, increased peripheral resistance, and increased
personal history of thyroid disease. vascular inflammation. Subclinical hypothyroidism also
The appropriate screening test for primary hypothy- is associated with adverse pregnancy outcomes, such as
roidism is a TSH level test. No other functional studies spontaneous abortion and fetal loss. An association with
are needed, such as testing for thyroxine or triiodothyro- impaired fetal neurocognitive development also has been
nine, if the TSH level is normal. In nonpregnant women found in some, but not all, studies.
who are not trying to conceive, routine assay for thyroid The U.S. Preventive Services Task Force supports
peroxidase antibodies remains controversial. A finding of treatment of all women (pregnant and nonpregnant) with
thyroid peroxidase antibodies helps define the diagnosis subclinical hypothyroidism who have a TSH level of
of Hashimoto thyroiditis. A finding of positive antibodies 10 mU/L or greater. Because of a lack of clinical trials,
in a woman with a normal TSH level also increases the the U.S. Preventive Services Task Force has made no
risk of subsequent development of hypothyroidism and specific recommendation regarding treatment of women
requires more frequent follow-up testing. In contrast, the with TSH levels below 10 mU/L.
finding of positive antibodies in a patient with an elevated In accordance with current data and recommendations,
TSH level does not affect the decision to treat. Evidence the most common course of action is to treat women
supports the testing of women with recurrent pregnancy who desire pregnancy and pregnant women with a TSH
loss for thyroid peroxidase antibodies because an associa- greater than 2.5 mU/L regardless of thyroid peroxidase
tion has been shown. Adverse pregnancy outcomes, such antibody status. In contrast, the treatment of women who
as preterm delivery before 34 weeks of gestation and neo- desire pregnancy and of pregnant women with positive
natal respiratory distress, have been observed in women thyroid peroxidase antibodies but with normal TSH
with thyroid peroxidase antibodies and normal TSH remains controversial.
levels (2.5 mU/L or less). No current consensus exists
regarding the benefit of treating women with normal TSH Cooper DS. Clinical practice. Subclinical hypothyroidism. N Engl J
Med 2001;345:260–5.
levels and elevated antibodies for recurrent pregnancy
Garber JR, Cobin RH, Gharib H, Hennessey JV, Klein I, Mechanick JI,
loss or to prevent pregnancy complications. et al. Clinical practice guidelines for hypothyroidism in adults: cospon-
Thyroid ultrasonography is not necessary in women sored by the American Association of Clinical Endocrinologists and
who have hypothyroidism and a normal thyroid examina- the American Thyroid Association. American Association Of Clinical
Endocrinologists And American Thyroid Association Taskforce
tion. Ultrasonography should be performed if a thyroid On Hypothyroidism In Adults [published errata appear in Thyroid
nodule is suspected to determine the need for pathologic 2013;23:251. Thyroid 2013;23:129]. Thyroid 2012;22:1200–35.
evaluation for cancer. Radioactive iodine uptake scan- Huber G, Staub JJ, Meier C, Mitrache C, Guglielmetti M, Huber P,
ning is used to evaluate for hyperthyroidism and is not et al. Prospective study of the spontaneous course of subclinical
hypothyroidism: prognostic value of thyrotropin, thyroid reserve, and
indicated in the evaluation of hypothyroidism. Iodine thyroid antibodies. J Clin Endocrinol Metab 2002;87:3221–6.
deficiency remains a cause of endemic goiter and hypo- Stagnaro-Green A, Abalovich M, Alexander E, Azizi F, Mestman J,
thyroidism in underdeveloped countries but rarely in Negro R, et al. Guidelines of the American Thyroid Association
developed countries. Because of a high incidence of goi- for the diagnosis and management of thyroid disease during preg-
nancy and postpartum. American Thyroid Association Taskforce on
ter in the Great Lakes region of the United States, iodine Thyroid Disease During Pregnancy and Postpartum. Thyroid 2011;
supplementation of salt was started in the 1920s. Iodine is 21:1081–125.
160 PROLOG
115
Ambiguous genitalia
You have been called to evaluate a neonate after delivery. On physical examination, the genitalia
are ambiguous. You suspect congenital adrenal hyperplasia (CAH). The neonate’s electrolytes are
normal and you await results of a karyotype, pelvic ultrasonography, and a steroid hormone profile.
The most appropriate step in the management of this neonate is to
The birth of an infant with sexual ambiguity is a medi- In the case of a sick infant with abnormal electro-
cal and psychosocial emergency for the infant, parents, lytes, most practitioners do not delay treatment to per-
and health care team. Congenital adrenal hyperplasia is form a cosyntropin stimulation test. The cosyntropin
an autosomal recessive disorder that occurs in approxi- stimulation test is the criterion standard for diagnosing
mately 1 in 16,000 births. If not diagnosed promptly, CAH. Cosyntropin stimulates the adrenal gland. A
CAH can be life threatening. More than 90% of cases newborn with salt-wasting disease will have the highest
are due to a 21-hydroxylase deficiency. This enzyme 17-hydroxyprogesterone levels whereas simple virilizing
is needed for cortisol synthesis in the adrenal cortex; newborns have lower 17-hydroxyprogesterone levels.
21-hydroxylase catalyzes conversion of 17-hydroxypro- The test takes an hour and, thus, treatment of a sick infant
gesterone to 11-deoxycortisol, which is a precursor for with salt-wasting CAH should not be delayed for the
cortisol. In addition, 21-hydroxylase catalyzes conver- cosyntropin test.
sion of progesterone to deoxycorticosterone, a precur- The most appropriate management for this newborn is
sor for aldosterone. Newborns with 21-hydroxylase to use a multidisciplinary team to assist in gender assign-
deficiency cannot synthesize cortisol, which causes the ment and work with the family. The multidisciplinary
adrenal gland to be stimulated by adrenocorticotropic team and the family should work together to formulate
hormone, driving excess cortisol precursors to produce the best strategy for clinical care. Ideally, a multidisci-
excess androgens (dehydroepiandrosterone and andro- plinary team would include a pediatric endocrinologist,
stenedione). These androgens cause ambiguous genitalia a pediatric surgeon, a pediatric urologist, a psychologist,
in the newborn. Approximately 95% of these cases result a pediatrician, and a coordinator. A careful family his-
in masculinization of a 46,XX offspring but do not sig- tory is needed to help the medical team formulate the
nificantly affect male sexual differentiation. proper diagnosis. This decision takes into account the par-
A defect in aldosterone biosynthesis (salt wasting) ents’ cultural background and desires when gender
causes excessive sodium excretion with the development assignment is made. To assign the preliminary gen-
of hypovolemia and hyperreninemia, resulting in hyper- der and meet with the family would not be the appropriate
kalemia. Cortisol deficiency causes increased antidiuretic next step. Instead, the multidisciplinary team and family
hormone secretion. These deficiencies together contrib- together should decide gender assignment.
ute to hyponatremic dehydration and shock if not diag- Consultation with a pediatric surgeon would not be the
nosed and treated quickly. best next step in the management of this newborn. Once
Initial newborn screening includes a physical examina- a diagnosis is established and the salt-wasting form of
tion, pelvic ultrasonography, and a karyotype, which can CAH is ruled out, the multidisciplinary team, including
be completed in 24 hours. If CAH is suspected, electro- the pediatric surgeon, should decide on a possible surgi-
lyte levels and a serum 17-hydroxyprogesterone level cal correction. Genital surgery is performed to maximize
should be obtained quickly. Newborns in some locales anatomy and sexual function. Single-stage surgical cor-
can be screened for CAH using dried blood on filter rection between the ages of 2 months and 6 months
paper to test for elevated 17-hydroxyprogesterone. If is advocated by some because at that age tissues are
electrolytes are abnormal and a positive newborn screen estrogenized and pliable. Early correction can mitigate
is obtained, treatment should be initiated immediately. stigmatization and minimize family concerns. However,
Reproductive Endocrinology and Infertility 161
some advocate waiting until the child is old enough to Houk CP, Hughes IA, Ahmed SF, Lee PA. Summary of consensus
statement on intersex disorders and their management. International
give informed consent before surgery is performed. Intersex Consensus Conference. Writing Committee for the
In the newborn with CAH who has signs of adrenal International Intersex Consensus Conference Participants. Pediatrics
crisis, such as vomiting, diarrhea, weight loss, hyperten- 2006;118:753–7.
sion, hyponatremia, hyperkalemia, or hypoglycemia, Sandberg DE, Gardner M, Cohen-Kettenis PT. Psychological aspects
of the treatment of patients with disorders of sex development. Semin
medical care must be initiated immediately. Electrolytes Reprod Med 2012;30:443–52.
are replaced, and, if 17-hydroxyprogesterone is elevated, White PC, Bachega TA. Congenital adrenal hyperplasia due to 21
a stress dose of steroids is given followed by a regimen hydroxylase deficiency: from birth to adulthood. Semin Reprod Med
of steroids until the newborn is stable. Diagnosing and 2012;30:400–9.
stabilizing this newborn is of primary concern and is Wilson JD, Rivarola MA, Mendonca BB, Warne GL, Josso N, Drop
SL, et al. Advice on the management of ambiguous genitalia to a
managed by the pediatric team. Once the newborn has young endocrinologist from experienced clinicians. Semin Reprod Med
a proper diagnosis and is medically stable, it is up to the 2012;30:339–50.
multidisciplinary team to assign gender.
116
Multifetal pregnancy reduction
A 31-year-old woman, gravida 1, para 0, underwent infertility treatment with gonadotropins and
intrauterine insemination and conceived a triplet pregnancy. She is very concerned about continu-
ing such a high-risk pregnancy and is considering multifetal pregnancy reduction. You explain to
her that the most frequent obstetric risk associated with a triplet pregnancy is
(A) miscarriage
* (B) preterm delivery
(C) fetal growth restriction
(D) preeclampsia
High-order multiple gestations, defined as pregnancies pregnancies. Multifetal gestations also are associated with
with three or more fetuses, are associated with an increased maternal morbidity and health care costs, given
increased risk of multiple adverse obstetric outcomes. that women with multiple gestations are six times more
In particular, preterm delivery is almost guaranteed; likely to be hospitalized with pregnancy complications.
approximately 75–100% of triplets are delivered preterm. Significant psychosocial complications also can follow
The risk of spontaneous loss of the entire pregnancy multiple births, including severe parenting stress, parental
is approximately 15% for triplets. The average gesta- depression, child abuse, and divorce.
tional age at delivery for triplets is 32.2 weeks (versus Fertility therapy has led to a dramatic increase in
35.3 weeks for twins). Compared with singleton pregnan- multifetal pregnancies. From 1980 to 2009, the twin rate
cies, cerebral palsy occurs 17 times more often in triplet increased by 76%. The high-order multifetal pregnancy
pregnancies. Approximately 50–60% of triplets exhibit rate increased by more than 400% between 1980 and 1998
growth restriction compared with 14–25% of twins. but decreased by 29% from 1998 to 2009. Preventive
In addition, 75% of triplets and 25% of twins require strategies to limit multifetal pregnancies, particularly
admission to the neonatal intensive care unit. The risk of high-order multiples, are essential in order to address this
preeclampsia is increased significantly in multiple gesta- issue. In recent years, there has been an increased empha-
tions and is more likely to occur earlier and to meet the sis by the American Society for Reproductive Medicine
criteria for severe or atypical preeclampsia. Women with on limiting the number of embryos transferred at the time
multiple gestations are also at increased risk of gestational of in vitro fertilization. Ideally, single embryo transfer
diabetes mellitus, which affects approximately 3–6% of should be considered in select patients. However, the
twin pregnancies and up to 22–39% of triplet pregnan- attempt to limit the incidence of multifetal pregnancies
cies. The risk of death in the first year of life is 20-fold with the use of controlled ovarian hyperstimulation with
higher in triplet pregnancies compared with singleton gonadotropins has proved more challenging. The risk of
162 PROLOG
high-order multiples with controlled ovarian hyperstimu- High-order multiple gestations present medical and
lation is approximately 9%, which is significantly higher ethical dilemmas. To maximize the safety of the preg-
than the risk with in vitro fertilization (approximately nancy, a maternal–fetal medicine specialist often is
1%). In the case of high-order multifetal pregnancies, consulted regarding selective reduction of a high-order
counseling should include discussion of multifetal preg- multiple pregnancy. For many patients, moral, religious,
nancy reduction, defined as a first-trimester or early social, cultural, and economic factors also may play a role
second-trimester procedure to reduce the total number of in this difficult decision-making process.
fetuses, by one or more, in a multifetal pregnancy.
In terms of the medical benefits of multifetal preg- Dodd JM, Crowther CA. Reduction of the number of fetuses for
women with a multiple pregnancy. Cochrane Database of Systematic
nancy reduction, the goal is to increase the probability Reviews 2012, Issue 10. Art. No.: CD003932. DOI: 10.1002/14651858.
of achieving at least one live birth and decrease the CD003932.pub2.
risk of spontaneous loss of the entire pregnancy. Estimates Evans MI, Dommergues M, Wapner RJ, Goldberg JD, Lynch L, Zador
of perinatal outcomes after fetal reduction include a loss IE, et al. International, collaborative experience of 1789 patients having
multifetal pregnancy reduction: a plateauing of risks and outcomes.
rate of 11–12% and a very early preterm delivery rate J Soc Gynecol Investig 1996;3:23–6.
of 4.5%. The loss rate is dependent on multiple factors,
Malizia BA, Dodge LE, Penzias AS, Hacker MR. The cumulative prob-
including the experience of the operator, the number of ability of liveborn multiples after in vitro fertilization: a cohort study of
fetuses present, and ease of access. Additionally, multi- more than 10,000 women. Fertil Steril 2013;99:393–9.
fetal pregnancy reduction is associated with a decreased Multifetal pregnancy reduction. Committee Opinion No. 553.
risk of preterm delivery, hypertension, preeclampsia, and American College of Obstetricians and Gynecologists. Obstet Gynecol
2013;121:405–10.
gestational diabetes mellitus. Patients may opt to have the
Multiple gestation: complicated twin, triplet, and high-order multifetal
fetuses tested for aneuploidy or genetic anomalies before pregnancy. ACOG Practice Bulletin No. 56. American College of
reduction is performed. Obstetricians and Gynecologists. Obstet Gynecol 2004;104:869–83.
117
Hypothalamic amenorrhea
A 31-year-old nulligravid woman comes to your office with an 18-month history of amenorrhea
after she stopped taking cyclical oral contraceptives (OCs). She runs 50 km (31.1 mi) a week
and has a body mass index of 19 (calculated as weight in kilograms divided by height in meters
squared). Her follicle-stimulating hormone (FSH) level is 5.0 mIU/mL, luteinizing hormone
(LH) level is 2.5 mIU/mL, prolactin level is 12 ng/mL, thyroid-stimulating hormone level is
2.1 mU/mL, and estradiol level is 15 pg/mL. She has no signs of hyperandrogenism. You determine
that the diagnosis is
amenorrhea. Waiting more than 6 months before ini- tropins is the treatment of choice. Clomiphene citrate
tiation of a formal evaluation would delay diagnosis and in this patient is unlikely to induce ovulation without an
treatment of her amenorrhea. intact hypothalamic–pituitary–ovarian axis.
Functional hypothalamic amenorrhea is due to sup- Polycystic ovary syndrome is also a diagnosis of
pression of gonadotropin-releasing hormone pulsatility exclusion. Features include clinical or biochemical evi-
with ultimate suppression of the hypothalamic–pituitary– dence of hyperandrogenism and chronic anovulation with
ovarian axis. Pituitary gonadotropin secretion is at basal or without polycystic-appearing ovaries.
levels, causing low estrogen levels. Functional hypo- Ovarian insufficiency (hypergonadotropic hypogonad-
thalamic amenorrhea usually is caused by weight loss, ism) in women younger than 40 years occurs in approxi-
exercise, or stress. Gonadotropin levels are low to normal mately 1% of women. Follicle-stimulating hormone levels
with low estrogen levels in the presence of normal brain above 20 mIU/mL and LH levels above 40 mIU/mL
imaging (normal pituitary and sellar magnetic resonance are diagnostic for ovarian insufficiency. The etiology of
imaging). This patient’s hypogonadism is due to strenu- ovarian insufficiency in most women is unknown. Iatro-
ous exercise. Most patients with hypothalamic amenor- genic causes of ovarian insufficiency include chemo-
rhea produce low gonadotropins. The described patient’s therapy and radiation for treatment of malignancies. In
FSH and LH levels are normal. The differences in gly- rare cases, a woman who discontinues OCs in order to
cosylation of the gonadotropins can result in decreased start family building and will have secondary amenor-
bioactivity and, thus, hypogonadism, even when gonado- rhea with elevated gonadotropins. In such a case, OC use
tropin levels are normal. Given that functional hypo- masks ovarian insufficiency developed while the patient
thalamic amenorrhea is a diagnosis of exclusion, other was taking OCs. This can be a devastating diagnosis,
causes of menstrual irregularity need to be ruled out. For especially when OCs are discontinued in the hope of
example, hyperprolactinemia and thyroid dysfunction are starting a family.
correctable causes of irregular menses and amenorrhea.
Long-distance runners often have low body weight and Fritz MA, Speroff L. Amenorrhea. In: Fritz MA, Speroff L, edi-
lean body mass. Among competitive athletes, distance tors. Clinical gynecologic and infertility. 8th ed. Philadelphia (PA):
Lippincott Williams & Wilkins; 2011. p. 435–93.
runners have the highest incidence of amenorrhea. Initial
Gordon CM. Clinical practice. Functional hypothalamic amenorrhea. N
counseling for this patient should include increasing Engl J Med 2010;363:365–71.
caloric intake and reducing her running intensity in con- Practice Committee of the American Society for Reproductive Medicine.
junction with nutritional counseling. After taking these Current evaluation of amenorrhea. Fertil Steril 2008;90:S219–25.
steps, menstrual function often will resume and patients Santoro N. Update in hyper- and hypogonadotropic amenorrhea. J Clin
can attempt pregnancy. If amenorrhea continues and Endocrinol Metab 2011;96:3281–8.
pregnancy is desired, ovulation induction with gonado-
164 PROLOG
118
Luteal phase deficiency
A 38-year-old woman, para 1, who has been trying to become pregnant visits your clinic. She is
concerned that she has shorter cycles than her peers. Her thyroid-stimulating hormone and prolac-
tin levels are normal. Her menstrual cycle intervals are typically 25–26 days. She has read about
luteal phase deficiency and wants to know if low progesterone may lead to infertility. The most
appropriate next step is
Progesterone is vital to the process of implantation and The endometrial biopsy was a central part of the basic
early embryonic development. After ovulation, proges- infertility workup until recently. The secretory endome-
terone is produced first by the corpus luteum and then trium has specific histologic features that mark each day
by the trophoblast. Progesterone maintains the integrity after ovulation. Endometrial biopsy allows experienced
of the endometrium in the secretory phase. Withdrawal pathologists to assign a date to endometrial specimens
of progesterone leads to rapid shedding of the secretory that correspond to the number of days past ovulation. If
endometrium. If pregnancy is established, embryonic the biopsy specimen shows a discrepancy of more than
growth and development is contingent upon adequate 2 days from menstrual dating, this is considered “out of
progesterone production from the corpus luteum. phase.” Two out-of-phase biopsies are considered proof
The importance of ovarian progesterone in implanta- of luteal phase deficiency. Until recently this test was
tion and early embryonic growth has been demonstrated accepted as the criterion standard for diagnosis of luteal
elegantly in several clinical models. If the corpus luteum phase deficiency.
is surgically removed before 7 weeks of gestation, a high Many flaws can be found in the interpretation of
risk of pregnancy loss exists, but if supplemental exoge- endometrial biopsy results. The standard endometrial
nous progesterone is given after luteectomy, a miscarriage dating criteria were derived from a population of infertile
before 7 weeks can be prevented. Exogenous progester- patients. This meant that the reference population was
one has enabled pregnancy by means of oocyte donation abnormal from the outset. Histologic dating shows sig-
in women who have ovarian failure. Mifepristone, a pro- nificant interobserver and intraobserver variation, which
gesterone antagonist, will induce a high rate of pregnancy raises questions about the validity of this test. Variations
loss if it is used before 7 weeks of gestation. seen among individual patients, different cycles within
Given the critical role that progesterone plays in nida- the same patient, and different observers of the same
tion, it is logical to think that inadequate endogenous histologic specimen were too large, showing that the
progesterone production would lead to insufficient endo- endometrial biopsy lacked precision.
metrial development for the support of early pregnancy. Controversy over the clinical relevance of luteal
Luteal phase deficiency was first described in 1949. phase deficiency is due in part to the lack of a reliable
Low circulating progesterone levels after ovulation were test to diagnose this disorder. In a multicenter random-
assumed to cause delayed endometrial maturation. A sig- ized controlled clinical trial of 847 women with regular
nificant delay was thought to cause asynchrony between cycles, abnormal histologic dating could not discriminate
the embryo and endometrium, leading to infertility or between infertile women and those with proven fertility.
early pregnancy wastage. Several means of diagnosing As an independent entity that causes infertility, luteal
luteal phase deficiency have been proposed: charting phase deficiency was disproven and endometrial biopsy
basal body temperature, measuring serum progesterone has been abandoned for this indication. Endometrial
levels in the luteal phase, monitoring luteal phase length biopsy remains valid for tissue confirmation of ovulation
from LH surge, and performing endometrial biopsy. or evaluation of pathologic states such as endometrial
Although measuring midluteal progesterone levels and cancer or hyperplasia. In terms of the most appropriate
charting basal body temperature are accurate in identify- next step for the described patient, you should coun-
ing recent ovulation, neither test is useful to judge the sel her that there is no reliable test for luteal phase
quality or adequacy of the luteal phase. deficiency.
Reproductive Endocrinology and Infertility 165
The clinical relevance of luteal phase deficiency: a committee opin- Progesterone supplementation during the luteal phase and in early preg-
ion. Practice Committee of the American Society for Reproductive nancy in the treatment of infertility: an educational bulletin. Practice
Medicine. Fertil Steril 2012;98:1112–7. Committee of the American Society for Reproductive Medicine. Fertil
Coutifaris C, Myers ER, Guzick DS, Diamond MP, Carson SA, Legro Steril 2008;89:789–92.
RS, et al. Histological dating of timed endometrial biopsy tissue is not Speroff L, Fritz MA. Female infertility. In: Speroff L, Fritz MA.
related to fertility status. NICHD National Cooperative Reproductive Clinical gynecologic endocrinology and infertility. 8th ed. Philadelphia
Medicine Network. Fertil Steril 2004;82:1264–72. (PA): Lippincott Williams and Wilkins; 2011. p. 1164–6.
Murray MJ, Meyer WR, Zaino RJ, Lessey BA, Novotny DB, Ireland Speroff L, Fritz MA. Recurrent early pregnancy loss. In: Speroff L,
K, et al. A critical analysis of the accuracy, reproducibility, and clinical Fritz MA. Clinical gynecologic endocrinology and infertility. 8th ed.
utility of histologic endometrial dating in fertile women. Fertil Steril Philadelphia (PA): Lippincott Williams and Wilkins; 2011. p. 1216–7.
2004;81:1333–43.
119
Donor sperm use
A 30-year-old woman, gravida 1, para 0, has been trying to conceive for more than 1 year with her
partner. As a teenager, she terminated a pregnancy she had conceived with a different partner. Her
current partner’s semen analysis reveals a normal volume of azoospermia on two separate centri-
fuged specimens. His urologic examination is notable for decreased testicular size. He has a total
testosterone level of 250 ng/dL, a follicle-stimulating hormone (FSH) level of 28 mIU/mL, negative
cystic fibrosis testing, 46,XY karyotype, and a Y chromosome AZFa microdeletion. In counseling
the couple, you explain that their best chance for conception is with
Approximately 15% of couples who try to conceive for a volume and an elevated FSH level. Approximately 12%
1-year period fail to do so. Up to 50% of those cases can of men with nonobstructive azoospermia will be found
be attributed to a male factor, sometimes manifested as to have karyotypic abnormalities, and 18% of these will
complete absence of sperm in the ejaculate, or azoosper- involve the Y chromosome. All men with azoospermia
mia. Azoospermia is confirmed when no sperm are found due to testicular failure should be offered genetic testing
in at least two separate centrifuged semen samples. with karyotype and Y chromosome microdeletion testing.
An azoospermic man should be evaluated with a com- Testing for terminal microdeletions of the long arm of the
plete history and physical examination. In addition, serum Y chromosome can help predict the likelihood of finding
total testosterone and FSH levels should be measured. sperm at the time of testicular exploration. Microdeletions
Men with hypogonadotropic hypogonadism also should of the Y chromosome will be found in approximately
have a serum prolactin level test and hypothalamic–pitu- 10–15% of men with severe oligospermia or azoosper-
itary imaging to exclude pituitary tumors. mia. In particular, three deletions in nonoverlapping
If the ejaculate volume is low, and testicular volume regions of the long arm of the Y chromosome (Yq11),
and FSH levels are normal, the patient may have obstruc- called the azoospermia factor (AZF), have been identi-
tive azoospermia. In cases of suspected obstructive azoo- fied: 1) AZFa (proximal), 2) AZFb (central), and 3) AZFc
spermia or severe oligospermia, patients should be (distal). Deletions involving the entire AZFa or AZFb
screened for cystic fibrosis. If at least one vas deferens is region predict a very poor prognosis for sperm retrieval,
palpable, a diagnostic testicular biopsy with simultaneous even with extensive testicular exploration. In men with
testicular sperm extraction can be performed. Possible AZFc deletions, reduced numbers of sperm may be pres-
reconstruction may subsequently be an option. ent in the ejaculate. Even if the ejaculate is azoospermic
In contrast to obstructive azoospermia, men with non- in a patient with an AZFc deletion, sperm extraction by
obstructive azoospermia often present with low testicular testicular biopsy still may be successful. In fact, men
166 PROLOG
with a known AZFc deletion have a greater probability futile. For this couple, donor sperm insemination would
of successful sperm retrieval than do men with idiopathic provide their best chance for conception. The couple also
azoospermia. If a man with an AZFc deletion success- might consider adoption.
fully conceives, any sons will inherit the abnormality.
Because the prognosis varies dramatically according to Hopps CV, Mielnik A, Goldstein M, Palermo GD, Rosenwaks Z,
Schlegel PN. Detection of sperm in men with Y chromosome micro-
the specific microdeletion, testing for Y chromosome deletions of the AZFa, AZFb and AZFc regions. Hum Reprod 2003;
microdeletion is strongly recommended for genetic and 18:1660 –5.
preoperative counseling in these patients. Practice Committee of American Society for Reproductive Medicine
In the described patient, testicular biopsy for sperm in collaboration with Society for Male Reproduction and Urology.
Evaluation of the azoospermic male. Fertil Steril 2008;90(Suppl):S74 –7.
procurement would be unsuccessful because of his AZFa
microdeletion; therefore, microsurgical testicular sperm Stahl PJ, Masson P, Mielnik A, Marean MB, Schlegel PN, Paduch DA.
A decade of experience emphasizes that testing for Y microdeletions is
extraction, testicular biopsy, epididymal sperm aspira- essential in American men with azoospermia and severe oligozoosper-
tion, and intracytoplasmic sperm injection would all be mia. Fertil Steril 2010;94:1753 –6.
120
Normal menopausal transition
A 39-year-old woman requests counseling for irregular menstrual cycles. She has no menopausal
symptoms at present but is concerned about the lengthening of her menstrual interval from 28 days
to 33 days with occasional missed cycles. The urine pregnancy test result is negative. You tell her
that if she develops irregular menses and menopausal symptoms before age 45 years, the most
appropriate next step is to check
Estimates of the median age at menopause typically range if the patient has a negative family history and a normal
from 50–52 years of age. The Massachusetts Women’s physical examination. Subclinical depression and midlife
Health Study (N=2,570) found the median age at meno- crisis are best assessed by careful attention to recent
pause to be 51.3 years. The transition to menopause nor- changes or stressors in the patient’s personal, social, or
mally is associated with alterations in menstrual bleeding professional life.
and the onset of vasomotor symptoms, such as night Menopausal symptoms that occur in a patient before
sweats. age 45 years are likely to be caused by something other
Many of the early symptoms of the normal menopausal than perimenopause. A patient who develops irregular
transition will overlap with other common conditions menses and menopausal symptoms before age 40 years
such as depression, mood swings, degenerative arthritis, should be investigated for primary ovarian insufficiency,
and hypothyroidism. The vast majority of women enter but other causes, such as hypothyroidism, are much more
menopause between ages 45 years and 56 years. Earlier common. When a patient has mild depression and fatigue
onset of menopause is associated with smoking, lower before this age, hypothyroidism is a much more likely
socioeconomic status, malnutrition, and a maternal his- diagnosis.
tory of early menopause. The described patient does not have symptoms at this
Discerning whether somatic and psychological symp- time but has asked what tests would be done if she were
toms in the fifth decade are a result of aging, depression, to develop irregular menses. You tell her that checking
menopause, hypothyroidism, midlife stressors, or chronic TSH and prolactin levels is the first step in the evalua-
illness can be clinically challenging. A normal TSH level tion of ovulatory dysfunction. For the patient to keep a
can rule out hypothyroidism. Joint pain and stiffness can menstrual calendar to monitor for shortening of the inter-
suggest degenerative arthritis, but this would be unlikely menstrual interval would not be appropriate as the next
Reproductive Endocrinology and Infertility 167
step. Similarly, checking estradiol and inhibin B levels Randolph JF Jr, Crawford S, Dennerstein L, Cain K, Harlow SD, Little
R, et al. The value of follicle-stimulating hormone concentration and
would constitute nonspecific tests in the detection of clinical findings as markers of the late menopausal transition. J Clin
menopause. If the patient’s TSH and prolactin levels are Endocrinol Metab 2006;91:3034 – 40.
normal, the FSH level should be checked next. However, Speroff L, Fritz MA. Menopause and the perimenopausal transition. In:
checking the FSH level would not be the first step. An Speroff L, Fritz MA. Clinical gynecologic endocrinology and infertil-
ity. 8th ed. Philadelphia (PA): Lippincott Williams and Wilkins; 2011.
FSH level in the menopausal range would identify pri- p. 686 –7.
mary ovarian insufficiency in the patient. Testing and interpreting measures of ovarian reserve: a committee
opinion. Practice Committee of the American Society for Reproductive
McKinlay SM, Brambilla DJ, Posner JG. The normal menopause transi- Medicine. Fertil Steril 2012;98:1407–14.
tion. Maturitas 1992;14:103–15.
121
Window of fertilization
A 28-year-old woman tells you that she would like to have her intrauterine device removed so she
can begin trying to conceive her first child. She is unsure of how to time intercourse and asks for
your advice. She has read that there is an optimal time to conceive known as the “window of fer-
tilization.” She and her husband plan to have intercourse every other day starting after cessation of
menstrual flow. You inform her that the time that the window of fertilization ends is
The luteinizing hormone (LH) surge typically occurs on for pinpointing the fertile window because of the normal
day 13 in a 28-day menstrual cycle. Follicular rupture fol- variation in cycle length and the fact that the temperature
lows 36–40 hours after the onset of the LH surge or up to rise accompanying ovulation usually is detected after the
24 hours after its peak. The usual reason to measure serum fact. Thus, the first day of temperature rise on the basal
or urinary LH is to predict ovulation. body temperature chart is not helpful to gauge the closing
Data suggest that the fertile window opens before fol- of the fertile window. When greater accuracy is needed,
licular rupture. The window begins the day before ovula- LH testing and ultrasonography may be employed.
tion and then declines rapidly thereafter. The fertilizable Serum LH measurements and ultrasonography need
life of an oocyte is 12–24 hours, meaning that the fertile to be performed serially and, as a result, are expensive.
window closes on the day of ovulation. Sperm, however, Therefore, these techniques usually are employed in con-
can survive in the reproductive tract for as long as 5 days, junction with other fertility treatments, such as insemina-
retaining the ability to fertilize an ovum even before ovu- tion or ovulation induction. Urinary LH kits are useful
lation occurs. The great majority of pregnancies occur to predict ovulation when compared with the other tests
when coitus takes place in the 3-day interval just before available. Fertility medications, polycystic ovary syn-
ovulation and the closure of the fertile window on the day drome, and the menopausal state can yield false-positive
of ovulation. results with over-the-counter urinary LH kits. Patient self-
Many options exist for couples who are trying to con- assessment of cervical mucus changes provides another
ceive and who need to know when to plan intercourse. accurate test that has essentially no cost.
Basal body temperature monitoring, calendar timing, and Peak production of cervical mucus occurs 2–3 days
ultrasonographic follicular monitoring, as well as serum before ovulation and does not mark the end of the fer-
LH testing and urinary LH testing, are options. Basal tility window. Similarly, the first day of the LH surge
body temperature charting and calendar timing have the would represent the midpoint of the fertile window, not
advantage of being low cost and noninvasive. For this the end. Serum progesterone levels are useful to confirm
reason, these methods are a logical starting place for that ovulation has taken place but have no role in predict-
patients. However, these methods are not very reliable ing ovulation.
168 PROLOG
Optimizing natural fertility. Practice Committee of American Society for Stanford JB, White GL, Hatasaka H. Timing intercourse to achieve
Reproductive Medicine in collaboration with Society for Reproductive pregnancy: current evidence. Obstet Gynecol 2002;100:1333–41.
Endocrinology and Infertility. Fertil Steril 2008;90:S1–6. Wilcox AJ, Weinberg CR, Baird DD. Timing of sexual intercourse in
Speroff L, Fritz MA. Regulation of the menstrual cycle. In: Speroff L, relation to ovulation. Effects on the probability of conception, survival
Fritz MA. Clinical gynecologic endocrinology and infertility. 8th ed. of the pregnancy, and sex of the baby. N Engl J Med 1995;333:1517–21.
Philadelphia (PA): Lippincott Williams and Wilkins; 2011. p. 199–242.
122
Use of stem cells in reproduction
The type of stem cells that presently hold the most promise for stem cell-based therapy for infertile
patients is
(A) hematopoietic
(B) embryonal
(C) embryonal carcinoma
* (D) induced pluripotent
The developing field of regenerative medicine attempts to leaves patients with the option of utilizing donated sperm
regenerate and replace damaged tissue. Research studies and oocytes. Several authors raised the possibility that
are investigating the creation of functional gametes from ESCs might provide a convenient and inexhaustible source
stem cells. It is thought that in the future this technology of oocytes. However, this approach would not allow
may help infertile couples to utilize their own genetic patients to have biologically related offspring. It also cre-
material to produce gametes for reproduction. ates an interesting ethical specter that a surplus embryo
Nearly every adult human tissue is believed to har- might become the “parent” of a multitude of offspring.
bor a population of stem cells, which are involved in Therefore, the use of ESCs as a therapy for infertility is
renewal (eg, replacement of blood cells), remodeling (eg, not ideal.
alterations to breast tissue before lactation), or repair Embryonal carcinoma cells are pluripotent cells derived
(eg, wound healing). Many of these cells are multipotent, from teratocarcinoma cells. Under the right conditions,
with the ability to differentiate into a number of cell they can be differentiated into germline stem cells.
types. Researchers have long sought to harness their However, zygotes created from embryonal carcinoma cell-
power for the treatment of human disease or injury. derived germ cells arrest at the 6–8-cell stage, presumably
Hematopoietic stem cells (HSCs) isolated from bone because of embryo imprinting abnormalities. Embryonal
marrow have been used successfully for the treatment carcinoma cells also do not allow for generation of patient-
of bone marrow and blood cancers for more than four specific therapies, and because of their teratocarcinoma
decades. However, adult stem cells such as HSCs appear origin, they may acquire additional undesirable genetic
to have a strong bias toward making cell types of the same mutations. Therefore, they are not currently a reasonable
lineage. This has limited their utility in basic research and source of gametes for infertile patients.
their therapeutic potential. Therefore, they are not typi- At present, induced pluripotent stem cells appear to be
cally seen as a viable option for generating gametes. the type of stem cells that offer the most promising stem
Embryonic stem cells (ESCs) offer greater promise cell-based therapy for infertile patients. Groundbreaking
because they are capable of differentiating into cells studies have demonstrated that retroviral-induced expres-
from all three germ layers: ectoderm, mesoderm, and sion of genes such as Oct4, Sox2, c-Myc, and Klf4 in
endoderm. A number of studies also have shown that dif- somatic cells is sufficient to convert them into a plu-
ferentiation of mouse and human ESCs into germline cells ripotent state. It is possible to transform a terminally
also is possible in vitro. Publication of these studies was differentiated skin fibroblast into a cell that resembles an
accompanied by speculation that ESC-derived gametes ESC by morphology, growth properties, and expression
might have a role in reproductive medicine. marker genes. This discovery won a Nobel Prize award in
Failure to conceive through assisted reproductive tech- 2012. This newly characterized class of stem cells opens
nologies because of the poor quality of their gametes the door for individualized therapies for a host of diseases
Reproductive Endocrinology and Infertility 169
and other health problems, including infertility. Induced Ichida JK, Blanchard J, Lam K, Son EY, Chung JE, Egli D, et al. A
small-molecule inhibitor of Tgf-β signaling replaces Sox2 in reprogram-
pluripotent stem cells can be differentiated into germ ming by inducing Nanog. Cell Stem Cell 2009;5:491–503.
cells that are capable of producing apparently healthy off- Maherali N, Sridharan R, Xie W, Utikal J, Eminli S, Arnold K, et al.
spring. One possible limitation of their utility is the fact Directly reprogrammed fibroblasts show global epigenetic remodeling
that viral modification of the genome is required for their and widespread tissue contribution. Cell Stem Cell 2007;1:55–70.
creation. These viral vectors are oncogenic and bring up Nayernia K, Li M, Jaroszynski L, Khusainov R, Wulf G, Schwandt
I, et al. Stem cell based therapeutical approach of male infertility by
safety concerns in this transgenic approach. Recent pub- teratocarcinoma derived germ cells. Hum Mol Genet 2004;13:1451–60.
lications have suggested that small molecules may soon
Nicholas CR, Haston KM, Grewall AK, Longacre TA, Reijo Pera RA.
replace the use of viruses, opening the door to therapeutic Transplantation directs oocyte maturation from embryonic stem cells
possibilities. and provides a therapeutic strategy for female infertility. Hum Mol
Genet 2009;18:4376–89.
Hayashi K, Ogushi S, Kurimoto K, Shimamoto S, Ohta H, Saitou M. Wernig M, Meissner A, Foreman R, Brambrink T, Ku M, Hochedlinger
Offspring from oocytes derived from in vitro primordial germ cell-like K, et al. In vitro reprogramming of fibroblasts into a pluripotent
cells in mice. Science 2012;338:971–5. ES-cell-like state. Nature 2007;448:318–24.
123
Bilateral tubal ligation and future fertility
A 28-year-old woman, gravida 1, para 1, is interested in tubal sterilization. She requests informa-
tion about the pros and cons of different approaches and tells you that she would prefer a method
that is reversible. You inform her that the sterilization method most amenable to surgical reversal is
Most women who choose sterilization do not regret their amount of tubal damage, and leaves the longest tubal
decision. Preoperative counseling should emphasize that segments for reanastomosis. Similarly, the small inert
the procedure is not intended to be reversible. Prospective synthetic ring produces minimal damage.
analysis of U.S. Collaborative Review of Sterilization The method of sterilization that is most amenable to
study data found that the cumulative probability of regret surgical reversal is laparoscopic application of a band or
over 14 years of follow-up was 12.7%. By age group, the clip. Laparoscopic electrocautery produces considerable
probability of regret was 20.3% for women aged 30 years tissue damage along the length of both tubes, and distal
or younger at the time of sterilization compared with 5.9% salpingectomy will not conserve distal tubal segments for
for women older than 30 years. reanastomosis.
Counseling should be comprehensive with the intent Hysteroscopic tubal occlusion can be performed in the
to minimize regret among individual women. Patients office setting and has an excellent efficacy and safety
who are younger than 30 years, who are of low parity, profile. Patients who undergo this procedure must rely
or who are in unstable relationships are at the highest on another method of contraception for at least 3 months
risk of regretting the decision for permanent sterilization. after the procedure. They also must undergo hysterosal-
Attention should be given to alternative methods such as pingography to confirm successful bilateral occlusion.
vasectomy or long-acting reversible contraception. This method of sterilization is designed to permanently
Two factors are associated with successful reversal of occlude the tubal lumen and leaves no chance for reversal
sterilization: 1) the amount of tissue damage produced of sterilization.
by the original procedure and 2) the length of proximal
Benefits and risks of sterilization. Practice Bulletin No. 133.
and distal tubal segments available for anastomosis. The American College of Obstetricians and Gynecologists. Obstet Gynecol
spring-loaded clip is only 5 mm wide, produces the least 2013;121:392–404.
170 PROLOG
Committee opinion: role of tubal surgery in the era of assisted reproduc- Peterson HB, Xia Z, Wilcox LS, Tylor LR, Trussel J. Pregnancy after
tive technology. The Practice Committee of the American Society for tubal sterilization with bipolar electrocoagulation. U.S. Collaborative
Reproductive Medicine. Fertil Steril 2012;97:539–45. Review of Sterilization Working Group. Obstet Gynecol 1999;94:
Hillis SD, Marchbanks PA, Tylor LR, Peterson HB. Post-sterilization 163–7.
regret: findings from the United States Collaborative Review of
Sterilization. Obstet Gynecol 1999;93:889– 95.
124
Pelvic inflammatory disease and reproductive dysfunction
A 27-year-old woman has a 2-year history of primary infertility. Her workup shows normal semen
analysis in her partner and cyclic menses along with left tubal patency and right proximal tubal
occlusion on hysterosalpingography (HSG). She recalls having several cervical or vaginal infec-
tions in her early 20s but does not believe she was ever diagnosed with pelvic inflammatory disease
(PID). The next test to determine if she has had PID would be
Tubal disease is one of the most common causes of In many women, PID is asymptomatic. Tubal disease
female infertility. It accounts for approximately 25–35% or peritoneal adhesions often are found in women with
of cases of infertility in U.S. women, with more than one infertility but no known prior history of PID. In one
half of cases resulting from salpingitis. Pelvic inflamma- study of patients with tubal factor infertility, only 31%
tory disease can be either an acute or chronic infection of patients could recall a prior episode of PID, but 79%
and is caused by the spread of pathologic bacteria from of patients had serologic evidence of prior C trachomatis
the lower to the upper genital tract with possible dissemi- infection. This often is referred to as “silent” PID.
nation into the peritoneal cavity. The infection typically The most reliable test to determine if the described
spreads to both fallopian tubes. According to the Centers patient has had PID in the past is to check her C tracho-
for Disease Control and Prevention, acute PID affects matis antibody levels. The left tube is patent, but she has
more than 1 million U.S. women per year, and more than a history of secondary infertility for the past year. Her
100,000 women per year will become infertile as a result clinical decision making will be influenced by evidence
of the infection. of prior PID, eg, if C trachomatis antibodies are present,
In addition to infertility, long-term sequelae of PID she may proceed to IVF.
include ectopic pregnancy and chronic pelvic pain. The Laparoscopy with chromotubation and HSG are the
incidence of post-PID ectopic pregnancy is approximately primary means of detecting tubal disease. Chlamydial anti-
4–9%, six times higher than the general population. The body testing is an indirect means of screening for tubal dis-
likelihood of ectopic pregnancy rises with each subse- ease. The test is noninvasive and relatively inexpensive. A
quent episode, and it is estimated that 50% of ectopic repeat HSG is unlikely to be helpful. Unilateral proximal
pregnancies are a consequence of prior PID. Similarly, tubal occlusion is commonly caused by spasm and rarely
the incidence of infertility increases with the severity and indicates pathology. A detailed history may not help in the
number of prior tubal infections. A single episode of PID case of silent PID. Cervical cultures would detect current
will cause infertility in approximately 8–13% of infected but not past infection. Laparoscopic surgery could be help-
women. A second episode will increase the incidence to ful but is more invasive than antibody testing.
20–36%, and a third episode causes infertility in up to
75% of infected women. Akande VA, Hunt LP, Cahill DJ, Caul EO, Ford WC, Jenkins JM. Tubal
damage in infertile women: prediction using chlamydia serology. Hum
Salpingitis is usually a polymicrobial process. The most Reprod 2003;18:1841–7.
common organisms found in salpingitis are C tracho-
Committee opinion: role of tubal surgery in the era of assisted repro-
matis, N gonorrhoeae, Staphylococcus, Streptococcus, ductive technology. Practice Committee of the American Society for
Mycoplasma, and Ureaplasma urealyticum. Reproductive Medicine. Fertil Steril 2012;97:539–45.
Reproductive Endocrinology and Infertility 171
den Hartog JE, Morre SA, Land JA. Chlamydia trachomatis-associated Veenemans LM, van der Linden PJ. The value of Chlamydia trachoma-
tubal factor subfertility: immunogenetic aspects and serological screen- tis antibody testing in predicting tubal factor infertility. Hum Reprod
ing. Hum Reprod Update 2006;12:719–30. 2002;17:695–8.
125
Obesity and pregnancy
A 31-year-old woman, gravida 2, para 1, aborta 1, with oligomenorrhea wishes to become preg-
nant. Her body mass index (BMI) is 36 (calculated as weight in kilograms divided by height in
meters squared). Her thyroid-stimulating hormone level is 2.2 mU/mL and her prolactin level is
14 ng/mL. The best recommendation for her is the use of
In 2008, 28.6% of U.S. women were overweight (BMI modification plus exercise. The most successful diets
of 25–29.9) and 35.5% were obese (BMI greater than educate patients on healthy food choices and portion
30). Obesity is the most common chronic disease in the control. Nutritionists teach patients how to read food
United States. The simplest measurement of weight is labels and substitute healthier food choices for those high
BMI. Obesity is associated with diabetes mellitus, hyper- in fat and glycemic content. Ovulatory function may
tension, hypertriglyceridemia, hypercholesterolemia, and return with weight loss, which will allow the patient to
musculoskeletal disease. have control over her menstrual cycle and to time inter-
Reproductive health is adversely affected by obesity. course more precisely.
Anovulation and oligo-ovulation are common in over- Clomiphene citrate is not the best initial recommenda-
weight and obese women. Menstrual cycle irregularities tion for this patient. Clomiphene is an ovulation induction
are attributed to alterations in gonadotropin-releasing agent for the anovulatory patient with normal thyroid-
hormone secretion leading to a dysfunctional hypotha- stimulating hormone and prolactin levels. This patient
lamic–pituitary–ovarian axis. Weight loss often will nor- is obese and, therefore, dietary and lifestyle changes are
malize ovulatory function and increase pregnancy rates in her first-line treatment strategy. Many obese patients who
obese patients. Obese patients who become pregnant have use clomiphene for ovulation induction do not respond to
an increased risk of miscarriage and birth defects. Causes treatment. In addition, a multiple pregnancy is a greater
for these adverse pregnancy outcomes may be attribut- risk for an obese patient because her comorbidities will
able to undiagnosed metabolic disturbances. Pregnancy be exacerbated. Obesity is a multifactorial problem with
complications, including gestational diabetes, diabetes behavioral and neurohormonal complexities that need
mellitus, preeclampsia, cesarean delivery, and hyperten- to be treated through a combination of lifestyle changes
sion, are common in overweight and obese patients. and sometimes by adding medications.
Fetuses of obese patients are at increased risk of congeni- Bariatric surgery is an option for the described patient
tal anomalies, stillbirth, macrosomia, difficult deliveries, if lifestyle modifications prove unsuccessful. The two
and childhood obesity. Maternal and fetal risks can be a most common procedures are the Roux-en-Y gastric
strong motivator for obese patients to lose weight before bypass and gastric banding. A number of studies have
conception. shown that 15% of patients who have undergone bariatric
This patient is obese with oligomenorrhea and has surgery maintain their reduced weight. Bariatric surgery
come in for a preconception consultation. The best recom- is best reserved for patients with a BMI greater than 40
mendation is for her to meet with a nutritionist to initiate a or greater than 35 with comorbidities. Surgery is the most
weight-loss program incorporating dietary and behavioral effective therapy for morbid obesity, decreasing and even
172 PROLOG
eliminating comorbidities while improving quality of life. is lifestyle modification. Once she has become regular
Menstrual cycle regularity and ovulation often return in and ovulatory, timing intercourse or using an ovulation
the oligo-ovulatory and anovulatory patient. predictor kit can be instituted.
Fecundability peaks in the 2 days before ovulation.
Holes-Lewis KA, Malcolm R, O’Neil PM. Pharmacotherapy of obesity:
In women with regular menstrual cycles, increasing the clinical treatments and considerations. Am J Med Sci 2013;345:284–8.
frequency of intercourse after the cessation of menses Kumbak B, Oral E, Bukulmez O. Female obesity and assisted reproduc-
increases the chances of pregnancy. In this patient, who tive technologies. Semin Reprod Med 2012;30:507–16.
has oligomenorrhea (menses occurring at intervals greater Lash MM, Armstrong A. Impact of obesity on women’s health. Fertil
than 35 days), an ovulation predictor kit is unlikely to be Steril 2009;91:1712–6.
accurate because her cycles are irregular and timing Moran LJ, Norman RJ. The effect of bariatric surgery on female repro-
ductive function. J Clin Endocrinol Metab 2012;97:4352–4.
the luteinizing hormone surge will be cumbersome. In
Obesity and reproduction: an educational bulletin. Practice Committee
addition, she has anovulatory cycles even though she is of American Society for Reproductive Medicine. Fertil Steril 2008;
menstruating. The best recommendation for this patient 90:S21–9.
126
Donor oocyte use
A 26-year-old woman, gravida 2, para 2, comes to see you after undergoing laparoscopic tubal
ligation. She is interested in serving as an oocyte donor. She has noticed that each clinic has a limit
on the number of cycles for each donor. She also has learned that there is a limit to the number of
times that a sperm donor may be used. You advise her that the most valid reason to limit the repeti-
tive use of donor gametes from males and females is to avoid the risk of
Data from the Centers for Disease Control and Prevention Certain risks to society are known to emanate from
and the Society for Assisted Reproductive Technologies gamete donation (either sperm or egg donation).
show a live-birth rate of 4.2% for patients older than Inadvertent consanguinity could occur if a donor has
42 years who use in vitro fertilization (IVF) with their donated to two or more families in a relatively confined
own oocytes. Oocyte donation has become an important population and the offspring are unaware of their specific
option with delayed childbirth having become a signifi- genetic heritage. For this reason, the American Society
cant trend over the last two decades. for Reproductive Medicine has published guidelines that
Indications for use of a donor egg are diminished ovar- advise an arbitrary limit of no more than 25 pregnancies
ian reserve, surgical or spontaneous menopause, genetic per sperm or oocyte donor per population of 800,000 to
abnormalities in the female (eg, balanced translocation or limit inadvertent consanguinity. It seems reasonable to
autosomal disorder carrier status), history of gonadotoxic assume that these limitations are more directly applicable
therapies, or multiple IVF failures using the patient’s own to sperm donors than egg donors. The most valid reason to
oocytes. In 2011, the Society for Assisted Reproductive limit the repetitive use of donor gametes from males and
Technologies database recorded a 54.9% live-birth rate females is to avoid the risk of inadvertent consanguinity.
for patients who used donor oocytes in fresh embryo Specific risks to the egg donor include surgical risks
transfers. For the patient who has diminished ovarian from repeated oocyte retrieval procedures and exposure
reserve and who has not insisted on the use of her own risks from repeated cycles of controlled ovarian hyper-
oocytes, oocyte donation represents the best chance to stimulation, notably the possible increased risk of ovar-
achieve live birth. The literature showing gamete donation ian cancer. At present, there is no documentation on any
to be safe and effective for the recipients of donor eggs is adverse long-term sequelae resulting from follicle aspira-
well documented. The risks of repetitive oocyte donation tion. The patient who undergoes repetitive retrieval pro-
are less well known. cedures is exposed to possible anesthetic complications
Reproductive Endocrinology and Infertility 173
and procedure-related pelvic infections, both of which limit of six stimulated cycles in order to protect the safety
are very small in number. Another risk is ovarian hyper- of oocyte donors.
stimulation syndrome. Donors are tested routinely for
HIV and hepatitis B. As a result, transmission of either de Boer A, Oosterwijk JC, Rigters-Aris CA. Determination of a maxi-
mum number of artificial inseminations by donor children per sperm
virus to the recipient is extremely rare. donor. Fertil Steril 1995;63:419–21.
Concern that ovarian stimulation might increase the risk Repetitive oocyte donation. Practice Committee of American Society
of ovarian cancer was first reported in 1994. Additional for Reproductive Medicine. Fertil Steril 2008;90(suppl):S194–5.
data analysis has either shown no association between the Society for Assisted Reproductive Technology Clinic Outcome
use of ovulation-inducing agents and ovarian cancer or Reporting System. IVF success rate reports: clinic summary report.
Birmingham (AL): SART; 2014. Available at: https://www.sartcor-
that the risk was only elevated when treatment exceeded sonline.com/rptCSR_PublicMultYear.aspx?ClinicPKID=0. Retrieved
12 cycles. For this reason, the American Society for March 14, 2014.
Reproductive Medicine has published guidelines to set a
127
Breast cancer
A 38-year-old white woman, gravida 3, para 2, comes to your office for her annual well-woman
examination. She informs you that her mother was treated for breast cancer when she was age
44 years and was found not to be a BRCA1 or BRCA2 mutation carrier. You decide to evaluate
her for a possible increased risk of developing breast cancer using the online Breast Cancer Risk
Assessment Tool. In addition to the age at which she gave birth to her firstborn child, the informa-
tion you need from her medical history is the patient’s
In recent years, several tools have been developed to help patients are at a significantly high risk of breast cancer
patients and health care providers assess risk of future development and should be referred directly to a genetic
cancer. These tools provide risk estimates compared with counselor or an oncology team that specializes in risk
the average risk of a specific age group over a certain assessment, screening, and prevention strategies.
period (often over the next 5–10 years). Most models Many factors can influence the risk of breast cancer,
combine many risk factors because the risk associated such as menstrual and reproductive history, age, ethnicity
with a single factor is usually too small to provide signifi- or race, diet, and alcohol use (Box 127-1). If the model
cant guidance about screening plans. indicates that the 5-year risk of breast cancer is 1.7% or
The most widely used model to help predict risk of greater, patients are encouraged to consider risk-reduction
breast cancer is the Breast Cancer Risk Assessment Tool, strategies, such as lifestyle modification, medical therapy
often called the Gail model. This model combines sev- with tamoxifen citrate, or surgery with mastectomy
eral risk factors for breast cancer and allows health care or bilateral salpingo-oophorectomy. If a woman has a
providers to stratify patients into risk categories. The tool life expectancy of less than 10 years, there is probably
enables better screening strategies for the optimal man- minimal benefit to screening to determine eligibility for
agement of individual patients. Although the Gail model risk-reduction strategies.
performs well in populations of women, it has been shown Prior use of OCs has been studied to assess the associa-
to be less useful for the rare woman who has a very sig- tion with breast cancer development. Many large case–
nificant family history of breast cancer. In patients who control studies of low-dose OCs have consistently shown
have a known BRCA mutation in their family or who no relation between breast cancer and current or past use
have a personal history of breast cancer, the Gail model of OCs. Information about OC use is not used to estimate
is not appropriate to triage their screening or care. Such risk in the Gail model.
174 PROLOG
128–130
Evaluation of gonads in androgen insensitivity syndrome
For each clinical scenario (128–130), choose the likely diagnosis (A–F).
Androgen insensitivity syndrome (AIS) involves a muta- TABLE 128–130-1. Categorization of Androgen
tion of the androgen receptor located on chromosome Insensitivity Syndrome
Xq11-12, resulting in complete loss of function (CAIS)
Grade Type Description
or partial loss of function (PAIS), also referred to as
Reifenstein syndrome. Approximately 2–5 women per 1 PAIS Normal male phenotype, possibly
100,000 will exhibit CAIS. More than 500 mutations gynecomastia or mild impairment
have been described, most of which are single-base sub- of virilization. Possible infertility.
stitutions. The androgen receptor is a transcription factor 2 PAIS Male phenotype, but small penis,
regulating gene transcription in response to androgens, penoscrotal hypospadias.
including 5α-dihydrotestosterone and testosterone. 3 PAIS Predominantly male phenotype
The presentation of AIS depends on the degree of loss with micropenis, perineal hypo-
of function of the androgen receptor. Typically, CAIS is spadia, cryptorchidism and
possibly bifid scrotum.
seen in teenagers with appropriate breast development,
reduced or absent pubic and axillary (underarm) hair, 4 PAIS Ambiguity of the external genitalia:
and primary amenorrhea. A less common presentation very large clitoris, urogenital sinus
with perineal opening and
is the finding of a newborn female after a prenatal chro- labioscrotal folds.
mosome analysis showed a normal male karyotype. In
5 PAIS Predominantly female phenotype:
a female infant or child, an inguinal hernia may sug- large clitoris, separate openings
gest CAIS, with an incidence of approximately 1.1%. of the urethra and vagina.
Age-appropriate evaluation should be considered in
6 PAIS Female phenotype, androgen-
this circumstance. Partial AIS has a varied presentation induced pubic and axillar hair
depending on the degree of receptor function and number growth at the time of puberty.
(Table 128–130-1). 50% inguinal hernia.
Complete loss of androgen receptor activity causes lack 7 CAIS Normal female phenotype. Lack
of pigmented sexual hair in the pubic and axillary regions, of androgen-induced pubic and
lack of male external genitalia and prostate development axillar hair growth at the time of
primarily under the control of 5α-dihydrotestosterone, puberty. 50% inguinal hernia.
and lack of internal male reproductive organs, including Abbreviations: CAIS, complete loss of function; PAIS, partial
epididymis, seminal vesicles, and vas deferens (wolff- loss of function.
ian ducts), primarily under the control of testosterone. Republished with permission of the Endocrine Society from
Because the testes normally produce antimüllerian hor- Endocrine Reviews. Quigley CA, De Bellis A, Marschke KB, el-
Awady MK, Wilson EM, French FS. Androgen receptor defects:
mone, also called müllerian-inhibiting substance, the historical, clinical, and molecular perspectives (Erratum in
müllerian ducts that result in the fallopian tubes, uterus, Endocr Rev 1995;16:546). Endocr Rev 1995;16:271–321.
cervix, and upper two thirds of the vagina are absent. Copyright 1995.
176 PROLOG
Estrogen production in patients with CAIS is approxi- Treatment for CAIS is directed at hormone replace-
mately twice that of normal males and originates from ment after removal of the gonads and development of a
extraglandular formation and secretion from the testes. functional vagina. An increased risk of gonadoblastoma
Estradiol is derived from direct secretion from the testes and dysgerminoma in the gonads is evident, with risk
and from extraglandular conversion from testosterone via greater in PAIS compared with CAIS (approximately
aromatization. Estrone is derived mainly from extraglan- 15% versus 1%, respectively). The risk increases with
dular conversion of androstenedione via aromatization age and is approximately 33% at age 50 years, although
and from metabolism of estradiol. An increase in testos- this has not been analyzed separately for each condition.
terone production from the testes is responsible for the Given the low risk for most individuals, it is reasonable to
increased estrogen levels and is due to higher luteinizing await completion of breast development in women with
hormone production by the pituitary. Increased pituitary CAIS before prophylactic gonadectomy. Earlier removal
luteinizing hormone production is secondary to lack of should be considered in women with PAIS, with con-
appropriate negative inhibition by testosterone on the sideration given to the degree of androgenization. After
hypothalamus. gonadectomy, age-appropriate hormone therapy should
With CAIS, physical examination generally shows be instituted.
Tanner stage 5 breast development, Tanner stage 1 pubic Creation of adequate vaginal caliber for intercourse
hair development, and a shortened, blind-ending vagina, should be based primarily on anticipated timing of sexual
typically 2.5–3.0 cm in length. Rectal examination or pel- activity. Use of vaginal dilators should be the first choice.
vic ultrasonography shows no evidence of a uterus. The More than 75% of women will achieve adequate vaginal
gonads are typically in the pelvis with CAIS, but with dimensions for successful intercourse by using vaginal
grades 4–6 PAIS, they may be located in the inguinal dilators. Vaginal depth should be obtained first with
canal or in the labia majora. Patients with CAIS have later more narrow dilators followed by dilators with increasing
closure of the epiphyses compared with normal women, breadth. Surgical therapy may be necessary in women
most likely due to lower total estrogen levels. Thus, final with failed attempts at dilation. A number of surgical
height is greater than normal females but less than nor- approaches are available, including the development of a
mal males. In general, patients with CAIS are not at risk neovagina with a skin graft (McIndoe procedure).
of any other congenital anomalies. Evaluation of other The attitude toward disclosure of the diagnosis of
organ systems is not necessary. Psychosexual function AIS has changed with time. Until the late 1990s, many
in patients with CAIS is the same as in normal females. physicians suggested that there was no need to explain
The diagnosis of CAIS is based on the physical the pathophysiology. Patients were simply told that they
examination findings along with supporting laboratory had a birth anomaly that resulted in the lack of a uterus
analysis. The karyotype is that of a normal male with a and the need for “ovary” removal due to increased risk of
serum testosterone level in the normal or high-normal cancer. Attitudes have changed because of more empha-
male range. In contrast, females born with pure gonadal sis on patient justice, nonmaleficence, beneficence, and
dysgenesis have a male karyotype but female levels of autonomy. Another contributing factor has been the
testosterone because the gonad typically does not secrete availability of health information on the Internet, which
hormones. Unrelated trisomies of the sex chromosomes can lead to some patients discovering the diagnosis with-
include 47,XXX (Trisomy X syndrome) and 47,XXY out the aid of counseling. At present, the recommendation
(Klinefelter syndrome). is to discuss the diagnosis with the assistance of counsel-
The differential diagnoses include imperforate hymen, ors trained in disorders of sexual development.
transverse vaginal septum, and müllerian agenesis,
also known as vaginal agenesis or Mayer–Rokitansky– Hiort O. Clinical and molecular aspects of androgen insensitivity.
Endocr Dev 2013;24:33–40.
Küster–Hauser syndrome. Imperforate hymen and trans-
MacDonald PC, Madden JD, Brenner PF, Wilson JD, Siiteri PK. Origin
verse vaginal septum typically occur with episodic pelvic of estrogen in normal men and in women with testicular feminization. J
pain, and a uterus is identifiable on rectal examination. Clin Endocrinol Metab 1979;49:905–16.
Patients with müllerian agenesis will differ from those Oakes MB, Eyvazzadeh AD, Quint E, Smith YR. Complete andro-
with CAIS by the presence of normal pubic and axillary gen insensitivity syndrome—a review. J Pediatr Adolesc Gynecol
2008;21:305–10.
hair, a normal XX karyotype, and a serum testosterone
level in the female range.
Reproductive Endocrinology and Infertility 177
131–133
Hereditary cancer syndromes
For each clinical scenario (131–133), choose the most likely genetic disorder (A–E).
131. B. A 53-year-old woman was recently diagnosed with breast cancer. She has a paternal grand-
mother with breast and ovarian cancer and a father with prostate cancer. She is most likely
to have a mutation associated with a region in exon 11 of the mutated gene.
132. D. Autosomal dominant disorder associated with mutations of the STK11 gene that causes
tumors of the gastrointestinal tract, breast, ovary, cervix, and testis as well as mucocutaneous
pigmentation.
133. C. Germline mismatch mutation of the MLH6 gene that confers a higher risk of endome-
trial cancer and a slightly lower risk of colorectal cancer compared with MLH1 and MSH2
mutations.
Approximately 10% of cases of breast cancer and 15% The normal BRCA1 gene suppresses estrogen-depen-
of cases of ovarian cancer are found to be associated dent transcription pathways. Paradoxically, mutations of
with a hereditary gene mutation. Although the most this gene are associated with tumors that are less likely to
common mutations found with breast and ovarian cancer express estrogen and progesterone receptors. The BRCA2
are mutations of the BRCA1 and BRCA2 genes, other gene mutations are associated with an increased risk of
less common mutations, such as hereditary nonpolypo- prostate cancer in male family members. In addition,
sis colorectal cancer and Peutz–Jeghers syndrome, can the BRCA2 gene has a region in exon 11 that appears
confer a similar risk profile, although each has unique to confer an increased risk of breast cancer (Clinical
characteristics. scenario 131).
BRCA1 and BRCA2 are tumor suppressor genes, which Peutz–Jeghers syndrome is a rare autosomal dominant
encode two types of proteins that have specific roles: syndrome caused by germline mutations in a serine/thre-
• Caretaker proteins maintain the integrity of the onine kinase (STK11) on chromosome 19p13.3 (Clinical
genome and gene stability. scenario 132). This syndrome is commonly associated
with hamartomatous polyps in the gastrointestinal tract
• Gatekeeper proteins regulate tumor growth directly
and mucocutaneous melanin pigmentation of the lips,
via mitosis and apoptosis.
buccal mucosa, fingers, and toes (Fig. 131–133-1; see
In encoding these two protein types, the genes limit cell color plate). Carriers are at increased risk of gastrointes-
replication. Mutations in tumor suppressor genes can tinal cancers (stomach, colorectal, small intestine, and
result in defects in the proteins’ caretaker and gatekeeper pancreas) as well as cancers of the lung, breast, uterus,
roles. and ovary.
The BRCA1 gene is located on chromosome 17q21, Lynch II syndrome is characterized by mutations
and to date more than 800 mutations of the gene have of the MLH1, MSH2, MSH6, and PMS2 genes, which
been identified. The BRCA2 gene is located on chromo- are involved in the function of DNA mismatch repair
some 13q12-13. Both types of gene mutations follow an enzymes. The syndrome has an autosomal dominant pat-
autosomal dominant pattern. Cumulative risk of breast tern and is associated with an increased risk of colon can-
cancer by age 70 years is different for BRCA1 mutations cer, especially in carriers younger than 50 years. There
than for BRCA2 mutations, approximately 57% (95% is also an elevated risk of cancers of the endometrium,
confidence interval [CI], 47–66%) and 49% (95% CI, ovary, gastrointestinal tract, upper urinary tract, and bile
40–57%), respectively. A patient’s personal risk must be duct (Clinical scenario 133). The risk of specific types of
individualized based on medical and reproductive history. cancer varies based on which of the four DNA mismatch
178 PROLOG
repair genes is mutated, ie, there is an increased risk of breast and ovarian cancer, with similar rates of occur-
endometrial cancer in families where the MLH6 gene is rence as those seen in carriers of BRCA1 and BRCA2
abnormal versus families with MLH1 and MSH2 muta- mutations.
tions. The lifetime risk of colon cancer is 30–70% and the
risk of ovarian cancer is 4–11%. Chen S, Parmigiani G. Meta-analysis of BRCA1 and BRCA2 pen-
etrance. J Clin Oncol 2007;25:1329–33.
More recently, other genetic mutations have been dis-
Daniels MS. Genetic testing by cancer site: uterus. Cancer J 2012;
covered that are associated with an increased risk of ovar- 18:338–42.
ian and breast cancer. For example, the RAD51C gene is National Comprehensive Cancer Network. NCCN Guidelines. Fort
involved in DNA double-strand break repair. Mutations Washington (PA): NCCN; 2014. Available at: http://www.nccn.org
in this gene have been associated with increased rates of professionals/physician_gls/f_guidelines.asp. Retrieved March 20, 2014.
134–138
Lifestyle choices and pregnancy outcome
Match the described patient (134–138) with the most likely outcome (A–E).
134. A. A woman has undergone bariatric surgery and has had a 35.4-kg (78-lb) weight loss and
frequent loose stools after eating.
135. C. An obese anovulatory woman has lost 5–10% of her body weight.
136. B. A 38-year-old woman has smoked one pack of cigarettes a day for 15 years.
137. D. A woman has severe maternal vitamin D deficiency during pregnancy.
138. E. A 37-year-old woman drank alcohol heavily in the first 10 weeks of gestation.
The preconception period provides a time to address loss and regular exercise are the first recommendations
and modify lifestyles that can increase the likelihood of for obese patients to reduce the incidence of these comor-
a healthy mother and baby. During the patient’s initial bidities. For patients who continue to have a BMI over 40
consultation, there should be an open dialogue concern- or a BMI over 35 with comorbidities after taking these
ing nutrition, exercise, and use of tobacco, alcohol, and steps, bariatric surgery is an option that decreases mater-
vitamins. nal and fetal pregnancy complications. Bariatric surgery
Obesity continues to be the most common chronic patients are at increased risk of nutritional deficiencies,
disease in the United States; it is defined as a body and an evaluation is indicated in the patient when there
mass index (BMI) of 30 or more (calculated as weight is a risk of a deficiency. Patient 134 has loose stools and
in kilograms divided by height in meters squared). In a history of bariatric surgery, so an evaluation for iron,
reproductive-aged patients, obesity often is associated vitamin D, calcium, vitamin B12, and folate deficiencies
with ovulatory dysfunction. Obese pregnant patients are should be initiated.
at increased risk of pregnancy complications, including More than 30% of obese patients have irregular
gestational diabetes mellitus, preeclampsia, hyperten- menstrual cycles caused by ovulatory dysfunction. This
sion, and cesarean delivery. Fetal risks of obese pregnant dysfunction leads to infertility and has been linked to
women include increased risks of pregnancy loss, pre- pregnancy loss and birth defects. Ovulation induction
maturity, macrosomia, and congenital anomalies. Weight with clomiphene citrate or gonadotropins has been used
Reproductive Endocrinology and Infertility 179
to help anovulatory infertility patients. Before using international units of vitamin D. Severe deficiency of vita-
these agents, loss of 5–10% of body weight in an obese min D in pregnancy has been associated with congenital
patient can trigger resumption of regular menstrual cycles. rickets and fractures in the newborn (Patient 137).
The mechanism for cycle resumption is decreased aro- Fetal alcohol syndrome results from the teratogenic
matization of androgens to estrogen in adipose tissue effects of alcohol (Patient 138). This syndrome is associ-
(Patient 135). ated with central nervous system abnormalities (impaired
Up to 30% of reproductive-aged women smoke ciga- intellectual development) and facial dysmorphia. The
rettes. Smoking is linked to infertility, decreased fecun- U.S. Surgeon General and the American College of
dity, pregnancy loss, and increased time to conception. Obstetricians and Gynecologists advise pregnant women
Tobacco smoke has toxins that compromise oocyte not to drink alcohol. Level of alcohol consumption can
quality. These toxins accelerate follicular depletion and be ascertained during preconception with the use of the
decrease reproductive function. Menopause starts earlier T-ACE questionnaire (Table 134–138-1). A patient who
in smokers. Patients who smoke should be counseled on drinks heavily should be counseled during preconception
the benefits of smoking cessation. Preconception care and pregnancy to curtail alcohol consumption.
presents an opportunity for such counseling to be success-
At-risk drinking and alcohol dependence: obstetric and gynecologic
ful, given that it is a time when patients may be motivated implications. Committee Opinion No. 496. American College of
to stop smoking (Patient 136). Obstetricians and Gynecologists. Obstet Gynecol 2011;118:383–8.
Vitamin D is produced in the skin from sunlight expo- Bariatric surgery and pregnancy. ACOG Practice Bulletin No. 105.
sure and obtained from dietary supplements, juice, fish American College of Obstetricians and Gynecologists. Obstet Gynecol
2009;113:1405–13.
oils, and fortified milk. Vitamin D deficiency is more
Obesity and reproduction: an educational bulletin. Practice Committee
prevalent in women who are vegetarians, women with of American Society for Reproductive Medicine. Fertil Steril 2008;
limited exposure to sunlight, and women with darker 90:S21–9.
skin. Prenatal vitamins usually contain 400 international Smoking and infertility: a committee opinion. Practice Committee of
units of vitamin D. For pregnant patients at risk of vita- the American Society for Reproductive Medicine. Fertil Steril 2012;
98:1400–6.
min D deficiency, serum levels of 25-OH-D-vitamin D
Vitamin D: screening and supplementation during pregnancy. Committee
should be drawn. If levels are less than 32 ng/mL, most Opinion No. 495. American College of Obstetricians and Gynecologists.
experts suggest daily supplementation with 1,000–2,000 Obstet Gynecol 2011;118:197–8.
139–142
Hormonal changes in pregnancy
Hormonal changes that take place during pregnancy are transportation of glucose to the fetus. These effects may
extensive. In pregnancy, a new endocrine organ, the pla- predispose the mother to gestational diabetes mellitus.
centa, plays a role in these hormonal changes by synthe- Maternal plasma cortisol increases in pregnancy sec-
sizing large precursor hormones, steroid hormones, and ondary to an increase in cortisol-binding globulin. The
various protein and peptide hormones. placenta produces CRH, which causes an increase in
Estrogen and progesterone levels increase significantly maternal cortisol levels. Typically, corticosteroids cause
in pregnancy. Maternal progesterone levels rise from a negative feedback effect on CRH production; however,
approximately 25 ng/mL in the luteal phase to 150 ng/mL in the placenta, corticosteroids have a positive feedback
at term. Fetal well-being does not affect progesterone lev- effect on the placental cell CRH output. It is believed that
els because production of progesterone by the placenta is CRH promotes myometrial contractility and may play a
dependent on maternal supply of low-density lipoprotein role in parturition.
cholesterol; the placenta lacks the ability to synthesize Thyroid gland size increases slightly in pregnancy.
cholesterol from acetate. Progesterone is thought to Despite marked increase in maternal serum total triio-
inhibit smooth muscle contractility and cause uterine qui- dothyronine and total T4, the pregnant woman remains
escence. Estrogens also increase markedly in pregnancy. euthyroid because of a parallel increase in thyroxine-
The corpus luteum is the initial source of estrogen in the binding globulin. Therefore, the levels of free triiodo-
first few weeks of pregnancy, but later the trophoblast of thyronine and T4 do not increase during pregnancy. It is
the placenta produces the bulk of estrogen seen in preg- believed that thyroxine-binding globulin is increased in
nancy. Estradiol and estrone are synthesized by the pla- response to estrogen exposure. Antimüllerian hormone
centa via conversion of dehydroepiandrosterone sulfate levels decrease during pregnancy. This decrease may be
(DHEAS), which reaches the placenta from the maternal related to ovarian suppression that normally takes place in
and fetal bloodstreams. However, estriol production is pregnant women and is independent of the woman’s age.
dependent upon conversion of DHEAS that is derived
Beshay V, Carr BR. Fertilization, implantation, and endocrinology of
from the fetal adrenal gland to 16α-hydroxy DHEAS. pregnancy. In: Kovacs WJ, Ojeda SR, eds. Textbook of endocrine phys-
This conversion occurs from the fetal liver. Because of iology. 6th ed. New York: Oxford University Press; 2012. p. 264–91.
the fetal contribution to estrogen production, estrogens, Beshay VE, Carr BR, Rainey WE. The human fetal adrenal gland,
especially estriol, have been used historically to monitor corticotropin-releasing hormone, and parturition. Semin Reprod Med
2007;25:14–20.
fetal well-being because their production is dependent on
Braunstein GD. Endocrine changes in pregnancy. In: Kronenberg HM,
fetal production of prehormones. Melmed S, Polonsky KS, Larsen PR, editors. Williams textbook of
Human placental lactogen is a product of the placenta endocrinology, 11th ed. Philadelphia (PA): Saunders Elsevier; 2008.
and has growth hormone-like activity. Its level increases p. 741–52.
with advancement of the pregnancy. Most actions of this Carr BR. The maternal–fetal–placental unit. In: Becker KL, editor.
hormone take place on the maternal side, causing lipoly- Principles and practice of endocrinology and metabolism. 3rd ed.
Philadelphia (PA): Lippincott Williams & Wilkins; 2001. p. 1059–72.
sis, which results in increased free fatty acids in maternal
Koninger A, Kauth A, Schmidt B, Schmidt M, Yerlikaya G, Kasimir-
and fetal circulation, as well as development of insulin Bauer S, et al. Anti-Müllerian-hormone levels during pregnancy and
resistance and inhibition of gluconeogenesis favoring the postpartum. Reprod Biol Endocrinol 2013;11:60.
Reproductive Endocrinology and Infertility 181
143–145
Statistical analysis
For each experiment (143–145), choose the most appropriate statistical analysis (A–E).
(A) Analysis of variance (ANOVA) with post-hoc Tukey honestly significant difference
testing
(B) Student t-test
(C) Kaplan–Meier curve
(D) Fisher exact test
(E) Mann–Whitney U test
143. D. A study to compare whether the uterus increased in mass (yes or no) in six mice fed a soy
diet versus six unrelated mice fed a regular diet
144. A. A laboratory study of breast cancer cell line proliferation rates after treatment with estradiol,
estradiol plus norethindrone, or vehicle
145. C. Longitudinal pregnancy rates in women with endometriosis after diagnostic versus operative
laparoscopy
Basic knowledge of statistical analyses includes under- Statistical testing can be defined as nonparametric
standing data types, distribution, and tests for com- and parametric. Nonparametric testing (distribution-free
parisons of two groups and of multiple groups. Data may testing) is used in ordinal data, nominal data, and
be qualitative or quantitative. Qualitative data are not quantitative data that are not normally distributed.
expressed in numbers and may be either ordinal or nomi- Parametric tests are used for normally distributed quanti-
nal. Ordinal data can be placed in some type of order, tative data. For quantitative data, nonparametric tests are
such as size (eg, small, medium, and large). Nominal data less powerful than parametric tests, making it more dif-
cannot be ordered (eg, gender, ethnicity, and occupation). ficult to find statistical significance. Thus, another option
Ordinal and nominal data usually are described in propor- is to mathematically transform nonnormal data into
tions with frequency tables. Quantitative data require a normally distributed data so that parametric tests may be
numerical scale. used. Various transformations include logarithms, square
The most common distribution for quantitative data is root, reciprocal, and arc–sine.
normal or gaussian distribution. This is recognized as a Statistical testing is used for many comparisons,
bell-shaped curve when data are plotted in a histogram. including comparing a group with a hypothetical value,
In this case, the observations cluster around a mean value comparing two unmatched groups, comparing two
and decline when they are further away from the mean. matched groups, comparing three or more unmatched
The standard deviation (SD) describes the distribu- groups, comparing three or more matched groups, deter-
tion of data around the mean. In a normal distribution, mining associations between two variables (correlation),
68% of the data will be contained within 1 SD, 95% of and predicting the value of a variable from another
data within 2 SD, and 99.7% of data within 3 SD. For measured variable (regression). Common statistical tests
example, in a test, the mean score was 50 points with for each situation and each data type are shown in
a SD of 10 points. If the scores were normally distrib- Table 143–145-1.
uted, 68% of students scored between 40 points and An example of comparing two groups of nominal data
60 points, 95% of students scored between 30 points would be a “yes” or “no” observation of whether uterine
and 70 points, and 99.7% scored between 20 points and mass increased in unmatched mice with a soy diet versus
80 points. Specific tests to determine normal distribution regular diet. The data can easily be placed into a 2 × 2
include the Kolmogorov–Smirnov and Shapiro–Wilk table for a Fisher exact test (Experiment 143). If the same
tests. Normally distributed quantitative data are described study was done with exact measurements of the uterus,
with the mean and standard deviation, whereas nonnormal it would comprise continuous data. If the data were nor-
quantitative data are described with median and range. mally distributed, the means would be compared with an
182 PROLOG
Type of Data
unpaired t-test. If the data were not normally distributed an overall statistical value to determine if there is a dif-
or transformed to normal data, a Mann–Whitney test ference in the means. Subsequent post-hoc testing, such
would be appropriate. as the Tukey honestly significant difference test, is used
Comparing the means of three or more groups is to determine which means are different.
commonly performed with the parametric ANOVA An outcome with time in one or more groups is com-
test (Experiment 144). This is preferable to multiple monly displayed with a Kaplan–Meier curve (Experiment
comparisons of two means (t-test), which would increase 145). This often is referred to as a survival curve because
the chance of concluding a significant difference when of its common use in conditions in which the outcome is
no significant difference exists (type 1 error). If there death. However, it can be used with any defined outcome,
was no significant difference noted with the existing data, such as the percent of women that conceive a pregnancy
a true difference still may exist and could be detected after two types of laparoscopic surgery. Most commonly,
with an increased sample size (type 2 error). A one- two or more groups with an intervention are compared
way ANOVA evaluates one factor whereas a two-way statistically on the Kaplan–Meier curve.
ANOVA looks for interactions between two factors. In
the example of breast cancer cells being treated with three Bland JM, Altman DG. Survival probabilities (the Kaplan–Meier
method). BMJ 1998;317:1572.
different hormones with determination of proliferation, a
Gordi T, Khamis H. Simple solution to a common statistical problem:
one-way ANOVA would be used. If the experiment was interpreting multiple tests. Clin Ther 2004;26:780 – 6.
performed with a sample of breast cancer cells grown for Greenhalgh T. How to read a paper. Statistics for the non-statistician. I:
1 week and a sample grown for 1 year, then a two-way Different types of data need different statistical tests [published erratum
ANOVA may be used to look at the interaction of the appears in BMJ 1997;315:675]. BMJ 1997;315:364 – 6.
age of the cells and the treatments. The ANOVA yields
Reproductive Endocrinology and Infertility 183
146–148
Mechanism of action of hormonal contraceptives
For each levonorgestrel (LNG)-containing contraceptive method (146–148), select the most likely
circulating serum level of LNG in pg/mL (A–E) in a 22-year-old woman with a body mass index
of 21 (calculated as weight in kilograms divided by height in meters squared).
(A) 100–200
(B) 500–600
(C) 2,000–3,000
(D) 5,000–6,000
(E) 10,000–15,000
146. C. 30-microgram ethinyl estradiol with 150-microgram LNG oral contraceptive (OC) pill
24 hours after ingestion
147. E. Single-dose progestin-only emergency contraceptive 2 hours after ingestion
148. A. A 5-year LNG intrauterine device (IUD)
Circulating serum levels of LNG vary among differ- In comparison with the LNG IUD, the average circulat-
ent methods of LNG-containing contraceptives. Levels ing serum levels of LNG in other contraceptives are much
are lowest in women who use the first-generation LNG higher. In normal-weight women, the average circulating
IUD compared with other LNG-containing contraceptive LNG level is 2,500 pg/mL 24 hours after ingestion of a
products. The first-generation LNG IUD was approved combination OC containing 30 micrograms of ethinyl
by the U.S. Food and Drug Administration in December estradiol and 150 micrograms of levonorgestrel. This
2000. It consists of a T-shaped polyethylene vertical level is 12.5–25 times higher than the circulating level
stem with a reservoir containing 52 mg of LNG. Two of LNG caused by the LNG IUD. In a comparison group
monofilament threads are attached to a loop at the end of obese women in the same pharmacokinetic study, the
of the vertical stem. The device contains barium sulfate, average peak LNG level 2 hours after ingestion of the
making it radiopaque. Initially, LNG is released at a con- combination OC was slightly lower than normal-weight
centration of 20 micrograms/d. The amount decreases to women (5,600 versus 7,000 pg/mL, respectively, P=.19),
11 micrograms/d after the device’s 5-year period of but their trough level 24 hours after OC ingestion was
approved use. The average circulating serum level of similar to that of the normal-weight women (2,600 pg/mL
LNG in women who use these IUDs is 100–200 pg/ versus 2,500 pg/mL, respectively, P=.56). Recently, a
mL. Despite the low circulating serum levels of LNG, 3-year LNG IUD was developed that contains 13.5 mg
the intrauterine location allows for the progestin to exert of LNG, a lower dose than the first-generation 5-year
an antiproliferative effect on the endometrium, leading LNG IUD. This amount does not appreciably change the
to decidualization and endometrial atrophy. The proges- circulating level of LNG.
tin further thickens the nearby cervical mucus, which The progestin-only emergency contraceptive was
decreases sperm penetration into the uterine cavity. These introduced in the United States in July 2009; it is a Food
two actions contribute to the contraceptive efficacy of and Drug Administration-approved method of emergency
the LNG IUD. Although it is possible to have systemic contraception available over the counter to women at risk
adverse effects from the intrauterine release of LNG, for an unintended pregnancy. The average peak serum
circulating serum levels are at a minimum compared with level of LNG 2 hours after ingestion of the progestin-
other contraceptive methods. Serum LNG levels further only emergency contraceptive is 14,600 pg/mL, approxi-
decrease with the patient’s increasing body mass index. mately 100-fold greater than the average circulating
Levonorgestrel is a lipophilic molecule that partitions serum level of LNG associated with the LNG IUD. The
into the fat tissue. Obese individuals have been shown to progestin-only emergency contraceptive is a single dose
have lower circulating serum LNG levels. Despite these of 1.5 mg of LNG. When taken orally within 72 hours of
lower levels, the incidence of vaginal bleeding does not unprotected sexual intercourse or contraceptive failure,
correlate with the circulating serum LNG level. this medication is effective in preventing pregnancy.
184 PROLOG
In a large multicenter trial for pregnancy prevention, a system [published erratum appears in Contraception 2013;88:194].
Contraception 2012;86:345–9.
single dose of 1.5 mg of LNG was associated with a 1.5%
von Hertzen H, Piaggio G, Ding J, Chen J, Song S, Bartfai G, et al.
failure rate. Low dose mifepristone and two regimens of levonorgestrel for emer-
gency contraception: a WHO multicentre randomised trial. Lancet
Kives S, Hahn PM, White E, Stanczyk FZ, Reid RL. Bioavailability of 2002;360:1803–10.
the Yuzpe and levonorgestrel regimens of emergency contraception:
vaginal vs. oral administration. Contraception 2005;71:197–201. Westhoff CL, Torgal AH, Mayeda ER, Pike MC, Stanczyk FZ.
Pharmacokinetics of a combined oral contraceptive in obese and
Mansour D. The benefits and risks of using a levonorgestrel-releasing normal-weight women. Contraception 2010;81:474–80.
intrauterine system for contraception. Contraception 2012;85:224–34.
Seeber B, Ziehr SC, Gschlieβer A, Moser C, Mattle V, Seger C, et al.
Quantitative levonorgestrel plasma level measurements in patients with
regular and prolonged use of the levonorgestrel-releasing intrauterine
Reproductive Endocrinology and Infertility 185
149–151
Differential diagnosis for hirsutism conditions
For each clinical scenario (149–151), choose the likely diagnosis for a woman with hirsutism
(A–E).
Hirsutism is the abnormal growth of pigmented hair A complete medical history and physical examina-
either in excessive amounts in normal locations or in tion are the keys to evaluation of hirsutism. Functional
locations not typically seen in most women. Significant disorders, such as PCOS and adult-onset CAH, charac-
ethnic differences exist in body hair growth, ie, hirsutism teristically have a peripubertal onset with slow, gradual
is scarce among East Asians but common in Southern worsening of symptoms. Cushing disease is rare and
Europeans. Differences in skin 5α-reductase activity par- often has other symptoms of corticosteroid excess,
tially explain these differences. including relatively rapid weight gain with truncal obe-
The Ferriman–Gallwey score is the most frequently sity, moon facies, buffalo hump, violaceous striae, and
used quantitative method to diagnose hirsutism (Appendix peripheral myopathy with weakness. Androgenization
D). The amount of pigmented hair is scored 0–4 (0 is no due to tumors would only coincidentally appear at
terminal hair and 4 is equivalent to a hairy male) in nine puberty and commonly has a more rapid course. Ovarian
body regions, including the upper lip, chin, chest, arms, androgen-producing tumors are vastly more common
upper abdomen, lower abdomen, upper back, lower back, than adrenal tumors, with Sertoli–Leydig cell tumors
and thighs. A score of greater than 8 is diagnostic of hir- being the most common (Fig. 149–151-1; see color
sutism. A score of 2 or more on the chin or lower abdo- plate). Physical examination should focus on acanthosis
men appears to be most predictive of hirsutism. nigricans (consistent with insulin resistance) and signs
Up to 80–90% of women with hirsutism will have of virilization (clitoromegaly, balding). Virilization is
an androgen-excess disorder such as PCOS, adult-onset concerning for a neoplasm because it is almost never seen
CAH, Cushing disease, or adrenal or ovarian androgen- with PCOS and is rarely observed with adult-onset CAH.
producing neoplasia. Occasionally, hirsutism will be seen Laboratory testing should include dehydroepiandros-
with primary hypothyroidism and hyperprolactinemia. In terone sulfate (DHEAS), testosterone, morning follicu-
most patients with hirsutism, the underlying diagnosis lar phase 17-hydroxyprogesterone, thyroid-stimulating
will be PCOS. Thus, hirsutism may be the presenting hormone, and prolactin levels. Some experts recom-
symptom for significant health issues. Associated health mend taking a free testosterone level instead of a total
problems may include insulin resistance with increased testosterone level because the free testosterone level
risk of type 2 diabetes mellitus as well as dyslipidemia has a greater sensitivity for a diagnosis of hyperan-
and increased vascular inflammation with increased risk drogenism. Approximately 10% of patients with PCOS
of arteriosclerosis. Less commonly, women may have will have isolated increases in DHEAS with a normal
idiopathic hirsutism characterized by normal androgen testosterone level; DHEAS is not typically elevated
levels, normal ovulatory cycles, and normal-appearing in adult-onset CAH and is not useful as a screening
ovaries on ultrasonography. Idiopathic hyperandrogen- test. A screening 17-hydroxyprogesterone level greater
ism refers to women with clinical and biochemical than 200 ng/dL requires an adrenocorticotropic hor-
androgen excess but normal ovulatory cycles and normal- mone stimulation test for further evaluation of adult-
appearing ovaries on ultrasonography. onset CAH. A screening 17-hydroxyprogesterone level
186 PROLOG
of 800 ng/dL does not require further testing. Initial bicalutamide. These drugs have received less use because
screening tests for Cushing syndrome include 24-hour of potential hepatotoxicity and the need to monitor liver
urinary collection for free cortisol, overnight 1-mg function. Examples of 5α-reductase inhibitors are fin-
dexamethasone suppression test, and late-night salivary asteride (inhibits 5α-reductase type 1) and dutasteride
cortisol test. Laboratory findings suggestive of an ovar- (inhibits 5α-reductase type 1 and 2). Finasteride has been
ian androgen-producing tumor include a total testoster- shown to be effective in the treatment of hirsutism. By
one level greater than 200 ng/dL and a reversal in the contrast, dutasteride has not been studied adequately for
androstenedione-to-testosterone ratio. In normal women the treatment of this condition.
and women with functional androgen excess (not caused None of the mentioned drugs, including OCs, are
by a tumor), total androstenedione will be greater than approved by the U.S. Food and Drug Administration
total testosterone. This relation often is reversed in cases for the treatment of hirsutism, although some OCs are
of ovarian tumors. approved for the treatment of acne. Antiandrogens (spi-
Treatment for hirsutism depends on the underlying ronolactone, flutamide, finasteride) sometimes are used
pathology. With idiopathic hirsutism and functional with combination OCs to increase efficacy if OC use
disorders such as PCOS and adult-onset CAH, the goals alone is not effective. All androgen receptor blockers
are to decrease free androgen levels and block andro- and 5α-reductase inhibitors are teratogenic and must be
gen action. Decreasing free androgen levels consists used with adequate contraceptive methods to prevent
of decreasing androgen production by the ovary and pregnancy.
decreasing the free fraction by increasing sex hormone- Eflornithine hydrochloride 13.9% cream is the only
binding globulin. Combination hormonal contraceptives FDA-approved product for the treatment of unwanted
remain the most cost-effective treatment for this purpose, facial hair. The chemical inhibits the enzyme ornithine
although to date, no studies have compared the efficacy decarboxylase, which is important in cell growth. Thus,
of combination oral contraceptives (OCs). An OC with a eflornithine does not remove hair but decreases hair
third-generation progestin (desogestrel and norgestimate) growth. Other topical therapies for hirsutism include
or drospirenone will result in the greatest increase in sex shaving, plucking, waxing, bleaching, depilatories, elec-
hormone-binding globulin. Decreased androgen action trolysis, laser, and intense pulsed light.
involves blockage of the androgen receptor with or
Escobar-Morreale HF, Carmina E, Dewailly D, Gambineri A, Kelestimur
without decreasing 5α-reductase activity converting tes- F, Moghetti P, et al. Epidemiology, diagnosis and management of hir-
tosterone to dihydrotestosterone. Spironolactone blocks sutism: a consensus statement by the Androgen Excess and Polycystic
the androgen receptor and decreases 5α-reductase activ- Ovary Syndrome Society [published erratum appears in Hum Reprod
Update 2013;19:207]. Hum Reprod Update 2012;18:146–70.
ity. Spironolactone treatment can result in more frequent
Paparodis R, Dunaif A. The Hirsute woman: challenges in evaluation
uterine bleeding. Uncommon adverse effects include and management. Endocr Pract 2011;17:807–18.
hyperkalemia and hypotension. Use of spironolactone Yildiz BO, Bolour S, Woods K, Moore A, Azziz R. Visually scoring
is contraindicated in women with renal insufficiency. hirsutism. Hum Reprod Update 2010;16:51–64.
Androgen receptor antagonists include flutamide and
Reproductive Endocrinology and Infertility 187
152–154
Testing for thyroid disease
Match the patient (152–154) with the thyroid function test results (A–D) that would help diagnose
her condition.
(A) Elevated thyroid-stimulating hormone (TSH) level and normal free thyroxine (T4)
level
(B) Low TSH level and elevated T4 level with thyroid antibodies
(C) High TSH level and low T4 level
(D) Low TSH, normal T4, and normal triiodothyronine (T3) level
152. C. A 33-year-old woman, gravida 1, para 1, with 40-day menstrual cycles has dry skin and
constantly feels tired. She is trying to become pregnant but is having difficulty interpreting
her ovulation predictor kit.
153. A. A 32-year-old woman, gravida 2, para 0, has had recurrent pregnancy loss. She is in good
health and has regular menses. Her recurrent pregnancy loss evaluation result is negative
except for subclinical hypothyroidism.
154. B. A 28-year-old nulligravid woman has a 6-month history of amenorrhea. She is feeling
frustrated because she wants to become pregnant and is not cycling. Her symptoms are heat
intolerance and palpitations. On physical examination, she has lid lag and patellar tendon
hyperreflexia. Her thyroid gland is diffusely enlarged.
The thyroid gland secretes T4 at 20 times the rate of T3. of hypothyroidism is chronic autoimmune thyroiditis,
Free T4 is converted to T3 mainly in the liver by removal which is 5–10 times more common in women than in
of an iodine atom. Triiodothyronine is the hormone men. Antithyroid antibodies infiltrate the thyroid because
responsible for most of the thyroid’s action on peripheral of abnormal regulation of the immune response. This
organs. Free T4 is controlled by the pituitary hormone impairs free T4 and free T3 production. In response to
TSH. Changes in T4 will modulate changes in TSH in
order to keep the appropriate balance of circulating thy-
TABLE 152–154-1. Clinical Characteristics of
roid hormone. The thyroid axis, in addition to the thyroid Hypothyroidism and Hyperthyroidism
gland and pituitary, includes TSH, which is produced by
the hypothalamus. These three glands work together to Hypothyroidism Hyperthyroidism
maintain normal thyroid function and peripheral action.
Firm irregular goiter Enlarged homogenous
Free T4 and T3 circulate bound and unbound to binding thyroid gland
proteins. It is the unbound hormones (free T4 and free T3)
Slow reflexes, decreased Hyperreflexia
that exert effects on target tissues. Thyroid dysfunction is energy
diagnosed by measuring TSH, free T4, and free T3.
Bradycardia Tachycardia, palpitations
The thyroid antibody tests for antithyroid peroxidase
Depression Nervousness, irritability,
and antithyroglobulin can help diagnose the etiology of anxiety, difficulty sleeping
thyroid dysfunction. In a patient with hypothyroidism,
Constipation Frequent bowel movements
antibodies help make the diagnosis of autoimmune
thyroiditis, also known as Hashimoto thyroiditis. When Muscle cramps, voice Hand tremors, muscle
changes weakness (upper arms/
hyperthyroidism is suspected, TSH receptor autoantibod- thighs)
ies point to autoimmune thyroid disease.
Weight gain Weight loss despite a good
The signs and symptoms of hypothyroidism (low thy- appetite
roid hormone) include dry skin, cold sensitivity, fatigue, Dry skin Sweating, brittle hair, warm
muscle cramps, constipation, and menstrual irregularities. moist skin
Patient 152 has elevated TSH and low T4 levels, which Irregular menstrual periods Irregular menstrual periods
confirm the diagnosis (Table 152–154-1). In the United
Cold intolerance Heat intolerance
States, where iodine is sufficient, the most common cause
188 PROLOG
low circulating thyroid hormone levels, the pituitary will thyroid gland. Symptoms include heat intolerance, diar-
try to compensate by increasing TSH levels to restore rhea, weight loss, sweating, palpitations, and nervousness.
thyroid hormone levels. When the pituitary is no longer As seen in Patient 154, physical examination includes lid
able to compensate by increasing TSH, thyroid hormone lag, goiter, hyperreflexia, moist skin, tachycardia, and
levels will decrease and the diagnosis of hypothyroidism tremor. Laboratory evaluation for Patient 154 is classic
is confirmed. Once hypothyroidism is treated, irregular for hyperthyroidism: low TSH level and elevated free
menstrual cycles often will become regular, allowing for T4 with thyroid antibodies. Treating her hyperthyroidism
more accurate timed intercourse. will most likely normalize her menstrual cycles. Patients
Subclinical hypothyroidism is early hypothyroidism with hyperthyroidism can have normal menstrual cycles,
before signs and symptoms present. It is a compensated oligo-ovulation, or amenorrhea.
state detected by an elevated TSH level and normal free
Bahn (Chair) RS, Burch HB, Cooper DS, Garber JR, Greenlee MC,
T4 level as observed in Patient 153. Subclinical hypo- Klein I, et al. Hyperthyroidism and other causes of thyrotoxico-
thyroidism progresses to overt hypothyroidism at a rate sis: management guidelines of the American Thyroid Association
of 2.6–4.3% per year depending on whether antithyroid and American Association of Clinical Endocrinologists. American
Thyroid Association and the American Association of Clinical
antibodies are present. Many experts recommend mea- Endocrinologists [published errata appear in Thyroid 2011;21:1169.
suring thyroid antibodies in the patient with subclinical Thyroid 2012;22:1195]. Thyroid 2011;21:593–646.
hypothyroidism to help predict and follow progression to Fritz MA, Speroff L. Reproduction and the thyroid. In: Fritz MA,
hypothyroidism. Treatment is recommended to avoid goi- Speroff L, editors. Clinical gynecologic and infertility. 8th ed.
Philadelphia (PA): Lippincott Williams & Wilkins; 2011. p. 885–905.
ter development and decrease the risk of pregnancy loss.
Garber JR, Cobin RH, Gharib H, Hennessey JV, Klein I, Mechanick JI,
Graves disease is the most common cause of hyperthy- et al. Clinical practice guidelines for hypothyroidism in adults: cospon-
roidism. It is 5–10 times more common in women than in sored by the American Association of Clinical Endocrinologists and
men. It is an autoimmune disorder caused by thyroid anti- the American Thyroid Association. American Association Of Clinical
Endocrinologists And American Thyroid Association Taskforce
body stimulation of the TSH receptor, increasing release On Hypothyroidism In Adults [published errata appear in Thyroid
of thyroid hormone with concomitant growth of the 2013;23:251. Thyroid 2013;23:129]. Thyroid 2012;22:1200–35.
Reproductive Endocrinology and Infertility 189
155–158
Obesity and pregnancy
For each set of medical conditions in a morbidly obese patient of reproductive age (155–158),
select the therapy (A–D) that is most likely to accomplish long-term improvement.
As of 2004, approximately 66% of all U.S. adults were 40 or more (calculated as weight in kilograms divided
either overweight or obese, making overweight and by height in meters squared) or those with a body mass
obesity problems of epidemic proportions. The National index of 35 or more with comorbidities are the optimal
Health and Nutrition Examination Survey published candidates to benefit from the procedure.
in 2009 found that more than one half of all pregnant Bariatric surgery produces rapid weight loss and can
women in the United States are overweight or obese. lead to an improvement in glucose tolerance and hir-
Obesity is highest among non-Hispanic black women sutism as well as normalization of menstrual cycles in
(50%) and Mexican American women (45%) compared obese women who have abnormalities as a consequence
with non-Hispanic white women (33%). of polycystic ovary syndrome. Therefore, bariatric sur-
Numerous consequences result from obesity. It is gery would be the best choice for Patient 155. The hor-
widely recognized that obesity has a central role in the monal profile associated with morbid obesity appears to
pathogenesis of cardiovascular disease, hypertension, reverse after surgical weight loss. Higher fertility rates
dyslipidemia, obstructive sleep apnea, and cancer. In have been consistently reported, but bariatric surgery
females, obesity can cause anovulation, menstrual irregu- should not yet be considered as a primary treatment for
larities, infertility, sexual dysfunction, and fetal loss. In infertility. Bariatric surgery is now being used to treat
pregnancy, overweight and obese women are at increased adolescents with obesity.
risk of preeclampsia, gestational diabetes, hypertension, As obesity becomes more prevalent in reproductive-
cesarean delivery, and postpartum weight retention. aged women, practitioners will encounter many patients
Delivery is complicated by increased rates of infectious who have had or are considering bariatric surgery.
morbidity, anesthetic complications, increased operative Preconception counseling and assessment are encour-
times, greater blood loss, and higher rates of thromboem- aged so the patient can be informed of specific maternal
bolism. Children born to obese women are at increased and fetal risks in pregnancy. Initial steps should include
risk of prematurity, stillbirth, congenital anomalies, mac- encouragement to follow a weight-reduction program.
rosomia, birth injury, and childhood obesity. Bariatric surgery is now recognized as the most effec-
Nonsurgical approaches to weight loss have tradition- tive weight-loss intervention for morbid obesity. It has
ally consisted of behavioral modification along with improved fertility, sexuality, quality of life, and preg-
changes in diet and exercise. These strategies have been nancy outcomes. However, bariatric surgery does carry
time-consuming and success rates have been low. Bari- specific risks. The procedure can lead to malabsorption
atric surgery has emerged as an option for rapid weight and nutritional deficiencies. A broad evaluation for
loss in reproductive-aged women. The annual rate of micronutrient deficiencies at the beginning of pregnancy
bariatric surgery in the United States increased 10-fold is recommended. After bariatric surgery with a malab-
from 1999 to 2005. Patients with a body mass index of sorptive component, patients and practitioners should
190 PROLOG
be aware that oral contraceptives may have an increased symptoms of hyperprolactinemia, which, once confirmed,
failure rate. If contraception is desired, a nonhormonal would best be treated by a dopamine agonist, such as cab-
method should be used. ergoline or bromocriptine.
Patient 156 has infertility with infrequent ovulation but
normal FSH, thyroid-stimulating hormone, and prolactin Bariatric surgery and pregnancy. ACOG Practice Bulletin No. 105.
American College of Obstetricians and Gynecologists. Obstet Gynecol
levels. The best therapy for this patient would be clomi- 2009;113:1405–13.
phene citrate, a selective estrogen receptor modulator Merhi ZO. Impact of bariatric surgery on female reproduction. Fertil
that increases production of FSH by inhibiting negative Steril 2009;92:1501–8.
feedback on the hypothalamus. Obesity in pregnancy. Committee Opinion No. 549. American College
Patient 157 has anemia, abnormal uterine bleeding, of Obstetricians and Gynecologists. Obstet Gynecol 2013;121:213–7.
and several 1-cm submucosal leiomyomas. Abnormal Pories WJ. Bariatric surgery: risks and rewards. J Clin Endocrinol
Metab 2008;93(suppl):S89–96.
uterine bleeding that results in anemia would be an indi-
Teitelman M, Grotegut CA, Williams NN, Lewis JD. The impact of
cation for surgery. Thus, Patient 157 would benefit from bariatric surgery on menstrual patterns. Obes Surg 2006;16:1457–63.
hysteroscopic myomectomy.
Patient 158 has experienced recurrent pregnancy loss,
galactorrhea, and a shortened luteal phase. These are
159–161
Laparoscopic surgery complications
For each patient’s set of symptoms (159–161), select the most likely complication (A–E) after
laparoscopic surgery.
Laparoscopic procedures are being performed with entry and Veress needle entry are compared. Open lapa-
greater frequency by gynecologists. Potentially prevent- roscopy was first described in 1971 and continues to be
able surgical errors have received a great deal of attention a safe procedure with regard to the avoidance of major
after the publication in 2010 by the Joint Commission of vascular injury or death. Lifting the umbilicus with towel
universal protocols for prevention of wrong-site, wrong- clips placed close to and inverting the umbilicus allows
patient, and wrong-person surgery. A systems approach significant elevation from the great vessels and also has
has been adopted that includes checklists, multiple safety been shown to be effective for safe trocar placement. The
stops, improved communication, and involvement of the reported rates of intestinal injury with open laparoscopy
patient in order to avoid errors. are similar to those with the use of blind trocar entry.
Complications are an inevitable part of laparoscopic Approximately 20–25% of laparoscopic complications
surgery. The incidence of complications has been esti- are not recognized intraoperatively. Early detection and
mated to be 3–6 per 1,000 cases, with up to one half of rapid response are the keys to reducing morbidity and pre-
all major complications shown to occur at the time of venting mortality from laparoscopic injury. Laparoscopic
surgical entry. The surgical literature does not provide injuries to different organ systems may have certain char-
an obvious consensus when the safety of direct trocar acteristic features that will aid in diagnosis.
Reproductive Endocrinology and Infertility 191
The close proximity of the abdominal wall to the ret- arterial or venous system. This life-threatening complica-
roperitoneal vessels (aorta, vena cava, and internal and tion requires immediate release of pneumoperitoneum
external iliac) is a risk factor, especially in thin patients. and resuscitation. Presenting symptoms include increased
The surgeon should have proper training and experience pulmonary artery pressure, tachycardia, hypotension,
with patient position, angle of insertion, abdominal wall mill-wheel murmur, decreased end tidal CO2, and rapidly
elevation, and degree of pneumoperitoneum. Blood on falling oxygen saturation (Patient 160).
aspiration of the Veress needle, intraperitoneal blood, Bladder injuries are much more common than ureteral
retroperitoneal hematoma, or unexplained sudden change injuries and are easier to recognize and more likely to be
in vital signs are all indicative of major vessel injury. repaired intraoperatively. Ureteral injury can be difficult
Bowel injuries account for almost one half of all major to identify intraoperatively; more than 70% of ureteral
complications of laparoscopy. Early recognition and injuries are diagnosed postoperatively. This injury is more
repair aids in the prevention of sequela, but less than 50% likely to occur with laparoscopic hysterectomy or removal
of bowel injuries are recognized at the time of surgery. of an entrapped ovary. Presenting signs and symptoms
Postoperative recognition of bowel injury is associated include hematuria, flank pain, leukocytosis, oliguria, and
with higher morbidity and significant mortality. Delayed an elevated C-reactive protein level (Patient 161).
diagnosis can lead to peritonitis, septicemia, and multi-
organ failure. Typical symptoms include pain at the Baggish M. Major laparoscopic complications: a review in two parts. J
trocar site nearest to the site of injury, fever, diarrhea, Gynecol Surg 2012;28:315–32.
distention, and leukocytosis (Patient 159). Presenting Bhoyrul S, Vierra MA, Nezhat CR, Krummel TM, Way LW. Trocar
injuries in laparoscopic surgery. J Am Coll Surg 2001;192:677–83.
signs may be subtle and varied. As a result, the surgeon
Goldberg JM, Chen CCG, Falcone T. Complication of laparoscopic
should err on the side of safety and admit any patient surgery. In: Falcone T, Goldberg JM, editors. Basic, advanced and
with a suspected bowel injury for observation and further robotic laparoscopic surgery. Philadelphia (PA): Saunders Elsevier,
assessment. 2010. p. 221–40.
Gas embolism is a rare but devastating complication Magrina JF. Complications of laparoscopic surgery. Clin Obstet
Gynecol 2002;45:469–80.
arising from the use of CO2 insufflation. Decreased
venous return and elevated pulmonary vascular pressures Makai G, Isaacson K. Complications of gynecologic laparoscopy. Clin
Obstet Gynecol 2009;52:401–11.
are known conditions that occur during laparoscopy and
Roy GM, Bazzurini L, Solima E, Luciano AA. Safe technique for
elevate CO2 levels during surgery. Gas embolism can laparoscopic entry into the abdominal cavity. J Am Assoc Gynecol
occur when large volumes of CO2 gain entry into the Laparosc 2001;8:519–28.
Reproductive Endocrinology and Infertility 193
Appendix A
Normal Values for Laboratory Tests*
*Values listed are specific for adults or women, if relevant, unless otherwise differentiated. (continued)
194 PROLOG
*Values listed are specific for adults or women, if relevant, unless otherwise differentiated. (continued)
Reproductive Endocrinology and Infertility 195
*Values listed are specific for adults or women, if relevant, unless otherwise differentiated. (continued)
196 PROLOG
*Values listed are specific for adults or women, if relevant, unless otherwise differentiated.
Reproductive Endocrinology and Infertility 197
Appendix B
198 PROLOG
Appendix C
Abnormal Uterine Bleeding (AUB)
• Heavy menstrual bleeding (AUB/HMB)
• Intermenstrual bleeding (AUB/IMB)
Appendix C. Basic PALM–COEIN classification system for the causes of abnormal uterine bleeding in nonpregnant
reproductive-aged women. This system, approved by the International Federation of Gynecology and Obstetrics, uses
the term “abnormal uterine bleeding” paired with terms that describe associated bleeding patterns (“heavy menstrual
bleeding” or “intermenstrual bleeding”), a qualifying letter (or letters) to indicate its etiology (or etiologies), or both.
(Data from Munro MG, Critchley HO, Broder MS, Fraser IS. FIGO classification system [PALM-COEIN] for causes of
abnormal uterine bleeding in nongravid women of reproductive age. FIGO Working Group on Menstrual Disorders. Int
J Gynaecol Obstet 2011;113:3–13.)
Reproductive Endocrinology and Infertility 199
Appendix D
1 2 3 4
1 2 3 4
1 2 3 4
1 2 3 4
1 2 3 4
1 2 3 4
1 2 3 4
1 2 3 4 1 2 3 4
Appendix D. The Ferriman–Gallwey score is used to quantify the degree of hirsutism by the use of representative
images as shown. The lowest score is 0, indicating no excessive hair growth, and the highest score is 32. (Hatch R,
Rosenfield RL, Kim MH, Tredway D. Hirsutism: implications, etiology, and management. Am J Obstet Gynecol 1981;
140:815–30. Copyright Elsevier 1981.)
Index
A American Diabetes Association, on metabolic APS. See Antiphospholipid syndrome
Ablation, uterine nerve, 42 syndrome, 104 Aromatase inhibitors
Abnormal uterine bleeding (AUB), 18, 109. See American Gastroenterological Association, on for androgen disorders, 61
also Heavy menstrual bleeding osteoporosis in celiac disease, 101 for leiomyoma in infertility, 36
in adolescents, 92 American Society for Reproductive Medicine, for ovulation induction, 90
causes of, 18 32 ART. See Assisted reproductive technology
leiomyomas and, 56 on bioidentical hormones, 113 Artificial insemination, for same-sex couples, 39
PALM–COEIN classification system for, on endometriosis, 37 Assisted reproductive technology (ART), ovarian
18, 92 “Guidelines for Gamete and Embryo hyperstimulation syndrome and, 8
polycystic ovary syndrome-related, 109 Donation” of, 39 AUB. See Abnormal uterine bleeding
serum reproductive hormone levels in, 109 on in vitro fertilization, 1 Aura, migraine with, 7
treatment for, 18, 56 on limiting number of embryos transferred at Autoimmune thyroiditis, 104, 114, 152–154
Abortion, 17, 26 time of in vitro fertilization, 116 Autonomy, patient, 1
Absolute risk on oocyte donor safety, 126 Autosomal recessive inherited disorder,
described, 57 on polycystic ovary syndrome, 59 congenital adrenal hyperplasia as, 60
relative risk versus, 57 on varicocele repair in treatment of male Autotransplantation, tissue cryopreservation
Acanthosis nigricans, in women with insulin factor infertility, 77 with, 75
resistance, 91 American Society for Reproductive Medicine AZF. See Azoospermia factor
Acetaminophen, for dysmenorrhea, 42 Ethics Committee guidelines, on donor sperm Azoospermia, 5, 119
Acne, 84 for same-sex couples, 39 nonobstructive, 119
ACTH. See Adrenocorticotropic hormone “Guidelines for Gamete and Embryo obstructive, 119
Acupuncture, for infertility-related issues, 85 Donation,” 39 Azoospermia factor (AZF), 119
Adhesion(s), labial, 30 American Society of Andrology, on male factor
Adolescent(s) infertility management, 77 B
abnormal uterine bleeding in, 92 American Thyroid Association, on hypothyroid- β-hCG, 40, 64
acne in, 84 ism screening, 114 Bariatric surgery, 9, 125
low bone mineral density in, 101 American Urological Association, on varicocele annual rate of, 155–158
Adoption, for patients with müllerian repair in treatment of male factor described, 155–158
dysgenesis, 27 infertility, 77 indications for, 78
Adrenal cortex, adrenal steroid biosynthesis Analysis of variance with post-hoc Tukey Basal body temperature monitoring, in
in, 62 honestly significant difference testing, conception, 121
Adrenal hyperplasia. See Congenital adrenal 143–145 Beck Depression Inventory, on infertility, 85
hyperplasia Androgen(s) Beckwith–Wiedemann syndrome, intracyto-
Adrenal steroid biosynthesis, in adrenal cortex, main action of, 84 plasmic sperm injection and, 11
62 maternal virilization due to, 68 Benzoyl peroxide, for acne, 84
Adrenarche synthetic, 68 Biestrogen, for menopausal symptoms, 113
normal, 62 Androgen disorders, 61, 103 Biguanide, for ovulation induction in patient
premature, 62 Androgen excess, prevalence of, 81 with polycystic ovary syndrome, 59
Adrenocorticotropic hormone (ACTH), in Androgen insensitivity syndrome (AIS), 48 Bilateral salpingectomy, 12
Cushing syndrome diagnosis, 110 categorization of, 128–130 Bilateral tubal ligation, fertility after, 123
Adrenocorticotropic hormone deficiency, complete, 128–130 Binge-eating disorder, 52
Sheehan syndrome and, 70 described, 128–130 Bioidentical hormones, 113
Adrenocorticotropic hormone stimulation test, features of, 103 Bisphosphonates
in androgen-secreting ovarian tumor gonad evaluation in, 128–130 in bone loss treatment, 94
evaluation, 98 partial, 128–130 for female athlete triad, 108
Air embolism, hysteroscopic resection and, 24 presentation of, 128–130 Bladder injuries, laparoscopic surgery and,
AIS. See Androgen insensitivity syndrome Androgen production, sexual hair growth 159–161
Alcohol use related to, 62 Bleeding. See also Hemorrhage
breast cancer associated with, 127 Androgen-secreting ovarian tumors, 98 abnormal uterine, 18, 56, 92, 109
pregnancy outcomes related to, 134–138 Anesthesia, for hysteroscopy, 24 after uterine fibroid embolization, 63
Alendronate sodium, for female athlete triad, 108 Aneuploidy screening, 71 estrogen breakthrough, 73
Alpha error, described, 57 Anorexia nervosa, nutritional management of, 54 heavy menstrual, 18, 80, 106
Ambiguity, sexual, 115 Anti-β2-glycoprotein I antibodies, in hysteroscopic resection and, 24
Ambiguous genitalia, 115 antiphospholipid syndrome, 3 postmenopausal uterine, 34
Amenorrhea, 97 Antiandrogens, for hirsutism, 112 BMD. See Bone mineral density
functional hypothalamic, 108, 117 Antibiotics, for acne, 84 Bone, physiology of, 94
hypothalamic, 117 Antibody(ies), anticardiolipin, 3 Bone loss, depot medroxyprogesterone acetate
primary, 27, 86, 103 Anticardiolipin antibodies, 3 and, 58
secondary, 10, 110, 117 Anticoagulant(s), lupus, 3 Bone mass
American College of Cardiology, on metabolic Antidepressant(s) low, 94
syndrome, 104 for anorexia nervosa, 54 screening for, 94
American College of Obstetricians and for infertility-related issues, 85 Bone mineral density (BMD)
Gynecologists. See The American Antimüllerian hormone level, in testing for in children and adolescents with newly
College of Obstetricians and ovarian reserve, 95 diagnosed celiac disease, 101
Gynecologists Antiphospholipid syndrome (APS), 3, 69 low, 58
Bowel injury, laparoscopic surgery and, Clomiphene citrate (continued) Cortisol deficiency, 115
159–161 for obese patients, 125, 155–158 Counseling, infertility-related, 85
BRCA gene mutations, 22, 43 in ovarian reserve testing, 95 Cryopreservation techniques, 32
BRCA1 gene mutations, 13, 131–133 for ovulation induction, 59, 90 embryo cryopreservation, 75
BRCA2 gene mutations, 131–133 for prolactin-induced sexual dysfunction, 38 oocyte cryopreservation, 75
Breakthrough bleeding, estrogen, 73 for unexplained infertility, 16 CT. See Computed tomography
Breast cancer, 127 Clonidine, for menopausal symptoms, 50 Cushing disease versus Cushing syndrome, 110
alcohol use and, 127 Coagulopathy, abnormal uterine bleeding in Cushing syndrome, 104, 110
risk factors for, 127 adolescents related to, 92 Cyst(s), dermoid, 100
screening for, 29 Cognitive–behavioral therapy, in bulimia Cystic fibrosis, 6, 88
in smokers, 127 nervosa management, 52
Breast Cancer Prevention Trial, of National Colles fractures, osteoporosis and, 94 D
Surgical Adjuvant Breast and Bowel Combination contraceptives D&E. See Dilation and evacuation
Project, 2 for hirsutism, 149–151 Dehydroepiandrosterone (DHEA), premature
Breast Cancer Risk Assessment Tool, in breast nonoral, 19 adrenarche and, 62
cancer risk prediction, 127 for patient with systemic lupus erythematosus, Dehydroepiandrosterone sulfate (DHEAS)
Breast development 66 elevated, 93
premature, 20 Combination estrogen–progestin therapy, for in nonclassic adrenal hyperplasia evaluation,
Tanner staging of, 20 BRCA1-positive patient with ovarian 81
Bromocriptine, in hyperprolactinemia cancer, 13 in pregnancy, 139–142
management, 97 Combination hormonal contraceptives premature adrenarche and, 62
Bulimia nervosa, 52 for hirsutism, 149–151 Depot medroxyprogesterone acetate (DMPA)
in hyperprolactinemia management, 97 bone loss and, 58
C Combination oral contraceptives (OCs) levonorgestrel, 19
Cabergoline, in hyperprolactinemia for hirsutism, 112 for patient with systemic lupus erythema-
management, 97 for patient with venous thromboembolism, tosus, 66
CAH. See Congenital adrenal hyperplasia 19 unscheduled uterine bleeding related to, 49
CAIS. See Complete androgen insensitivity “Common migraine,” 7 Depression, infertility-related, 85
syndrome Complete androgen insensitivity syndrome Dermoid cysts, 100
Calcitonin, in bone loss treatment, 94 (CAIS), 128–130 Dexamethasone, in congenital adrenal
Calcium supplementation, in bone loss differential diagnosis of, 128–130 hyperplasia prevention, 60
treatment, 94 treatment of, 128–130 DHEA. See Dehydroepiandrosterone
Calendar timing, in conception, 121 Computed tomography (CT), in androgen- DHEAS. See Dehydroepiandrosterone sulfate
Cancer(s). See also specific types secreting ovarian tumor evaluation, 98 Diabetes mellitus
breast, 29, 127 Conception, monitoring during period of, 121 contraception for, 44, 67
cervical, 29 Condom(s), failure of, 65 diagnosis of, 91
endometrial, 22, 34 Congenital adrenal hyperplasia (CAH), 45, 60 impaired glucose tolerance and, 91
fertility preservation in patient undergoing causes of, 115 type 2
treatment for, 32 described, 60 impaired glucose tolerance and, 51, 91
ovarian, 13, 22 late-onset, 81, 104 metformin hydrochloride for, 78
screening for, 29 management of, 115 Dienogest, in pregnancy termination, 26
Cardiovascular disease, Framingham Point newborn screening for, 115 Dilation and evacuation (D&E), pregnancy ter-
Score for, 104 nonclassic, 81 mination by, 26
CDC. See Centers for Disease Control and prevalence of, 115 Disordered proliferative endometrium, poly-
Prevention prevention of, 60 cystic ovary syndrome and, 73
Celiac disease, 101 Congenital rickets, 134–138 Distal radial fractures, osteoporosis and, 94
Centers for Disease Control and Prevention Contraception. See also Contraceptives; Oral Distention media, for diagnostic hysteroscopy,
(CDC) contraceptives; specific types 24
on in vitro fertilization with own oocytes, 126 condoms, 65 Distribution-free testing, 143–145
on pelvic inflammatory disease, 124 for diabetes mellitus, 44, 67 DMPA. See Depot medroxyprogesterone acetate
Central precocious puberty, causes of, 82 emergency, 65 DNA fragmentation testing, 71
Cerebral palsy, in triplet pregnancies, 116 morning-after, 65 Donor egg in vitro fertilization (IVF), 29
Cervical cancer, screening for, 29 for older adult smoker, 55 Donor oocytes
Cervical laceration, hysteroscopic resection for patient with BRCA gene mutation, 22 müllerian dysgenesis and, 27
and, 24 for patient with systemic lupus erythematosus, uses for, 126
Cervical mucus, peak production of, 121 66 Donor sperm
Children for patient with venous thromboembolism, insemination, 119
labial adhesions in, 30 19 for same-sex couples, 39
low bone mineral density in, 101 Contraceptive(s). See also Contraception; Oral use, 119
Chlamydia trachomatis antibody levels, in pelvic contraceptives Dopamine agonists
inflammatory disease detection, 124 combination, 13, 19, 66, 112, 149–151 for macroadenoma, 46
Chronic malodorous vaginal discharge, after hormonal, 97, 146–148, 149–151 for obese patients, 155–158
uterine fibroid embolization, 63 in hyperprolactinemia management, 97 for prolactin-induced sexual dysfunction, 38
Chronic pelvic pain, 96 levonorgestrel-containing, 146–148 Dual energy X-ray absorptiometry (DXA) scan
Cigarette smoking, bone mineral density loss long-acting reversible, 17 of hip, 94
related to, 58 obesity effects of, 17 of spine, 94
Cleavage-stage embryo biopsy, 71 progestin-only, 19, 57, 84, 146–148 Dwarfism, Laron, 84
Clinical hypothyroidism, 114 Copper intrauterine device, for patient with sys- DXA scan. See Dual energy X-ray
Clomiphene citrate temic lupus erythematosus, 66 absorptiometry scan
for anorexia nervosa, 54 Corticosteroids, for acne, 84 Dysmenorrhea, 42
in male factor infertility management, 77 Cortisol, Sheehan syndrome and, 70 Dysmorphia, facial, 134–138
Hormone therapy (HT) (continued) Hysteroscopic tubal occlusion, 123 Intrauterine devices (IUDs) (continued)
for menopausal symptoms, 113 Hysteroscopically placed permanent contracep- mechanism of action of, 146–148
postmenopausal, 72 tive microinsert, female sterilization for obese women, 17
postmenopausal uterine bleeding and, 34 with, 35 Intrauterine insemination, 16, 111
risks and benefits of, 31 Hysteroscopy Intrauterine microinsert
types of, 72 anesthesia for, 24 follow-up, 87
Women’s Health Initiative study on, 31 complications of, 24 pregnancy and, 35
HSCs. See Hematopoietic stem cells diagnostic, 24 Irregular menstrual cycles, 120, 127
HSG. See Hysterosalpingography for endometrial polyps, 4 Isosexual precocious puberty, evaluation of, 82
HT. See Hormone therapy operative, 24 Isotretinoin, for acne, 84
Human chorionic gonadotropin (hCG) IUDs. See Intrauterine devices
β-, 40 I IVF. See In vitro fertilization
in early pregnancy, 64 ICSI. See Intracytoplasmic sperm injection
Human placental lactogen, in pregnancy, Idiopathic premature adrenarche, 62 K
139–142 IGF-1. See Insulin-like growth factor 1 Kallman syndrome, 90, 103
Hydrosalpinges, in vitro fertilization–embryo IGT. See Impaired glucose tolerance Kaplan–Meier curve, 143–145
transfer and, 12 Impaired glucose tolerance (IGT), diabetes Karotypic abnormalities, oligospermia related
11β-Hydroxylase deficiency, 60 mellitus and, 51, 91 to, 11
17α-Hydroxylase deficiency, 60 Imprinting disorders, 11 Karyotype, in premature ovarian failure
21-Hydroxylase deficiency, congenital adrenal In vitro fertilization (IVF), 1 evaluation, 83
hyperplasia related to, 60, 115 donor egg, 29 Karyotype abnormalities, premature ovarian
17α-Hydroxyprogesterone, morning follicular goal of, 11 failure related to, 83
serum, 81 intracytoplasmic sperm injection and, 11 Karyotype analysis, in recurrent pregnancy loss
3β-Hydroxysteroid dehydrogenase type 2, 60 limiting number of embryos transferred at evaluation, 69
Hyperandrogenism, in pregnancy, 100 time of, 116 Klinefelter syndrome, 88
Hypergonadism, hypergonadotropic, 117 ovarian hyperstimulation syndrome and, 11 Kolmogorov–Smirnov test, 143–145
Hypergonadotropic hypogonadism, 117 with own oocytes, 126
Hyperinsulinemic euglycemic clamp, in insulin patient autonomy in, 1 L
resistance evaluation in patients with for unexplained infertility, 16 Labial adhesions, in children, 30
polycystic ovary syndrome, 91 In vitro fertilization and gestational carrier, for Labial agglutination, in children, 30
Hyperlipidemia, 104 müllerian dysgenesis, 27 Laceration(s), cervical, 24
Hyperplasia. See Congenital adrenal In vitro fertilization–embryo transfer, hydrosal- Laparoscopic applications of band or clip, in
hyperplasia pinges and, 12 surgical reversal, 123
Hyperprolactinemia, 97 In vitro maturation, 32 Laparoscopic electrocautery, 123
erectile dysfunction and, 38 Induced pluripotent, in reproduction, 122 Laparoscopic oophorectomy, for ovarian
galactorrhea due to, 10 Infection(s) androgen-secreting tumor, 98
menstrual irregularity and galactorrhea intrauterine device-related, 102 Laparoscopic surgery, complications of,
related to, 81 after uterine fibroid embolization, 63 159–161
non-tumor-related, 97 Infertility Laparoscopy, in endometriosis, 14
Hyperreactio luteinalis counseling related to, 85 Laron dwarfism, 84
defined, 68 depression related to, 85 Laser hair removal, 112
maternal virilization in, 68, 100 endometriosis and, 14 Late-onset congenital adrenal hyperplasia, 81,
Hyperstimulation, ovarian, 100 female, 124 104
Hyperthyroidism, 74 leiomyoma in, 36 Legal issues, in vitro fertilization-related, 1
causes of, 74 male factor, 5, 38, 77, 119 Leiomyoma(s)
clinical characteristics of, 152–154 prevalence of, 85 abnormal uterine bleeding in adolescents
Graves disease and, 74, 152–154 psychologic effects of, 85 related to, 92
during pregnancy, 53 tubal disease and, 124 heavy menstrual bleeding and, 56
prevalence of, 74 unexplained, 16 in infertility, 36
transient, 53 Insemination Leptin
Hypogonadism donor sperm, 119 negative energy balance effects on, 15
primary hypogonadotropic, 88 intrauterine, 16, 111 in reproductive function, 15
secondary hypogonadotropic, 88 for same-sex couples, 39 Letrozole
Hypogonadotropic hypogonadism Insulin-like growth factor 1 (IGF-1), 10, 84 for leiomyoma in infertility, 36
primary, 88 Insulin resistance for ovulation induction in patient with
secondary, 88 described, 91 polycystic ovary syndrome, 59
Hypothalamic amenorrhea, 108, 117 obesity and, 78 Leuprolide, indications for, 89
Hypothyroidism, 114 in polycystic ovary syndrome, 91 Levonorgestrel-containing contraceptive
causes of, 114 syndrome, 104 methods, 146–148
clinical, 114 Insulin-sensitizing agents Levonorgestrel intrauterine devices
during pregnancy, 53 for hirsutism, 112 in chronic pelvic pain management, 96
screening for, 114 for ovulation induction in patient with for heavy menstrual bleeding, 80
signs and symptoms of, 152–154 polycystic ovary syndrome, 59 in heavy menstrual bleeding management, 106
subclinical, 114, 152–154 Intracytoplasmic sperm injection (ICSI) for leiomyoma in infertility, 36
Hysterosalpingography (HSG), 79 Beckwith–Wiedemann syndrome and, 11 mechanism of action of, 146–148
complications of, 89 in vitro fertilization and, 11 in systemic lupus erythematosus patient, 66
described, 89 Intrauterine devices (IUDs) in venous thromboembolism patient, 19
Hysteroscope(s), 24 complications of, 102 Levonorgestrel-only method, for emergency
Hysteroscopic proximal tubal catheterization, copper, 66 contraception, 65
79 levonorgestrel, 19, 36, 66, 80, 96, 106, Lifestyle choices, pregnancy outcomes related
Hysteroscopic resection, of endometrial polyp, 4 146–148 to, 134–138
Oral corticosteroids, for acne, 84 Placental aromatase deficiency, 68 Primary amenorrhea, 103
Osteopenia, primary ovarian insufficiency and, Pluripotent, induced, 122 gonadal dysgenesis and, 86
83 Pluripotent stem cells, in reproduction, 122 müllerian dysgenesis and, 27
Osteoporosis Polycystic ovary(ies), assisted reproductive Primary dysmenorrhea, 42
celiac disease and, 101 technology and, 8 Primary hypogonadotropic hypogonadism, 88
fractures related to, 94 Polycystic ovary syndrome (PCOS), 104 Primary ovarian insufficiency, 6, 83
functional hypothalamic amenorrhea and, 108 acne and, 84 Primary subclinical hypothyroidism, 114
primary ovarian insufficiency and, 83 abnormal uterine bleeding and, 109 Progesterone
Ovarian androgen-secreting tumor, 98 clinical manifestations of, 78 cyclic, 25
Ovarian cancer cyclic progesterone administration for, 25 in implantation and early embryonic
in BRCA1-positive patient, 13 described, 117 development, 118
oral contraceptives in decreasing risk of, 22 estrogen breakthrough bleeding in, 73 ovarian, 118
Ovarian failure, premature, 83 evaluation of, 78, 98 Progesterone levels, in pregnancy, 139–142
Ovarian follicular depletion, pregnancy out- hirsutism and, 112 Progestin, 19, 25
comes related to, 134–138 insulin resistance testing in, 91 Progestin implant, for obese women, 17
Ovarian hyperstimulation, 100 irregular menstrual pattern with, 127 Progestin-only contraceptives
Ovarian hyperstimulation syndrome obesity and, 51, 78 for acne, 84
assisted reproductive technology and, 8 ovulation induction in patient with, 59 mechanism of action of, 146–148
in vitro fertilization and, 11 pathophysiology of, 109 for patient with venous thromboembolism, 19
Ovarian insufficiency, 117 premature adrenarche and, 62 Progestin-only oral contraceptives, venous
Ovarian progesterone, in implantation and early prevalence of, 59, 73, 78, 109 thromboembolism related to, 57
embryonic development, 118 Polyp(s) Prolactin, in nonclassic adrenal hyperplasia
Ovarian reserve, testing for, 95 endometrial, 4 evaluation, 81
Ovarian Sertoli–Leydig cell tumor, 107 tamoxifen citrate and, 2 Prolactin-induced sexual dysfunction, dopamine
Ovarian stimulation, risks associated with, Positive predictive value (PPV), 57 agonist therapy for, 38
126 Postembolization syndrome, after uterine fibroid Prolactin levels, irregular menstrual cycles and,
Ovarian tissue freezing, 32 embolization, 63 120
Ovarian transposition, 32 Postmenopausal uterine bleeding, hormone Prolactinomas, 10
Ovarian tumors therapy and, 34 Protein levels, reproductive function related
androgen-secreting, 98 Postmenopausal women to, 15
during pregnancy, 68 low bone mass in, 94 Proximal tubal occlusion, 79
Ovary(ies), polycystic, 8 low-dose estrogen therapy in, 72 Proximal tubal repair, 79
Overnight dexamethasone suppression test Postpartum infarction of pituitary, 70 Psychologic counseling, infertility-related, 85
in androgen-secreting ovarian tumor Postpartum thyroiditis, 53 Psychologic effects, of infertility, 85
evaluation, 98 PPV. See Positive predictive value Pubarche, premature, 62
in Cushing syndrome diagnosis, 110 Precocious puberty, 61, 82 Puberty, precocious, 61, 82
Ovulation induction Preconception evaluation Pure gonadal dysgenesis, 128–130
methods of, 90 before donor egg in vitro fertilization, 29
polycystic ovary syndrome and, 59 for obese patients, 125 Q
Ovulation predictor kit, oligomenorrhea and, Pregnancy Quality of life, after bariatric surgery, 9
125 abnormal uterine bleeding in adolescents
related to, 92 R
P alcohol consumption during, 134–138 RAD51C gene mutation, 131–133
Pain, chronic pelvic, 96 complications of, 99 Radioiodine uptake, in hyperthyroidism
PAIS. See Partial androgen insensitivity early, 64, 99 evaluation, 74
syndrome ectopic, 64, 99, 124 Reciprocal translocations, in couples with
PALM–COEIN classification system, for hormonal changes in, 139–142 recurrent pregnancy loss, 21
abnormal uterine bleeding, 18, 92 hyperandrogenism in, 100 Recurrent pregnancy loss, 21, 69
Parathyroid hormone, for female athlete triad, hyperthyroidism in, 53 Relative risk
108 hypothyroidism in, 53 absolute risk versus, 57
Partial androgen insensitivity syndrome (PAIS), intrauterine microinsert and, 35 described, 57
128–130 lifestyle choices effects on, 134–138 Reproduction
Patient autonomy, 1 multifetal, 116 pelvic inflammatory disease effects on, 124
PCOS. See Polycystic ovary syndrome obesity effects on, 51, 125, 134–138, stem cells in, 122
Pelvic inflammatory disease (PID) 155–158 Reproductive function, nutritional status and
Centers for Disease Control and Prevention ovarian tumors during, 68 protein levels related to, 15
on, 124 termination of, 26 Reproductive health, obesity effects on, 125
long-term sequelae of, 124 tubal, 64 Reproductive history, in breast cancer risk
reproductive dysfunction related to, 124 Pregnancy failure, early, 64 prediction, 127
Pelvic pain, chronic, 96 Pregnancy loss, recurrent, 21, 69 Reproductive Medicine Network, on polycystic
Perforation, hysteroscopic resection and, 24 Pregnancy luteomas, 100 ovary syndrome, 59
Perimenopausal changes, 28 Pregnancy reduction, multifetal, 116 Retinoids, for acne, 84
Perimenopause, 28 Preimplantation genetic screening, 71 Retrograde ejaculation, 111
Peripheral precocious puberty, 82 Premature adrenarche, 62 Reversible contraceptives, long-acting, 17
Perrault syndrome, 48 Premature development of secondary sexual Rickets, congenital, 134–138
Peutz–Jeghers syndrome, 131–133 characteristics, evaluation of, 61 Risk(s), absolute versus relative, 57
Phantom β-hCG, results of, 40 Premature ovarian failure, 83 Robertsonian translocations, in couples with
PID. See Pelvic inflammatory disease Premature pubarche, 62 recurrent pregnancy loss, 21
Pioglitazone, for ovulation induction in patient Premature thelarche, 20 Robotic-assisted surgery, complications of, 105
with polycystic ovary syndrome, 59 Presacral neurectomy, for dysmenorrhea, 42 Rosiglitazone, for ovulation induction in patient
Pituitary gland, postpartum infarction of, 70 Preterm delivery, in triplet pregnancy, 116 with polycystic ovary syndrome, 59
Roux-en-Y gastric bypass, 9, 78, 125 Sterilization decision, U.S. Collaborative Trophectoderm biopsy, in preimplantation
Rubella, 29 Review of Sterilization study on, 123 genetic screening, 71
Subclinical hypothyroidism, 114, 152–154 TSH levels. See Thyroid-stimulating hormone
S Surgical reversal, sterilization-related, 123 levels
Salpingectomy, bilateral, 12 Swyer syndrome, 48, 86, 103, 128–130 Tubal disease
Salpingitis, 124 Syndrome X, 68, 104 detection methods, 124
Salpingitis isthmica nodosa, 79 Synthetic androgen female infertility due to, 124
Salt wasting, 115 Synthetic ethinyl estradiol, for patient with Tubal ligation
Same-sex couples, donor sperm for, 39 venous thromboembolism, 19 by clip, 79
SCOFF questionnaire, 54 Systemic lupus erythematosus (SLE), 66 fertility after, 47, 123
Sebum, 84 in vitro fertilization after, 1
Secondary amenorrhea, 6, 10, 110, 117 T modified Pomeroy, 79
Secondary dysmenorrhea, 42 T-ACE Problem Drinking Screen, 134–138 Tubal obstruction, hysterosalpingography and,
Secondary hypogonadotropic hypogonadism, Tamoxifen citrate 89
88 polyps due to, 2 Tubal occlusion, proximal, 79
Secondary sexual characteristics, premature safety of, 2 Tubal pregnancy, intramuscular methotrexate
development of, 61 ultrasonographic findings in patient taking, 2 use in, 64
Sensitivity, described, 57 Tanner staging of breast development, 20 Tubal repair, proximal, 79
Sequential vaginal dilators, in creating func- Teratoma(s), mature cystic, 100 Tubal sterilization, for patient with diabetes
tional vagina, 23 Teriparatide, in bone loss treatment, 94 mellitus, 67
Sertoli–Leydig cell tumor Testosterone Tumor(s). See also specific types
ovarian, 107 for prolactin-induced sexual dysfunction, 38 granulosa cell, 100
virilization due to, 107 virilization and ovarian mass effects on, 107 ovarian, 68, 98
Serum reproductive hormone levels, abnormal Tetracycline, for acne, 84 Sertoli–Leydig cell, 107
uterine bleeding-related, 109 The American College of Obstetricians and sex steroid-producing, 61
Sex steroid-producing tumor, 61 Gynecologists Turner syndrome, 33, 103
Sexual ambiguity, 115 on alcohol consumption during pregnancy, described, 33
Sexual dysfunction, prolactin-induced, 38 134–138 mosaic, 103
Sexual hair growth, androgen production and, on bioidentical hormones, 113 phenotypic findings in, 33
62 on emergency contraception, 65 premature ovarian failure related to, 83
Shapiro–Wilk test, 143–145 Thelarche presentation of, 33
Sheehan syndrome, 70 described, 20 Type 2 diabetes mellitus. See Diabetes mellitus
Sildenafil citrate, for prolactin-induced sexual premature, 20 Type I error, described, 57
dysfunction, 38 Thiazolidinediones, for ovulation induction in
SLE. See Systemic lupus erythematosus patient with polycystic ovary syndrome, U
Small-bowel obstruction, robotic-assisted 59 Ulipristal acetate, as morning-after
surgery and, 105 Three-dimensional ultrasonography, in müllerian contraception, 65
Smoker(s) anomaly evaluation, 76 Ultrasonographic follicular monitoring, in
bone mineral density loss in, 58 Thromboembolism, venous, 19 conception, 121
breast cancer in, 127 Thrombosis, in obese women, 17 Ultrasonography
contraception for older adult, 55 Thyroid antibody tests, 152–154 in androgen-secreting ovarian tumor
Smoking, pregnancy outcomes related to, Thyroid disease, 53, 114 evaluation, 98
134–138 oligo-ovulation screening for, 54 in hypothyroidism screening, 114
Smoking cessation, in bone mineral density loss testing for, 152–154 tamoxifen citrate findings on, 2
prevention, 58 Thyroid gland, in pregnancy, 139–142 three-dimensional, 76
Society for Assisted Reproductive Thyroid-stimulating hormone (TSH) levels Unexplained infertility, 16
Technologies, on in vitro fertilization in hypothyroidism screening, 114 Unscheduled uterine bleeding, depot medroxy-
with own oocytes, 126 irregular menstrual cycles and, 120 progesterone acetate and, 49
Society of Gynecologic Oncologists, on BRCA in Sheehan syndrome, 70 Ureteral injury, laparoscopic surgery and,
gene mutation screening, 43 Thyroiditis 159–161
Specificity, described, 57 autoimmune, 104, 114, 152–154 U.S. Collaborative Review of Sterilization
Sperm Hashimoto, 114, 152–154 study, on sterilization decision, 123
donor use of, 119 postpartum, 53 U.S. Department of Defense, on robotic
fructose in, 5 Thyrotoxicosis, causes of, 74 platform, 105
Spine, dual-energy X-ray absorptiometry scan Thyroxine (T4), in pregnancy, 139–142 U.S. Food and Drug Administration (FDA), 35
of, 94 Tissue cryopreservation with later autotrans- on combination OCs for acne, 84
Spironolactone plantation, in fertility preservation, 75 on donor sperm for same-sex couples, 39
for acne, 84 Tissue freezing, ovarian, 32 on emergency contraception, 65
for hirsutism, 112, 149–151 Total testosterone level, in nonclassic adrenal on estrogen therapy in postmenopausal
Stages of Reproductive Aging Workshop, 28 hyperplasia, 81 women, 72
Stanford Research Institute, on robotic platform, Tranexamic acid on hirsutism treatment, 149–151
105 for abnormal uterine bleeding, 18, 56 on robotic platform, 105
Statistical analysis, 143–145 for heavy menstrual bleeding, 56 on tranexamic acid for heavy menstrual
Statistical testing, 143–145 Transient hyperthyroidism, 53 bleeding, 56
Stem cell(s) Translocation(s), in couples with recurrent U.S. Preventive Services Task Force
in reproduction, 122 pregnancy loss, 21 on nutritional supplementation in bone loss
types of, 122 Triestrogen, for menopausal symptoms, 113 management, 94
Sterilization Triiodothyronine, 152–154 on thyroid-stimulating hormone levels, 114
female, 35 Triplet pregnancy, diseases associated with, U.S. Surgeon General, on alcohol consumption
reversal of, 123 116 during pregnancy, 134–138
tubal, 67 Trisomy X syndrome, 128–130 Uterine artery embolization, 63
FIG. 68-1
FIG. 91-1
FIG. 100-1. Hyperreactio luteinalis showing ovaries
enlarged by multiple thin-walled cysts. (Courtesy
PathologyOutlines.com, Inc., AFIP Fascicle 3rd Series,
Vol. 23.)
C D
FIG. 149–151-1. A patient who presented with hirsutism and clitoromegaly (A) underwent laparoscopic
oophorectomy (B) showing a solid tumor (C) with final pathology (D) consistent with a Sertoli–Leydig cell
tumor.
Acknowledgments
Fig. 10-1 was originally published in Beshay VE, Beshay JE, Halvorson LM. Pituitary
tumors: diagnosis, management, and implications for reproduction. Semin Reprod Med
2007;25:388–401.
Fig. 61-1 was originally published in Dumitrescu CE, Collins MT. McCune–Albright syn-
drome. Orphanet J Rare Dis 2008;3:12.
Fig. 68-1 provided courtesy of Jason S. Yeh, MD, Duke University Medical Center.
Fig. 70-1 was originally published in Sarafoff N, Baur DM, Gaa J, von Beckerath N.
Images in cardiovascular medicine. Recurrent syncope due to torsades de pointes in
a 41-year-old woman with an empty sella, anterior pituitary insufficiency, and a long-QT
interval. Circulation 2009;120:e127–9.
Fig. 91-1 was supplied courtesy of Task Force member J. Ricardo Loret de Mola, MD.
Fig. 149–151-1 was supplied by Task Force member Thomas M. Price, MD.
Appendix B figure showing hydrosalpinx was reproduced courtesy of Gary S. Berger,
MD. This figure previously appeared at http://www.tubal-reversal.net/blog/2011/dr-berger/
diagnosing-and-fixing-blocked-tubes.html.